വിമർശനം:
സൂര്യനാണല്ലൊ ഭൂമിയിലെ ഋതുക്കൾക്ക് കാരണം. അപ്പോൾ നരകത്തിൻ്റെ ശ്വാസമൊ/വിടുതിയൊ ആണ് കാലാവസ്ഥകൾക്ക് കാരണം എന്ന ഹദീസ് ശാസ്ത്ര വിരുദ്ധമല്ലെ ? കൂടാതെ പനി നരകത്തിൻ്റെ ചൂടാണെന്നും, മലക്ക് മേഘങ്ങളെ തെളിച്ചു കൊണ്ടു പോകുമ്പോഴുള്ള ശബ്ദവും വെളിച്ചവുമാണ് ഇടിമിന്നൽ എന്നും ഹദീസുകളിൽ വിവരിക്കപ്പെട്ടതെല്ലാം യാഥാർത്ഥ്യത്തോടും യുക്തിയോടും നിരക്കാത്തതല്ലെ ?
വിമർശനവിധേയമായ ഹദീസ്:
اشْتَكَتِ النَّارُ إلى رَبِّهَا، فَقالَتْ: يا رَبِّ أَكَلَ بَعْضِي بَعْضًا، فأذِنَ لَهَا بنَفَسَيْنِ، نَفَسٍ في الشِّتَاءِ، وَنَفَسٍ في الصَّيْفِ، فَهْوَ أَشَدُّ ما تَجِدُونَ مِنَ الحَرِّ، وَأَشَدُّ ما تَجِدُونَ مِنَ الزَّمْهَرِيرِ.“നരകം തൻ്റെ രക്ഷിതാവിനോട് പരാതിപ്പെട്ടു: “എൻ്റെ ചില ഭാഗം ചിലതിനെ തിന്നു”. അപ്പോൾ രണ്ട് ആശ്വാസം/ശ്വാസം* സമയം നരകത്തിന് അനുവദിക്കപ്പെട്ടു. ഒരു ആശ്വാസം/ശ്വാസം ശൈത്യകാലത്തും മറ്റൊന്ന് ഉഷ്ണകാലത്തുമാണ്. അത് നിങ്ങൾ (ഈ കാലാവസ്ഥകളിൽ) അനുഭവിക്കുന്ന തീക്ഷ്ണമായ ചൂടും തണുപ്പുമാണ്.” (സ്വഹീഹുൽ ബുഖാരി: 3260, സ്വഹീഹു മുസ്ലിം: 617)
* ഹദീസിലെ (النَّفَسُ) എന്ന പദത്തിന് ആശ്വാസം/ശ്വാസം എന്നീ രണ്ട് അർത്ഥവും കൽപ്പിക്കാവുന്നതാണ്.
മറുപടി:
“പാവ”ലോകത്ത്, ഒരു “വിശ്വാസി- നിഷേധി” സംവാദം നടക്കുന്നു!
വിശ്വാസികളുടെ പക്ഷത്തു നിന്ന് വിഷയമവതരിപ്പിച്ച “പാവ” പണ്ഡിതൻ തൻ്റെ അവതരണത്തിന് ഇപ്രകാരം വിരാമം കുറിച്ചു:
“ചുരുക്കത്തിൽ, നാം പാവകൾ സൃഷ്ടിക്കപ്പെട്ടിരിക്കുന്നത് മനുഷ്യ കുഞ്ഞുങ്ങൾക്ക് കളിക്കാനാണ്.
രാവിലെ പെൺകുട്ടിയാണ് നമ്മളെയും കൊണ്ട് കളിക്കുന്നത്. അതുകൊണ്ടാണ് അധികം പരിക്കൊന്നും നമുക്ക് ഉണ്ടാവാത്തത്. വൈകുന്നേരം നമ്മളെയും കൊണ്ട് കളിക്കുന്നത് ആൺകുട്ടിയാണ്. അതുകൊണ്ടാണ് വൈകുന്നേരം നമ്മുടെ പരിക്കുകൾ കൂടുന്നതും.
എന്തു കൊണ്ടാണ് നാം പാട്ട് പാടുന്ന പാവകളായത് ? മനുഷ്യ കുട്ടികൾക്ക് പാട്ടു കേൾക്കാൻ ഇഷ്ടമായതു കൊണ്ടാണത്.
എന്തുകൊണ്ടാണ് -പാവകളായ- നമ്മുടെ ശരീരം മിനുസമുള്ളതായത് എന്ന് ചിന്തിച്ചു നോക്കിയിട്ടുണ്ടോ? കൂർത്ത ഭാഗങ്ങളുണ്ടായാൽ, കുഞ്ഞുങ്ങളുടെ മൃദുലമായ കൈകളിൽ മുറിവേൽക്കും എന്നതുകൊണ്ടാണത്.
നമ്മുടെ വലിപ്പം ഇത്രയായി തിട്ടപ്പെടുത്തപ്പെട്ടു എന്നതിനും ഒരു കാരണമുണ്ട്. നാം നന്നേ ചെറുതായാൽ കുട്ടികൾ നമ്മെ വിഴുങ്ങിക്കളഞ്ഞേക്കും, നാം വളരെ വലുതായാൽ കുട്ടികൾക്ക് നമ്മെ താങ്ങാൻ കഴിയില്ല എന്നതിനാലുമാണത്.
നമ്മുടെ പ്രകൃതത്തിനും സൃഷ്ടിപ്പിനും ഓരോ ചലനത്തിനു പോലും അർത്ഥവും ലക്ഷ്യവുമുണ്ട് എന്ന് ചുരുക്കം…”
എതിർ പക്ഷത്തു നിന്ന് എഴുന്നേറ്റ് നിന്ന, നിഷേധിയായ പാവ പരിഹാസപൂർവ്വം ചിരിച്ചു കൊണ്ട് പറഞ്ഞു:
“യാഥാർത്ഥ്യങ്ങൾക്ക് നിരക്കാത്ത, ശാസ്ത്ര വിരുദ്ധമായ എത്ര വലിയ വിഡ്ഢിത്തങ്ങളാണ് മറുപക്ഷം ഇവിടെ വിശദീകരിച്ചിരിക്കുന്നത്!
രാവിലെ പെൺകുട്ടിയും വൈകുന്നേരം ആൺകുട്ടിയുമാണ് നമ്മളെയും കൊണ്ട് കളിക്കുന്നത് എന്നതിനാലല്ല നമ്മുക്ക് പരിക്ക് കൂടുകയും കുറയുകയും ചെയ്യുന്നത്. മറിച്ച് രാവിലെ നമ്മുടെ ശരീരം സ്വഭാവികമായും ശാന്തമായതു കൊണ്ടാണ് നമ്മുക്ക് പരിക്കുകൾ കുറയുന്നത്. വൈകുന്നേരങ്ങളിലാകട്ടെ നമ്മുടെ ശരീരം പ്രകൃത്യാ ചലനാത്മകമാണ്. അതുകൊണ്ടാണ് പരിക്കുകൾ കൂടുന്നത്. ആൺകുട്ടിയും പെൺകുട്ടിയും മാറി മാറി കളിക്കുന്നതിനാലാണ് പരിക്കുകളിൽ ഏറ്റക്കുറച്ചിലുകൾ ഉണ്ടാവുന്നത് എന്ന അന്ധവിശ്വാസം (നമ്മുടെ പരിക്കുകളെ കുറിച്ചും പ്രതിവിധികളെ കുറിച്ചുമുള്ള) വൈദ്യ ശാസ്ത്രത്തിൻ്റെ പുരോഗമനത്തിന് തടസ്സമാണ്.
മനുഷ്യ കുട്ടികൾക്ക് പാട്ടു കേൾക്കാൻ ഇഷ്ടമായതു കൊണ്ടാണ് നാം പാട്ടുകാരായത് എന്ന വാദം വിചിത്രം തന്നെ! മെക്കാനിക്കൽ ഫോണോഗ്രാഫ്, ടേപ്പ് റെക്കോർഡറുകൾ, ഇലക്ട്രോണിക് സിന്തസൈസർ എന്നിവയൊക്കെയാണ് നാം പാട്ടുകാരാവാൻ കാരണം എന്ന് ശാസ്ത്രം ഇന്ന് തെളിയിച്ചു കഴിഞ്ഞിട്ടുണ്ട്.
കുട്ടികളുടെ കൈകളിൽ മുറിവാവരുത് എന്നതാണ് നമ്മുടെ ശരീരത്തിൻ്റെ മിനുസത്തിന് കാരണം എന്ന വിശ്വാസം അതിനേക്കാൾ വലിയ തമാശ! സിലിക്കണു കൊണ്ട് നിർമ്മിക്കപ്പെട്ടതിനാലാണ് നമ്മുടെ ശരീരം ഇത്ര മൃദുലം എന്ന വിവരം പോലും ഇവർക്കില്ലെ?!
പിന്നെ, നമ്മളെ വിഴുങ്ങാൻ മാത്രം വലുതാണ് ഈ കുട്ടികൾ എങ്കിൽ നമ്മളെ താങ്ങാനും അവർക്ക് കഴിയേണ്ടതല്ലെ?…
ചുരുക്കത്തിൽ, നമ്മുടെ സൃഷ്ടിപ്പും പ്രകൃതിയും ചലനങ്ങളാസകലവും ആകസ്മികതകൾ മാത്രമാണ്. ഒന്നിനും വസ്തുനിഷ്ഠമായ ഒരു അർത്ഥവും ലക്ഷ്യവും ഇല്ല…
സംവാദം കരഘോഷങ്ങളുടെ ആരവങ്ങളോടെ തുടർന്നുകൊണ്ടിരുന്നു…
ഭൗതിക ലോകത്തെയും പ്രകൃതി പ്രതിഭാസങ്ങളെയും വിശകലനം ചെയ്യുമ്പോൾ (ഇസ്ലാം) മതവും ശാസ്ത്രവും മിക്കവാറും രണ്ട് വ്യത്യസ്ത മേഖലകളെയും കാരണങ്ങളെയുമാണ് ചർച്ച ചെയ്യുന്നത് എന്ന് പാവ ലോകത്തെ ഈ വിശ്വാസി-നിഷേധി സംവാദത്തിലൂടെ വായനക്കാർക്ക് മനസ്സിലായിരിക്കുമല്ലൊ. ശാസ്ത്രം പ്രപഞ്ചത്തിലെ ഭൗതിക കാരണങ്ങളെ കുറിച്ച് സംസാരിക്കുമ്പോൾ മതം ആ പ്രപഞ്ചത്തിൻ്റെ ഭൗതിക കാരണങ്ങൾക്ക് പിന്നിലെ ദൈവിക ലക്ഷ്യത്തെയും ഉദ്ദേശ്യത്തേയുമാണ് കൈകാര്യം ചെയ്യുന്നത്. ഈ രണ്ട് വിശദീകരങ്ങളും തമ്മിൽ വൈരുദ്ധ്യമില്ലെന്ന് “പാവ സംവാദ” ത്തിലൂടെ മനസ്സിലാക്കാമല്ലൊ. പാവകൾ വിനോദ ഉപകരണങ്ങളാണെന്നും അവയുടെ സൃഷ്ടിപ്പിലെ ഓരോ ഭൗതിക കാരണങ്ങൾക്കും (ആകാരം, മൃതുലത, പാട്ട്, പരിക്കുകൾ) പിന്നിൽ അഭൗതിക (പാവേതര) കാരണങ്ങൾ ഉണ്ട് എന്നും അംഗീകരിക്കാനും, ഈ ഭൗതികവും അഭൗതികവുമായ കാരണങ്ങളെ പരസ്പരം സംയോജിപ്പിക്കാനും ഒരു നിഷേധിക്ക് കഴിയില്ല. അതിന് കാരണം നിഷേധി, പാവലോകത്തിനപ്പുറം ഉള്ളതെല്ലാം നിഷേധിക്കുന്നു എന്നത് മാത്രമല്ല. ഒരു പ്രതിഭാസത്തിന് ഒരേ സമയം പല കാരണങ്ങളും വ്യാഖ്യാനങ്ങളും ഉണ്ടാവൽ ബുദ്ധിപരമാണ്, എന്ന് തത്ത്വത്തിലെങ്കിലും അംഗീകരിക്കാനുള്ള യുക്തിയൊ വിനയമൊ ഇല്ലാത്തതു കൊണ്ടു കൂടിയാണത്. പാവയുടെ മെയ്യഴകിന് കാരണം സിലിക്കണെന്ന ഭൗതിക കാരണമാണ് എന്നതോടൊപ്പം തന്നെ, പാവ കൊണ്ട് കളിക്കുന്ന കുട്ടിയുടെ പിഞ്ചു കൈയ്യിൽ മുറിവേൽക്കാതിരിക്കുക എന്ന ഉദ്ദേശ്യ-ലക്ഷ്യപരമായ (Intentional- Purposefull) കാരണം കൂടി ഉണ്ട് എന്ന് ഒരാൾ അംഗീകരിച്ചാൽ അവ തമ്മിൽ എങ്ങനെ വൈരുദ്ധ്യമാവും ?! പാവ ലോകത്തെ ശാസ്ത്രത്തിൻ്റെ വളർച്ചക്കോ പുരോഗമനത്തിനൊ ഈ ഉദ്ദേശ്യ-ലക്ഷ്യപരമായ കാരണം എങ്ങനെ തടസ്സമാവും ?!
പാവലോകത്തിനപ്പുറം ഒരു ലോകമില്ലെന്നും, പാവപ്രകൃതിക്ക് ഒരു ഉദ്ദേശ്യ-ലക്ഷ്യവും ഇല്ലെന്നും ഒരു പാവക്ക് കാരണങ്ങളൊന്നും കൂടാതെ വാദിക്കാം. പക്ഷെ ഈ നിഷേധത്തിന് തെളിവായി, പാവ ലോകത്തെ ഭൗതിക കാരണങ്ങൾ ഉദ്ധരിക്കുന്നത് ന്യായ വൈകല്യം മാത്രമാണ്. സിലിക്കൺ പദാർത്ഥം കൊണ്ടാണ് ഉണ്ടാക്കപ്പെട്ടിരിക്കുന്നത് എന്ന ഭൗതിക കാരണം, അതിന് പിന്നിലെ, കുട്ടിയുടെ കൈയ്യിൽ മുറിവേൽക്കാതിരിക്കുക എന്ന ഉദ്ദേശ്യ-ലക്ഷ്യപരമായ (Intentional- Purposefull) കാരണത്തെ തെളിയിക്കുകയല്ലെ ചെയ്യുന്നത്? അല്ലാതെ ഈ ഉദ്ദേശ്യ-ലക്ഷ്യപരമായ കാരണത്തെ നിഷേധിക്കാൻ, സിലിക്കൺ പദാർത്ഥം കൊണ്ട് ഉണ്ടാക്കപ്പെട്ടിരിക്കുന്നു എന്ന ഭൗതിക കാരണം എങ്ങനെ ന്യായമാവും ?!
***********************ഇനി വിമർശന വിധേയമായ ഹദീസിലേക്ക് വരാം… ഭൗമിക കാലാവസ്ഥകളുടെ ഭൗതിക കാരണങ്ങൾ വിശകലനം ചെയ്യുകയല്ല ഹദീസ് ചെയ്യുന്നത് എന്ന് വിമർശകർ മനസ്സിലാക്കണം. പ്രകൃതിശാസ്ത്രങ്ങളുടെ ഭൗതിക വിശദീകരണങ്ങൾക്കപ്പുറം അദൃശ്യവും മനുഷ്യർക്ക് അപ്രാപ്യവുമായ ആത്മീയ അന്തസാരങ്ങൾ പങ്കു വെക്കുകയാണ് (ഇസ്ലാം) മതം ചെയ്യുന്നത്. ശാസ്ത്രം, പ്രകൃതി പ്രതിഭാസങ്ങളുടെ ഭൗതിക കാരണങ്ങൾ (material cause) ചർച്ച ചെയ്യുമ്പോൾ, മതം പ്രകൃതി പ്രതിഭാസങ്ങളുടെ അന്തിമ കാരണമാണ് (Final cause) അനാവരണം ചെയ്യുന്നത്; ഒരു പ്രകൃതി പ്രതിഭാസത്തിൻ്റെ ദൈവിക ഉദ്ദേശ്യം അല്ലെങ്കിൽ ദൈവികലക്ഷ്യം… അല്ലെങ്കിൽ ഒരു കാര്യം സ്വാഭാവികമായി വികസിച്ചെത്തുന്ന അതിൻ്റെ അവസാനം, ഇതൊക്കെയാണ് മതത്തിൻ്റെ ചർച്ചാവിഷയം. (കൂടുതൽ വിവരങ്ങൾക്ക്: https://www.snehasamvadam.org/തെറ്റിദ്ധരിക്കപ്പെട്ട-9/)
ഭൗതിക കാരണവും (material cause) അന്തിമ കാരണവും (Final cause) തമ്മിലുള്ള വ്യത്യാസം മനസ്സിലാവാത്തവർ ശാസ്ത്ര പുസ്തകങ്ങളിൽ മനപാഠമാക്കിയ വരികളിൽ നിന്നും “വ്യത്യസ്തമായ” എന്ത് വിശദീകരണങ്ങളും വർണനകളും കണ്ടാലും അതൊക്കെ ശാസ്ത്ര “വിരുദ്ധമായി” തെറ്റിദ്ധരിക്കുന്നു എന്ന് മാത്രം.
വിമർശനവിധേയമായ ഹദീസിനെ മുസ്ലിം സമൂഹവും ആദ്യകാല ഹദീസ് ശാസ്ത്രജ്ഞരും പല രീതിയിലും മനസ്സിലാക്കിയതായി പൗരാണിക ഹദീസ് ഗ്രന്ഥങ്ങളിൽ നിന്നും മനസ്സിലാവുന്നു. വിശദാംശങ്ങളിൽ ശാസ്ത്രത്തിൻ്റെ മേഖലക്ക് അപ്പുറമുള്ള വിവരങ്ങൾ ഉണ്ടാവാമെങ്കിലും ഈ വ്യാഖ്യാനങ്ങളിൽ ഏത് സ്വീകരിച്ചാലും ബുദ്ധിക്കൊ ശാസ്ത്രത്തിനൊ എതിരായി ഒന്നും ഹദീസിൽ കണ്ടെത്താൻ കഴിയില്ല എന്നതാണ് വസ്തുത.
വ്യാഖ്യാനം 1:
ഹദീസ് അനാവരണം ചെയ്യുന്നത് ഭൗമികമായ കാലാവസ്ഥയെ സംബന്ധിച്ച് അല്ലേയല്ല. പ്രത്യുത, നരകത്തിലെ കഠിനമായ ചൂടിൻ്റെയും തണുപ്പിൻ്റെയും ഭീകരത എത്രത്തോളമാണ് എന്ന് ബോധ്യപ്പെടുത്തുകയാണ് ഹദീസിൻ്റെ ഉദ്ദേശ്യം.
അതായത്, വർഷത്തിൽ രണ്ട് തവണ നരകത്തിന് ആശ്വാസം/ശ്വാസം അല്ലാഹു അനുവദിച്ചു. ആ ആശ്വാസ/ശ്വാസ വേളയിൽ നരകത്തിൻ്റെ ചൂടിൻ്റെയും തണുപ്പിൻ്റെയും കാഠിന്യം കുറയുമല്ലൊ. എന്നാൽ നരകത്തിലെ ഈ കാഠിന്യം കുറഞ്ഞ ചൂടും തണുപ്പും പോലും, ഭൗമികമായി ശൈത്യകാലത്തും ഉഷ്ണകാലത്തും നാം അനുഭവിക്കുന്ന ഏറ്റവും തീക്ഷ്ണമായ ചൂടും തണുപ്പിനോടും തുല്യമായിരിക്കും. അഥവാ, ഭൗമിക കാലാവസ്ഥകളിൽ നാം അനുഭവിക്കുന്ന ഏറ്റവും തീക്ഷ്ണമായ ചൂടിനോടും തണുപ്പിനോടും തുല്യമായിരിക്കും നരകത്തിലെ ഏറ്റവും കാഠിന്യം കുറഞ്ഞ ചൂടും തണുപ്പും. എങ്കിൽ നരകത്തിൻ്റെ സ്വാഭാവിക താപവും കുളിരും എത്ര അസഹനീയമായിരിക്കും !!!
فَهْوَ أَشَدُّ ما تَجِدُونَ مِنَ الحَرِّ، وَأَشَدُّ ما تَجِدُونَ مِنَ الزَّمْهَرِيرِ. “അത് നിങ്ങൾ (ഈ കാലാവസ്ഥകളിൽ) അനുഭവിക്കുന്ന തീക്ഷ്ണമായ ചൂടും തണുപ്പുമാണ്.” എന്ന ഹദീസിലെ ഭാഗം ഇതാണ് സൂചിപ്പിക്കുന്നത്.
ഇസ്ലാം വിരോധികളുടെ ഹദീസ് നിരൂപണങ്ങൾ ഉദയം കൊള്ളുന്നതിന് നൂറ്റാണ്ടുകൾക്ക് മുമ്പ് തന്നെ ഇത്തരമൊരു വ്യാഖ്യാനം, മുസ്ലിം ഹദീസ് പണ്ഡിതർ ഹദീസിന് നൽകിയിട്ടുണ്ട് എന്ന് നാം മനസ്സിലാക്കണം. ഉദാഹരണത്തിന്, ഹിജ്രാബ്ദം 476 ൽ (1083 CE) ഭൂജാതനായ കാദി ഇയാദ് എഴുതിയത് കാണുക: “…ഭൗമികമായ ഉഷ്ണകാലത്തെ നരകത്തോട് സദൃശ്യപ്പെടുത്തുകയും ഉപമിക്കുകയുമാണ് ഹദീസ്, അതിനാൽ നരകത്തിൻ്റെ കഠിനമായ ഉഷ്ണത്തെ ഭയപ്പെടുക, അത് വന്നു ഭവിക്കുന്ന അവസ്ഥയെ ഇല്ലാതാക്കുക.” (ശർഹു മുസ്ലിം: ഇമാം നവവി :5:120, ശർഹു സുനനു ഇബ്നു മാജ: സുയൂത്വി: 1:321, ഫത്ഹുൽ ബാരി: ഇബ്നു ഹജ്ർ: 10: 186)
രണ്ട് ശ്വാസങ്ങൾ/ആശ്വാസങ്ങളിലൂടെ, നരകത്തിലെ രണ്ട് ഘട്ടങ്ങളിലുണ്ടാവുന്ന ഏറ്റവും കുറഞ്ഞ ചൂടും തണുപ്പും, ഭൂമിയിലെ ഏറ്റവും കഠിനമായ ഉഷ്ണ കാലത്തെയും ശൈത്യകാലത്തെയും പോലെയായിരിക്കും എന്ന് ചുരുക്കം.
വ്യാഖ്യാനം 2:
“നരകം” ഒരു ഭാഗത്ത് ഉറച്ചു നിൽക്കുന്ന ഒരു അസ്തിത്വത്തെ മാത്രം വിളിക്കുന്ന സാങ്കേതിക പദമല്ല എന്നതാണ് ഇസ്ലാമിക വീക്ഷണം.
1. എല്ലാ ശിക്ഷയെയും ശിക്ഷാ ഹേതുവെയും “നരകം” എന്ന് വിളിക്കാം എന്നാണ് ഹദീസുകളിൽ നിന്ന് മനസ്സിലാവുന്നത്.
അന്യായമായ ഭൂമി അപഹരിച്ചാൽ “നരക”ത്തിൻ്റെ ഒരു കണ്ടമാണ് ഒരാൾ സ്വന്തമാക്കുന്നത് എന്ന് നബി (സ) പറഞ്ഞത് ഉദാഹരണം: فإنَّما أقْطَعُ له قِطْعَةً مِنَ النَّارِ. (സ്വഹീഹുൽ ബുഖാരി: 7169)2. പ്രയാസകരമായ ഭൗമീകാനുഭവങ്ങൾ ഒരു വിശ്വാസിയുടെ പാപങ്ങളെ മായ്ച്ചു കളയാനും ഇല്ലാതാക്കാനും ഉതകുന്നതാണ്. അതിലൂടെ നരകശിക്ഷയിൽ നിന്നും ഒരു വിശ്വാസി മോക്ഷം നേടുന്നു. ഇത്തരം പ്രയാസങ്ങളെയും ക്ലേശങ്ങളെയും (ഉദാഹരണം, കഠിനമായ വെയിൽ, പനി…) “നരകം” എന്നു തന്നെയാണ് ഹദീസുകൾ പരിചയപ്പെടുത്തിയിട്ടുള്ളത്… മനുഷ്യരുടെ പാപങ്ങളെ കരിച്ചു കളയുന്നതായ എല്ലാ ചൂടും പ്രയാസവും ക്ലേശങ്ങളും നരകമാണ്…
لسَّفَرُ قِطْعَةٌ مِنَ العَذَابِ“യാത്ര ശിക്ഷയിൽ നിന്നുള്ള ഒരു കഷ്ണമാണ്…” (സ്വഹീഹുൽ ബുഖാരി: 1804 )
فإنَّ شِدَّةَ الحَرِّ مِن فَيْحِ جَهَنَّمَ “കൊടും ചൂട് നരകത്തിൻ്റെ ചൂടിൽ നിന്നുമുള്ളതാണ്…” (സ്വഹീഹു മുസ്ലിം: 617)
الْحُمَّى مِنْ فَيْحِ جَهَنَّمَ فَابْرُدُوهَا بِالْمَاءِ “പനി നരകത്തിൻ്റെ ചൂടിൽ നിന്നും ഉള്ളതാണ്…” (സ്വഹീഹുൽ ബുഖാരി: 5725)
ഇബ്നുൽ ക്വയ്യിം (ജനനം ഹിജ്രാബ്ദം: 751 : CE 1292) എഴുതി:
“രണ്ടു രീതിയിലാണ് ഈ ഹദീസ് മനസ്സിലാക്കാവുന്നത്:
a) ഭൗമികമായ ക്ലേശങ്ങളും പനി പോലെയുള്ള രോഗങ്ങളുമെല്ലാം നരക ശിക്ഷക്ക് ചെറിയ സാമ്യതയോടെ അല്ലാഹു നിശ്ചയിച്ചത് മനുഷ്യർക്ക് നരകാനുഭവത്തിൻ്റെ ഒരു ലഘുവായ അനുഭൂതി മനസ്സിലാക്കാനും പാഠം ഉൾക്കൊള്ളാനുമാണ്. (Final cause – ലേ)
ഈ ഉദ്ദേശ്യത്തെ അടിസ്ഥാനപ്പെടുത്തി പിന്നീട് അതിന് അനുയോജ്യമായ ഭൗതിക കാരണങ്ങൾ (meterial cause -ലേ) അല്ലാഹു പനിക്കും രോഗത്തിനും നിർണയിച്ചു.
ആശ്വാസം, വിടുതി , സന്തോഷം, ആസ്വാദനങ്ങൾ എന്നിവ പരലോകത്തെ സ്വർഗത്തിൻ്റെ ഉള്ളടക്കങ്ങളാണ്. എന്നാൽ അവയെ കുറിച്ച് മനസ്സിലാക്കാനും ഉൾകൊള്ളാനും സങ്കൽപ്പിക്കാനും മനുഷ്യർക്ക് കഴിയാനായി അതിന് അനുയോജ്യമായ സുഖ-സന്തോഷാനുഭവങ്ങളും ഭൂമിയിൽ അല്ലാഹു നിശ്ചയിച്ചു.
b) പനിയുടെ രൂക്ഷതയെ നരകത്തിൻ്റെ കാഠിന്യവുമായി ഉപമിക്കുകയാണ് ഹദീസ് ചെയ്തത്. നരക ശിക്ഷയെ കുറിച്ച ബോധം നമ്മിൽ അങ്കുരിക്കപ്പെടുകയാണ് ഈ ഉപമയിലൂടെ നബി (സ) ഉദ്ദേശിച്ചത്. ( അല്ലാതെ പനിയുടെ ഭൗതിക കാരണം (meterial cause) വിശദീകരിക്കുകയല്ല.) (അത്ത്വിബ്ബുന്നബവിയ്: ഇബ്നുൽ ക്വയ്യിം: 21)
ഇബ്നു ഹജർ (ജനനം ഹിജ്രാബ്ദം: 773 : CE 1371) എഴുതി:
“പനിയെ നരകത്തോട് ചേർത്തി പറഞ്ഞതിൻ്റെ വ്യാഖ്യാനം പലതാണ്: പനി അക്ഷരാർത്ഥത്തിൽ നരകത്തിൽ നിന്നുള്ളതാണ് എന്നാണ് ഒരു വ്യാഖ്യാനം. പനിയിലൂടെ ശരീരത്തിൽ ഉളവാകുന്ന താപം, മനുഷ്യർ നരകത്തെ ലഘുവായി അനുഭവിക്കാനും ഉൽബുദ്ധരാവാനും വേണ്ടി അല്ലാഹു നിശ്ചയിച്ചതാണ്. ആ ഉദ്ദേശ്യത്തിൻ്റെ അടിസ്ഥാനത്തിൽ അല്ലാഹു ഭൗതികമായ കാരണങ്ങൾ പനിക്ക്, പിന്നീട് നിശ്ചയിച്ചു നൽകി… ബസ്സാർ തൻ്റെ മുസ്നദിൽ പ്രവാചക പത്നി ആഇശയിൽ (റ) നിന്ന് നിവേദനം ചെയ്തതായി വന്ന ഹദീസിലും, ഇമാം അഹ്മദ്, ത്വബ്റാനി എന്നിവർ പ്രവാചകാനുചരന്മാരായ ഇബ്നു മസ്ഊദ്, അബൂ ഉമാമ എന്നിവരിൽ നിന്നും ഉദ്ധരിച്ച ഹദീസിൽ ഇപ്രകാരം വ്യക്തമായി പ്രസ്ഥാവിക്കപ്പെട്ടിരിക്കുന്നു: الحمى حظ المؤمن من النار. “പനി, വിശ്വാസിക്ക് നരകത്തിൽ നിന്നും നൽകപ്പെടുന്ന ഒരു വിഹിതമാണ്.”…” (പാപങ്ങൾ പൊറുക്കപ്പെടാനുള്ള കാരണമായി പനിയെ പരിഗണിക്കുകയും നരക ശിക്ഷയുടെ വിഹിതം പനിയായി ലഘൂകരിച്ച് ഇഹലോകത്ത് തന്നെ അല്ലാഹു നൽകുകയും ചെയ്യുന്നു. അപ്പോൾ പനി അക്ഷരാർത്ഥത്തിൽ നരകത്തിൽ നിന്നും ഉള്ളതാണെന്ന് പറയാമല്ലൊ.” (ഫത്ഹുൽ ബാരി: ഇബ്നു ഹജ്ർ: 10: 186)
പനി എന്തുകൊണ്ട് ചൂടുള്ളതായി ? എന്ന ചോദ്യത്തിന് പല കാരണങ്ങളും പല വീക്ഷണകോണിൽ നിന്ന് കൊണ്ട് പറയാവുന്നതാണ്. പനിയുടെ ഭൗതിക കാരണം (meterial cause), ഹദീസിൽ പറയപ്പെട്ട ആത്മീയ കാരണത്തോട് ഒരിക്കലും വിരുദ്ധമാവുന്നില്ല. ഹദീസ് സംസാരിക്കുന്നത് പനിയുടെ അവസാന കാരണം അല്ലെങ്കിൽ അന്തിമ കാരണത്തെ (Final cause) സംബന്ധിച്ചാണ് എന്ന് എത്രയോ നൂറ്റാണ്ടുകൾക്ക് മുമ്പ് തന്നെ ഇമാം ഇബ്നുൽ ക്വയ്യിമും ഇബ്നു ഹജറും വ്യക്തമാക്കിയിരിക്കുന്നു !!
ഈ കാരണങ്ങൾ തമ്മിൽ വ്യത്യസ്തമാണ് എങ്കിലും പരസ്പര വിരുദ്ധമല്ല. പനിയുടെ ഭൗതിക കാരണങ്ങളെ സംബന്ധിച്ച തുടർച്ചയായ അന്വേഷണങ്ങളിലൂടെ ഇത് ഏതൊരാൾക്കും മനസ്സിലാക്കാവുന്നതാണ്:
എന്തു കൊണ്ട് നമ്മെ പനി ബാധിക്കുന്നു?
അണുബാധ മൂലം ശരീര താപനിലയിലെ താൽക്കാലിക വർദ്ധനവാണ് പനി. ശരീരത്തിൻ്റെ പ്രതിരോധ സംവിധാനത്തിൽ നിന്നുള്ള മൊത്തത്തിലുള്ള പ്രതികരണത്തിൻ്റെ ഒരു ഭാഗമാണിത്. ( https://www.mayoclinic.org/diseases-conditions/fever/symptoms-causes/syc-20352759 )
എങ്കിൽ, പനിക്ക് എന്തുകൊണ്ട് ചൂട് ?
സാധാരണയായി വൈറസ്, ബാക്റ്റീരിയ ബാധയെ പ്രതിരോധിക്കാനും മുക്തമാക്കാനും വേണ്ടിയാണ് ശരീരം താപനിലയെ താൽക്കാലിക വർദ്ധിപ്പിക്കുന്നത്.
( https://www.mayoclinic.org/diseases-conditions/fever/symptoms-causes/syc-20352759 )
എങ്കിൽ, വൈറസ്, ബാക്റ്റീരിയ ബാധയെ ചൂട് എന്തു കൊണ്ട് പ്രതിരോധിക്കുന്നു?
വൈറസ് ബാധ മൂലമാണ് വൈറൽ പനി ഉണ്ടാകുന്നത്. വൈറസുകൾ വളരെ ചെറിയ പകർച്ചവ്യാധി ഏജൻ്റുകളാണ്. അവ നിങ്ങളുടെ ശരീരത്തിലെ കോശങ്ങളെ ബാധിക്കുകയും പെരുകുകയും ചെയ്യുന്നു. വൈറസിനെതിരെ പോരാടാനുള്ള നിങ്ങളുടെ ശരീരത്തിൻ്റെ മാർഗമാണ് പനി. പല വൈറസുകളും താപനിലയിലെ മാറ്റങ്ങളോട് സംവേദനക്ഷമമാണ്, അതിനാൽ നിങ്ങളുടെ ശരീര താപനിലയിലെ പെട്ടെന്നുള്ള വർദ്ധനവ് നിങ്ങളെ വൈറസുകൾക്ക് കോശങ്ങളിലുള്ള ആതിഥ്യം തടസ്സപ്പെടുത്തുന്നു.
(https://www.healthline.com/health/viral-fever)
എങ്കിൽ, വൈറസുകൾ എന്തുകൊണ്ട് താപനിലയിലെ മാറ്റങ്ങളോട് സംവേദനക്ഷമമായി ?
വൈവിധ്യമാർന്ന വൈറസുകൾ വ്യത്യസ്തമായ താപനില സംവേദനം പ്രകടിപ്പിക്കുന്നു. ഓരോ വൈറസും ആതിഥേയ കോശത്തിൽ ഉൽപ്പാദിപ്പിക്കുന്ന മിക്ക പ്രോട്ടീനുകളും താപനില സംവേദനം പ്രകടിപ്പിക്കുന്നു.
എന്തു കൊണ്ട് ?
the mechanism of temperature sensitivity is not known…
താപനില സംവേദനക്ഷമതയുടെ സംവിധാനം ഇതു വരെ നമുക്ക് അറിയില്ല. പക്ഷേ വൈറൽ എൻസൈമുകളുടെ ഘടനയിലെ ചെറിയ മാറ്റങ്ങളുമായി ബന്ധപ്പെട്ടതായിരിക്കാം അത്…
(https://www.ncbi.nlm.nih.gov )
ശാസ്ത്രമെത്ര വികസിച്ചാലും, പ്രകൃതി പ്രതിഭാസങ്ങളുടെ വികാസത്തിലെ ഒരു ഘട്ടത്തിലെ ഭൗതിക കാരണം (meterial cause) മാത്രം വിശദീകരിക്കാനെ അതിന് കഴിയു. അതിൻ്റെ ആത്യന്തികമായ കാരണം (അഥവാ ദൈവിക ലക്ഷ്യം) വിശദീകരിക്കാൻ ശാസ്ത്രത്തിന് കഴിയില്ല. കാരണം ശാസ്ത്രത്തിൻ്റെ വിഷയത്തിനും പരിമിതിക്കും അപ്പുറമാണത്.
എന്തുകൊണ്ട് പനി ചൂടായി. തണുപ്പൊ മറ്റെന്തെങ്കിലും ആവാമായിരുന്നില്ലെ. വൈറസുകളിൽ എന്തുകൊണ്ട് ചൂടിനോട് തന്നെ സംവേദനമുണ്ടായി? എന്തുകൊണ്ട് എന്നതിനുള്ള ഉത്തരം മനുഷ്യ ശരീരത്തെയും വൈറസിനെയും സൃഷ്ടിച്ച ദൈവത്തിനല്ലെ പറയാനാകു? ദൈവത്തിൻ്റെ ഉദ്ദേശ്യം അല്ലെങ്കിൽ ലക്ഷ്യമാണ് ആത്യന്തികമായി ഒരു പ്രതിഭാസത്തിൻ്റെ ഏറ്റവും അടിസ്ഥാനപരമായ പ്രകൃതത്തെ നിർണയിക്കുക.
ആ ദൈവം പറയുന്നു… നരകത്തെ നിങ്ങൾ ഓർക്കാനും, പാപങ്ങൾ പൊറുത്തു നൽകാനും വേണ്ടി നരകച്ചൂടിൻ്റെ ഒരു പൊട്ട് ചൂട് നിങ്ങൾക്ക് നൽകാനാണ് പനി ചൂടായി നിശ്ചയിച്ചത്. ഈ ദൈവിക ഉദ്ദേശ്യത്തെ ഭൗതികമായി നടപ്പാക്കാൻ ഭൗതികമായ കാരണങ്ങൾ ദൈവം നിശ്ചയിക്കുകയും ചെയ്തു. ആ ഭൗതികാരണമാണ് ശാസ്ത്രത്തിലൂടെ മനുഷ്യർ കണ്ടെത്തുന്നത്. അതിനു പിന്നിലെ ദൈവിക ഉദ്ദേശ്യമാണ് മതത്തിലൂടെ മനുഷ്യർ കണ്ടെത്തുന്നത്. ഈ രണ്ട് കാരണങ്ങളും ആശയപരമായി പരസ്പരം എതിരല്ല. അവ രണ്ടും തമ്മിൽ വ്യത്യസ്തമായിരിക്കാം, പക്ഷെ തമ്മിൽ തെല്ലും വൈരുധ്യമില്ല.
സമാനമായ രീതിയിൽ തന്നെയാണ്, മറ്റു പ്രതിഭാസങ്ങളുടെ ആത്മീയ വ്യാഖ്യാനങ്ങളെ മനസ്സിലാക്കാൻ.
“മലക്ക് മേഘങ്ങളെ തെളിച്ചു കൊണ്ടുപോകുമ്പോളുള്ള ശബ്ദവും വെളിച്ചവുമാണ് ഇടിമിന്നൽ…” എന്ന് ചില നിവേദനങ്ങൾ കാണാം. ഹദീസ് സ്വഹീഹാണെന്ന് വന്നാൽ തന്നെ ഇടിമിന്നൽ എന്ന പ്രതിഭാസത്തിൻ്റെ ഭൗതിക കാരണം വിശദീകരിക്കുകയല്ല ഹദീസിലെ ഉദ്ദേശ്യം. മറിച്ച് പ്രകൃതി പ്രതിഭാസങ്ങളുടെ ഭൗതിക കാരണങ്ങൾക്കപ്പുറം അഭൗതികവും ആത്യന്തികവുമായ ദൈവിക ഉദ്ദേശ്യമാണ് ഹദീസിൽ വർണിക്കപ്പെടുന്നത്.
ഇടിയുടെയും മിന്നലിൻ്റെയും ഭൗതിക കാരണങ്ങളുടെ (Meterial Cause) അറ്റം തേടി പോയാൽ ഭൗതിക കാരണങ്ങൾക്കപ്പുറം മറ്റെന്തെങ്കിലുമൊരു കാരണം ആവശ്യമായി വരുന്ന ഘട്ടം എത്തുന്നുണ്ട്. ഇത് എല്ലാ പ്രകൃതി പ്രതിഭാസങ്ങളുടെ കാര്യത്തിലും കണ്ടെത്താനാവും. ഉദാഹരണമായി ഇടി-മിന്നൽ ചർച്ച ചെയ്യാം.
എന്താണ് (What) മിന്നൽ ?
മേഘങ്ങൾ, വായു, അല്ലെങ്കിൽ ഭൂമി എന്നിവയ്ക്കിടയിലുള്ള അന്തരീക്ഷത്തിൽ വൈദ്യുതിയുടെ ഒരു ഭീമാകാരമായ തീപ്പൊരിയാണ് മിന്നൽ.
(https://www.nssl.noaa.gov/education/svrwx101/lightning)
എങ്ങനെയാണ് (How) വൈദ്യുതി ഉടലെടുക്കുന്നത്?
ഈ പ്രകൃതി പ്രതിഭാസത്തിൻ്റെ വികസനത്തിൻ്റെ പ്രാരംഭ ഘട്ടത്തിൽ, മേഘത്തിലെ പോസിറ്റീവ്, നെഗറ്റീവ് ചാർജുകൾക്കിടയിലും മേഘത്തിനും ഭൂമിക്കും ഇടയിൽ വായു ഒരു ഇൻസുലേറ്ററായി പ്രവർത്തിക്കുന്നു; ചാർജുകളിലെ വ്യത്യാസം വളരെ വലുതാകുമ്പോൾ, വായുവിൻ്റെ ഈ ഇൻസുലേറ്റിംഗ് കപ്പാസിറ്റി തകരുകയും വൈദ്യുതിയുടെ ദ്രുതഗതിയിലുള്ള പുറം തള്ളൽ ഉണ്ടാകുകയും ചെയ്യുന്നു, ഇതിനെ മിന്നൽ എന്നറിയപ്പെടുന്നു. (https://www.weather.gov/safety/lightning-science-overview)
എങ്കിൽ എങ്ങനെയാണ് (How) ഈ പോസിറ്റീവ്, നെഗറ്റീവ് ചാർജുകൾ മേഘങ്ങളിൽ ഉണ്ടാവുന്നത്?
മേഘങ്ങൾക്കുള്ളിൽ, ജലബാഷ്പം ചുറ്റി സഞ്ചരിക്കുന്നത് തുടരുന്നു. ജലതന്മാത്രകൾ പരസ്പരം കൂട്ടിമുട്ടുകയും പരസ്പരം ഉരസുകയും ഇലക്ട്രോണുകൾ പുറം തള്ളുകയും ചെയ്യും, അതാണ് വൈദ്യുതി: ഇലക്ട്രോണുകൾ. ചാർജുകളുടെ ശേഖരണത്തോടെ മേഘവും ചാർജ്ജ് ഉള്ളതായി പരിണമിക്കുന്നു, മുകളിൽ പോസിറ്റീവ് ചാർജും അടിയിൽ നെഗറ്റീവ് ചാർജും. (https://scied.ucar.edu/learning-zone/storms/thunder-and-lightning)
മേഘങ്ങളുടെ അടിത്തട്ടിലുള്ള നെഗറ്റീവ് ചാർജുകൾ ഉപരിതലത്തിലെ പോസിറ്റീവ് ചാർജിലേക്ക് ആകർഷിക്കപ്പെടുന്നു. വിപരീത ചാർജ്ജുള്ള വസ്തുക്കൾ inverse-square law അനുസരിച്ച് ആകർഷിക്കപ്പെടുന്നുവെന്ന് കൂലോംബിൻ്റെ നിയമം (Coulomb’s law) പഠിപ്പിക്കുന്നു. (https://www.scu.edu/illuminate/thought-leaders/phil-kesten/when-lightning-strikes.)
എന്തുകൊണ്ട് (Why) പോസിറ്റീവ്, നെഗറ്റീവ് ചാർജുകൾ തമ്മിൽ ആകർഷിക്കപ്പെടുന്നു ?
ഒരു പോസിറ്റീവ് ചാർജിൽ അധിക പ്രോട്ടോണുകൾ അടങ്ങിയിരിക്കുന്നു, അതേസമയം നെഗറ്റീവ് ചാർജിൽ കുറഞ്ഞ പ്രോട്ടോണുകളാണ് അടങ്ങിയിട്ടുള്ളത്. പോസിറ്റീവ് ചാർജിനെ നെഗറ്റീവ് ചാർജിനോട് അടുപ്പിക്കുമ്പോൾ, പോസിറ്റീവ് ചാർജിൽ നിന്നുള്ള അധിക സംഖ്യ പ്രോട്ടോണുകൾ നെഗറ്റീവ് ചാർജിലേക്ക് മാറ്റപ്പെടും, അങ്ങനെ രണ്ട് ചാർജുകളിലെയും പ്രോട്ടോണുകളുടെ എണ്ണം സന്തുലിതമാകും. ഒരു പോസിറ്റീവ് ചാർജിൽ നിന്ന് നെഗറ്റീവ് ചാർജിലേക്കുള്ള പ്രോട്ടോണുകളുടെ ഈ വെർച്വൽ ട്രാൻസ്ഫർ അവയ്ക്കിടയിൽ ആകർഷകമായ ബലത്തിന് കാരണമാകുന്നു.
എങ്കിൽ, നെഗറ്റീവ് പോസറ്റീവ് ചാർജുകൾക്കിടയിലെ ആകർഷണീയതയെ വിശദീകരിക്കുക മാത്രമാണ് പ്രോട്ടോണുകളുടെ സംഖ്യാ സന്തുലിതവൽക്കരം വിവരിക്കുന്നതിലൂടെ ചെയ്യുന്നുള്ളു. എന്തുകൊണ്ടാണ് (Why) നെഗറ്റീവ് പോസറ്റീവ് ചാർജുകൾക്കിടയിലെ ആകർഷണീയത ? പോസിറ്റീവ് ചാർജിൽ നിന്നുള്ള അധിക സംഖ്യ പ്രോട്ടോണുകൾ നെഗറ്റീവ് ചാർജിലേക്ക് മാറ്റപ്പെടുന്നത് എന്തു കൊണ്ടാണ് (Why)?
ഭൗതികശാസ്ത്ര ചർച്ചകൾ കൊണ്ട് സജീവമായ ഒരു വിഖ്യാത വെബ്സൈറ്റിൽ ഈ, “എന്തു കൊണ്ട് (Why)?” എന്ന ചോദ്യത്തിന് നൽകപ്പെട്ട ചില മറുപടികൾ രസകരമാണ്:
മറുപടി 1:
“മൗലികശക്തികൾ പോലുള്ളവ എന്തിനാണ് നിലനിൽക്കുന്നതെന്ന് വിശദീകരിക്കാൻ ഭൗതികശാസ്ത്രം ശ്രമിക്കുന്നില്ല, അവ എങ്ങനെ പെരുമാറുന്നുവെന്ന് വിശദീകരിക്കാൻ മാത്രമാണ് ഭൗതികശാസ്ത്രം ശ്രമിക്കുന്നത്. ഈ ശക്തികൾ സ്പേയ്സിൻ്റെ വിവിധ symmetryകൾ മൂലമാണെന്ന് നമുക്ക് പറയാം. ഉദാഹരണത്തിന്, വൈദ്യുത ചാർജ് ഒരു U(1) U(1) gauge symmetry യുമായി ബന്ധപ്പെട്ടിരിക്കുന്നു എന്ന് വേണമെങ്കിൽ പറയാം. എന്നാൽ ആ പ്രത്യേക symmetry നിലനിൽക്കുന്നത് എന്തുകൊണ്ടാണെന്ന് അപ്പോഴും വിശദീകരിക്കപ്പെടുന്നില്ല.”
മറുപടി 1:
“എന്തുകൊണ്ട്” എന്ന ചോദ്യത്തിനുള്ള ഉത്തരം നൽകാൻ ഭൗതികശാസ്ത്രം ശ്രമിക്കുന്നില്ലെങ്കിലും, ഒരു പാർശ്വഫലമായി അത് പലപ്പോഴും “എന്തുകൊണ്ട്” എന്നത് വിശദീകരിക്കാറുണ്ട്. “എന്തുകൊണ്ട്” എന്നാൽ “എന്താണ് കാരണം”, ഭൗതികശാസ്ത്രം സംഭവങ്ങൾ തമ്മിലുള്ള ബന്ധത്തെ വിവരിക്കുന്ന ശാസ്ത്രമാണ്. അതിനാൽ, അത് കാരണങ്ങളെ (cause) വിവരിക്കുന്നത്, ദാർശനിക അർത്ഥത്തിലല്ല, പ്രായോഗിക അർത്ഥത്തിൽ മാത്രമാണ്…”
മറുപടി 3:
“സ്കൂളിൽ, നാം ലളിതമായിട്ടാണ് ശാസ്ത്രത്തെ കുറിച്ച് പഠിച്ചത്. തത്ത്വചിന്തകർ “ജീവതത്ത്വശാസ്ത്രമായ” (ontological) അർത്ഥം എന്ന് വിളിക്കുന്ന, പ്രപഞ്ചത്തെക്കുറിച്ചുള്ള സത്യം വിശദീകരിക്കുന്നതുപോലെയാണ് നാം ശാസ്ത്രത്തെക്കുറിച്ച് സ്കൂളിൽ സംസാരിച്ചിരുന്നത്. വാസ്തവത്തിൽ, ഭൗതികശാസ്ത്രം എന്നത് കാര്യങ്ങൾ വിശദീകരിക്കുകയും കാര്യങ്ങൾ പ്രവചിക്കുകയും ചെയ്യുന്ന മാതൃകകൾ ഉണ്ടാക്കുകയും ചെയ്യുന്ന ദൗത്യമാണ്…”
4. “It just so happens that…”
“എന്തൊ അങ്ങനെയാണ് രണ്ട് ചാർജുകളുടെയും പ്രകൃതി…”
(https://physics.stackexchange.com/questions/535448/why-do-positive-charges-attract-negative-charges )
മുകളിൽ വിവരിച്ച അന്വേഷണങ്ങളിലെ ചോദ്യങ്ങൾ ഒരു തവണ കൂടി ശ്രദ്ധിച്ചു നോക്കൂ…
എന്താണ് (What) മിന്നൽ ?
എങ്ങനെയാണ് (How) വൈദ്യുതി ഉടലെടുക്കുന്നത്?
എങ്കിൽ എങ്ങനെയാണ് (How) ഈ പോസിറ്റീവ്, നെഗറ്റീവ് ചാർജുകൾ മേഘങ്ങളിൽ ഉണ്ടാവുന്നത്?
എന്തുകൊണ്ട് (Why) പോസിറ്റീവ്, നെഗറ്റീവ് ചാർജുകൾ തമ്മിൽ ആകർഷിക്കപ്പെടുന്നു ?
എന്ത് (what)? എന്ന ചോദ്യത്തിലൂടെ തുടങ്ങി, എങ്ങനെ (How)? എന്ന ചോദ്യത്തിൽ ഭൗതിക ശാസ്ത്ര വിശദീകരണങ്ങൾ അവസാനിക്കുന്നു. എന്തുകൊണ്ട് (Why)? എന്ന ചോദ്യം എത്തിയപ്പോഴേക്കും തത്ത്വശാസ്ത്രത്തെയും (Philosophy) ജീവതത്ത്വശാസ്ത്രത്തെയും (ontology) കുറിച്ചൊക്കെ സംസാരിക്കേണ്ടതൊ വിമർശിക്കേണ്ടതൊ ആയി വരുന്നു. അല്ലെങ്കിൽ ഈ എങ്ങനെ (How)? തന്നെയാണ് എന്തുകൊണ്ട് (Why)? എന്ന് അങ്ങ് “വിശ്വസി”ച്ച് നാസ്തികരെ പോലെ സമാധാനമടയാൻ ശ്രമിക്കുക എന്ന ദുർഗതിയിലെത്തുന്നു.
ഇനി ഇടിവെട്ടിനെ കുറിച്ച് ഭൗതിക ശാസ്ത്ര ചർച്ചയിലേക്ക് വന്നാലും ഇടിവെട്ട് ഏറ്റവനെ പാമ്പ് കടിച്ച അവസ്ഥയായിരിക്കും നാസ്തികർക്ക് !
എന്താണ് (What) ഇടി ?
മിന്നൽ മൂലമുണ്ടാകുന്ന ശബ്ദമാണ് ഇടി.
മിന്നൽ എങ്ങനെയാണ് (How) ഈ ശബ്ദം ഉണ്ടാക്കുന്നത്?
മിന്നൽ വായുവിലൂടെ കടന്നുപോകുമ്പോഴാണ് ഇടിമുഴക്കം ഉണ്ടാകുന്നത്. മിന്നൽ, വായുവിനെ പൊടുന്നനെ ചൂടാക്കുകയും വായു വികസിക്കുകയും ചെയ്യുന്നു. മിന്നൽ ചാലിലെ വായുവിൻ്റെ താപനില 50,000 ഡിഗ്രി ഫാരൻഹീറ്റിലെത്തിയേക്കാം, സൂര്യൻ്റെ ഉപരിതലത്തേക്കാൾ 5 മടങ്ങ് ചൂടാണിത്. ഫ്ലാഷിനുശേഷം, വായു തണുക്കുകയും വേഗത്തിൽ ചുരുങ്ങുകയും ചെയ്യുന്നു. ഈ ദ്രുതഗതിയിലുള്ള വികാസവും സങ്കോചവും, നാം ഇടിമുഴക്കം പോലെ കേൾക്കുന്ന ശബ്ദ തരംഗത്തെ സൃഷ്ടിക്കുന്നു. (https://www.weather.gov/safety/lightning-science-thunder)
മിന്നൽ ചൂടാക്കുമ്പോൾ വായു എങ്ങനെയാണ് (How) വികസിക്കുന്നത് ?
കാരണം, വായുവിൻ്റെ കണികകൾ ചൂടാകുമ്പോൾ, പരസ്പരം അകന്നുപോകുകയും കൂടുതൽ ഇടത്തിലേക്ക് വ്യാപിക്കുകയും ചെയ്യുന്നു. ചൂടാകുമ്പോൾ വായു കൂടുതൽ ഇടം പിടിക്കുന്നുവെന്ന് നമുക്ക് പറയാം. കണികകൾ പരസ്പരം അകന്നു പോകുന്നതിനാൽ, വായുവിൻ്റെ സാന്ദ്രത കുറയുകയും ഭാരം കുറയുകയും ചെയ്യുന്നു. തൽഫലമായി, ചൂടുള്ള വായു ഉയരുന്നു. നേരെമറിച്ച്, തണുപ്പിക്കുമ്പോൾ വായുവിൻ്റെ കണികകൾ പരസ്പരം അടുക്കുന്നു. ഇക്കാരണത്താൽ, വായുവിൻ്റെ അളവ് കുറയുന്നു, വായു കൂടുതൽ സാന്ദ്രവും ഭാരമുള്ളതുമാകുന്നു. (https://www.toppr.com/guides/science/winds-storms-and-cyclones/air-expands-on-heating/)
ചൂടാക്കുമ്പോൾ തന്മാത്രകൾ വേഗത്തിൽ നീങ്ങുകയും ചലിക്കുകയും ചെയ്യും. അവ ചലിക്കുമ്പോൾ പരസ്പരം അകന്നുപോകുകയും ചെയ്യുന്നു. അതിനാൽ, മറ്റ് പദാർത്ഥങ്ങളെപ്പോലെ വായുവും ചൂടാക്കുമ്പോൾ വികസിക്കുകയും തണുപ്പിക്കുമ്പോൾ ചുരുങ്ങുകയും ചെയ്യുന്നു. തന്മാത്രകൾക്കിടയിൽ കൂടുതൽ ഇടം ഉള്ളതിനാൽ, വായുവിന് ചുറ്റുമുള്ള പദാർത്ഥത്തേക്കാൾ സാന്ദ്രത കുറവാണ്, ചൂടുള്ള വായു മുകളിലേക്ക് പൊങ്ങിക്കിടക്കുന്നു. (https://www.grc.nasa.gov)
എന്തുകൊണ്ടാണ് (Why) ചൂട് തന്മാത്രകളെ ചലിപ്പിക്കുന്നത്?
താപനില കൂടുന്നതിനനുസരിച്ച് കണികകൾ ഗതികോർജ്ജം (kinetic energy) നേടുകയും വേഗത്തിൽ നീങ്ങുകയും ചെയ്യുന്നു. (https://www.education.vic.gov)
താപനില കൂടുന്നതിനനുസരിച്ച് കണികകൾ ഗതികോർജ്ജം നേടുന്നു എന്ന വാചകവും, താപനില തന്മാത്രകളെ ചലിപ്പിക്കുന്നു എന്ന വാചകവും ഒന്നു തന്നെയാണ്. ആദ്യം പറഞ്ഞ കാര്യത്തെ രണ്ടാമത്തെ വാചകത്തിൽ സാങ്കേതിക പദം ഉപയോഗിച്ച് പറഞ്ഞു എന്ന് മാത്രം ! ചോദ്യം അപ്പോഴും അവശേഷിക്കുന്നു.
താപനില കൂടുന്നത് എന്തുകൊണ്ട് (Why) ചലനത്തെ ത്വരിതപ്പെടുത്തുന്നു?!
ഉത്തരം വിശ്വാസികളെ സംബന്ധിച്ചിടത്തോളം ലളിതമാണ്. പ്രകൃതി പ്രതിഭാസങ്ങളുടെ ഭൗതിക കാരണങ്ങൾ ശാസ്ത്രം വിശദീകരിക്കട്ടെ. എന്നാൽ പ്രകൃതി പ്രതിഭാസങ്ങളെല്ലാം ആശ്രിത വസ്തുക്കളാണ് (contingent). ഒന്ന് മറ്റൊന്നിനെ ആശ്രയിച്ചാണ് പ്രപഞ്ചം മുഴുവൻ നിലനിൽക്കുന്നത്. പല നിലക്കും ആവാമായിരുന്ന പ്രപഞ്ചത്തിലെ വസ്തുക്കൾ/പ്രതിഭാസങ്ങൾ എന്തുകൊണ്ട് നിലവിലെ സ്ഥിതിയിൽ തന്നെ ആയി ?! ഋതുക്കളും പനിയും ഇടിയും മിന്നലുമെല്ലാം എങ്ങനെയും ആകാമായിരുന്നു. തീർത്തും ഇല്ലാതിരിക്കുകയും ചെയ്യാമായിരുന്നു. എന്നിട്ടും അവ ഉണ്ടാവാനും നിലവിലെ പ്രകൃതിയിൽ ആകാനുമുള്ള കാരണമെന്താണ്?! അതിനുള്ള ഉത്തരം നിരാശ്രിതമായ ഒരു അസ്തിത്വം (ദൈവം) അവയെയെല്ലാം പ്രത്യേക ലക്ഷ്യത്തോടെ, താൻ ഉദ്ദേശിക്കുന്ന രൂപത്തിലാണ് ഉണ്ടാക്കിയത് എന്നതാണ്. ഈ പ്രകൃതി പ്രതിഭാസങ്ങളിലെ ദൈവിക ഉദ്ദേശ്യങ്ങളെ സംബന്ധിച്ചാണ് മതവും ഹദീസുകളും എല്ലാം സംസാരിക്കുന്നത്.
വ്യാഖ്യാനം 3:
സൂര്യൻ നരകത്തിൻ്റെ ഭാഗമായി പരിണമിക്കപ്പെടും എന്ന് മുഹമ്മദ് നബി (സ) വ്യക്തമാക്കിയിട്ടുണ്ട്. إن الشمس والقمر ثوران في النار يوم القيامة സൂര്യനും ചന്ദ്രനും നരകത്തിൻ്റെ ഭാഗമാക്കപ്പെടും എന്ന് വ്യക്തമായി ഹദീസിൽ വിശദീകരിക്കപ്പെട്ടിട്ടുണ്ട്. (ബസ്സാർ: 15: 243, സിൽസിലതു സ്വഹീഹ: 1: 242)
അപ്പോൾ, ഫലത്തിൽ സൂര്യനിൽ നിന്നുള്ള സ്വാധീനങ്ങൾ നരകത്തിൽ നിന്നുള്ള പ്രതിഫലനങ്ങളായി പറയാവുന്നതാണ്. ഈ ഭാഷാ പ്രയോഗത്തിനെ അറബി അലങ്കാരശാസ്ത്രത്തിൽ പരിചയപ്പെടുത്തപ്പെടുക اعتبار ما يكون എന്നാണ്. (വസ്തു എന്തായി മാറുമെന്നു പരിഗണിച്ച് അതിനെ പരിചയപ്പെടുത്തുക.) (കിതാബു ഇൽമുൽ ബയാൻ: അൽ മജാസുൽ മുർസൽ: അബ്ദുൽ അസീസ് അതീക്: പേജ്: 161)
സൂര്യനാണല്ലൊ ഭൗമിക താപത്തിൻ്റെ കേന്ദ്രം. സൂര്യനും നരകമാണെങ്കിൽ, ഋതുക്കളുടെ കാരണം നരകമാണെന്ന് സ്വഭാവികമായും പറയാവുന്നതാണല്ലൊ.
ഭൂമിയുടെ ഭ്രമണമാണല്ലൊ രാപ്പകലുകളുടെ കാരണം (Cause). രാത്രിയിൽ സൂര്യൻ ദൈവ സിംഹാസനത്തിനടിയിൽ സുജൂദ് (സാഷ്ടാംഗം) ചെയ്യാൻ പോവുകയാണ് എന്ന് ഹദീസിൽ വന്നിരിക്കുന്നത് അശാസ്ത്രീയത അല്ലെ?
……………………………വിമർശന വിധേയമായ ഹദീസ്:
അബൂ ദർറ് (റ) പറഞ്ഞു: സൂര്യൻ അസ്തമിച്ചപ്പോൾ പ്രവാചകൻ (സ) അബൂ ദർറിനോട് (റ) ചോദിച്ചു: എവിടേക്കാണ് സൂര്യൻ പോകുന്നത് എന്ന് താങ്കൾക്കറിയാമോ? ഞാൻ (അബൂ ദർറ് (റ) ) പറഞ്ഞു: അല്ലാഹുവും അവന്റെ ദൂതനുമാണ് ഏറ്റവും കൂടുതൽ അറിയുന്നത്. പ്രവാചകൻ (സ) പറഞ്ഞു: ദൈവത്തിൻ്റെ സിംഹാസനത്തിന് കീഴിൽ സാഷ്ടാംഗം ചെയ്യാനാണ് അതിൻ്റെ പ്രയാണം. എന്നിട്ട് അത് സമ്മതം ചോദിക്കും, അപ്പോൾ സമ്മതം നൽകപ്പെടും. (സമ്മതം നൽകപ്പെടുകയും “നീ വന്നിടത്തേക്ക് മടങ്ങുകയും ചെയ്യുക” എന്ന് പറയപ്പെടുകയും ചെയ്താൽ അത് മടങ്ങുകയും ഉദയസ്ഥാനത്തു തന്നെ ഉദിക്കുകയും ചെയ്യും.) അത് സാഷ്ടാംഗം ചെയ്യുകയും സമ്മതം നൽകപ്പെടാതിരിക്കുകയും ചെയ്യുന്ന ഒരു കാലം (ലോകാവസാനം) അടുത്തിരിക്കുന്നു. അന്ന് അതിനോട് പറയപ്പെടും: നീ വന്നിടത്തേക്ക് മടങ്ങുക. (പടിഞ്ഞാറ് നിന്ന് ഉദിക്കുകയും ചെയ്യുക.) അപ്പോൾ അത് പടിഞ്ഞാറു നിന്ന് ഉദിക്കും. (ലോകാവസാനം സംഭവിക്കും.) അതാണ് അല്ലാഹു ഈ വാക്യത്തിൽ പറഞ്ഞത്: ”സൂര്യന് അതിന് സ്ഥിരമായുള്ള ഒരു സ്ഥാനത്തേക്ക് സഞ്ചരിക്കുന്നു. പ്രതാപിയും സര്വ്വജ്ഞനുമായ അല്ലാഹു കണക്കാക്കിയതാണത്.” (ക്വുർആൻ: 36: 38) (സ്വഹീഹുൽ ബുഖാരി: 7424)
മറുപടി:
ഞാൻ ഒരു കസേരയിലിരുന്നാണ് ഈ ലേഖനം എഴുതുന്നത്. “കസേര”യുടെ കാരണം (Cause) എന്താണ് എന്ന് ഞാൻ അരിസ്റ്റോട്ടിലിനോട് ചോദിച്ചാൽ, വ്യത്യസ്തമായ നാല് കാര്യകാരണങ്ങൾ (Cause) കസേരയ്ക്കുണ്ട് എന്ന് അരിസ്റ്റോട്ടിൽ പറയും. മരവും കോടാലിയും ആണിയും ചുറ്റികയുമാണ് കസേരയുടെ കാരണം (Cause) എന്ന് പറയാവുന്നതാണ്. ഇതിന് material cause എന്നാണ് പറയുക.
കസേരക്ക് ആവശ്യമായ വസ്തുക്കളുടെ ക്രമീകരണം, ആകൃതി അല്ലെങ്കിൽ പ്രത്യേകമായ രൂപത്തിലേക്ക് പരിവർത്തനം എന്നിവയെല്ലാമാണ് കസേരയുടെ കാരണം (Cause) എന്നും പറയാവുന്നതാണ്. അഥവാ നിർമ്മിക്കപ്പെടുന്ന ചട്ടക്കൂട് നൽകുന്ന വസ്തുവിൻ്റെ പ്രത്യേക സത്ത അല്ലെങ്കിൽ സ്വഭാവമാണ് ആ കാര്യത്തിൻ്റെ കാരണം എന്നർത്ഥം. ഇതിന് Formal cause എന്നാണ് പറയുക.
കസേരയുടെ കാരണം (Cause), ആശാരി അല്ലെങ്കിൽ വാസ്തുവിദ്യ ആണ് എന്നും പറയാമല്ലൊ. അഥവാ പ്രാവർത്തികമായൊ ആശയപരമായൊ അതിനെ യാഥാർത്ഥ്യവൽക്കരിക്കുന്നത് എന്തോ/ആരോ അതാണ് അതിൻ്റെ കാരണം (Cause) എന്ന്. ഇതിനെ efficient cause അല്ലെങ്കിൽ moving cause എന്ന് പറയുന്നു.
സ്റ്റൈലിൽ കാലിൻമേൽ കാലും കയറ്റി വച്ച് ഇരിക്കാൻ ഒരു സംഗതി വേണം എന്ന ലക്ഷ്യമൊ ഉദ്ദേശ്യമൊ ആണ് കസേരയുടെ കാരണം (Cause) എന്ന് പറയുന്നതും ശരിയാണ്. ഇതിനെ final cause എന്നാണ് വിളിക്കപ്പെടുന്നത്. ഒരു വസ്തു/കാര്യം ഉണ്ടാവാനുള്ള ആത്യന്തികമായ കാരണമാണ് final cause.
(Western Philosophy I: page: 43, SGOU-SLM – MA PHILOSOPHY)
ഇങ്ങനെ പല രീതിയിലും ഒരു കാര്യത്തിൻ്റെ കാരണം വിശദീകരിക്കാവുന്നതാണ്. കാര്യത്തിൻ്റെ ഉള്ളടക്കത്തിലെ ഏത് ഘടകത്തെയും ഭാഗത്തെയും പരിഗണിച്ചാണ് നമ്മൾ കാരണം (Cause) പറയാനുദ്ദേശിക്കുന്നത്, അതിനനുസൃതമായി പല രീതിയിലും ഒരു കാര്യത്തിന് കാരണങ്ങൾ പറയാം എന്നർത്ഥം. അതിൽ ഒരു രീതിയിൽ കാരണം (Cause) പറയുന്നത് മറ്റൊരു രീതിയിൽ കാരണം (Cause) പറയുന്നതിനോട് എതിരാവുന്നില്ലല്ലൊ. ഉദാഹരണത്തിന് കസേരയുടെ കാരണം മരവും കോടാലിയും ആണിയും ചുറ്റികയുമടങ്ങുന്ന വസ്തുക്കളാണ് എന്ന് പറയുന്നത് കസേരയുടെ കാരണം ആശാരിയൊ അല്ലെങ്കിൽ വാസ്തുവിദ്യയൊ ആണ് എന്ന് പറയുന്നതിന് എതിരല്ലല്ലൊ. രണ്ടും ശരിയാണ്. ഈ സാമാന്യ ബുദ്ധി ഉപയോഗിച്ച് വേണം, ഒരു കാര്യത്തിന് മതം മുന്നോട്ടു വെക്കുന്ന, ആത്മീയമൊ അഭൗതികമൊ ആയ കാരണങ്ങളെ മനസ്സിലാക്കാൻ. “ദൈവം മഴ നൽകി” എന്ന് ഒരു മത വിശ്വാസി പറയുമ്പോൾ “ഏഴാനാകാശത്തിനപ്പുറം ദൈവം കുറച്ച് വെള്ളമുണ്ടാക്കി, നിലത്തേക്ക് എറിയുന്നതാണ് മഴ… അല്ലാതെ മേഘങ്ങൾ, സൂര്യൻ, താപം, ബാഷ്പീകരണം, സാന്ദ്രീകരണം എന്നിവയൊന്നും മഴയുടെ കാരണമല്ല!” എന്നാണ് വിശ്വാസി അല്ലെങ്കിൽ മതം പറയുന്നത് എന്ന് തെറ്റിദ്ധരിക്കാൻ മാത്രം ബുദ്ധിശൂന്യരാണോ നാസ്തികർ ?! അങ്ങനെയാണെങ്കിൽ അതേ മതം തന്നെ മഴയുടെ കാരണങ്ങളായി കാർമേഘത്തെയും സാന്ദ്രീകരണത്തെയും (ക്വുർആൻ: 7: 57) പലയിടത്തും അംഗീകരിക്കുന്നത് എന്ത് കൊണ്ടാണ് ?! കാര്യം സാമാന്യ ബുദ്ധിയുള്ളവർക്ക് (അഥവാ വിശ്വാസികൾക്ക്) വ്യക്തമാണ്. ഒരിടത്ത് മഴയുടെ material cause അല്ലെങ്കിൽ Formal cause പറയുമ്പോൾ മറ്റൊരിടത്ത് അതിൻ്റെ efficient cause അല്ലെങ്കിൽ final Cause ആണ് വിശദീകരിക്കപ്പെടുന്നത്. രോഗം ശമിപ്പിക്കുന്നത് ദൈവമാണ് എന്ന് ക്വുർആൻ പറയുന്നു. (ക്വുർആൻ: 26: 80). എല്ലാ രോഗങ്ങൾക്കും മരുന്നുണ്ട് എന്നും, രോഗം വന്നാൽ ചികിത്സിച്ച് ശമിപ്പിക്കണം എന്നും മുഹമ്മദ് നബി (സ) പറയുന്നു. (സ്വഹീഹുൽ ബുഖാരി: 5678). അപ്പോൾ രോഗശമനത്തിന് കാരണം ദൈവമാണൊ? അതൊ മരുന്നാണൊ ? എന്ന് ഒരു വിശ്വാസിയും ഇതുവരെ സംശയമുന്നയിച്ചിട്ടില്ല. രോഗശമനത്തിൻ്റെ കാരണം ദൈവമാണ് എന്ന് പറയുമ്പോൾ അതിൻ്റെ ആത്യന്തികമായ കാരണം (Cause) ദൈവമാണെന്നാണ് ഉദ്ദേശ്യം. രോഗശമനത്തിൻ്റെ കാരണം മരുന്നാണ് എന്ന് പറയുമ്പോൾ അതിൻ്റെ material cause മരുന്നാണ് എന്നുമാണ് ഉദ്ദേശ്യം എന്നത് സാമാന്യ ബോധമാണ്.
അപ്പോൾ ഇത്തരം കാരണവാദങ്ങൾ വ്യത്യസ്തമാണെങ്കിലും വൈരുദ്ധ്യമല്ല എന്ന് ആദ്യമെ മനസ്സിലാക്കി വേണം വിഷയത്തിലേക്ക് കടക്കാൻ.
സൂര്യനും -ഗ്രഹങ്ങൾ പോലെ തന്നെ – രണ്ട് തരം ചലനങ്ങളുണ്ട്. ഒന്ന് അതിൻ്റെ സ്വയം ഭ്രമണം. രണ്ട്, മിൽകിവേയ് ഗാലക്സിയുടെ കേന്ദ്രത്തിന് ചുറ്റുമുള്ള അതിൻ്റെ അതി ദീർഘമായ പ്രയാണം/സഞ്ചാരം.
സൂര്യൻ സ്വന്തം അച്ചുതണ്ടിൽ കറങ്ങുന്നത്, മധ്യരേഖയിൽ 25.6 ദിവസവും ധ്രുവങ്ങളിൽ 33.5 ദിവസവും എന്ന ഭ്രമണ കാലയളവോടെയാണ്.
സൂര്യൻ ക്ഷീരപഥ ഗാലക്സിയുടെ മധ്യഭാഗത്ത് ചുറ്റുന്നത് മണിക്കൂറിൽ ശരാശരി 828,000 കിലോമീറ്റർ വേഗതയിലാണ്. എന്നാൽ ഇത്രയും ഉയർന്ന നിരക്കിൽ പോലും, ക്ഷീരപഥത്തിന് ചുറ്റും ഒരു പൂർണ്ണ ഭ്രമണപഥം വെക്കാൻ സൂര്യന് ഏകദേശം 230 ദശലക്ഷം വർഷങ്ങൾ ആവശ്യമാണ്! (starchild.gsfc.nasa.gov)
സൂര്യൻ്റെ സ്വയംഭ്രമണവും സഞ്ചാരവും ദൈവത്തിനുള്ള ആരാധനയും സുജൂദും (സാഷ്ടാംഗം) ആണെന്ന് പറയുമ്പോൾ സൂര്യ ചലനത്തിൻ്റെയൊ രാപ്പകലുകളുടെയോ material cause അല്ല ഹദീസ് അവതരിപ്പിക്കുന്നത്. മറിച്ച് പ്രപഞ്ചത്തിൻ്റെ ഇത്തരമൊരു ഘടനക്കും, രൂപത്തിനും, സ്വഭാവത്തിനും ഒരു ദൈവികമായ ഉദ്ദേശ്യമുണ്ട്. ആ ദൈവിക ഇച്ഛയെ പ്രപഞ്ചത്തിലെ സകല സൃഷ്ടിപ്പിൻ്റെയും പ്രതിഭാസത്തിൻ്റെയും Final cause ആയി പറയാമല്ലൊ. പ്രത്യേക ഉദ്ദേശ്യത്തോടെയും ലക്ഷ്യത്തോടെയുമാണ് സർവ്വതും സൃഷ്ടിച്ചത്. അപ്പോൾ പ്രപഞ്ചത്തിലെ സകല സൃഷ്ടിപ്പിൻ്റെയും പ്രതിഭാസത്തിൻ്റെയും പരമവും ആത്യന്തികവുമായ Final cause ദൈവത്തിൻ്റെ ഉദ്ദേശ്യവും ലക്ഷ്യവുമാണ്.*
(* ഈ അർത്ഥത്തിൽ, ചർച്ചയുടെ ഒഴുക്കിനായി, ഭൗതിക പ്രതിഭാസങ്ങളുടെ അഭൗതികവും ദൈവിക ഉദ്ദേശ്യ പ്രകാരവുമുള്ള കാരണങ്ങളെ Final cause എന്നാണ് തുടർന്നുള്ള ഭാഗങ്ങളിൽ വിശേഷിപ്പിക്കുക.)
അങ്ങനെ വരുമ്പോൾ അത് ഭൗതികമായ കാര്യകാരണങ്ങളിൽ നിന്നും വ്യത്യസ്തമായിരിക്കാം, പക്ഷെ വിരുദ്ധമല്ല. ശാസ്ത്രത്തിൻ്റെ ചർച്ചയും വിഷയവും പ്രപഞ്ചത്തിൻ്റെ material cause കളിൽ കേന്ദ്രീകരിച്ചു കൊണ്ടാണ് എന്ന് ഓർക്കുക.
ഉദാഹരണത്തിന്, സൂര്യൻ്റെ സ്വയംഭ്രമണം അതിൻ്റെ പൂർവ്വസ്ഥിതിയിലെ കോണീയ ആക്കം നിലനിർത്തുന്നത് മൂലമാണ്. അഥവാ, സൂര്യൻ രൂപംകൊണ്ട വാതക മേഘത്തിന് കോണീയ ആക്കം ഉണ്ടായിരുന്നു, ആ കോണീയ ആക്കം അത് രൂപപ്പെടുമ്പോൾ സൂര്യനിലേക്ക് കൈമാറി, അത് -ഇന്ന് നാം നിരീക്ഷിക്കുന്ന ഭ്രമണം- സൂര്യന് നൽകി എന്ന് ഒരു ഭൗതികശാസ്ത്രജ്ഞൻ വിശദീകരിക്കും. ഈ ശാസ്ത്ര വിശദീകരണം സൂര്യഭ്രമണത്തിൻ്റെ material cause അല്ലെങ്കിൽ efficient cause മാത്രമാണ്. ഈ ശാസ്ത്ര വിശദീകരണം സൂര്യഭ്രമണത്തിൻ്റെ അവസാന കാരണം (final cause) എങ്ങനെയാണ് ആവുക? തീർച്ചയായും അത് അവസാന കാരണം ആവില്ല. കാരണം സൂര്യൻ്റെ ഭ്രമണം, വാതക മേഘത്തിൽ നിന്നുളള കോണീയ ആക്കം നിലനിർത്തുന്നത് മൂലമാണ് എന്നതാണ് അവസാന കാരണമെങ്കിൽ വാതക മേഘത്തിൻ്റെ കോണീയ ആക്കത്തിന് കാരണമെന്താണ്? അതിന് കാരണമായി ഒരു material cause ശാസ്ത്രീയമായി വിശദീകരിക്കപ്പെട്ടു എന്ന് കരുതുക. എങ്കിൽ അതിന് കാരണമെന്താണ് എന്ന് വീണ്ടും ചോദിക്കപ്പെടാം. അതിനു കാരണമായി ഒരു material cause ശാസ്ത്രീയമായി വിശദീകരിക്കപ്പെട്ടു എന്ന് കരുതുക. എങ്കിൽ അതിന് കാരണമെന്താണ് എന്ന് വീണ്ടും ചോദിക്കപ്പെടാം. ഇങ്ങനെ കാരണാന്വേഷണം തുടന്നു തുടർന്നു പോയാൽ, material cause ന് അപ്പുറം, ഒരു അവസാന കാരണം വിശദീകരിക്കപ്പെട്ടെ മതിയാവൂ. ഈ അവസാന കാരണമാണ് മതം “ദൈവേച്ഛ” യിലൂടെ അവതരിപ്പിക്കുന്നത്. അതെങ്ങനെ ശാസ്ത്രം വിശദീകരിക്കുന്ന material cause ന് എതിരാവും?! ഒരു വസ്തുവിൻ്റെ അവസാന കാരണം (final cause) ആ വസ്തു തന്നെ (material cause) ആവൽ ബുദ്ധിപരമായി അസാധ്യമാണ് എന്ന് മതവും തത്ത്വശാസ്ത്രവും നമ്മെ പഠിപ്പിച്ചു.
അപ്പോൾ സൂര്യൻ്റെ സ്വയംഭ്രമണം, വാതക മേഘത്തിൽ (Gas cloud) നിന്നുള്ള കോണീയ ആക്കം നിലനിർത്തുന്നത് മൂലമാണ് എന്ന ശാസ്ത്രീയ വീക്ഷണം സൂര്യഭ്രമണത്തിൻ്റെ material cause ആയി മതവും മതവിശ്വാസിയും അംഗീകരിക്കുന്നു. എന്നാൽ സൂര്യഭ്രമണത്തിന് ഒരു അവസാന കാരണമുണ്ട്. അത്, സൂര്യനെ സൃഷ്ടിച്ച ദൈവത്തിൻ്റെ കൽപ്പനകൾക്ക് വിധേയമായി അവനെ ആരാധിക്കലും അവന് സാഷ്ടാംഗം ചെയ്യലും അനുവാദം വാങ്ങലുമൊക്കെയാണ്. ഇതെങ്ങനെയാണ് ശാസ്ത്ര വിരുദ്ധമാവുക എന്ന് ബുദ്ധിപരമായൊ ഫിലോസഫിക്കലായൊ തെളിയിക്കാൻ ഏത് നാസ്തികനാണ് സാധിക്കുക?!
അപ്പോൾ സൂര്യഭ്രമണത്തിൻ്റെയും സൂര്യസഞ്ചാരത്തിൻ്റെ Final cause ദൈവത്തിന് സാഷ്ടാംഗം ചെയ്യലും അനുവാദം വാങ്ങലുമൊക്കെയാണ് എന്ന് ഹദീസിൽ വന്നതിൽ ശാസ്ത്ര “വിരുദ്ധമായി” ഒന്നും തന്നെ ഇല്ല. ശാസ്ത്രത്തിന് “അപ്രാപ്യമായ” വിഷയമാണത് എന്ന് വേണമെങ്കിൽ പറയാമെന്ന് മാത്രം.
*****************************ഇനി, ദൈവത്തിൻ്റെ സിംഹാസനത്തിന് അടിയിൽ സൂര്യൻ പോവുന്നു എന്ന് ഹദീസ് പറയുമ്പോൾ, സൂര്യൻ അതിൻ്റെ ഭ്രമണപഥത്തിൽ നിന്ന് നീങ്ങി സഞ്ചരിക്കുന്നു എന്ന് വരില്ലെ എന്നതാണ് അടുത്ത ചോദ്യം. ഇല്ല എന്നതാണ് ഉത്തരം. കാരണം, ദൈവ സിംഹാസനം “ആകാശഭൂമികളെ മുഴുവന് ഉള്കൊള്ളുന്നതാകുന്നു” എന്ന് ക്വുർആൻ (2: 255) വ്യക്തമാക്കിയിട്ടുണ്ട്. അപ്പോൾ ഏത് അച്ചുതണ്ടിൽ സഞ്ചരിച്ചും ദൈവത്തിന് സുജൂദും (സാഷ്ടാംഗവും) ആരാധനകളും നിർവ്വഹിക്കാം. സഞ്ചരിക്കുക പോയിട്ട് ഒരു ചലനം പോലും കൂടാതെ ദൈവത്തിന് സുജൂദ് (സാഷ്ടാംഗം) നിർവഹിക്കാം. കയ്യും കാലും അവയവങ്ങളുമില്ലാതെയും ദൈവത്തിന് സുജൂദ് (സാഷ്ടാംഗം) നിർവഹിക്കാം എന്നാണ് ഇസ്ലാം വിശദീകരിക്കുന്നത്. എല്ലാം ഓരോ സൃഷ്ടികളുടെയും രൂപവും പ്രകൃതിയും സൃഷ്ടിപ്പും അനുസരിച്ചാണ് അവ സുജൂദ് (സാഷ്ടാംഗം) നിർവ്വഹിക്കുക; മനുഷ്യർ നിർവ്വഹിക്കുന്നത് പോലെയല്ല. ക്വുർആൻ പറയുന്നു:
“ആകാശങ്ങളിലുള്ളവരും ഭൂമിയിലുള്ളവരും, സൂര്യനും ചന്ദ്രനും നക്ഷത്രങ്ങളും, പര്വ്വതങ്ങളും വൃക്ഷങ്ങളും ജന്തുക്കളും, മനുഷ്യരില് കുറെപേരും അല്ലാഹുവിന് സുജൂദ് (സാഷ്ടാംഗവും) ചെയ്തുകൊണ്ടിരിക്കുന്നു എന്ന് നീ കണ്ടില്ലേ?” (ക്വുർആൻ: 22: 18)
“ആകാശങ്ങളിലും ഭൂമിയിലുമുള്ളവരും, ചിറക് നിവര്ത്തിപ്പിടിച്ചു കൊണ്ട് പക്ഷികളും അല്ലാഹുവിന്റെ മഹത്വം പ്രകീര്ത്തിച്ചു കൊണ്ടിരിക്കുന്നു എന്ന് നീ കണ്ടില്ലേ? *ഓരോരുത്തര്ക്കും തന്റെ പ്രാര്ത്ഥനയും കീര്ത്തനവും എങ്ങനെയെന്ന് അറിവുണ്ട്.* അവര് പ്രവര്ത്തിക്കുന്നതിനെപ്പറ്റി അല്ലാഹു അറിയുന്നവനത്രെ.” (ക്വുർആൻ: 24: 41)
ചന്ദ്രനും നക്ഷത്രങ്ങളും, പര്വ്വതങ്ങളും വൃക്ഷങ്ങളും എന്തിനേറെ നിഴലുകൾ പോലും ദൈവത്തിന് സാഷ്ടാംഗം നമസ്കരിക്കുന്നു (13: 15) എന്നാണ് ക്വുർആൻ പറയുന്നത്! നിഴലുകൾക്ക് ശരീരമുണ്ടൊ? അവയവങ്ങളുണ്ടോ? അവയൊന്നും കൂടാതെ തന്നെ, ഓരോ സൃഷ്ടിക്കും അതിൻ്റെതായ സുജൂദ് (പ്രമാണം) അർപ്പിക്കാൻ സാധിക്കുന്നതാണ്.
وكل من ذل وخضع لما أمر به، فقد سجد“ആർ താൻ കൽപ്പിക്കപ്പെട്ട ഒരു കാര്യത്തിനും മുന്നിൽ പരമമായ വിധേയത്വവും കീഴൊതുക്കവും പ്രകടിപ്പിച്ചുവൊ അയാൾ ‘സുജൂദ്’ ചെയ്തു…” എന്ന് പൗരാണിക അറബി ഭാഷാ പണ്ഡിതരെല്ലാം, സുജൂദ് എന്ന അറബി പദത്തെ വിശദീകരിച്ചിട്ടുണ്ട്. (ലിസാനുൽ അറബ്: ഇബ്നു മൻളൂർ (ജനനം: 630 ഹിജ്രാബ്ദം.1232 CE) 3: 206 )
സുജൂദിന് തലയൊ കാലോ അവയവങ്ങളൊ നിശ്ചലാവസ്ഥയൊ ശരീരം വെക്കാനുള്ള പ്രതലമൊ ഒന്നും നിർബന്ധമില്ല എന്നർത്ഥം. ഓരോ സൃഷ്ടികൾക്കുമനുസരിച്ച് അവരുടെ സുജൂദും വ്യത്യസ്തമാണ്. മനുഷ്യർ അർപ്പിക്കുന്ന സുജൂദ് തന്നെ അവസ്ഥകൾക്കനുസരിച്ച് ആപേക്ഷിക രൂപമാറ്റങ്ങൾ ഉണ്ടാവാം. മനസ്സു കൊണ്ട് പോലും സുജൂദ് ചെയ്യുന്ന അവസ്ഥയുണ്ടാവാം. അവയങ്ങൾ കൊണ്ട് സാഷ്ടാംഗം ചെയ്യുമ്പോഴും ആശയപരമായി അത് സുജൂദ് അല്ലാതിരിക്കുന്ന അവസ്ഥകളും ഉണ്ട്. അത്തരം സന്ദർഭങ്ങളിൽ അത് ഭാഷാപരമായ സുജൂദ് മാത്രമാണ്; ആദമിന് മലക്കുകൾ അർപ്പിച്ച “സുജൂദ്” (ആദരപ്രകടം) പോലെ.
ഹദീസ് വായിക്കുമ്പോൾ വിമർശകരുടെ മനസ്സിൽ സൂര്യൻ ഒരു മൃഗമൊ, മനുഷ്യനൊ, കൈകാലുകളും അവയവങ്ങളുമുള്ള ഒരു ജീവിയൊ ഒക്കെ ആയി പ്രത്യക്ഷപ്പെടുന്നു. അത്തരമൊരു സൂര്യൻ “പോകുന്നതും” “വണങ്ങുന്നതും” “മടങ്ങുന്നതും” എല്ലാം അവർ സങ്കൽപ്പിക്കുന്നു. അതുകൊണ്ടാണ് സൂര്യനെ സംബന്ധിച്ച ഈ പരാമർശങ്ങളൊന്നും വിമർശകർക്ക് ഉൾക്കൊള്ളാൻ കഴിയാത്തത്. എന്നാൽ സൂര്യൻ അതിൻ്റെ രൂപത്തിലും പ്രകൃതിയിലും ഭ്രമണത്തിലും ഇവയെല്ലാം ചെയ്യുന്നുണ്ട് എന്നതാണ് ഇസ്ലാമിക വീക്ഷണം. പ്രപഞ്ചം മുഴുവൻ വിശാലമായ ദൈവത്തിൻ്റെ സിംഹാനത്തിന് ചുവട്ടിൽ സുജൂദ് ചെയ്യാൻ… ഭ്രമണപഥത്തിൽ നിന്ന് മാറി സഞ്ചരിക്കുകയൊ, ശരീരാവയങ്ങൾ ഉണ്ടാവുകയൊ ഒന്നും വേണ്ടതില്ല. അവയൊക്കെ വേണമെന്ന്, നാം സങ്കൽപ്പിക്കുന്നു എന്ന് മാത്രം. കാരണം മനുഷ്യർ സുജൂദ് ചെയ്യാറുള്ളത് അപ്രകാരമാണണല്ലൊ. അതുകൊണ്ട് സൂര്യൻ്റെ സുജൂദും അങ്ങനെ ആയെ തീരൂ എന്ന് വിമർശകരുടെ ദുർബുദ്ധി വാശി പിടിക്കുന്നു.
ശൈഖ് സ്വലിഹ് അൽമുനജ്ജിദ് എഴുതി:
“സൂര്യൻ അതിൻ്റെ ഭ്രമണപഥത്തിലൂടെ അനുസ്യൂതം സഞ്ചരിക്കുകയും കറങ്ങുകയും ചെയ്യുമ്പോൾ തന്നെ സുജൂദ് നിർവഹിക്കുന്നുണ്ട്. ഇതാണ് ഹദീസിൻ്റെ പദങ്ങൾ പ്രത്യക്ഷത്തിൽ സൂചിപ്പിക്കുന്നത്. تجري لا مستقرا لها സൂര്യൻ യാതൊരു ഭംഗവുമില്ലാതെ “അനുസ്യൂതം സഞ്ചരിക്കുന്നു” എന്ന പ്രവാചകശിഷ്യരായ ഇബ്നു അബ്ബാസ്, ഇബ്നു മസ്ഊദ് എന്നിവരുടെ (ജാമിഉ അഹ്കാമിൽ ക്വുർആൻ: 15/28-29) വ്യാഖ്യാനവും ഇക്കാര്യത്തെ ഉറപ്പിക്കുന്നു.
ഇത് തന്നെയാണ് ഇബ്നു ഹജറിൻ്റെ (ജനനം: 773 ഹിജ്രാബ്ദം.1371 CE) വ്യാഖ്യാനത്തിൽ നിന്നും മനസ്സിലാവുന്നത്. സൂര്യൻ ദൈവത്തിന് സുജൂദ് ചെയ്യുന്നു എന്നതുകൊണ്ട് അതിൻ്റെ സഞ്ചാരം നിർത്തി ഒരു സ്ഥലത്ത് തങ്ങുന്നു എന്ന് അതിനർത്ഥമില്ല. ദൈവ സിംഹാസനത്തിന് കീഴിൽ സുജൂദ് ചെയ്യുന്നു എന്നത് കൊണ്ട് ജനങ്ങളുടെ കാഴ്ച്ചയിൽ നിന്ന് അകന്ന് മറ്റെവിടെയെങ്കിലും പോവണം എന്നുമില്ല. കാരണം ദൈവിക സിംഹാസനം ആകാശ ഭൂമികൾക്കും സൂര്യനുമെല്ലാം മുകളിലും എല്ലാത്തിനെയും ഉള്കൊള്ളുന്നതുമാകുന്നു (ക്വുർആൻ 2: 255)
സൂര്യൻ ആകാശങ്ങൾക്കപ്പുറം ഉയർന്ന് പോയി സിംഹാസനത്തിന് താഴെ സുജൂദ് ചെയ്യുന്നു എന്ന് ഹദീസ് സൂചിപ്പിക്കുന്നില്ല, ഭ്രമണപഥത്തിലൂടെ സഞ്ചരിക്കുമ്പോൾ തന്നെ, സൂര്യൻ സുജൂദ് ചെയ്യുന്നു എന്നാണ് ഹദീസ് സൂചിപ്പിക്കുന്നത് എന്ന് ഇമാം ഇബ്നു കസീർ (ജനനം: 701 ഹിജ്രാബ്ദം.1373 CE) നൂറ്റാണ്ടുകൾക്കപ്പുറം വ്യക്തമാക്കിയിട്ടുണ്ട്. (അൽബിദായ വന്നിഹായ: 1:33)”
(https://www.google.com/amp/s/www.islamweb.net/amp/ar/fatwa/99520/#ip=1)
******************************സൂര്യൻ ദൈവത്തിന് മുന്നിൽ സാഷ്ടാംഗം ചെയ്യാൻ “പോകുന്നു”… ദൈവ സമ്മതം ആരായുന്നു. ദൈവം സമ്മതം നൽകുന്നു. തുടർന്നുള്ള സഞ്ചാരത്തിലേക്ക് മടങ്ങി പോകാൻ പറയുന്നു. സൂര്യൻ പ്രയാണം തുടരുന്നു… സൂര്യൻ്റെ “പോക്കും ” “മടക്കവും” തുടങ്ങിയ പരാമർശങ്ങളൊക്കെ അശാസ്ത്രീയമല്ലെ എന്നതാണ് മറ്റൊരു ചോദ്യം.
സൂര്യൻ ദൈവത്തിനടുത്തേക്ക് “പോകുന്നു”, തൻ്റെ തുടർ സഞ്ചാരത്തിനുള്ള അനുവാദം ലഭിക്കുമ്പോൾ “മടങ്ങുന്നു”. ഈ “പോക്കും”, “മടക്കവും” ആണ് സൂര്യസഞ്ചാരം അല്ലെങ്കിൽ കറക്കം. !
ഗാലക്സിയുടെ കേന്ദ്രത്തിന് ചുറ്റും ഏകദേശം വൃത്താകൃതിയിൽ എന്ത് കൊണ്ട് സൂര്യൻ സഞ്ചരിക്കുന്നു എന്ന് ചിന്തിച്ച് നോക്കൂ. സൂര്യനു ചുറ്റും ഗ്രഹങ്ങളും ഭൂമിക്ക് ചുറ്റും ചന്ദ്രനും എന്തിന് കറങ്ങി കൊണ്ടിരിക്കുന്നു എന്ന് ചിന്തിച്ച് നോക്കൂ. അവ നേർ രേഖയിലൊ, അലക്ഷ്യമായൊ സഞ്ചരിക്കാത്തതെന്ത്. ഇതിന് കാരണം ആകേന്ദ്രബലമാണ് (centripetal force). ആകേന്ദ്രബലം ഒരു വസ്തുവെ, അത് സഞ്ചരിക്കുമ്പോൾ വൃത്തത്തിൻ്റെ മധ്യഭാഗത്തേക്ക് വലിക്കുകയോ തള്ളുകയോ ചെയ്യുന്നു, ഇത് കോണീയമോ വൃത്താകൃതിയിലുള്ളതോ ആയ ചലനത്തിന് കാരണമായി മാറുന്നു. ഗാലക്സിയുടെ മധ്യത്തിലേക്കുള്ള നക്ഷത്രങ്ങളിലെ ആകേന്ദ്രബലം, ചന്ദ്രനെ ഭൂമിക്ക് ചുറ്റുമുള്ള ഭ്രമണപഥത്തിലും ഭൂമിയെ സൂര്യനുചുറ്റും ഭ്രമണപഥത്തിൽ നിലനിർത്തുന്ന അതേ അപകേന്ദ്രബലമാണ്: അത് ഗുരുത്വാകർഷണ ബലമാണ്. അഥവാ, നമ്മുടെ ഗാലക്സിയിൽ ധാരാളം പിണ്ഡം (Mass) അടങ്ങിയിരിക്കുന്നു. അതിൽ നക്ഷത്രങ്ങൾ, വാതകം, ഗ്രഹങ്ങൾ, ഇരുണ്ട ദ്രവ്യം (dark matter) എന്നിവ ഉൾപ്പെടുന്നു. നമ്മുടെ ഗാലക്സിയുടെ കേന്ദ്രത്തിൽ ഒരു തമോഗർത്തവും (black hole) ഉണ്ട്. ഗാലക്സിയുടെ ആകെ പിണ്ഡത്തിൻ്റെ ഒരു ദശലക്ഷത്തിൽ ഒന്ന് മാത്രമാണ് കേന്ദ്രത്തിലെ തമോഗർത്തം. പിണ്ഡം (mass) ഗുരുത്വാകർഷണത്തിനും (gravity) ഗുരുത്വാകർഷണം ഭ്രമണപഥങ്ങൾക്കും (Orbit) കാരണമാകുന്നതിനാൽ, ഗാലക്സിയിലെ സൂര്യനടക്കം എല്ലാ വസ്തുക്കളുടെയും ഭ്രമണപഥങ്ങൾ ഗാലക്സിയുടെ ആകെ പിണ്ഡം മൂലമാണ് ഉണ്ടാകുന്നത് എന്ന് പറയാം.
അപ്പോൾ സൂര്യൻ ഗാലക്സിയുടെ കേന്ദ്രത്തിലേക്ക് ഗുരുത്വാകർഷണം മൂലം അടുക്കുമ്പോൾ അതേസമയം അപകേന്ദ്രബലം (centrifugal force) സൃഷ്ടിക്കപ്പെടുന്നു, അത് സൂര്യനെ ഗാലക്സിയുടെ കേന്ദ്രത്തിൽ നിന്ന് അകറ്റുന്നു. ഒരു പിണ്ഡം (Mass) തിരിക്കുമ്പോൾ അതിന്മേലുള്ള പ്രത്യക്ഷമായ പുറത്തേക്കുള്ള ബലമാണ് അപകേന്ദ്രബലം.
മൊത്തം ആകേന്ദ്രബലവും (centripetal force) അപകേന്ദ്രബലവും (centrifugal force) തുല്യവും എതിർദിശയിലുള്ളതുമാണ്, അതിനാൽ അവ പരസ്പരം റദ്ദാക്കുകയും സൂര്യൻ അതിൻ്റെ ഭ്രമണപഥത്തിൽ (orbit) ഗാലക്സിയുടെ മധ്യത്തെ ചുറ്റുകയും ചെയ്യുന്നു. ഭൂമി അതിൻ്റെ ഭ്രമണപഥത്തിൽ (orbit) സൂര്യനെ ചുറ്റുന്നത് പോലെ.
അപ്പോൾ ഒരു സൂര്യസഞ്ചാരം അല്ലെങ്കിൽ സൂര്യൻ്റെ ഭ്രമണപഥം (orbit) എന്നാൽ ഒരു കേന്ദ്രത്തിൽ നിന്നും ഒരേ സമയം ആകർഷിക്കപ്പെടുകയും വികർഷിക്കപ്പെടുകയും (അടുക്കുകയും അകന്നുപോകുകയും) ചെയ്യലല്ലെ ?! സൂര്യഭ്രമണത്തിൻ്റെ ഭ്രമണപഥ രൂപീകരണത്തെ സംബന്ധിച്ച അത്ഭുതകരമായ ഈ ശാസ്ത്രീയ സൂചന നൽകി എന്നത് ഹദീസിൻ്റെ മൗലികതയെ ഊട്ടി ഉറപ്പിക്കുന്നു.
അപ്പോൾ സൂര്യൻ്റെ Rotation (കറക്കം, സഞ്ചാരം) ഒരു പോക്കും വരവും തന്നെയാണ്. ഗാലക്സിയുടെ കേന്ദ്രത്തിലേക്കുളള “മടക്കവും” അതിൽ നിന്ന് തെന്നിമാറിയുള്ള “പോക്കും” ആകുന്നു. ഈ പോക്കും മടക്കവും സൃഷ്ടിക്കുന്ന സൂര്യൻ്റെ ഭ്രമണപഥ കറക്കത്തിൻ്റെ material cause അല്ലെങ്കിൽ efficient cause ആകേന്ദ്രബലവും (centripetal force) അപകേന്ദ്രബലവും (centrifugal force) ആകുന്നു. പക്ഷെ അതിൻ്റെ അവസാന കാരണം ദൈവ സന്നിധിയിലേക്കുള്ള “പോക്കി”ൻ്റെയും “വരവി”ൻ്റെയും വിരാമമൊ ഭംഗമൊ ഇല്ലാത്ത പരമ്പരയാണ് എന്ന് ഹദീസ് വ്യാഖ്യാനിക്കുന്നു.
ആകേന്ദ്രബലവും അപകേന്ദ്രബലവും ഗുരുത്വാകർഷണ ബലവുമൊക്കെ എങ്ങനെ അവസാന കാരണമാവും?! ആ കാരണങ്ങളെല്ലാം അവസാനിക്കാത്ത ചോദ്യങ്ങൾക്ക് വിധേയമാവുന്നുണ്ട്. ഗുരുത്വാകർഷണ ബലത്തിന് കാരണമെന്താണ്? വസ്തുക്കളുടെ പിണ്ഡമാണെങ്കിൽ, പിണ്ഡമെന്തിന് ആകർഷണം ഉണ്ടാക്കുന്നു? പിണ്ഡം സ്പേസ്ടൈം എന്ന് വിളിക്കപ്പെടുന്ന, പ്രപഞ്ചത്തിൻ്റെ നിർമ്മാണത്തെ (fabric of the universe) വളക്കുന്നത് കാരണമാണ് എന്നാണ് ഉത്തരമെങ്കിൽ, അടുത്ത ചോദ്യം പ്രപഞ്ചത്തിൻ്റെ നിർമ്മാണത്തെ പിണ്ഡം എന്തിന് വളക്കുന്നു? എന്നാണ്. ഇതിന് ഒരു പ്രമുഖ ഭൗതികശാസ്ത്ര വെബ്സൈറ്റിൽ വന്ന ഒരു ഉത്തരം കാണുക:
“എന്തുകൊണ്ട്? (why) എന്ന ചോദ്യങ്ങൾക്ക് ഭൗതികശാസ്ത്രം അന്തിമ ഉത്തരം നൽകേണ്ടതുണ്ടോ?എൻ്റെ അഭിപ്രായത്തിൽ, അത് ഭൗതികശാസ്ത്രജ്ഞൻ്റെ ലക്ഷ്യമല്ല, മാത്രമല്ല അത് ഭൗതികശാസ്ത്രത്തിൻ്റെ പരിധിക്കപ്പുറവുമാണ്.
ചുറ്റുമുള്ള അനന്തമായ സങ്കീർണ്ണമായ ലോകത്തിൻ്റെ ചില വശങ്ങൾ മനസ്സിലാക്കുന്നതിനും പ്രവചിക്കുന്നതിനുമുള്ള നമ്മുടെ ഉപകരണങ്ങളായാണ് ഭൗതികശാസ്ത്രം കൂടുതലും സിദ്ധാന്തങ്ങളെ നിർമ്മിക്കുന്നത്. ചിലപ്പോഴൊക്കെ നമ്മൾ ഒരു പൊതു പ്രയോഗക്ഷമതയുള്ള ഒരു സിദ്ധാന്തത്തിലേക്ക് എത്തുന്നു, അത് ഒന്നിലധികം വിച്ഛേദിക്കപ്പെട്ട നിരീക്ഷണങ്ങൾക്ക് ഗംഭീരമായ സൈദ്ധാന്തിക ഏകീകരണം നൽകുന്നു…” (https://physics.stackexchange.com/questions/250709/why-does-matter-curve-space-time)
ഇതു തന്നെയല്ലെ മതവിശ്വാസികൾ കാലാകാലങ്ങളായി തൊണ്ട കീറി പറഞ്ഞു മനസ്സിലാക്കാൻ ശ്രമിച്ചു കൊണ്ടിരുന്നത്?! ശാസ്ത്രം ഉത്തരം അന്വേഷിക്കുന്നത് How (എങ്ങനെ?) ചോദ്യങ്ങൾക്കാണ്. മതത്തിൻ്റെയും തത്ത്വശാസ്ത്രത്തിൻ്റെയും മേഖലയാണ് എന്തുകൊണ്ട്? (why) എന്ന ചോദ്യങ്ങൾ.
സൂര്യപ്രയാണവുമായി ബന്ധപ്പെട്ട, പ്രത്യേകം ശ്രദ്ധേയമായ ഒരു വിവരം കൂടി ഈ ചർച്ചയിൽ കൂട്ടിവായിക്കാൻ അർഹതപ്പെടുന്നു: “സൂര്യൻ ക്ഷീരപഥത്തിന് ചുറ്റും കറങ്ങുമ്പോൾ, മറ്റ് താരാപഥങ്ങളെ അപേക്ഷിച്ച് സൂര്യൻ മുകളിലേക്കും താഴേക്കും നീങ്ങുന്നതായി കാണപ്പെടുന്നു”. (https://public.nrao.edu, www.forbes.com)
സഞ്ചാരത്തിനിടയിൽ സൂര്യൻ്റെ ഈ മുകളിലേക്കും താഴേക്കും നീങ്ങലുകൾ എന്തായിരിക്കും ?! മുകളിലേക്കും താഴേക്കും നീങ്ങലുകൾക്കും material/efficient cause കൾക്കപ്പുറം ആത്യന്തികമായ ഒരു ഉദ്ദേശ്യ സമ്പൂർണമായ കാരണം ഉണ്ടാവും എന്നതാണ് ഒരു വിശ്വാസി കരുതുന്നത്. കാരണം പ്രപഞ്ചത്തിൽ ഒന്നും അലക്ഷ്യവും ആകസ്മികവുമായിട്ടില്ല. ദൈവം എല്ലാം സൃഷ്ടിച്ചത് ലക്ഷ്യപൂർവ്വമാണ്. അവയിൽ ചിലത് നാം അറിയുന്നു, മിക്കവയും നാം അറിയുന്നില്ല. “അറിവില് നിന്ന് അല്പമല്ലാതെ നിങ്ങള്ക്ക് നല്കപ്പെട്ടിട്ടില്ല.” (ക്വുർആൻ: 17: 85)
******************************** ഇനി, സൂര്യ സഞ്ചാരവും രാപ്പകലുകളുടെ നിർമ്മിതിയുമായി ബന്ധപ്പെട്ട വിഷയത്തിലേക്ക് അടുത്തതായി വരാം.
“ഭൂമിയുടെ ഭ്രമണമാണ് രാപ്പകലുകളുടെ കാരണം (cause); അല്ലാതെ സൂര്യ സഞ്ചാരം അല്ല. അപ്പോൾ ഹദീസിലെ അത്തരമൊരു വിശദീകരണം ശാസ്ത്രീയമായി തെറ്റാണ്” എന്നതാണ് വിമർശനം. ഈ വിമർശനവും പല കാരണങ്ങളാലും ബാലിശമാണ്:
1. സൂര്യൻ നമ്മുടെ ഗാലക്സിയുടെ കേന്ദ്രത്തിന് ചുറ്റും ചലിച്ച് കൊണ്ടിരിക്കുന്നു എന്ന, ഒരുപാട് നാൾ ശാസ്ത്രജ്ഞർക്ക് അജ്ഞമായ ഒരു വിവരമാണ് ഹദീസ് സംസാരിക്കുന്നത് എന്ന അത്ഭുതകരമായ വസ്തുത വിമർശകർ “ശ്രദ്ധാപൂർവ്വം” അവഗണിക്കുന്നു.
രാവും പകലും ആവർത്തിക്കുന്നതിൽ ഭൂമിയുടെ സ്വയം ഭ്രമണം മാത്രം പങ്കു വഹിക്കുന്നുള്ളു എന്ന വാദം ഈ ലോകത്ത് ഒരു ശാസ്ത്ര വിജ്ഞാനിയും പറയില്ല. കാരണം, ഇവയിൽ സൂര്യൻ്റെ പ്രയാണത്തിനാണല്ലൊ പ്രാഥമികമായ പങ്ക്. ഗാലക്സിയുടെ ക്രേന്ദഭാഗത്തിന് ചുറ്റും കോണീയ ആക്കത്തോടെ സൂര്യൻ സഞ്ചരിച്ചു കൊണ്ടിരിക്കുന്നത് കൊണ്ടല്ലെ -അടിസ്ഥാനപരമായി – രാപ്പകലുകളുടെ ആവർത്തനം ഉണ്ടാവുന്നത്? അഥവാ സൂര്യൻ്റെ സഞ്ചാരം പൊടുന്നനെ നിന്നാൽ… അല്ലെങ്കിൽ സൂര്യൻ പൊടുന്നനെ എതിർ ദിശയിലേക്ക് സഞ്ചരിക്കുകയും ഗ്രഹങ്ങൾ സൂര്യൻ്റെ ഗുരുത്വാകർഷണ വലയത്തിനപ്പുറത്തേക്ക് മുന്നോട്ട് നീങ്ങിയാൽ പിന്നെ രാവും പകലും ഉണ്ടോ? ഒരിക്കലുമില്ല. സൂര്യൻ്റെ പെട്ടെന്നുള്ള സ്തംഭനം സംഭവിക്കുകയും ഭൂമിയും മറ്റു ഗ്രഹങ്ങളും മുന്നോട്ട് ഗമിക്കുകയും ചെയ്യുന്നതോടെ ഭൂമിയുടെ ഒരു ഭാഗത്ത് മാത്രം പകലും മറുവശത്ത് രാത്രി മാത്രം ദിവങ്ങളോളമൊ ആഴ്ച്ചകളോളം നിലനിൽക്കുകയും പിന്നീട് ഭൂമി എന്നെന്നേക്കുമായ ഇരുട്ടിലേക്കും (രാത്രിയിലേക്കും) അനന്തമായ ശൂന്യാകാശത്തേക്ക് ഒഴുകി പോവുകയും ചെയ്യും. (സൂര്യൻ പടിഞ്ഞാറ് നിന്ന് കിഴക്കോട്ട് സഞ്ചരിക്കുന്നതായി ഭൂമിയിൽ അനുഭവപ്പെടും.)
അപ്പോൾ ദൈവ സമ്മതം നിരന്തരം ലഭിച്ച് സൂര്യൻ അതിൻ്റെ പ്രയാണം അനുസ്യൂതം തുടരുന്നു എന്നതിനാലാണ് അടിസ്ഥാനപരമായി രാപ്പകലുകൾ ഉണ്ടാവുന്നത് എന്ന് വിശദീകരിക്കുന്നതിൽ ശാസ്ത്രീയമായി ഒരു അബദ്ധവുമില്ല.
സൂര്യൻ്റെ ഈ സഞ്ചാരം നിലച്ചാൽ എന്താണ് സംഭവിക്കുക:
“സൂര്യൻ അത്ഭുതകരമായി അപ്രത്യക്ഷമായാൽ, ഭൂമിയും (സൗരയൂഥത്തിലെ മറ്റെല്ലാ വസ്തുക്കളും) അവയുടെ -ഏതാണ്ട് വൃത്താകൃതിയിലുള്ള- ഭ്രമണപഥങ്ങൾ പിന്തുടരുന്നതിനുപകരം, ബഹിരാകാശത്തേക്ക് ഒരു നേർരേഖയിൽ മുന്നോട്ട് നീങ്ങുന്നത് തുടരും… ഭൂമി ബഹിരാകാശത്ത് ഒഴുകുന്നത് തുടരും. അങ്ങനെ സംഭവിച്ചാൽ പോലും, സൂര്യൻ ഭൂമിയിൽ നിന്ന് അപ്രത്യക്ഷമായി ആഴ്ചകൾക്കുള്ളിൽ നാം മരവിച്ച് മരണമടയും!
(https://www.sciencefocus.com/space/what-would-happen-to-earths-orbit-if-the-sun-vanished)
സൂര്യൻ്റെ ചലനം നിന്നാൽ എന്ത് സംഭവിക്കും? എന്ന ചോദ്യത്തിന് Brainly ലെ ഒരു ശാസ്ത്ര കുതുകി എഴുതിയ മറുപടിയിലെ വാചകങ്ങൾ ശ്രദ്ധിക്കൂ:
സൂര്യൻ്റെ ചലനം നിന്നാൽ ഒരു ദിശയിലേക്ക് മാത്രം സൂര്യോദയം നിലനിൽക്കും, അതിലൂടെ മഞ്ഞ് മലകൾ ഉരുകാൻ തുടങ്ങുകയും വെള്ളപ്പൊക്കത്താൽ നാം കഷ്ടപ്പെടുകയും ചെയ്യും… (https://brainly.in/question/43106937)
ഭൂമി എതിർ ദിശയിൽ കറങ്ങിയാൽ സൂര്യൻ പടിഞ്ഞാറ് നിന്ന് ഉദിക്കുകയും കിഴക്ക് അസ്തമിക്കുകയും ചെയ്യുന്നതായി ഭൗമികമായി അനുഭവപ്പെട്ടേക്കാം. എന്നാൽ അത് ഒരു ലോകാവസാനത്തിലേക്ക് എത്തി ചേരില്ല എന്നാണ് ശാസ്ത്രസൂചന.
അങ്ങനെ സംഭവിച്ചാൽ, “ഭൂമി വ്യത്യസ്തമായിരിക്കുമെങ്കിലും ജീവയോഗ്യം തന്നെയായിരിക്കും” എന്നാണ് പഠനങ്ങൾ പറയുന്നത്. ( https://www.sciencefocus.com/planet-earth/what-would-happen-if-earth-spun-the-other-way )
എന്ന് മാത്രമല്ല കൂടുതൽ മനോഹരമായ ഒരു ലോകമാണ് അതിലൂടെ സംജാതമാവുക എന്നു കൂടി പഠനങ്ങൾ നിഗമിക്കുന്നു:
“മൊത്തത്തിൽ, പിന്നിലേക്ക് കറങ്ങുന്ന ഭൂമി പച്ചനിറമുള്ള ഭൂമിയാണെന്ന് ഗവേഷകർ കണ്ടെത്തി. ആഗോള മരുഭൂമിയുടെ വിസ്തീർണം ഏകദേശം 16 ദശലക്ഷം ചതുരശ്ര മൈലിൽ (42 ദശലക്ഷം ചതുരശ്ര കിലോമീറ്റർ) നിന്ന് ഏകദേശം 12 ദശലക്ഷം ചതുരശ്ര മൈൽ (31 ദശലക്ഷം ചതുരശ്ര കിലോമീറ്റർ) ആയി ചുരുങ്ങും. മുൻ മരുഭൂപ്രദേശങ്ങളിൽ പകുതിയോളം പുല്ലുകൾ മുളച്ചു, മറുപകുതിയിൽ മരച്ചെടികൾ ഉയർന്നുവരും…” ( https://www.livescience.com/62405-what-if-earth-rotation-reversed.html )
അപ്പോൾ, സൂര്യൻ പടിഞ്ഞാറ് ഭാഗത്ത് നിന്ന് ഉദിച്ച് കിഴക്ക് അസ്തമിക്കുകയും അതോടൊപ്പം ലോകാവസാനവും ഭൂമിയുടെ സർവ്വനാശവും ഉണ്ടാവുക എന്ന ഒരു പ്രതിഭാസം സംഭവിക്കണമെങ്കിൽ സൂര്യൻ്റെ പ്രയാണത്തിൽ മാറ്റം സംഭവിച്ചെ തീരൂ. ഈ മാറ്റമാണ് ഹദീസിൽ സൂചിപ്പിക്കപ്പെട്ടത്, ഭൂമിയുടെ സഞ്ചാരഗതിയിലുള്ള മാറ്റമല്ല എന്നത് ഹദീസിൻ്റെ ശാസ്ത്രീയതക്ക് മാറ്റ് കൂട്ടുന്നു.
2. സൗരയൂഥത്തിൻ്റെ ഘടന, ഭൂമി, സൂര്യൻ ചന്ദ്രനക്ഷത്രാദികൾ, അവയുടെ രൂപം, അവയുടെ സഞ്ചാരം എന്നിങ്ങനെ ഖഗോളശാസ്ത്രവുമായി ബന്ധപ്പെട്ട മിക്കവാറും ശാസ്ത്രീയ വിവരങ്ങളും വസ്തുതകളും ക്വുർആനിലൂടെയും പ്രവാചകനിലൂടെയും മുസ്ലിംകൾക്ക് നൽകപ്പെട്ടിരിരുന്നു. ഉദാഹരണത്തിന് ക്വുർആനിലെ കേവലം മൂന്ന് വചനങ്ങളിലൂടെ മുസ്ലിംകൾക്ക് ലഭിച്ച ഖഗോളശാസ്ത്ര വിജ്ഞാനങ്ങൾ എന്തൊക്കെയായിരുന്നു എന്ന് നമ്മുക്കൊന്ന് പരിശോധിക്കാം:
“അവനത്രെ രാത്രി, പകല്, സൂര്യന്, ചന്ദ്രന് എന്നിവയെ സൃഷ്ടിച്ചത്. ഓരോന്നും ഓരോ ഭ്രമണപഥത്തിലൂടെ നീന്തിക്കൊണ്ടിരിക്കുന്നു.” (ക്വുർആൻ: 21:33)
“സൂര്യന് ചന്ദ്രനെ പ്രാപിക്കാനാവില്ല. രാവ് പകലിനെ മറികടക്കുന്നതുമല്ല. ഓരോന്നും ഓരോ (നിശ്ചിത) ഭ്രമണപഥത്തില് നീന്തിക്കൊണ്ടിരിക്കുന്നു.”(ക്വുർആൻ: 36:40)
“രാവിനെ അവന് പകലില് പ്രവേശിപ്പിക്കുന്നു. പകലിനെ രാവിലും പ്രവേശിപ്പിക്കുന്നു. സൂര്യനെയും ചന്ദ്രനെയും അവന് (തന്റെ നിയമത്തിന്) വിധേയമാക്കുകയും ചെയ്തിരിക്കുന്നു. അവയോരോന്നും നിശ്ചിതമായ ഒരു പരിധി വരെ സഞ്ചരിക്കുന്നു. അങ്ങനെയുള്ളവനാകുന്നു നിങ്ങളുടെ രക്ഷിതാവായ അല്ലാഹു.” (ക്വുർആൻ: 35: 13)
ഈ മൂന്ന് ക്വുർആൻ വചനങ്ങൾ താഴെ പറയുന്ന ഖഗോളശാസ്ത്ര വസ്തുതകളെ നൂറ്റാണ്ടുകൾക്ക് മുമ്പ് തന്നെ പ്രഖ്യാപിച്ചു:
* ഭൂമി, സൂര്യൻ, ചന്ദ്രൻ എന്നിങ്ങനെ എല്ലാ ഗോളങ്ങളും തുടർ പ്രയാണത്തിലും ചലനത്തിലുമാണ്.
* كُلࣱّ فِی فَلَكࣲ یَسۡبَحُونَ“…ഓരോന്നും ഓരോ ഭ്രമണപഥത്തിലൂടെ നീന്തിക്കൊണ്ടിരിക്കുന്നു.” (ക്വുർആൻ: 21: 33) എന്ന വാചകത്തിലെ ‘യസ്ബഹൂൻ'(يسبحون) എന്ന പദത്തിലൂടെ സൂര്യചന്ദ്രനക്ഷത്രാദികൾ ഭ്രമണപഥത്തിലൂടെ സഞ്ചരിക്കുക മാത്രമല്ല, സ്വയം തിരിയുക കൂടി ചെയ്യുന്നുവെന്ന് സൂചിപ്പിക്കുന്നു.
* എല്ലാ ഗോളങ്ങളും ഒരു ഭ്രമണപഥത്തിലൂടെ (orbit) യാണ് സഞ്ചരിക്കുന്നത്.
* ഭ്രമണപഥങ്ങൾ (orbit) ഏതാണ്ട് വൃത്താകൃതിയിലാണ്; ദീർഘവൃത്താകൃതി (elliptical).
ക്വുർആനിൽ ഭ്രമണപഥം എന്നതിന് പ്രയോഗിക്കപ്പെട്ട “ഫലക്” (الفَلَك) എന്ന പദം തന്നെ വൃത്താകൃതിയെ സൂചിപ്പിക്കുന്നുണ്ട്.
الفَلَك: مَدَار النجوم. والجمع: أفلاك. وفَلَكَ كل شيء: مُستداره ومٌعظَمه. وفَلَكَ البحر: مَوْجه المستدير المتردّدഫലക് (الفَلَك) എന്ന പദം വട്ടത്തെയും വളഞ്ഞ, കറങ്ങി വരുന്ന വഴിയെയുമെല്ലാമാണ് സൂചിപ്പിക്കുന്നത് എന്ന് പൗരാണിക അറബി ഭാഷാ പണ്ഡിതൻമാർ വ്യക്തമാക്കിയിട്ടുണ്ട്. (ഫിൽ മുഹ്കമുൽ മുഹീത്വ്: ഇബ്നു സീദ:)
ഫലക് (الفَلَك) എന്ന പദം വൃത്താകൃതിയെ സൂചിപ്പിക്കുന്നു എന്ന് പ്രവാചക ശിഷ്യനായ ഇബ്നു അബ്ബാസും വിശദീകരിച്ചിട്ടുണ്ട്. (അസ്വവാഇകു ശ്ശദീദ അലാ ഇത്തിബാഇൽ ഹൈഅതിൽ ജദീദ: ഹമൂദ് ബിൻ അബ്ദുല്ല അത്തുവൈജിരി, ഫൈദുൽ ക്വദീർ: മുനാവി:1:647)
* كُلࣱّ فِی فَلَكࣲ یَسۡبَحُونَ“…ഓരോന്നും ഓരോ ഭ്രമണപഥത്തിലൂടെ നീന്തിക്കൊണ്ടിരിക്കുന്നു.” (ക്വുർആൻ: 21: 33) എന്ന വാചകത്തിലെ “ഓരോന്നും ഓരോ ഭ്രമണപഥത്തിലൂടെ” (كُلࣱّ فِی فَلَكࣲ) എന്നത് ഒരു ഉച്ഛാരണപരമായ അനുലോമവിലോമ വാചകമാണ്. അനുലോമവിലോമപദം/വാചകം (palindrome) എന്നാൽ, madam അല്ലെങ്കിൽ racecar പോലെയുള്ള മുന്നോട്ടും പിന്നോട്ടും ഒരു പോലെ വായിക്കാവുന്ന വാക്കുകളും, സംഖ്യകളും, ശൈലികളും ഒക്കെ ഉൾക്കൊള്ളുന്ന ഒരു ശ്രേണിയാണ്.
كُلࣱّ فِی فَلَكࣲ ك ل ف ي ف ل كഈ അനുലോമവിലോമ വാചകളിലെ അക്ഷരങ്ങൾ തമ്മിൽ വരച്ചു ചേർത്താൽ ഗോളങ്ങളുടെ ഭ്രമണപഥത്തിൻ്റെ (orbit) രൂപം ദൃശ്യമാവും!
* ഭൂമിയും മറ്റു ഗ്രഹങ്ങളും ഉപഗ്രഹങ്ങളുമെല്ലാം ഉരുണ്ട ആകൃതിയിലാണ് എന്നും ഈ മൂന്ന് വാക്യങ്ങളിൽ നിന്നും തെളിയുന്നു. അതുകൊണ്ട് തന്നെ ഭൂമിയും ഗ്രഹങ്ങളും ഉപഗ്രഹങ്ങളുമെല്ലാം ഗോളാകൃതിയിലാണ് എന്ന കാര്യത്തിൽ പ്രവാചക കാലഘട്ടം മുതൽക്കെ, മുസ്ലിം പണ്ഡിതന്മാർക്കിടയിൽ ഇജ്മാഅ് (ഏകാഭിപ്രായം) ഉണ്ടായിരുന്നു. ഇക്കാര്യത്തിൽ മുസ്ലിം പണ്ഡിതന്മാർക്കിടയിൽ ഇജ്മാഅ് (ഏകാഭിപ്രായം) ഉണ്ടെന്നും, യാതൊരു അഭിപ്രായ വ്യത്യാസവും അവർക്കിടയിൽ നിലവിലുള്ളതായി അറിയില്ല എന്നും ഹിജ്റാബ്ദം രണ്ടാം ഒന്നാം നൂറ്റാണ്ടുകാരായ, ഇമാം അഹ്മദുബ്നു ഹമ്പലിൻ്റെ ശിഷ്യഗണങ്ങൾ, ഹിജ്റാബ്ദം 256 മരണപ്പെട്ട (869 CE) ഇബ്നു മുനാദി, ഹിജ്റാബ്ദം 384 മരണപ്പെട്ട (994 CE) ഇബ്നു തൈമിയ, ഹിജ്റാബ്ദം 661 മരണപ്പെട്ട (1263 CE) ഇബ്നു തൈമിയ, ഹിജ്റാബ്ദം 543 ൽ ഭൂജാതനായ ഇമാം റാസി (1149 CE) തുടങ്ങി ഒട്ടനവധി പൗരാണിക പണ്ഡിതന്മാർ വ്യക്തമാക്കിയിട്ടുണ്ട്. (മജ്മൂഉൽ ഫതാവാ: 25/195, 6/586, അൽഫസ്ൽ ഫിൽ മിലലി വൽഅഹ്വാഇ വന്നിഹൽ:2/78)
ക്വുർആനിലും (39: 5) ഹദീസിലും വന്ന (യുകവ്വിറു يكور പന്ത് പോലെ ഉരുട്ടുന്നു എന്ന) പ്രസ്താവനകളിൽ നിന്ന് അറിവിൽ നേതൃപദവി (ഇമാം) അലങ്കരിക്കുന്ന സർവ്വ പണ്ഡിതരും ഭൂമിയുടെ ഗോളാകൃതി അംഗീകരിക്കുന്നവരായിരുന്നു എന്നും ഇമാം ഇബ്നു ഹസം വിശദീകരിക്കുകയുണ്ടായി. (അൽഫസ്ൽ ഫിൽ മിലലി വൽഅഹ്വാഇ വന്നിഹൽ: 2/78)
ഖഗോളശാസ്ത്രവുമായി ബന്ധപ്പെട്ട മിക്കവാറും ശാസ്ത്രീയ വിവരങ്ങളും വസ്തുതകളും ക്വുർആനിലൂടെയും പ്രവാചകനിലൂടെയും മുസ്ലിംകൾക്ക് നൽകപ്പെട്ടിരിരുന്നു എന്നർത്ഥം. ഇത് ഈ കാലഘട്ടത്തിലെ മുസ്ലിംകൾ മുന്നോട്ട് വെക്കുന്ന ഒരു അവകാശവാദമല്ല എന്ന് പ്രവാചകശിഷ്യന്മാരുടെയും പൗരാണിക മുസ്ലിം പണ്ഡിതന്മാരുടെയും ക്വുർആൻ വ്യാഖ്യാനത്തിൽ നിന്നും തന്നെ ഇവിടെ തെളിയിച്ചു കഴിഞ്ഞു. സൗരയൂഥത്തിൻ്റെ ഘടന, ഭൂമി, സൂര്യൻ ചന്ദ്രനക്ഷത്രാദികൾ, അവയുടെ രൂപം, അവയുടെ സഞ്ചാരം എന്നിങ്ങനെ സർവ്വതും മുസ്ലിംകൾ പണ്ടു മുതലേ -ക്വുർആനിൽ നിന്നും ഹദീസുകളിൽ നിന്നും – മനസ്സിലാക്കിയിരുന്നു. എന്നിട്ടും ഹദീസിലെ സൂര്യൻ്റെ സാഷ്ടാംഗവും അവർക്ക് അവരുടെ ഖഗോളശാസ്ത്ര വിജ്ഞാനങ്ങൾക്ക് എതിരായി ഒരു കാലഘട്ടത്തിലും തോന്നിയില്ല. അതിന് കാരണം സൂര്യഭ്രമണത്തിൻ്റെ material cause നെ പറ്റി പഠിപ്പിക്കുകയല്ല മതത്തിൻ്റെ ആത്യന്തികമായ ലക്ഷ്യം എന്ന തിരിച്ചറിവാണ്. ഈ പ്രകൃതി പ്രതിഭാസങ്ങളുടെ ആത്മീയ മണ്ഡലവും പരമമായ കാരണങ്ങളും (ദൈവിക ലക്ഷ്യം) വിശദീകരിക്കുകയാണ് മതം ചെയ്യുന്നത് എന്ന കേവല ബുദ്ധിയും തത്ത്വജ്ഞാനവും അവർക്കെല്ലാം ഉണ്ടായിരുന്നു. അനുഭവം മാത്രമാണ് ജ്ഞാനത്തിനു കാരണമെന്നുള്ള എൻപിരിസസ (empiricism) ജ്വരവും ശാസ്ത്രത്തെ മതവൽകരിക്കുന്ന സയൻ്റിസമൊ (scientism) മുസ്ലിംകളിലെ തത്ത്വജ്ഞാന ബോധത്തെയും യുക്തിയെയും ഊറ്റി കളഞ്ഞിട്ടില്ല എന്നതാണ് ഈ തിരിച്ചറിവിന് കാരണം.
ചുരുക്കത്തിൽ, അശാസ്ത്രീയവും അയുക്തികരവുമായ ഒന്നും ഹദീസിൻ്റെ ഉള്ളടക്കത്തിൽ ഇല്ല. ശാസ്ത്രീയ പ്രവചനങ്ങളുടെയും സൂചനകളുടെയും ഒരു അത്ഭുത കലവറയാണ് യഥാർത്ഥത്തിൽ ഈ ഹദീസ്.
വിമർശനം:
സൂര്യനെ സൃഷ്ടിക്കപ്പെടുന്നതിന് മുമ്പ് ചെടികളും മരങ്ങളും സൃഷ്ടിക്കപ്പെട്ടു എന്ന ബൈബിളിലെ ഉൽപ്പത്തി പ്രതിബാധനത്തിലെ അബദ്ധം മുഹമ്മദ് നബിയും കോപ്പിയടിച്ചു. തന്നിമിത്തം ഹദീസിലേയും ഉൽപ്പത്തി പ്രതിബാധനത്തിൽ ധാരാളം അബദ്ധങ്ങൾ ഉള്ളടങ്ങിയിട്ടില്ലെ ?!
മറുപടി:
ക്വുർആനിൽ, പ്രവാചകൻ ഇബ്റാഹീമിന്റെ (അബ്രഹാം) കാലത്തെ ഈജിപ്തിലെ ഭരണാധികാരിയെ സംബന്ധിച്ച് “അല്ലാഹു അധികാരം നൽകിയ” വ്യക്തി (آتَاهُ اللَّهُ الْمُلْكَ) (ക്വുർആൻ: 2: 258) എന്നാണ് പറയപ്പെടുന്നത്… പ്രവാചകൻ യൂസുഫിന്റെ (അ) (ജോസഫ്) കാലത്തെ ഈജിപ്തിലെ ഭരണാധികാരിയെ സംബന്ധിച്ച് “രാജാവ്” (ملك King) എന്നാണ് പറയപ്പെടുന്നത്; മൂസാ നബിയുടെ(അ) (മോശെ) കാലത്തെ ഭരണാധികാരിയെ “ഫിർഔൻ” (Pharaoh فرعون ഫറവോൻ) എന്നുമാണ് പറയപ്പെടുന്നത്.
ജോസഫിന്റെ മരണത്തിന് ശേഷമാണ് ഈജിപ്തിലെ ഭരണാധികാരികളെ സൂചിപ്പിക്കാൻ “ഫറവോൻ” എന്ന പദവി ഉപയോഗിക്കാൻ തുടങ്ങിയത്.
എന്നാൽ ബൈബിളിൽ, ജോസഫിന്റെ കാലത്തെ ഭരണാധികാരിയെയും “ഫറവോൻ” എന്ന പേരിലാണ് പരിചയപ്പെടുത്തുന്നത്.
ജോസഫിനും മുമ്പ്, അബ്രഹാമിന്റെ കാലത്തെ ഭരണാധികാരിയേയും “ഫറവോൻ” എന്ന പേരിലാണ് (ആറു തവണ) പരിചയപ്പെടുത്തുന്നത്. (ഉല്പത്തി: 12: 10-20)
ജോസഫിന്റെ കാലഘത്തിലെ ഭരണാധികാരിയേയും “ഫറവോൻ” എന്ന പേരിൽ തന്നെയാണ് (തൊന്നൂറ് തവണ) പരിചയപ്പെടുത്തുന്നത്. (ഉല്പത്തി: 41: 14, 25, 46…)
അതിന് കാരണം, എല്ലാ കാലഘട്ടത്തിലും, ഈജിപ്റ്റിലെ ഭരണാധികാരികളെ എല്ലാം “ഫറവോൻ” എന്ന പേരിലാണ് വിളിക്കപ്പെട്ടിരുന്നത് എന്ന തെറ്റിദ്ധാരണയിൽ, മനുഷ്യരാൽ രചിക്കപ്പെട്ടതാണ് എന്നതിനാലാണ് ഈ ചരിത്രപരമായ് സ്കലിതം ബൈബിളിൽ വരാൻ കാരണം.
ക്വുർആൻ, മനുഷ്യ നിർമ്മിതമായിരുന്നെങ്കിൽ, ബൈബിൾ നോക്കി പകർത്തിയതായിരുന്നെങ്കിൽ ഇത്രയും സൂക്ഷ്മവും, സങ്കീർണവും, നൂറ്റാണ്ടുകൾക്ക് ശേഷം തിരിച്ചറിഞ്ഞതുമായ ഈ സ്കലിതം ക്വുർആനിലും സംഭവിക്കുമായിരുന്നില്ലെ? പക്ഷെ അങ്ങനെ സംഭവിച്ചില്ല ! ക്വുർആനിൽ പ്രവാചകൻ ഇബ്റാഹീമിന്റെ (അ) കാലഘട്ടത്തിലെ ഈജിപ്ത്യൻ ഭരണാധികാരിയെ “അല്ലാഹു അധികാരം നൽകിയ” വ്യക്തി (آتَاهُ اللَّهُ الْمُلْكَ) എന്നും (ക്വുർആൻ:2:258), പ്രവചകൻ യൂസുഫിന്റെ കാലഘട്ടത്തിലെ ഈജിപ്ത്യൻ ഭരണാധികാരിയെ “രാജാവ്” (ملك) എന്നും (ക്വുർആൻ: 12: 76) അഭിസംബോധനം ചെയ്ത അത്യത്ഭുതകരമായ സൂക്ഷ്മതയിൽ നിന്ന് രണ്ട് കാര്യങ്ങൾ സുതരാം വ്യക്തമാവുന്നു: ഒന്ന്, ക്വുർആൻ ബൈബിളിന്റെ കോപിയല്ല. ബൈബിളിൽ ദിവ്യബോധനത്തിൽ നിന്നുമുള്ള ഭാഗങ്ങൾ ഉള്ളടങ്ങിയിട്ടുണ്ട് എന്നതിനാൽ തന്നെ ബൈബിളും ക്വുർആനും വിഷയാതിഷ്ടിതമായ സാമ്യതകൾ ഉണ്ടാകാം എന്ന് മാത്രം.
രണ്ട്, ബൈബിളിൽ പരാമർശിക്കപ്പെട്ട സങ്കീർണവും അതിസൂക്ഷ്മവും മാനുഷികവുമായ തെറ്റ് ക്വുർആൻ ആവർത്തിക്കാതിരുന്നതും ക്വുർആന്റെ അത്യത്ഭുതകരമായ ചരിത്ര കൃത്യതയും തെളിയിക്കുന്നത് ക്വുർആന്റെ അമാനുഷികതയെയാണ്. ഇത് മനുഷ്യനിൽ നിന്നല്ല എന്നർത്ഥം.
സമാനമാണ് വിമർശന വിധേയമായ ഹദീസിന്റെയും അവസ്ഥ. ബൈബിളിൽ നിന്നുള്ള കോപ്പിയടിയാണ് ഹദീസും, അതിനാൽ അതിൽ ധാരാളം അബദ്ധങ്ങൾ ആവർത്തിക്കപ്പെട്ടു എന്നല്ല വിമർശന വിധേയമായ ഹദീസിൽ നിന്നും സൂക്ഷ്മ പരിശോധനയിൽ നിന്നും മനസ്സിലാവുക. പ്രത്യുത, ബൈബിളിൽ ഉൽപ്പത്തിയെ സംബന്ധിച്ച് പറയപ്പെട്ട വിശദീകരണത്തിലെ സൂക്ഷ്മമായ അബദ്ധങ്ങളും അശാസ്ത്രീയതകളും അത്യൽഭുതകരമായ വിധം ഹദീസിലെ ഉൽപ്പത്തി വിശദീകരണത്തിൽ ഒഴിവാക്കപ്പെട്ടിരിക്കുകയാണ് !
ഇത് മനസ്സിലാക്കാനായി ആദ്യം നമ്മുക്ക് ബൈബിളിലെ, ഉൽപ്പത്തി പുസ്തകം: അദ്ധ്യായം ഒന്നിലെ, 1 മുതൽ 31 വരെയുളള വാക്യങ്ങൾ വായിച്ചു നോക്കാം:
1 ആദിയിൽ ദൈവം ആകാശവും ഭൂമിയും സൃഷ്ടിച്ചു. 2 ഭൂമി പാഴായും ശൂന്യമായും ഇരുന്നു; ആഴത്തിന്മീതെ ഇരുൾ ഉണ്ടായിരുന്നു. ദൈവത്തിന്റെ ആത്മാവ് വെള്ളത്തിന്മീതെ പരിവർത്തിച്ചു കൊണ്ടിരുന്നു. 3 വെളിച്ചം ഉണ്ടാകട്ടെ എന്നു ദൈവം കല്പിച്ചു; വെളിച്ചം ഉണ്ടായി. 4 വെളിച്ചം നല്ലത് എന്നു ദൈവം കണ്ടു; ദൈവം വെളിച്ചവും ഇരുളും തമ്മിൽ വേർപിരിച്ചു. 5 ദൈവം വെളിച്ചത്തിനു പകൽ എന്നും ഇരുളിനു രാത്രി എന്നും പേരിട്ടു. സന്ധ്യയായി ഉഷസ്സുമായി, ഒന്നാം ദിവസം. 6 ദൈവം: വെള്ളങ്ങളുടെ മധ്യേ ഒരു വിതാനം ഉണ്ടാകട്ടെ; അതു വെള്ളത്തിനും വെള്ളത്തിനും തമ്മിൽ വേർപിരിവായിരിക്കട്ടെ എന്നു കല്പിച്ചു. 7 വിതാനം ഉണ്ടാക്കിയിട്ടു ദൈവം വിതാനത്തിൻകീഴുള്ള വെള്ളവും വിതാനത്തിന്മീതെയുള്ള വെള്ളവും തമ്മിൽ വേർപിരിച്ചു; അങ്ങനെ സംഭവിച്ചു. 8 ദൈവം വിതാനത്തിന് ആകാശം എന്നു പേരിട്ടു. സന്ധ്യയായി ഉഷസ്സുമായി, രണ്ടാം ദിവസം. 9 ദൈവം: ആകാശത്തിൻകീഴുള്ള വെള്ളം ഒരു സ്ഥലത്തു കൂടട്ടെ; ഉണങ്ങിയ നിലം കാണട്ടെ എന്നു കല്പിച്ചു; അങ്ങനെ സംഭവിച്ചു. 10 ഉണങ്ങിയ നിലത്തിനു ദൈവം ഭൂമി എന്നും വെള്ളത്തിന്റെ കൂട്ടത്തിനു സമുദ്രം എന്നും പേരിട്ടു; നല്ലത് എന്നു ദൈവം കണ്ടു. 11 ഭൂമിയിൽനിന്നു പുല്ലും വിത്തുള്ള സസ്യങ്ങളും ഭൂമിയിൽ അതതുതരം വിത്തുള്ള ഫലം കായ്ക്കുന്ന വൃക്ഷങ്ങളും മുളച്ചുവരട്ടെയെന്നു ദൈവം കല്പിച്ചു; അങ്ങനെ സംഭവിച്ചു. 12 ഭൂമിയിൽനിന്നു പുല്ലും അതതുതരം വിത്തുള്ള ഫലം കായ്ക്കുന്ന വൃക്ഷങ്ങളും മുളച്ചുവന്നു; നല്ലത് എന്നു ദൈവം കണ്ടു. 13 സന്ധ്യയായി ഉഷസ്സുമായി, മൂന്നാം ദിവസം. 14 പകലും രാവും തമ്മിൽ വേർപിരിവാൻ ആകാശവിതാനത്തിൽ വെളിച്ചങ്ങൾ ഉണ്ടാകട്ടെ; അവ അടയാളങ്ങളായും കാലം, ദിവസം, സംവത്സരം എന്നിവ തിരിച്ചറിവാനായും ഉതകട്ടെ; 15 ഭൂമിയെ പ്രകാശിപ്പിപ്പാൻ ആകാശവിതാനത്തിൽ അവ വെളിച്ചങ്ങളായിരിക്കട്ടെ എന്നു ദൈവം കല്പിച്ചു; അങ്ങനെ സംഭവിച്ചു. 16 പകൽ വാഴേണ്ടതിനു വലിപ്പമേറിയ വെളിച്ചവും രാത്രി വാഴേണ്ടതിനു വലിപ്പം കുറഞ്ഞ വെളിച്ചവും ആയി രണ്ടു വലിയ വെളിച്ചങ്ങളെ ദൈവം ഉണ്ടാക്കി; നക്ഷത്രങ്ങളെയും ഉണ്ടാക്കി. 17 ഭൂമിയെ പ്രകാശിപ്പിപ്പാനും പകലും രാത്രിയും വാഴുവാനും വെളിച്ചത്തെയും ഇരുളിനെയും തമ്മിൽ വേർപിരിപ്പാനുമായി 18 ദൈവം അവയെ ആകാശവിതാനത്തിൽ നിർത്തി; നല്ലത് എന്നു ദൈവം കണ്ടു. 19 സന്ധ്യയായി ഉഷസ്സുമായി, നാലാം ദിവസം. 20 വെള്ളത്തിൽ ജലജന്തുക്കൾ കൂട്ടമായി ജനിക്കട്ടെ; ഭൂമിയുടെമീതെ ആകാശവിതാനത്തിൽ പറവജാതി പറക്കട്ടെ എന്നു ദൈവം കല്പിച്ചു. 21 ദൈവം വലിയ തിമിംഗലങ്ങളെയും വെള്ളത്തിൽ കൂട്ടമായി ജനിച്ചു ചരിക്കുന്ന അതതുതരം ജീവജന്തുക്കളെയും അതതുതരം പറവജാതിയെയും സൃഷ്ടിച്ചു; നല്ലത് എന്നു ദൈവം കണ്ടു. 22 നിങ്ങൾ വർധിച്ചു പെരുകി സമുദ്രത്തിലെ വെള്ളത്തിൽ നിറവിൻ; പറവജാതി ഭൂമിയിൽ പെരുകട്ടെ എന്നു കല്പിച്ചു ദൈവം അവയെ അനുഗ്രഹിച്ചു. 23 സന്ധ്യയായി ഉഷസ്സുമായി, അഞ്ചാം ദിവസം. 24 അതതുതരം കന്നുകാലി, ഇഴജാതി, കാട്ടുമൃഗം ഇങ്ങനെ അതതുതരം ജീവജന്തുക്കൾ ഭൂമിയിൽനിന്ന് ഉളവാകട്ടെ എന്നു ദൈവം കല്പിച്ചു; അങ്ങനെ സംഭവിച്ചു. 25 ഇങ്ങനെ ദൈവം അതതുതരം കാട്ടുമൃഗങ്ങളെയും അതതുതരം കന്നുകാലികളെയും അതതുതരം ഭൂചരജന്തുക്കളെയും ഉണ്ടാക്കി; നല്ലത് എന്നു ദൈവം കണ്ടു. 26 അനന്തരം ദൈവം: നാം നമ്മുടെ സ്വരൂപത്തിൽ നമ്മുടെ സാദൃശ്യപ്രകാരം മനുഷ്യനെ ഉണ്ടാക്കുക; അവർ സമുദ്രത്തിലുള്ള മത്സ്യത്തിന്മേലും ആകാശത്തിലുള്ള പറവജാതിയിന്മേലും മൃഗങ്ങളിന്മേലും സർവഭൂമിയിന്മേലും ഭൂമിയിൽ ഇഴയുന്ന എല്ലാ ഇഴജാതിയിന്മേലും വാഴട്ടെ എന്നു കല്പിച്ചു. 27 ഇങ്ങനെ ദൈവം തന്റെ സ്വരൂപത്തിൽ മനുഷ്യനെ സൃഷ്ടിച്ചു, ദൈവത്തിന്റെ സ്വരൂപത്തിൽ അവനെ സൃഷ്ടിച്ചു, ആണും പെണ്ണുമായി അവരെ സൃഷ്ടിച്ചു. 28 ദൈവം അവരെ അനുഗ്രഹിച്ചു: നിങ്ങൾ സന്താനപുഷ്ടിയുള്ളവരായി പെരുകി ഭൂമിയിൽ നിറഞ്ഞ് അതിനെ അടക്കി സമുദ്രത്തിലെ മത്സ്യത്തിന്മേലും ആകാശത്തിലെ പറവജാതിയിന്മേലും സകല ഭൂചരജന്തുവിന്മേലും വാഴുവിൻ എന്ന് അവരോടു കല്പിച്ചു. 29 ഭൂമിയിൽ എങ്ങും വിത്തുള്ള സസ്യങ്ങളും വൃക്ഷത്തിന്റെ വിത്തുള്ള ഫലം കായ്ക്കുന്ന സകല വൃക്ഷങ്ങളും ഇതാ, ഞാൻ നിങ്ങൾക്കു തന്നിരിക്കുന്നു; അവ നിങ്ങൾക്ക് ആഹാരമായിരിക്കട്ടെ; 30 ഭൂമിയിലെ സകല മൃഗങ്ങൾക്കും ആകാശത്തിലെ എല്ലാ പറവകൾക്കും ഭൂമിയിൽ ചരിക്കുന്ന സകല ഭൂചരജന്തുക്കൾക്കും ആഹാരമായിട്ടു പച്ചസസ്യമൊക്കെയും ഞാൻ കൊടുത്തിരിക്കുന്നു എന്നു ദൈവം കല്പിച്ചു; അങ്ങനെ സംഭവിച്ചു. 31 താൻ ഉണ്ടാക്കിയതിനെയൊക്കെയും ദൈവം നോക്കി, അത് എത്രയും നല്ലത് എന്നു കണ്ടു. സന്ധ്യയായി ഉഷസ്സുമായി, ആറാം ദിവസം.”
ഏഴ് ദിവസങ്ങളിലെ ഓരോ ദിവസവും നടന്ന സൃഷ്ടിപ്പിനൊപ്പം “സന്ധ്യയായി ഉഷസ്സുമായി, ഒന്നാം ദിവസം…”, “സന്ധ്യയായി ഉഷസ്സുമായി, രണ്ടാം ദിവസം…” എന്ന് ആവർത്തിക്കപ്പെടുന്നത് ശ്രദ്ധിക്കുക. ആ ദിവസങ്ങളിലെ സൃഷ്ടിപ്പുകൾ കാലക്രമം അനുസരിച്ച് (chronological order) നടന്നു എന്നാണ് ബൈബിൾ വാദിക്കുന്നത്. എന്ന് വെച്ചാൽ… രണ്ടാം ദിവസം ഭൂമിയിൽ പുല്ലും വിത്തുള്ള സസ്യങ്ങളും വൃക്ഷങ്ങളും മുളപ്പിച്ചു… മൂന്നാം ദിവസം… പകലും രാവും തമ്മിൽ വേർപിരിവാനും കാലം, ദിവസം, സംവത്സരം എന്നിവ തിരിച്ചറിവാനായും… വെളിച്ചങ്ങൾ (സൂര്യ ചന്ദ്രൻമാരെ) സൃഷ്ടിച്ചു. നാലാം ദിവസം വെള്ളത്തിലെ ജലജന്തുക്കളെയും സൃഷ്ടിച്ചു… എന്നിങ്ങനെ കാലക്രമം അനുസരിച്ചാണ് സൃഷ്ടിപ്പ്. അപ്പോൾ സൂര്യനും ചന്ദ്രനും ഇല്ലാതെ ഒന്നാം ദിവസം ഉഷസ്സും സന്ധ്യയും എങ്ങനെ ആയി? സൂര്യനും ചന്ദ്രനും സൃഷ്ടിക്കപ്പെടുന്നതിന് മുമ്പ് രാവും പകലും എങ്ങനെ ഉണ്ടായി ?!സൂര്യപ്രകാശം സൃഷ്ടിക്കുന്നതിന് മുമ്പ് സസ്യങ്ങളും വൃക്ഷങ്ങളും എങ്ങനെ നിലനിൽക്കുന്നു ?! ഇങ്ങനെ ഒരുപാട് വൈരുധ്യങ്ങളും ശാസ്ത്രാബദ്ധങ്ങളും ബൈബിളിലെ ഉൽപ്പത്തി പ്രതിബാധനത്തിൽ വന്നു ഭവിക്കുന്നു.
എന്നാൽ ഹദീസിലെ ഉൽപ്പത്തി വിവരണത്തിൽ ഈ അബദ്ധങ്ങളും ശാസ്ത്ര വിരുദ്ധതയും ഒഴിവാക്കപ്പെട്ടിരിക്കുന്നു:
ദൈവം ശനിയാഴ്ച്ച മണ്ണ് സൃഷ്ടിച്ചു. ഞായറാഴ്ച്ച പർവ്വതങ്ങളെ ഉണ്ടാക്കി. തിങ്കളാഴ്ച്ച മരങ്ങളെ സൃഷ്ടിച്ചു. ചൊവ്വാഴ്ച്ച മക്റൂഹ് (വെറുക്കപ്പെട്ടത്, ദോഷകരമായ വസ്തുക്കൾ) സൃഷ്ടിച്ചു. ബുധനാഴ്ച്ച പ്രകാശം (ഗുണകരമായ വസ്തുക്കൾ) സൃഷ്ടിച്ചു. വ്യാഴാഴ്ച്ച ജന്തുക്കളെ ഭൂമിയിൽ പരത്തി. വെള്ളിയാഴ്ച്ചയുടെ അവസാന സന്ധിയിൽ, അസറിനും രാത്രിക്കും ഇടയിൽ ആദമിനെ (സ) സൃഷ്ടിച്ചു. (സ്വഹീഹു മുസ്ലിം: 2789) എന്നാണ് ഹദീസിലെ ഉള്ളടക്കം. ഇവിടെ ദിവസങ്ങളിലെ സൃഷ്ടിപ്പുകൾ കാലക്രമം അനുസരിച്ച് (chronological order) നടന്നതാണ് എന്ന ബൈബിളിലെ അബദ്ധം ഒഴിവാക്കപ്പെട്ടിരിക്കുന്നു ! ഒന്നാം ദിവസം, രണ്ടാം ദിവസം… തുടങ്ങിയ കാലക്രമമൊ, സന്ധ്യയായി ഉഷസ്സായി തുടങ്ങി തുടർച്ചയെയൊ പ്രസ്താവിച്ചില്ല എന്നത് അത്ഭുതകരമാം വിധം സൂക്ഷ്മത അടങ്ങുന്നതാണ്.
“എന്റെ മകൻ ജനിച്ചത് ഞായറാഴ്ച്ചയാണ്. അതുകഴിഞ്ഞ് രണ്ടാം ദിവസമാണ് ഞാൻ ജനിച്ചത്. അതു കഴിഞ്ഞ് മൂന്നാം ദിവസമാണ് എന്റെ അച്ഛൻ ജനിച്ചത്. അതുകഴിഞ്ഞ് നാലാം ദിവസം എന്റെ അമ്മ ജനിച്ചു…” എന്ന് പറയൽ ശുദ്ധ അസംബന്ധമാണ് എന്ന് പ്രത്യേകം തെളിയിക്കേണ്ടതില്ലല്ലൊ. ഈ ഉൽപ്പത്തി പ്രതിബാധനമനുസരിച്ച് എന്റെ മകൻ എന്നേക്കാൾ മൂത്തതും, ഞാൻ ജനിച്ച ശേഷമാണ് എന്റെ അച്ചനും അമ്മയും ജനിച്ചത് എന്നും വരുന്നു ! ഇത്തരമൊരു ഉൽപ്പത്തി പ്രതിബാധനമാണ് ബൈബിൾ അവതരിപ്പിക്കുന്നത്.
അതേ സമയം ഒരാൾ ഇപ്രകാരം പറഞ്ഞു എന്ന് കരുതുക:
“എന്റെ മകൻ ജനിച്ചത് ഞായറാഴ്ചയാണ്. ഞാൻ ജനിച്ചത് തിങ്കളാഴ്ചയാണ്. എന്റെ അച്ഛൻ ജനിച്ചത് ചൊവ്വാഴ്ചയാണ്. എന്റെ അമ്മ ജനിച്ചത് ബുധനാഴ്ചയാണ്.”
ഇവിടെ ഉൽപ്പത്തിയുടെ ക്രമമൊ കാല തുടർച്ചയൊ പറയപ്പെടുന്നില്ല; ദിവസങ്ങൾക്ക് മാത്രമെ കാലക്രമം സംജാതമാകുന്നുള്ളു. ഇതാണ് ഹദീസിലെ ഉൽപ്പത്തി പ്രതിബാധന രീതിയുടെ വ്യത്യാസം.
ഹദീസിലെ ഉൽപ്പത്തി പ്രതിബാധനത്തിലെ മറ്റൊരു വ്യത്യാസം കൂടി ശ്രദ്ധിക്കേണ്ടതുണ്ട്:
ഇബ്നുൽ അസീർ (റ) പറഞ്ഞു: “മരങ്ങളെ സൃഷ്ടിച്ചു. ചൊവ്വാഴ്ച്ച മക്റൂഹ് (വെറുക്കപ്പെട്ടത്) എന്നത് കൊണ്ട് ഇവിടെ ഉദ്ദേശ്യം ദോഷകരമായ കാര്യങ്ങളാണ്. കാരണം ഹദീസിൽ തുടർന്ന് പറയുന്നത് “ബുധനാഴ്ച്ച പ്രകാശം സൃഷ്ടിച്ചു” എന്നാണ്. പ്രകാശം എന്നാൽ ഉപകാര പ്രധമായ കാര്യങ്ങളാണ്. ദോഷകരമായ കാര്യങ്ങളെ മക്റൂഹ് (വെറുക്കപ്പെട്ടത്) എന്ന് വിളിക്കപ്പെടാൻ കാരണം, അവ ‘പ്രിയങ്കരമായ’ കാര്യങ്ങൾക്ക് വിപരീതമായത് കൊണ്ടാണ്.” (അന്നിഹായ: 4:169)
ചുരുക്കത്തിൽ, ബൈബിളിലെ ഉൽപ്പത്തി വിവരണത്തിൽ നിന്നുള്ള കോപ്പിയടിയായിരുന്നു ഹദീസും എങ്കിൽ അതിലും ബൈബിളിലെ അബദ്ധങ്ങൾ ആവർത്തിക്കപ്പെടുമായിരുന്നു. എന്നാൽ വിമർശന വിധേയമായ ഹദീസിന്റെ സൂക്ഷ്മ പരിശോധനയിൽ നിന്നും മനസ്സിലാവുക, ബൈബിളിൽ ഉൽപ്പത്തിയെ സംബന്ധിച്ച് പറയപ്പെട്ട വിശദീകരണത്തിലെ സൂക്ഷ്മമായ അബദ്ധങ്ങളും അശാസ്ത്രീയതകളും – വിശിഷ്യാ കാലക്രമം – അത്യൽഭുതകരമായ വിധം ഹദീസിലെ ഉൽപ്പത്തി വിശദീകരണത്തിൽ ഒഴിവാക്കപ്പെട്ടിരിക്കുന്നു എന്നാണ് ! ഇത് ഹദീസുകളുടെ ദിവ്യബോധന അടിത്തറയെയും സത്യതയെയുമാണ് തെളിയിക്കുന്നത്.
**********************************വിമർശനം: ആകാശഭൂമികളെ ആറ് ദിവസം കൊണ്ട് സൃഷ്ടിച്ചു എന്ന് ക്വുർആൻ പറയുന്നു. ആകാശത്തെ രണ്ട് ദിവസങ്ങൾ കൊണ്ടും ഭൂമിയെ രണ്ട് ദിവസങ്ങൾ കൊണ്ടും ഭൂമിയിലെ വിശദാംശങ്ങളെ രണ്ട് ദിവസങ്ങൾ കൊണ്ടും സൃഷ്ടിച്ചു. അപ്പോൾ പിന്നെ ഹദീസിൽ ഏഴ് ദിവസങ്ങൾ കൊണ്ട് നടന്ന സൃഷ്ടിപ്പ് ക്വുർആനിക പരാമർശത്തോട് എതിരല്ലെ ?
മറുപടി: ഒരിക്കലുമല്ല. കാരണം, ക്വുർആൻ സംസാരിക്കുന്നത് ഭൂമി സൃഷ്ടിക്കപ്പെടുന്നതിന് മുമ്പുള്ള, ഭൂമിയുടെ സൃഷ്ടിപ്പു കൂടി ഉൾപ്പെടുന്ന അഭൗമികമായ (ആറ്) ദിവസങ്ങളെ സംബന്ധിച്ചാണ്. ചർച്ച വിഷയകമായ ഹദീസിൽ പരാമർശിക്കുന്നത് ഭൂമി സൃഷ്ടിക്കപ്പെട്ടതിന് ശേഷം, ഭൂമിയിലെ വിശദാംശങ്ങളുടെ സൃഷ്ടിപ്പ് നടന്ന ഭൗമികമായ (ഏഴ്) ദിവസങ്ങളെ സംബന്ധിച്ചുമാണ്.
അറബി പദമായ يوم (യവ്മ്, ദിവസം) ഭൗമികമായ ദിവസത്തെ സൂചിപ്പിക്കാൻ മാത്രമല്ല ഉപയോഗിക്കപ്പെടുന്നത്. കാലത്തിന്റെ ഏതു തരം ഘട്ടങ്ങളെയും ഭാഗങ്ങളെയും യവ്മ് (ദിവസം) എന്ന പദം കൊണ്ട് വിവക്ഷിക്കാവുന്നതാണ്. ഇതിന് ഉദാഹരണങ്ങൾ ക്വുർആനിൽ തന്നെ കാണാവുന്നതാണ്:
“പിന്നീട് ഒരു ദിവസം കാര്യം അവങ്കലേക്ക് ഉയര്ന്ന് പോകുന്നു. നിങ്ങള് കണക്കാക്കുന്ന തരത്തിലുള്ള ആയിരം വര്ഷമാകുന്നു ആ ദിവസത്തിന്റെ അളവ്.” (ക്വുർആൻ: 32: 5)
“അമ്പതിനായിരം കൊല്ലത്തിന്റെ അളവുള്ളതായ ഒരു ദിവസത്തില് മലക്കുകളും ആത്മാവും അവങ്കലേക്ക് കയറിപ്പോകുന്നു.” (ക്വുർആൻ: 70: 4)
ആയിരം ഭൗമിക വർഷങ്ങളോടു തുല്യമായ ഒരു യവ്മ് (ദിവസം), ഭൗമികമായ അമ്പതിനായിരം കൊല്ലത്തിന്റെ അളവുള്ളതായ ഒരു യവ്മ് (ദിവസം) എന്നൊക്കെ ക്വുർആൻ പറഞ്ഞതിൽ നിന്നും കാലത്തിന്റെ ഏതു തരം ഘട്ടങ്ങളെയും ഭാഗങ്ങളെയും യവ്മ് (ദിവസം) എന്ന പദം കൊണ്ട് വിവക്ഷിക്കാവുന്നതാണ് എന്ന് വ്യക്തം.
ഈ വസ്തു മനസ്സിലാക്കിയതിന് ശേഷം ശൈഖ് സ്വാലിഹ് അൽ മുനജ്ജിദ് എഴുതുന്നത് വായിക്കുക:
“അല്ലാഹു ആകാശങ്ങളെ രണ്ട് അഭൗമികമായ ദിവസം കൊണ്ട് (അഥവാ രണ്ട് കാല- ഘട്ടം കൊണ്ട്) സൃഷ്ടിച്ചു. ശേഷം ഭൂമിയെ രണ്ട് അഭൗമികമായ ദിവസം കൊണ്ട് (അഥവാ രണ്ട് കാല- ഘട്ടം കൊണ്ട്) സൃഷ്ടിച്ചു. ശേഷം ഭൂമിയിലെ വിശദാംശങ്ങൾ രണ്ട് അഭൗമികമായ ദിവസം കൊണ്ട് (അഥവാ രണ്ട് കാല- ഘട്ടം കൊണ്ട്) സൃഷ്ടിച്ചു. അപ്പോൾ മൊത്തം ആറ് അഭൗമികമായ ദിവസങ്ങൾ കൊണ്ട് ആകാശ ഭൂമികളെ സൃഷ്ടിച്ചു.
ഭൂമിയിലെ വിശദാംശങ്ങൾ സൃഷ്ടിച്ച രണ്ട് അഭൗമികമായ ദിവസങ്ങൾ, ഏഴ് ഭൗമികമായ ദിവസങ്ങൾക്ക് തുല്യമാണ്. ഈ രണ്ട് അഭൗമികമായ ദിവസങ്ങളിൽ ( അഥവാ ഏഴ് ഭൗമികമായ ദിവസങ്ങളിൽ ) നടന്ന സൃഷ്ടിപ്പിനെ സംബന്ധിച്ചാണ് ഹദീസിൽ ചർച്ച ചെയ്യപ്പെടുന്നത്.
ആകാശ ഭൂമികളെ ആറ് ദിവസങ്ങൾ കൊണ്ട് സൃഷ്ടിച്ചു എന്ന് ക്വുർആൻ പറയുന്ന ദിവസങ്ങൾ സൂര്യ ചന്ദ്ര ഉദയാസ്തമങ്ങൾ കൊണ്ട് അളക്കപ്പെടുന്ന ഭൗമികമായ ദിവസങ്ങൾ അല്ല. ആ സമയത്ത് സൂര്യനും ഭൂമിയും സൃഷ്ടിക്കപ്പെട്ടിട്ടു തന്നെയില്ലല്ലൊ; പിന്നെ അതെങ്ങനെ ഭൗമികമായ ദിവസങ്ങൾ ആവും ?! യവ്മ് (ദിവസം) എന്നാൽ കാലത്തിന്റെ അപേക്ഷികമായ ഭാഗങ്ങൾ അല്ലെങ്കിൽ ഘട്ടങ്ങൾ മാത്രമാണ്.”
ശൈഖ് നാസ്വിറുദ്ധീൻ അൽബാനി പറഞ്ഞു:
“ഹദീസിൽ പറയപ്പെട്ടിട്ടുള്ള ഏഴ് ദിവസങ്ങൾ, (ആകാശഭൂമികൾ സൃഷ്ടിക്കപ്പെട്ട,) ക്വുർആനിൽ പ്രസ്ഥാപിക്കപ്പെട്ട ആറ് ദിവസങ്ങളല്ല. ഹദീസിൽ പ്രസ്ഥാവിക്കപ്പെട്ടിരിക്കുന്നത് ഭൂമിൽ അല്ലാഹു നടപ്പാക്കിയ സംവിധാനങ്ങളുടെ വിശതമായ വിവരണമാണ്. ക്വുർആനിൽ പ്രസ്ഥാപിക്കപ്പെട്ടതിന് പുറമെ – അധികമായി – നടന്ന സൃഷ്ടിപ്പുകളും സംവിധാനങ്ങളുമാണ് ഹദീസിലെ പ്രമേയം; അല്ലാതെ ക്വുർആനിൽ പ്രസ്ഥാപിക്കപ്പെട്ടതിന് എതിരായ വിവരങ്ങളല്ല… ” (മുഖ്തസ്വറുൽ ഉലുവ് ലിഅലിയ്യുൽ അളീം: 112)
ക്വുർആനിലെ ആറും ഹദീസിലെ ഏഴും, ഇവ രണ്ടും (ഭൗമികവും അഭൗമികവുമായ) രണ്ട് തരം ദിവസങ്ങളും, രണ്ട് വ്യത്യസ്ഥ വിഷയങ്ങളുമാണ് എന്ന് ഹദീസുകളിൽ തന്നെ വ്യക്തമാക്കപ്പെട്ടിട്ടുണ്ട്:
ഇമാം നസാഈയുടെ “സുനനുൽ കുബ്റാ” യിലെ (6/427/11392) ഒരു ഹദീസിൽ മുഹമ്മദ് നബി (സ), തന്റെ അനുചരനായ അബൂ ഹുറൈറയോട് (റ) ഇപ്രകാരം പറഞ്ഞതായി വന്നിരിക്കുന്നു: “അല്ലയൊ അബൂ ഹുറൈറ ! അല്ലാഹു അകാശഭൂമികളെയും അവക്കിടയിലുള്ളവയെയും ആറ് (അഭൗമിക) ദിവസങ്ങളിലായി (കാല ഘട്ടങ്ങളായി) സൃഷ്ടിച്ചു. ഏഴാമത്തെ ദിവസം അവൻ സിംഹാസനസ്ഥനായി. (ഭൂമിയിലെ വിശദാംശങ്ങളുടെ സൃഷ്ടിപ്പ് ഭൗമികമായ ദിവസങ്ങളിൽ പറഞ്ഞാൽ) മണ്ണ് ശനിയാഴ്ച്ച അവൻ പടച്ചു…(ശേഷം ഏഴ് ഭൗമികമായ ദിവസങ്ങളിലെ സൂക്ഷ്മായ സൃഷ്ടിപ്പുകൾ പ്രവാചകൻ (സ) എണ്ണി പറഞ്ഞു…”
ഈ ഹദീസിൽ നിന്ന് കാര്യങ്ങൾ വ്യക്തമായി മനസ്സിലാക്കാം. ആകാശ ഭൂമികളുടെ സൃഷ്ടിപ്പ് നടന്നത് അഭൗമികമായ ആറ് ദിവസങ്ങളിൽ (ഘട്ടങ്ങളിൽ) ആയാണ്. അതിൽ രണ്ട് അഭൗമിക ഘട്ടങ്ങൾ ഭൗമികമായ ഏഴ് ദിവസങ്ങൾക്ക് തുല്യമാണ്. ശേഷം ഭൗമികമായ ഓരോ ദിവസവും സൃഷ്ടിക്കപ്പെട്ട ഓരോ സൃഷ്ടിപ്പും പ്രവാചകൻ (സ) എണ്ണി പറഞ്ഞു. ശനിയാഴ്ച്ച മണ്ണ് സൃഷ്ടിച്ചു. ഞായറാഴ്ച്ച പർവ്വതങ്ങളെ ഉണ്ടാക്കി… അപ്പോൾ ക്വുർആനും ചർച്ചാ വിഷയകമായി ഹദീസും തമ്മിൽ വൈരുദ്ധ്യമില്ല എന്ന് മാത്രമല്ല. രണ്ട് തരം ദിവസങ്ങളെയും കൂട്ടിയിണക്കി തന്നെ ഒരു ഹദീസ് കാര്യങ്ങൾ വ്യക്തമാക്കുന്നുണ്ട്.
*******************************വിമർശനം:
ആകാശ ഭൂമികളെ ആറ് ദിവസങ്ങളിലായി സൃഷ്ടിച്ചു എന്ന് ക്വുർആനിൽ പലയിടത്തും പറയുന്നത്. എന്നാൽ ഇതിനോട് വിരുദ്ധമായി ആകാശ ഭൂമികളെ ഏട്ട് ദിവസങ്ങളിലായി സൃഷ്ടിച്ചു എന്ന് ക്വുർആനിൽ ഒരു അധ്യായത്തിൽ പരാമർശിക്കുന്നില്ലേ ?
മറുപടി:
“നീ പറയുക: രണ്ടുദിവസങ്ങളിലായി ഭൂമിയെ സൃഷ്ടിച്ചവനില് നിങ്ങള് അവിശ്വസിക്കുകയും അവന്ന് നിങ്ങള് സമന്മാരെ സ്ഥാപിക്കുകയും തന്നെയാണോ ചെയ്യുന്നത്? അവനാകുന്നു ലോകങ്ങളുടെ രക്ഷിതാവ്. അതില് (ഭൂമിയില്) – അതിന്റെ ഉപരിഭാഗത്ത് – ഉറച്ചുനില്ക്കുന്ന പര്വ്വതങ്ങള് അവന് സ്ഥാപിക്കുകയും അതില് അഭിവൃദ്ധിയുണ്ടാക്കുകയും, അതിലെ ആഹാരങ്ങള് അവിടെ വ്യവസ്ഥപ്പെടുത്തി വെക്കുകയും ചെയ്തിരിക്കുന്നു. നാലു ദിവസങ്ങളിലായിട്ടാണ് (അവനത് ചെയ്തത്.)ആവശ്യപ്പെടുന്നവര്ക്ക് വേണ്ടി ശരിയായ അനുപാതത്തില്. (ക്വുർആൻ: 41: 9, 10)
ഈ ആയത്തുകൾ പ്രകാരം രണ്ടുദിവസങ്ങളിലായി ഭൂമിയെ സൃഷ്ടിച്ചു. പര്വ്വതങ്ങള്, അഭിവൃദ്ധി, ആഹാരങ്ങള് തുടങ്ങിയ വിശദമായ സൃഷ്ടിപ്പുകൾ നാല് ദിവസങ്ങൾ കൊണ്ട് നടത്തി. അതിലേക്ക് ആകാശങ്ങളെ സൃഷ്ടിച്ച രണ്ട് ദിവസങ്ങൾ കൂടി ചേർത്താൽ… 2+4+2 എട്ട് ദിവസങ്ങൾ ആയില്ലെ ? ക്വുർആനിൽ മറ്റു പലയിടത്തും പറയുന്ന ആറ് ദിവസ കണക്കിന് എതിരല്ലെ ? എന്നതാണ് വിമർശനം.
ഈ വിമർശനം തെറ്റായ കണക്കുകൂട്ടലിനെ അടിസ്ഥപ്പെടുത്തിയാണ് വിമർശകർ ഉന്നയിക്കുന്നത്. ശരിയായ കണക്ക് ഇപ്രകാരമാണ്:
രണ്ടുദിവസങ്ങളിലായി ഭൂമിയെ സൃഷ്ടിച്ചു. തുടർന്ന് പര്വ്വതങ്ങള്, അഭിവൃദ്ധി, ആഹാരങ്ങള് തുടങ്ങിയ വിശദമായ സൃഷ്ടിപ്പുകൾ നടത്തി. فِي أَرْبَعَةِ أَيَّامٍ “നാലു ദിവസങ്ങളിലായിട്ടാണ് (അവനത് ചെയ്തത്.)” (ക്വുർആൻ: 41:10) അഥവാ നാലു ദിവസങ്ങളിലായിട്ടാണ് ഭൂമിയെയും തുടർന്ന് പര്വ്വതങ്ങള്, അഭിവൃദ്ധി, ആഹാരങ്ങള് തുടങ്ങിയ വിശദമായ കാര്യങ്ങളെയും സൃഷ്ടിച്ചത്. ഇതിലേക്ക് ആകാശങ്ങളെ സൃഷ്ടിച്ച രണ്ട് ദിവസങ്ങൾ കൂടി ചേർത്താൽ… 2+2+2 ആറ് ദിവസങ്ങൾ. 2+2(അഥവാ 4)+2 = 6.
ഇസ്ലാം വിമർശനങ്ങൾ ഉയർന്നു വരുന്നതിന് നൂറ്റാണ്ടുകൾ എത്രയൊ മുമ്പ് ഭാഷാ പണ്ഡിതരായ ക്വുർആൻ വ്യാഖ്യാതാക്കൾ ഈ കണക്ക് വ്യക്തമായി നമ്മെ കൂട്ടി പഠിപ്പിച്ചിട്ടുണ്ട്. അൽപം ചില ഉദാഹരണങ്ങൾ ഇവിടെ ചേർക്കാം:
ഇമാം ക്വുർതുബി (ജനനം: 1214 CE) പറഞ്ഞു: “ഭൂമിയും അതിലെ വിശദാംശങ്ങളും മൊത്തം നാല് ദിവസങ്ങൾ (ഘട്ടങ്ങൾ) കൊണ്ട് സൃഷ്ടിച്ചു എന്നാണ് ഇവിടെ ഉദ്ദേശിച്ചിരിക്കുന്നത്. ഉദാഹരണത്തിന് ഒരാൾ ഇപ്രകാരം പറഞ്ഞു എന്ന് കരുതുക: “ഞാൻ ബസ്വറയിൽ നിന്ന് ബഗ്ദാദിലേക്ക് പത്ത് ദിവസം കൊണ്ട് യാത്ര ചെയ്ത് എത്തി. കൂഫയിലേക്ക് പതിനഞ്ച് ദിവസങ്ങൾ കൊണ്ട് എത്തി. മൊത്തം പതിനഞ്ച് ദിവസം യാത്ര എടുത്തു.” (അൽ ജാമിഉ ലി അഹ്കാമിൽ ക്വുർആൻ: 15: 343)
കാരണം ബസ്വറയിൽ നിന്നും കൂഫയിലേക്കുള്ള യാത്രക്ക് ഇടയിലാണ് ബഗ്ദാദ്. ബഗ്ദാദ് യാത്ര, കൂഫയിലേക്കുള്ള യാത്രയുടെ ഒരു ഭാഗം മാത്രമാണ്. നമ്മുടെ നാട്ടിലെ ഒരു ഉദാഹരണം പറഞ്ഞാൽ ഇങ്ങനെയുണ്ടാവും: ഞാൻ കാസർകോട് നിന്നും മലപ്പുറം വരെ അഞ്ച് ദിവസം കൊണ്ട് യാത്ര ചെയ്തു. കാസർകോട് നിന്നും തിരുവനന്തപുരം വരെ പത്ത് ദിവസം കൊണ്ട് യാത്ര ചെയ്തു. മൊത്തം പത്ത് ദിവസങ്ങളാണ് യാത്ര; പതിനഞ്ചല്ല.
ഇമാം ബഗ്വി (ജനനം: 1044 CE) പറയുന്നു: ഭൂമിയും അതിലെ വിശദാംശങ്ങളും മൊത്തം നാല് ദിവസങ്ങൾ (ഘട്ടങ്ങൾ) കൊണ്ട് സൃഷ്ടിച്ചു എന്നാണ് ഇവിടെ ഉദ്ദേശിച്ചിരിക്കുന്നത്. ഭൂമി രണ്ട് ദിവസങ്ങൾ (ഘട്ടങ്ങൾ) കൊണ്ട് സൃഷ്ടിച്ചു. ഭൂമിയിലുള്ള സൂക്ഷ്മമായ സൃഷ്ടിപ്പുകൾ -ഭൂമി സൃഷ്ടിച്ച രണ്ട് ദിവസങ്ങൾ കൂടി ചേർത്ത് – നാല് ദിവസങ്ങൾ കൊണ്ട് സൃഷ്ടിച്ചു. അപ്പോൾ മൊത്തം നാല് ദിവസങ്ങൾ കൊണ്ട് അതിലെ വിശദാംശങ്ങളും സൃഷ്ടിച്ചു എന്നർത്ഥം; ആറ് ദിവസങ്ങൾ കൊണ്ടല്ല. (തഫ്സീറുൽ ബഗ്വി: 7:165)
അസ്സജാജ് പറഞ്ഞു: (ജനനം: 855 CE) وقال الزجاج : في تتمة أربعة أيام ، يريد بالتتمة اليومين . “രണ്ട് (അഭൗമിക) ദിവസങ്ങൾ കൊണ്ട് ഭൂമിയും രണ്ട് (അഭൗമിക) ദിവസങ്ങൾ കൊണ്ട് ഭൂമിയിലെ വിശദാംശങ്ങളും സൃഷ്ടിച്ചു. മൊത്തം നാല് (അഭൗമിക) ദിവസങ്ങൾ കൊണ്ട് ഭൂമിയും ഭൂമിയിലെ സൂക്ഷ്മമായ കാര്യങ്ങളും സൃഷ്ടിച്ചു എന്നർത്ഥം.”ഇത് അറബി ഭാഷാപണ്ഡിതനായ സമഖ്ശരി (ജനനം: 1074 CE) തന്റെ “കശ്ശാഫ്” എന്ന ഗ്രന്ഥത്തിൽ (3: 444) ഉദ്ധരിക്കുന്നു.
മനുഷ്യർക്ക് ഉപദ്രവമുണ്ടാക്കുന്ന ചില ജീവികളെ ആവശ്യ സന്ദർഭത്തിൽ കൊല്ലാനുള്ള അനുവാദം പ്രവാചകൻ (സ) നൽകിയിട്ടുണ്ട്. ഇത്തരം ഉപദ്രവകാരികളായ ജീവികളെ ‘ഫവാസിക്‘ (فَوَاسِقُ) എന്നാണ് ഹദീസുകളിൽ വിളിച്ചിട്ടുള്ളത്. ഇത്തരം ജീവികളെ സാധാരണയായി Animal rights activist കളല്ലാത്ത എല്ലാവരും – മതത്തിന്റെയൊ ആദർശത്തിന്റെയൊ വ്യത്യാസമില്ലാതെ കൊല്ലാറുമുണ്ട്. (പക്ഷെ ഇത്തരം ഉപദ്രവകാരികളായ ജീവികളെ കൊല്ലാൻ, മുഹമ്മദ് നബി (സ) അനുവാദം നൽകി എന്നതുകൊണ്ട് ‘താൽകാലിക’ അഹിംസ വാദികളായി ഇസ്ലാം വിമർശകർ നാട്യം കളിക്കാറുണ്ടെന്ന് മാത്രം.) ഫവാസിക് (فَوَاسِقُ) ഉപദ്രവകാരികളായ ജീവികൾ എന്ന് പ്രവാചകൻ (സ) എണ്ണിയവ ഹദീസുകളിൽ നിന്ന് വായിക്കാം: خَمْسٌ مِنَ الدَّوَابِّ كُلُّهَا فَوَاسِقُ تُقْتَلُ فِي الْحَرَمِ: الْغُرَابُ، وَالْحِدَأَةُ، وَالْكَلْبُ الْعَقُورُ، وَالْعَقْرَبُ، وَالْفَارَةُ.
“മൃഗങ്ങളിൽ നിന്നുള്ള അഞ്ചെണ്ണം ‘ഫവാസികുകൾ’ (ഉപദ്രവകാരികൾ) ആകുന്നു. അവയെ ഹറമിൽ വെച്ചായാൽ (പോലും) കൊല്ലൽ അനുവദനീയമാണ്. അവ: കാക്ക, ഗരുഡൻ, കടിക്കുന്ന നായ്, തേൾ, എലി എന്നിവയാണ്.”
ചില നിവേദനത്തിൽ തേളിന് പകരം സർപ്പത്തെ പറയപ്പെട്ടിരിക്കുന്നു. മറ്റൊരു നിവേദനത്തിൽ കാക്കയെ ”അൽ ഗുറാബുൽ അബ്കഅ്” (وَالْغُرَابُ الأَبْقَعُ) എന്ന് പ്രത്യേകമായി വിശേഷിപ്പിക്കുകയും ചെയ്തിരിക്കുന്നു.
“അവയെ കൊല്ലുന്നതിൽ പാപമില്ല” എന്ന് ഒരു നിവേദനത്തിൽ കാണാം. (ബുഖാരി: 1829, മുസ്ലിം: 1198, തുർമുദി: 837, നസാഈ: 2888, ഇബ്നുമാജ: 3087)
മറ്റു ചില നിവേദനങ്ങളിൽ ഈ ‘ഫവാസിക്കു’കളുടെ കൂട്ടത്തിൽ ചെന്നായ, പുലി എന്നിവയെയൊക്കെ പ്രസ്ഥാവിച്ചതായും വന്നിരിക്കുന്നു(ഫത്ഹുൽ ബാരി: 4: 30) എന്നതിൽ നിന്നെല്ലാം ഇവയെ കൊല്ലാൻ അനുവാദം നൽകിയതിന്റെ കാരണം വ്യക്തമാണ്. അവ സാധാരണ വളർത്തു മൃഗങ്ങളിൽ നിന്നും വ്യത്യസ്ഥമായി ഉപദ്രവകാരികളാണ് എന്നതാണത്.
ഇമാം മാലിക് പറഞ്ഞു: മനുഷ്യനെ കടിക്കുകയും ആക്രമിക്കുകയും ഭയപ്പെടുത്തുകയും ചെയ്യുന്ന പുലി, സിംഹം, ചെന്നായ എന്നിവയും കടിക്കുന്ന നായയുടെ സ്ഥാനത്ത് തന്നെയാണ്. (അൽ മുൻതകാ ശർഹുൽ മുവത്വഅ് 2: 262)
ഇമാം ഐനി പറഞ്ഞു: “ഫവാസിക്കുകളിൽ കാക്കയെ എണ്ണിയപ്പോൾ, “വെള്ളയും കറുപ്പും നിറം കലർന്ന കാക്ക” (ഗുറാബുൽ അബ്കഅ് وَالْغُرَابُ الأَبْقَعُ) എന്ന് ഒരു ഹദീസിൽ പ്രത്യേകമായി വിശേഷിപ്പിച്ചിരിക്കുന്നു. കാക്കയെ കൊല്ലാൻ അനുവാദം നൽകിയത് കാക്ക ഇങ്ങോട്ട് ഉപദ്രവിക്കുന്നു എന്നതിനാലാണ്. “വെള്ളയും കറുപ്പും നിറം കലർന്ന കാക്ക”കളാണ് ഇങ്ങോട്ട് ഉപദ്രവിക്കുക. അതല്ലാത്ത, ഉപദ്രവകാരികളല്ലാത്ത കാക്കകളെ കൊല്ലാനും പാടില്ല എന്ന് ഹദീസിൽ നിന്ന് മനസ്സിലാക്കാം.” (ഉംദത്തുൽ കാരി 10:180)
ഇമാം ഇബ്നു ഹജർ പറഞ്ഞു: “ഹദീസിന്റെ ആശയത്തിൽ നിന്ന് കൊല്ലാൻ അനുവാദം നൽകിയതിന് കാരണം മനുഷ്യരെ ഉപദ്രവിക്കുക എന്നതാണ് എന്ന് വരുന്നു. അപ്പോൾ മനുഷ്യരെ ഉപദ്രവിക്കുന്ന ഏത് ജീവിയേയും ആവശ്യഘട്ടത്തിൽ ഫവാസിക്ക് എന്നതിൽ ഉൾപ്പെടുത്താം.” (ഫത്ഹുൽ ബാരി: 4: 30)
ഈ ഒരു അടിസ്ഥാനത്തിൽ നിന്നുകൊണ്ട്, പല്ലി ശല്യം അധികരിച്ചാൽ അവയുടെ ഉപദ്രവകാരണത്താൽ അവയെ കൊല്ലാനും പ്രവാചകൻ (സ) അനുവാദം നൽകി. പല്ലിയുൾപ്പെടെയുള്ള ‘ഫവാസിക്കു’കളെ തേടിപ്പിടിച്ച് കൊല്ലാനല്ല പ്രവാചക കൽകപ്പനയുടെ ഉദ്ദേശമെന്ന് “അവയെ കൊല്ലുന്നതിൽ കുറ്റമില്ല.” (لا حرج على من قتلهن) എന്ന ഹദീസിലെ (ബുഖാരി: 1828) വാചകത്തിൽ നിന്ന് മനസ്സിലാക്കാം. വേട്ടയാടേണ്ട ഒന്നല്ല പല്ലി എന്ന കാര്യത്തിൽ മുസ്ലിം പണ്ഡിതന്മാരെല്ലാം ഏകോപിച്ചിരിക്കുന്നു എന്ന് അബൂ അംറ് അൽ കുർതുബി (മരണം: 463 ഹി) പറയുന്നു. (അത്തംബീഹ് ലിമാ ഫിൽ മുവത്വഅ് മിനൽ മആനി വൽ അസാനീദ് 15:187)
കൂടാതെ, “പല്ലികളെ കൊല്ലുവാൻ പ്രവാചകൻ (സ) കൽപ്പിച്ചതായി ‘ഞാൻ’ കേട്ടിട്ടില്ല” എന്ന് പ്രവാചക പത്നി ആഇശ (റ) പറഞ്ഞതിന്റെ (സ്വഹീഹുൽ ബുഖാരി: 1831) അർത്ഥമെന്താണ്? പ്രിയ പത്നി ആഇശയോട് പല്ലികളെ കൊല്ലുന്നതിനെ സംബന്ധിച്ച് പ്രവാചകൻ (സ) സംസാരിച്ചിട്ടേയില്ല എന്നാണ് ! അഥവാ പല്ലിയെ കൊല്ലാൻ പറഞ്ഞത് പല്ലി ശല്യവും ഉപദ്രവവും ഉള്ളവരോട് മാത്രമാണ്. പ്രവാചകൻ (സ) സ്വയം പല്ലിയെ കൊന്നതായും ഒരു ഹദീസിലും ഇല്ല !!! (പല്ലിയെ കൊല്ലൽ മതത്തിൽ ഒരു പുണ്യകർമമായിരുന്നെങ്കിൽ പ്രവാചകൻ (സ) അത് നിരന്തരമായി അനുഷ്ടിക്കാതിരിക്കില്ലല്ലൊ.) ഇതും സൂചിപ്പിക്കുന്നത് പല്ലിയെ കൊല്ലൽ പല്ലി ശല്യവും ഉപദ്രവവും ഉള്ളവർക്കുള്ള ഒരു സ്വഭാവിക അനുമതി മാത്രമാണ്. അല്ലാതെ പല്ലിയെ കൊല്ലൽ ഒരു പ്രമേയമായോ കാമ്പയിനായോ അദ്ദേഹം അവതരിപ്പിച്ചിട്ടില്ല എന്നർത്ഥം.
പല്ലിയെ കൊല്ലാൻ അനുവാദം നൽകിയ ഹദീസുകളിൽ അതിനുള്ള കാരണവും വ്യക്തമാക്കിയിട്ടുണ്ട്. അവ ‘ഫുവൈസിക്ക്’ അഥവാ ‘കുറിയ ഉപദ്രവകാരികളാണ്’ എന്നതാണത്.
أَنَّ رَسُولَ اللَّهِ صَلَّى اللهُ عَلَيْهِ وَسَلَّمَ، قَالَ لِلْوَزَغِ: فُوَيْسِقٌ…
“അല്ലാഹുവിന്റെ തിരുദൂതൻ (സ) പല്ലികളെ സംബന്ധിച്ച് ‘ഫുവൈസിക്ക്’ (കുറിയ ഉപദ്രവകാരി) എന്ന് പറഞ്ഞു…” (സ്വഹീഹുൽ ബുഖാരി: 1831)
أَنَّ النَّبِيَّ صَلَّى اللهُ عَلَيْهِ وَسَلَّمَ” أَمَرَ بِقَتْلِ الْوَزَغِ ، وَسَمَّاهُ فُوَيْسِقًا”
“പല്ലികളെ കൊല്ലാൻ (അനുവാദം നൽകിക്കൊണ്ട്) പ്രവാചകൻ (സ) കൽപ്പന പുറപ്പെടുവിച്ചു. അതിനെ ‘ഫുവൈസിക്ക്’ (കുറിയ ഉപദ്രവകാരി) എന്ന് അദ്ദേഹം വിളിച്ചു.” (സ്വഹീഹു മുസ്ലിം: 2238)
ഒരു അന്ധവിശ്വാസത്തിന്റെയൊ മിഥ്യാ ധാരണയുടെയൊ അടിസ്ഥാനത്തിലല്ല പല്ലികളെ കൊല്ലാൻ പ്രവാചകൻ (സ) അനുവാദം നൽകിയത്. അവ മനുഷ്യർക്ക് ശല്യവും ഉപദ്രവവുമായി മാറുന്നതിന്റെ അടിസ്ഥാനത്തിലാണ് എന്ന് ഹദീസുകളിൽ വ്യക്തമായി പ്രഖ്യാപിച്ചിട്ടുണ്ട് എന്ന് ചുരുക്കം.
ഇമാം ദിംയരി പറഞ്ഞു: “പല്ലിയെ സംബന്ധിച്ച് ‘ഫുവൈസിക്ക്’ (കുറിയ ഉപദ്രവകാരി) എന്ന് വിശേഷിപ്പിക്കാൻ കാരണം, ഹറമിലും അല്ലാത്തിടത്തും കൊല്ലാൻ അനുവാദം നൽകപ്പെട്ട ഉപദ്രവകാരികളായ (ഫവാസിക്ക്) ജീവികളിൽ പല്ലി പെടുന്നു എന്നതിനാലാണ്. ‘ഫിസ്ക്’ (الفسق) എന്ന പദത്തിന്റെ അടിത്തറ ‘ഖുറൂജ് ‘ (الخروج പുറത്തുപോവുക) എന്നതാണ്. ഇപ്പറഞ്ഞ (അഞ്ച് ഉപദ്രവകാരികളായ ജീവികളും) മനുഷ്യനെ കടിച്ചും ഉപദ്രവിച്ചും ശാരീരിക അപായങ്ങൾ വരുത്തിയും ഭൂരിഭാഗം ജീവികളുടെയും സ്വഭാവത്തിൽ നിന്നും പ്രകൃതത്തിൽ നിന്നും ‘പുറത്തുപ്പോവുന്നു’ എന്നതിനാലാണ് അവക്ക് ഫവാസിക്ക് എന്ന പേര് നൽകപ്പെട്ടത്. (ഹയാത്തുൽ ഹയവാനുൽ കുബ്റാ: 2:546)
ശൈഖ് മുനജ്ജിദ് പറഞ്ഞു: فعلة قتله :الأذى والضرر. “അപ്പോൾ അവയെ കൊല്ലാനുള്ള കാരണം മനുഷ്യന് ഉപദ്രവങ്ങളും ശാരീരിക അപായങ്ങളും വരുത്തുന്നവയാണ് അവ എന്നതാണ്.” (ഇസ്ലാം: സുആൽ വൽജവാബ്: 289055)
“പല്ലിയെ ‘ഫുവൈസിക്ക’ എന്നാണ് വിളിക്കപ്പെട്ടത്. ത്വയ്യിബി പറഞ്ഞു: പല്ലിയെ ഇപ്രകാരം വിളിക്കാൻ കാരണം (ഫവാസിക്) ഉപദ്രവകാരികളായ അഞ്ച് ജീവികളെ പോലെ പല്ലിയിൽ നിന്നും ഉപദ്രവമുണ്ടാകാം എന്നതിനാലാണ്.” (ശർഹു സുനനു ഇബ്നുമാജ: 1: 232)
പല്ലികളെ കൊല്ലാൻ അനുവദിച്ചതിലെ കാരണം ചർച്ച ചെയ്യവെ ഇമാം നവവി പറഞ്ഞു: പല്ലികൾ പല ഉപദ്രവങ്ങളുമുണ്ടാക്കുന്ന ജീവിയാണെന്നതിൽ ഏകാഭിപ്രായമുണ്ട്… അവയെ കൊല്ലുന്നത് പ്രവാചകൻ (സ) പ്രോത്സാഹിപ്പിക്കാൻ കാരണം അവയിലെ ഉപദ്രവങ്ങളാണ്.” (ശർഹു മുസ്ലിം: 14:236)
അവ വെള്ള പാത്രങ്ങളിൽ മനുഷ്യന് ഉപദ്രവകരമായ പലതും നിക്ഷേപിച്ചു കൊണ്ടും ഗുരുതരമായ രോഗങ്ങളും വിഷങ്ങളും പടർത്തിക്കൊണ്ടും ഉപദ്രവങ്ങൾ ഏൽപ്പിക്കുന്നുവെന്ന് ഇമാം ഐനി വ്യക്തമാക്കുന്നു. (ഉംദത്തുൽ കാരി: 15: 250)
അബൂബക്കർ ഇബ്നുൽ അറബി പറഞ്ഞു: മൃഗങ്ങൾ രണ്ടു വിതമുണ്ട്. ഉപദ്രവിക്കുന്നവയും ഉപദ്രവിക്കാത്തവയും. ഉപദ്രവിക്കുന്നവയെ കൊല്ലാം. ഉപദ്രവിക്കാത്തവയെ കൊല്ലരുത്. പല്ലിയെ കൊല്ലാൻ അനുവദിച്ചത് അവ ഉപദ്രവകാരിയാണ് എന്ന അടിസ്ഥാനത്തിലാണ്. (ആരിദത്തുൽ അഹ്വദി: 6:276)
പല്ലികളെ കൊല്ലാനുള്ള ഭൗതീകമായ, മുഖ്യ കാരണം അവയിലെ ഉപദ്രവമാണ് എന്ന് ചുരുക്കം. ഇതാകട്ടെ ശാസ്ത്രത്തിന്റെ വെളിച്ചത്തിൽ ഒരു മണ്ടത്തരമൊ മിഥ്യയൊ ആണെന്ന വ്യാഖ്യാനത്തിനെതിരായാണ് വസ്തുതകൾ സംസാരിക്കുന്നത്.
പല്ലികളുൾപ്പെടെ ഗൗളിവർഗ ജീവികളിൽപ്പെട്ട (Lizard) അയ്യായിരത്തിലധികം വർഗങ്ങൾ ലോകത്തുണ്ട്. ഹദീസിലെ ‘വസഗ് ‘ (الْوَزَغِ ،الوَزَغَة) എന്ന പദം പല്ലി വർഗത്തിൽപ്പെട്ട (Lizard) ആയിരത്തോളം വരുന്ന വിഭാഗങ്ങളെ വിശേഷിപ്പിക്കാവുന്ന പേരാണ്; വീട്ടു പല്ലികളെ സംബന്ധിച്ച് മാത്രമല്ല. (https://mawdoo3.com/%D9%85%D8%A7_%D9%87%D9%88_%D8%AD%D9%8A%D9%88%D8%A7%D9%86_ %D8%A7%D9%84%D9%88%D8%B2%D8%BA) ഗെക്കോ (പല്ലി), പല്ലി വർഗത്തിൽപ്പെട്ട (Lizard) ആയിരത്തിലധികം ഇനം പല്ലികളിൽ ഏതെങ്കിലുമാണെന്ന് എൻസൈക്ലോപീഡിയ ബ്രിട്ടാനിക്ക വ്യക്തമാക്കുന്നു. (https://www.britannica.com/animal/gecko)
സൗദി അറേബ്യയിൽ കുറഞ്ഞത് 100 പല്ലികളും ഗൗളിവർഗ (Lizard) ഇനങ്ങളുണ്ട്, അവയിൽ പലതും പാശ്ചാത്യർക്ക് അജ്ഞാതമാണ്. സൗദി അറേബ്യ, ഉരകങ്ങളുടെ ഒരു അപൂർവ്വ കലവറയാണ്. മറ്റൊരു വാചകത്തിൽ പറഞ്ഞാൽ, പൗരാണിക കാലഘട്ടത്തിൽ, പൂർണമായും നാഗരികമായിട്ടില്ലാത്ത മരുഭൂ പ്രദേശങ്ങളിൽ ജീവിക്കുന്നവരെ സംബന്ധിച്ചിടത്തോളം നൂറു കണക്കിന് പല്ലി വർഗങ്ങൾ ശല്യം കൊണ്ടും ഉപദ്രവങ്ങൾ കൊണ്ടും അവരുടെ ദുസ്വപ്നമായി (Nightmare) അവ മാറിയിട്ടുണ്ടാവണം.
“വടക്കുകിഴക്കൻ ആഫ്രിക്ക മുതൽ തെക്കുപടിഞ്ഞാറൻ ഏഷ്യ വരെയുള്ള അറേബ്യൻ പ്രദേശങ്ങളിൽ ഗെക്കോകളുടേയും മറ്റു പല്ലി വർഗങ്ങളുടേയും സമൃദ്ധി നിലനിന്നിരുന്നു. പ്രോജക്റ്റ് സൈറ്റുകൾ ഇൻഡസ്ട്രിയൽ കോംപ്ലക്സുകൾ എന്നിവയുടെ ഭാഗമായ ശുദ്ധീകരണവും നഗരവികസനവും മൂലമുണ്ടായ പാരിസ്ഥിതിക മാറ്റങ്ങൾ, പല്ലി വർഗങ്ങളുടെ പ്രകൃതിദത്ത ആവാസവ്യവസ്ഥയുടെ ദ്രുതഗതിയിലുള്ള ഇടിവിന് കാരണമായി. എത്രത്തോളമെന്നാൽ തുരൈഫ് പ്രദേശത്തെ പല്ലി വൈവിധ്യത്തിന്റെ -പഠന കാലയളവിലെ- 16 ഇനം പല്ലികളെ രേഖപ്പെടുത്തപ്പെട്ടിരുന്നത്, ഇപ്പോഴത്തെ സർവേയിൽ -ഏറ്റവും സമൃദ്ധമായ കുടുംബം- 9 ഇനങ്ങളുള്ള ലാസെർട്ടിഡേ ആയി ചുരുങ്ങി.” (https://www.ncbi.nlm.nih.gov/pmc/articles/PMC4992096/)
വിവിധ തരം മരുഭൂ പല്ലികൾ, ഓന്തുകൾ, അരണകൾ തുടങ്ങിയ പല്ലിവർഗങ്ങളുടെ (Lizards) ആധിക്യത്തിൽ കേവലം പതിറ്റാണ്ടുകൾ കൊണ്ട് സംഭവിച്ച ഇടിവാണ് ഈ പഠനം സൂചിപ്പിക്കുന്നത്. എങ്കിൽ 14 നൂറ്റാണ്ടുകൾക്ക് മുമ്പ്, നാഗരിക വികസനങ്ങൾ സംഭവിക്കുന്നതിനപ്പുറം പൗരാണിക അറേബ്യയിലെ ജനങ്ങൾ ജീവിച്ചിരുന്ന മരുഭൂവന്യതയിൽ നിന്നു കൊണ്ടാകണം ഉപദ്രവകാരികളായ പല്ലികളെ കൊല്ലാൻ പ്രവാചകൻ (സ) അനുവാദം നൽകിയതിനെ സംബന്ധിച്ച് ചിന്തിക്കാൻ.
“പല്ലിവർഗങ്ങളുമായും പാമ്പുകളുമായുള്ള മനുഷ്യന്റെ ഇടപെടലിന്റെ വർദ്ധനവ് മനുഷ്യ സാൽമൊണെല്ലോസിസ് രോഗത്തിന്റെ വ്യാപനത്തിൽ പ്രാധാന പങ്കു വഹിക്കുന്നതായി പഠനങ്ങൾ സൂചിപ്പിക്കുന്നു, പ്രത്യേകിച്ചും ചെറിയ കുട്ടികളിൽ കണ്ടുവരുന്ന കൂടുതൽ ആക്രമണാത്മക അണുബാധകൾക്ക് കാരണമിതാണ്.” (https://www.ncbi.nlm.nih.gov/pmc/articles/PMC5617995/)
ഊഷര ഭൂമിയിൽ, ഒട്ടകപ്പുറത്ത് ദിവസങ്ങളോളവും മാസങ്ങളോളവും യാത്ര ചെയ്തും, മരച്ചുവട്ടിലും ഓലപ്പുരയിലും കിടന്നുറങ്ങിയുമെല്ലാം ജീവിച്ചിരുന്ന പൗരാണിക അറബിയെ വലച്ചിരുന്ന പ്രശ്നത്തെയാണ് പ്രവാചകൻ (സ) അഭിസംബോധന ചെയ്യുന്നത്. അല്ലാതെ ഇന്ന്, മിനുമിനുത്ത, റബ്ബറൈസ്ഡ് എക്സ്പ്രസ് ഹൈവേകളും, ശീതീകരിച്ച ആഡംഭര കാറുകളും, അംബരചുമ്പികളായ കെട്ടിടങ്ങളുമെല്ലാം ജീവിത ചിത്രങ്ങളായി പരിണമിച്ച പരിഷ്കൃത നഗകവാസികളോട്, കെട്ടിടത്തിന്റെ ഏതോ നിലയിലെ, ഏതോ മുറിയിൽ… ഏതോ മൂലയിൽ ആരുമറിയാതെ പതുങ്ങിയിരിക്കുന്ന പല്ലിയെ തേടിപ്പിടിച്ച് ‘ശിക്ഷിക്കാനു’ള്ള ആഹ്വാനമല്ല അത്. അങ്ങനെ ആ ഹദീസിനെ വ്യഖ്യാനിക്കുമ്പോൾ മാത്രമാണ് തെറ്റിദ്ധാരണകൾ ഉടലെടുക്കുന്നത്.
അറേബ്യൻ ഉപദ്വീപിലെ മരുഭൂമികളിൽ ഏറ്റവും കൂടുതൽ കണ്ടുവരുന്ന ഗൗളിവർഗ ജീവികളിൽ പല്ലികളോട് അടുത്ത ഒന്നാണ്, ഇരുണ്ട പുള്ളികളുള്ള, മണൽ നിറമുള്ള ‘ഡെസേർട്ട് മോണിറ്റർ’. അവ മരുഭൂമിയിലെ അന്തരീക്ഷവുമായി മികച്ച രീതിയിൽ ഇണങ്ങിച്ചേരുന്നു. മരുഭൂമിയിലെ ഏറ്റവും ആക്രമണാത്മക ഉരഗങ്ങളിൽ ഒന്നാണിത്. ഭീഷണി നേരിടുന്നതായി അനുഭവപ്പെട്ടാൽ അവ ‘ശരീരം വായു കൊണ്ട് വീർപ്പിക്കുകയും ഉച്ചത്തിൽ ചീറ്റുകയുകയും’ ചെയ്യും. പ്രതിരോധത്തിനായി വാൽ ഉപയോഗിച്ച് ചാട്ടവാറടി പോലെ വീശിയടിക്കുമെന്നതിന് പുറമെ വേദനാജനകമായ അവയുടെ കടി പലപ്പോഴും രോഗബാധയായി പരിണമിക്കാറുമുണ്ട്. ‘ഡെസേർട്ട് മോണിറ്റർ’ ഒരു സജീവ വേട്ടക്കാരനാണ്, വേട്ടയാടി പിടിക്കാനും കീഴടക്കാനും കഴിയുന്ന എന്തും അവ ഭക്ഷിക്കും; ഇതിൽ പെരുച്ചാഴി, എലി, അണ്ണാൻ, എട്ടുകാലി തുടങ്ങി മറ്റ് ഉരഗങ്ങൾ, ചെറിയ സസ്തനികൾ, പക്ഷികൾ, ചീഞ്ഞുനാറുന്ന ശവം വരെ ഉൾപ്പെടുന്നു. അറേബ്യൻ ഉപദ്വീപിലുടനീളം അതിന്റെ വിഹാര പരിധി വ്യാപിച്ച് കിടക്കുന്നു. (https://www.ddcr.org/FloraFauna/Detail.aspx?Class=Reptiles&Order=Reptiles&Referrer=Monitors& Subclass=Lizard%20Family&Name=Desert%20Monitor&Id=169)
മറ്റു ചില പല്ലികളും അവയുടെ ഉപദ്രവങ്ങളും പഠനങ്ങളിൽ നിന്ന് നമുക്ക് വായിക്കാം:
“ഹവായ് ദ്വീപിലുടനീളം നടത്തപ്പെട്ട ചില പഠനഫലങ്ങൾ സാൽമൊണെല്ല രോഗം ബാധിച്ച പല്ലികൾ ദ്വീപുകളിൽ വ്യാപകമായി വസിച്ചു വരുന്നുവെന്ന് കാണിക്കുന്നു. ഹവായ് ദ്വീപുകളിലെ പല്ലികൾ, പഴയ കെട്ടിടങ്ങളിൽ, ഇരുമ്പ് മേൽക്കൂരയുടെ ആവരണത്തിനും ചുമരിനും ഇടയിലും മതിലുകളിലെ വിള്ളലുകളിലും പ്രധാനമായും താമസിക്കുന്നു. മാത്രമല്ല, ലൈറ്റ് ഫർണിച്ചറുകളേയും ജനലുകളേയും ചുറ്റിപ്പറ്റി അവ ജീവിക്കുന്നു. സർവേയിൽ ഉൾപ്പെടുത്തിയ 13 സൈറ്റുകളിൽ, 76.9 ശതമാനം സാൽമൊണെല്ല ബാധിച്ച 10 പല്ലികളെ കണ്ടെടുക്കപ്പെട്ടു. ഇവിടെയുള്ള 9 വീടുകളിൽ 23ൽ 7 പല്ലികളുടെ കാഷ്ടത്തിൽ (30.4 ശതമാനം) സാൽമൊണെല്ല പോസിറ്റീവ് ആയി കണ്ടെത്തി. ഈ വീടുകളിൽ 63 പല്ലികളിൽ 27 എണ്ണം സാൽമൊണെല്ലക്ക് (42.9 ശതമാനം) പോസിറ്റീവ് ആണ് എന്ന് 1981 ൽ ഹെൽമ് കണ്ടെത്തി. ദ്വീപുകളിൽ പല്ലികളിൽ നിന്ന് മനുഷ്യരിലേക്ക് സാൽമൊണെല്ല പകരുന്നത് ഭക്ഷണവും വെള്ളവും അവയുടെ കാഷ്ടത്തിലൂടെ മലിനീകരിക്കപ്പെടുന്നതിലൂടെയാണ്.”
(Salmonella in Two Gecko Species on the Island of Hawaii: John G. Chan, Charlene Shero, Laura Young, Barney Bareng, Biology Discipline: University of Hawaii at Hilo: Hilo, Hawaii 96720)
മലേഷ്യയിൽ നടത്തിയ ഒരു പഠനത്തിൽ 83.3% വീട്ടിൽ വളർത്താനായി പിടിക്കപ്പെടുന്ന പല്ലികളും (Iguanidae, Agamidae, Scincidae, Gekkonidae, Varanidae) 25% കാട്ടു പല്ലികളും (Agamidae, Scincidae, Gekkonidae) സാൽമൊണെല്ല അണുബാധ വാഹകരാണെന്ന് തെളിയിക്കപ്പെട്ടു.
ജപ്പാനിൽ ഒരു വളർത്തുമൃഗ സ്റ്റോറിൽ നിന്ന് മാത്രം 66% (47/71) പല്ലിവർഗവും (Lizards) 100% (23/23) പാമ്പുകളും സാൽമൊണെല്ലയ്ക്ക് കാരണകാരികളായി കണ്ടെത്തപ്പെട്ടു.
ക്രൊയേഷ്യയിൽ, ഒരു സ്വകാര്യ ഉടമയുടെ അടുക്കലുണ്ടായിരുന്ന 48.4% വീട്ടിൽ വളർത്താനായി പിടിക്കപ്പെട്ട പല്ലിവർഗങ്ങളും, 8.9% പിടിക്കപ്പെട്ട പാമ്പുകളും സാൽമൊണെല്ലയ്ക്ക് പോസിറ്റീവായതായി കണ്ടെത്തി.
പോളണ്ടിലെ മൃഗശാലകളിലും സ്വകാര്യ സൂക്ഷിപ്പുകാരുടെ അടുക്കലുമുണ്ടായിരുന്ന മുപ്പത്തൊമ്പത് ശതമാനം (58/149) പല്ലിവർഗങ്ങളും, 29% (31/106) പാമ്പുകളും സാൽമണെല്ലയ്ക്ക് പോസിറ്റീവ് ആണെന്ന് കണ്ടെത്തി. കാനഡയിൽ, പോസ്റ്റ്മോർട്ടത്തിനായി സമർപ്പിച്ച 51% വളർത്തുമൃഗ പാമ്പുകളും 48% വളർത്തുമൃഗ പല്ലിവർഗങ്ങളും സാൽമൊണെല്ലയ്ക്ക് പോസിറ്റീവ് ആണെന്ന് കണ്ടെത്തി, സാൽമൊണെല്ല പോസിറ്റീവ് ആയ മൃഗങ്ങളിൽ മൂന്നിലൊന്നിന്റെയും മരണത്തിന് കാരണമായത് ‘സാൽമൊനെല്ലോസി’സാണ്. (https://www.ncbi.nlm.nih.gov/pmc/articles/PMC5617995/)
അമേരിക്കയിലെ ഒരു പഠനത്തിൽ കാട്ടിൽ നിന്ന് പിടികൂടി യു.എസ്.എയിലേക്ക് ഇറക്കുമതി ചെയ്ത 80% (88/110) ഇന്തോനേഷ്യൻ ‘ടോക്കെയ് ഗെക്കോസ്’ പല്ലികൾ (ഗെക്കോ ഗെക്കോ) ‘സാൽമൊണെല്ലയ്ക്ക് പോസിറ്റീവാണെന്ന് കണ്ടെത്തി. ഇതിൽ 14 വ്യത്യസ്ത സെറോഗ്രൂപ്പുകളും, 17 പ്രത്യേക സെറോടൈപ്പുകളും ഉൾപ്പെടുന്നു, അവയിൽ പലതും ആൻറിബയോട്ടിക്കുകളെ ചെറുക്കാൻ ശേഷിയുള്ളവയാണ്. (https://pubmed.ncbi.nlm.nih.gov/22607081/)
വിയറ്റ്നാമിലെ മെകോംഗ് ഡെൽറ്റയിൽ നിന്ന് 201 കാട്ടു പല്ലികളെ ശേഖരിച്ചു, അവയുടെ കാഷ്ടത്തിൽ സാൽമൊണെല്ലയുടെ അതിജീവന കാല പരിധിയെ നിർണ്ണയിക്കാൻ നടത്തിയ പഠനത്തിൽ, പരിശോധിച്ച 101 സാമ്പിളുകളിൽ 24 എണ്ണം (23.8%) സാൽമൊണെല്ല പോസിറ്റീവ് ആയിരുന്നു.
വിയറ്റ്നാമിലെ ഊഷ്മാവിൽ, പല്ലി കാഷ്ടത്തിലെ സാൽമൊണെല്ലയ്ക്ക് 6 ആഴ്ച അതിജീവിക്കാൻ കഴിയും. തെക്ക് കിഴക്കൻ ഏഷ്യൻ രാജ്യങ്ങളിലെ മനുഷ്യരിൽ സാൽമൊണെല്ലയുടെ സംഭരണത്തിലും സാൽമൊണെല്ല അണുബാധയുടെ ഉറവിടമായി വർത്തിക്കുന്നതിലും കാട്ടു പല്ലികൾ ഒരു പ്രധാന പങ്ക് വഹിക്കുന്നുവെന്ന് ഫലങ്ങൾ തെളിയിക്കുന്നു. (https://www.jstage.jst.go.jp/article/jvms/80/8/80_18-0233/_article)
‘സാൽമൊണെല്ല ബാക്ടീരിയ പ്രതിവർഷം 19,000 പേരെ ആശുപത്രികളിലേക്കും 380 മരണങ്ങളിലേക്കും നയിച്ചുവെന്ന് അമേരിക്കയിലെ Centers for Disease Control and Prevention റിപ്പോർട്ട് ചെയ്യുന്നു.
പല്ലികളെ വളർത്തുമൃഗങ്ങളായി ഉപയോഗിക്കപ്പെടുന്നത് വ്യാപകമായതോടെ, യുഎസിലെ 16 സംസ്ഥാനങ്ങളിൽ അപകടകരമായ സാൽമൊണെല്ല പൊട്ടിപ്പുറപ്പെടലുകൾ ഉണ്ടായി എന്ന് പുതിയ വാർത്താ റിപ്പോർട്ടുകൾ പുറത്തു വരുന്നു.’ (abcnews.go.com)
പ്രവാചക കാലഘട്ടത്തിൽ മരുഭൂവാസികളായ പൗരാണിക അറബികൾക്ക്, ഈ പല്ലികളിലും ഗൗളിവർഗ ജീവികളിലും (Lizard) ഉപദ്രവകാരികളായ വിഭാഗങ്ങളിൽ നിന്നും നിരന്തരം ഉപദ്രവമേൽക്കുന്നവരായിരുന്നു എന്നത് മുന്നിൽ വെച്ചാണ് ഹദീസിനെ സമീപിക്കേണ്ടത്. ഇത്തരം പല്ലികളെയാണ് കൊല്ലാൻ അനുവാദം നൽകപ്പെട്ടത് എന്നതാണ് പല ഗവേഷകരുടേയും പണ്ഡിതരുടെയും വീക്ഷണം. അറേബ്യ നാഗരീകമായി വികസിച്ചിട്ടില്ലാത്ത അക്കാലഘട്ടത്തിൽ ഇവയുടെ വിഹാര പരിധിയും തോതും ഇന്നത്തേക്കാൾ എത്രയൊ ഇരട്ടി കൂടുതലായിരിക്കും എന്ന് ചിന്തിക്കാവുന്നതെയുള്ളു.
ഈ ഗൗളിവർഗ ജീവികളെ (Lizards) സംബന്ധിച്ചാണ് ഹദീസ് സംസാരിക്കുന്നത് എങ്കിലും -ഉപദ്രവ ഹേതുവാണെങ്കിൽ – വീട്ടു പല്ലി ഉൾപ്പെടെ ഏത് പല്ലിയെയും കൊല്ലലും ഹദീസിലെ അനുവദിക്കപ്പെട്ട വിഭാഗമായി പരിഗണിക്കപ്പെടും എന്ന മറ്റൊരു വീക്ഷണവുമുണ്ട്. രണ്ടിലും തെറ്റൊന്നും കാണുന്നില്ല.
നമ്മെ ചുറ്റിപറ്റി ജീവിക്കുന്ന പല്ലികളിൽ വളരെ സാധാരണയായി കാണപ്പെടുന്ന ഒരു വിഭാഗമാണ് House geckos എന്നറിയപ്പെടുന്ന, വീട്ടു പല്ലികൾ അല്ലെങ്കിൽ ചുമർ പല്ലികൾ. ഹദീസിൽ കൊല്ലാൻ അനുവാദം നൽകപ്പെട്ടത് ഈ പല്ലി വർഗത്തെ സംബന്ധിച്ചു കൂടിയാണ് എന്ന് മനസ്സിലാക്കിയാൽ തന്നെ, മനുഷ്യർക്ക് അറപ്പുളവാക്കുകയും നിരന്തരം ശല്യപ്പെടുത്തുകയും ചെയ്യുന്നവയാണ് അവ എന്നതിലുപരി മനുഷ്യരിൽ ഗുരുതരമായ രോഗങ്ങൾക്ക് കാരണമായ സാൽമൊണെല്ല (Salmonella) എന്ന അണുക്കളുടെ വാഹകർ കൂടിയാണ് അവ എന്നും നാം തിരിച്ചറിയേണ്ടതുണ്ട്. ഉരഗങ്ങളുമായുള്ള സമ്പർക്കം പുലർത്തുന്നതിലൂടെയും സാൽമൊണെല്ല അണുബാധ ഉണ്ടാകാം, അവ സമ്പർക്കം പുലർത്തിയ പാത്രങ്ങൾ, ഭക്ഷണം, വെള്ളം ഉൾപ്പെടെ എന്തിൽ നിന്നും അണുബാധ ഉണ്ടാകാം. (https://www.cdc.gov/healthypets/diseases/salmonella.html) ഉഭയജീവികളുമായോ (ഉദാ. തവളകൾ), ഉരഗങ്ങളുമായോ (ഉദാ. പല്ലികൾ) അല്ലെങ്കിൽ അവയുടെ വിസർജ്ജത്തിൽ നിന്നോ കാഷ്ടത്തിൽ നിന്നോ നേരിട്ടോ അല്ലാതെയോ സമ്പർക്കം പുലർത്തുന്നതിലൂടെ സാൽമൊണെല്ല പടരാം. സാൽമൊണെല്ല ബാക്റ്റീരിയ സാധാരണയായി കുടലിനെയും, ചിലപ്പോഴെല്ലാം രക്തപ്രവാഹത്തെയും ബാധിക്കുന്നു. പല്ലികൾ ഈ ബാക്ടീരിയകളെ കുടൽ, വായ, കാഷ്ടം എന്നിവയിൽ വഹിക്കാറുണ്ട്.
ഈ ബാക്ടീരിയകൾ വയറിളക്കരോഗത്തിനുള്ള ഏറ്റവും സാധാരണമായ കാരണങ്ങളിലൊന്നാണ്. ന്യൂയോർക്ക് സ്റ്റേറ്റിൽ ഓരോ വർഷവും ഇക്കാരണത്താലുണ്ടാകുന്ന ആയിരക്കണക്കിന് കേസുകൾ റിപ്പോർട്ട് ചെയ്യപ്പെടുന്നു. മിക്ക കേസുകളും വേനൽക്കാലത്ത് സംഭവിക്കുന്നു. ചിലരിൽ ജീവനു ഭീഷണിയാവുന്ന അപകടങ്ങളും സൃഷ്ടിച്ചേക്കും. (https://www.health.ny.gov/diseases/communicable/zoonoses/salmonella/amphibian_reptilian_questions_and_answers.htm)
സാൽമൊണെല്ലയെ ഒരു ഭക്ഷ്യ രോഗകാരണമായ അണുവായാണ് കണക്കാക്കപ്പെടുന്നത്. അവ മൂലം മലിനമായ ഭക്ഷണത്തിലൂടെ -ലോകത്ത്- പ്രതിവർഷം 80 ദശലക്ഷം സാൽമൊനെലോസിസ് കേസുകൾ റിപ്പോർട്ട് ചെയ്യപ്പെടുന്നു. യു.എസ്.എയിൽ 6% സ്പോറാഡിക് സാൽമൊനെലോസിസ് കേസുകളും, 21 വയസ്സിന് താഴെയുള്ളവരിൽ 11% കേസുകളും ഉരഗങ്ങളും ഉഭയജീവികളുമായുള്ള സമ്പർക്കം മൂലമാണെന്ന് കണക്കാക്കപ്പെടുന്നു.
നൈജീരിയയിലെ സുക്കയിൽ, വീട്ടു പല്ലികളിലുള്ള സാൽമൊണെല്ല അണുബാധയെ സംബന്ധിച്ച അന്വേഷണ പഠനത്തിൽ തൊന്നൂറിൽ ഇരുപത് പല്ലികളിൽ സാൽമൊണെല്ല സാന്നിദ്ധ്യം കണ്ടെത്തി; 30 ശതമാനം വാഹക നിരക്കിൽ. (https://pubmed.ncbi.nlm.nih.gov/3833829 /)
150 ചുമർ പല്ലികളുടെ (Hemidactylus brookei) കുടലിലെ എയറോബിക് ബാക്ടീരിയ വ്യൂഹത്തെ കുറിച്ചു പഠനം നടത്തപ്പെട്ടപ്പോൾ സാൽമൊണെല്ലയുടെ 35 ഇൻസുലേറ്റുകളും എന്ററോബാക്ടീരിയേസിയിലെ (Enterobacteriaceae) മറ്റ് പല ഇനങ്ങളും ഉൾപ്പെടെ വിവിധതരം ബാക്ടീരിയകൾ കണ്ടെടുത്തു. ഷിഗെല്ല സോനെയി – 2, എഡ്വേർഡീസെല്ല ടാർഡ – 4, എന്റർടോബാക്റ്റർ എസ്പിപി – 8, സിട്രോബാക്റ്റർ ഫ്രോയിഡി – 3, സെറാട്ടിയ മാർസെസെൻസ് – 3, പ്രോട്ടിയസ് എസ്പിപി – 35, ക്ലെബ്സില്ല ന്യൂമോണിയ – 13, എസ്ഷെറിച്ച കോളി – 17, ഇൻസുലേറ്റുകൾ. എട്ട് സാൽമൊണല്ല സെറോടൈപ്പുകൾ എന്നിവ തിരിച്ചറിഞ്ഞു, അവയിൽ പ്രധാനം എസ്. വിറ്റിംഗ്ഫോസ് (S. hvittingfoss), എസ്. ടൈഫിമുറിയം (S.typhimurium) എന്നിവയാണ്. മനുഷ്യ ശരീരത്തിലെ എന്ററോപാഥോജനുകളുടെ വ്യാപനവുമായി ഈ കണ്ടെത്തലുകൾക്കുള്ള ബന്ധം വളരെ പ്രസക്തമാണ്. (https://pubmed.ncbi.nlm.nih.gov/3729372/)
“എല്ലാ ഉരഗങ്ങളിലും ബാക്ടീരിയ, വൈറസ്, പരാന്നഭോജികൾ, പുഴുക്കൾ എന്നിവയുൾപ്പെടെ നിരവധി അണുക്കൾ ഉണ്ട്. ഇവയിൽ പലതും ഉരഗ ഉടമകളുടെ കുടുംബത്തിലേക്ക് പകരാം. ഇവയിൽ ഏറ്റവും പ്രധാനപ്പെട്ടവ ഇനി പറയുന്നവയാണ്:
സാൽമൊണെല്ല: സാൽമൊണെല്ല സാധാരണയായി എല്ലാത്തരം ഉരഗങ്ങളിലും കാണപ്പെടുന്നു. ഉരഗങ്ങളുടെ കാഷ്ടവുമായി സമ്പർക്കത്തിൽ വന്ന എന്തെങ്കിലും വായിൽ വെക്കുമ്പോൾ ഉരഗങ്ങളിൽ നിന്ന് മനുഷ്യരിലേക്ക് ഇത് വ്യാപിച്ചേക്കാം. ഉദാഹരണത്തിന് ഉരഗങ്ങൾ/ ഉരഗജീവികളുമായുള്ള സമ്പർക്കം വഴി മലിനമായ ഫോർമുല കുപ്പികൾ കുടിക്കുന്നതിലൂടെ ശിശുക്കൾക്ക് സാൽമൊണെല്ല ബാധിക്കാം. സാൽമൊണല്ല അണുബാധ വയറിളക്കം, തലവേദന, പനി, വയറു വേദന എന്നിവയ്ക്ക് കാരണമാവുകയും സെപ്റ്റിസീമിയ (രക്തത്തിലെ വിഷബാധ) ഉണ്ടാവുകയും ചെയ്യാം. കഠിനമായ നിർജ്ജലീകരണവും സംഭവിക്കാം. 2008 ൽ 449 സാൽമൊനെലോസിസ് കേസുകൾ ഉണ്ടായിരുന്നു, ഇതിൽ പതിനഞ്ച് കേസുകൾ ഉരഗങ്ങളുമായി അടുത്തിടെ സമ്പർക്കമുണ്ടായ ആളുകളായിരുന്നു. ഈ പതിനഞ്ച് കേസുകളിൽ ഒമ്പത് പേർ ഒരു വയസ്സിന് താഴെയുള്ള കുഞ്ഞുങ്ങൾ ആയിരുന്നു.
ബോട്ടുലിസം: പക്ഷാഘാതത്തിനും മരണത്തിനും കാരണമാകുന്ന ക്ലോസ്ട്രിഡിയം ബാക്ടീരിയം പുറത്തുവിടുന്ന വിഷവസ്തു മൂലമുണ്ടാകുന്ന ഗുരുതരവും ജീവന് ഭീഷണിയുമായ രോഗമാണ് ബോട്ടുലിസം.
ക്യാംപിലോബാക്ടീരിയോസിസ് (മലവിസർജ്ജനം), ലെപ്റ്റോസ്പിറോസിസ് (കരൾ രോഗം), ട്രിച്ചിനെല്ലോസിസ് (നാഡീവ്യവസ്ഥ, ഹൃദയം, ശ്വാസകോശം, പേശികൾ എന്നിവയെ ബാധിക്കുന്ന രോഗം) എന്നിവ ഉരഗങ്ങളെ വളർത്തുന്നതുമായി ബന്ധപ്പെട്ടിരിക്കുന്നു. മിക്കതും ചികിത്സിക്കാവുന്നവയാണെങ്കിലും ചിലത് വളരെ ഗുരുതരമാണ്.” (https://www.hpsc.ie/a-z/zoonotic/reptilesandrisksofinfectiousdiseases/)
57 വീടുകളിൽ നിന്ന്, നൂറ് ‘ഏഷ്യൻ ഹൗസ് ഗെക്കോ’ പല്ലികളെ ശേഖരിച്ച് നടത്തിയ പഠനങ്ങളിൽ സാൽമൊണല്ലയുടെ മൂന്ന് സെറോടൈപ്പുകൾ കണ്ടെത്തി. അതിൽ ഒന്നായ ‘സാൽമൊണെല്ല വിർചോവ്’ (ഫേജ് തരം 8) ആക്രമണാത്മക രോഗവുമായി ബന്ധപ്പെട്ടിരിക്കുന്നതിനാൽ സാൽമൊനെലോസിസ് എന്ന പകർച്ചവ്യാധിയിൽ ഏഷ്യൻ ഹൗസ് ഗെക്കോയ്ക്ക് ഒരു പ്രധാന പങ്കുണ്ടെന്ന് കണ്ടെത്തി. (https://pubmed.ncbi.nlm.nih.gov/20973656/)
നമ്മെ ചുറ്റിപറ്റി ജീവിക്കുന്ന മറ്റൊരു പല്ലി വർഗമായ, iguanas പല്ലികൾ വീടിനകത്ത് എത്തുകയൊ, മുറ്റത്ത് കറങ്ങി നടക്കുകയൊ ചെയ്യാറുണ്ട്. അവയുടെ വേദനയേറിയ ദംശനം, മാംസം കീറുകയും, അവയുടെ പല്ലുകൾ ത്വക്കിൽ തറച്ചിരിക്കുകയും ചെയ്യും.
ഇവയിലൂടെ മനുഷ്യരിലേക്ക് എത്തുന്ന സാൽമൊനെലോസിസ്, അസുഖകരമായ ഇൻഫ്ലുവൻസക്ക് പുറമെ, ജീവനു ഭീഷണിയാവുന്ന അപകടങ്ങളും സൃഷ്ടിച്ചേക്കും. (https://www.crittercontrol.com/wildlife/lizard/lizards-in-the-house)
iguanas പല്ലികൾ പാമ്പുകളേയും അപകടകരമായ വേട്ട ജീവികളേയും വിഷജന്തുക്കളേയും വീട്ടിലേക്കും മുറ്റത്തേക്കും ആകർഷിച്ചേക്കാം. (https://www.crittercontrol.com/wildlife/lizard/lizards-in-the-house)
വേലി പല്ലികൾ, ഗെക്കോകൾ, ഏങ്കിൾസുകൾ എന്നിവ വൃത്തികെട്ട കാഷ്ടങ്ങൾ കൊണ്ട് പരിസരങ്ങളും വസ്ത്രങ്ങളും ഭക്ഷണപദാർത്ഥങ്ങളും മലിനമാക്കുകയും, പൊടുന്നനെയുള്ള ചലനങ്ങളിലൂടെ ആളുകളെ സംഭ്രമത്തിലാക്കുകയും ഭീതിപ്പെടുത്തുകയും ചെയ്യുന്നു.
വലിയ ഇഗുവാന പല്ലികൾക്ക് കടിക്കാനും മാന്താനും വാലുകൾ കൊണ്ട് അടിക്കാനും കഴിയും. അവ പുൽത്തകിടികൾ മാന്തികുഴിക്കുകയും നടപ്പാതകൾ തകർക്കുകയും ഫലങ്ങൾ കഴിക്കുകയും, പൂന്തോട്ടങ്ങളിൽ നാശങ്ങളുണ്ടാക്കുകയും ചെയ്യുന്നു.
പാശ്ചാത്യ വേലി പല്ലികൾ, ഗെക്കോകൾ, തവിട്ട്/പച്ച അനോളുകൾ തുടങ്ങിയ പല്ലി വർഗങ്ങൾ ജനലുകൾ, വാതിലുകൾ, തറകൾ എന്നിവയ്ക്ക് ചുറ്റുമുള്ള വിള്ളലുകളിലൂടെ സഞ്ചരിക്കുന്നതിൽ വിദഗ്ധരാണ്. (https://www.crittercontrol.com/wildlife/lizard/lizards-in-the-house)
ഇവക്കു പുറമെ ദോഷകാരികളും അപകടകാരികളുമായ അനേകം ഇനം പല്ലികൾ വേറെയുമുണ്ട്. ഉദാഹരണത്തിന്, ടോക്കെ ഗെക്കോ (Tokay gecko) പല്ലി വർഗം കടിക്കുന്നവയാണ്. ടോക്കെ പല്ലികൾ വളരെ ആക്രമണാത്മക സ്വഭാവമുള്ള പല്ലിയാണിത്. ഭീഷണിപ്പെടുത്തുമ്പോഴോ, ഭയപ്പെടുമ്പോഴോ മാത്രമല്ല അവ കോപിക്കുമ്പോഴും സമ്മർദ്ദം അനുഭവിക്കുമ്പോഴും കടിക്കും. പൂർണ്ണമായി വളർന്ന ടോക്കെയ് പല്ലിക്ക് ശക്തമായ താടിയെല്ലുണ്ട്, ഇത് ചർമ്മത്തിൽ മുറുകെ പിടിക്കുന്നു. അവയെ വെള്ളത്തിൽ മുക്കിയാലല്ലാതെ അവ കടി വിടുകയില്ല. ചർമ്മത്തിൽ നിന്ന് അവയെ വലിച്ചെടുക്കാൻ ശ്രമിക്കുന്നതോടെ അവ കടി മുറുക്കുകയെ ഉള്ളു. ടോക്കെ പല്ലികൾ രോഗകാരികളായ സൂക്ഷ്മ ജീവികളുടേയും വൈവിധ്യമാർന്ന അണുക്കളുടേയും വാഹകരാവാം. ഇവയിൽ ഭൂരിഭാഗവും മനുഷ്യർക്ക് അപകടകരമല്ല എങ്കിലും അവയിൽ ചിലത് ദോഷകരമായ ബാക്ടീരിയകളാണ്. കൂടാതെ ടോക്കെയുടെ തുളച്ചുകയറുന്ന ഒരു കടിയിലൂടെ ദോഷകരമായ പ്രോട്ടോസോവകളും കടന്നുപോയേക്കാം. ഇത്തരം അണുബാധകൾ കുട്ടികളെ എളുപ്പത്തിൽ അപകടത്തിലേക്ക് എത്തിച്ചേക്കും. (https://tokaygeckoguide.com/why-you-dont-want-to-get-bitten-by-a-tokay-gecko/1603/)
ഇബ്റാഹിം നബിയെ ശത്രുക്കൾ അഗ്നിയിലേക്ക് എറിഞ്ഞപ്പോൾ പല്ലികൾ തീ ഊതി ആളി കത്തിക്കാൻ ശ്രമിച്ചതിനാലാണ് തലമുറകൾക്കിപ്പുറവും പല്ലികളെ മുസ്ലിംകൾ കൊന്നു കൊണ്ടിരിക്കുന്നത് എന്ന വിമർശനത്തിന് യാതൊരു യാഥാർത്ഥ്യവുമില്ല. ഒരാൾ ചെയ്ത തെറ്റിന് അയാളുടെ സന്ധതികളിൽ കുറ്റമാരോപിക്കുന്ന മൗഢ്യതയെ നിശിതമായി വിമർശിച്ച മതമാണ് ഇസ്ലാം.
മുഹമ്മദ് നബി (സ) പറഞ്ഞു:
لا تَجْني نفسٌ على الأخرى
“ഒരാളുടെ കുറ്റം മറ്റൊരാളുടെ മേൽ ചുമത്തപ്പെടില്ല.” (സുനനു നസാഈ: 4833, ത്വബ്റാനി: 1384, മഅ്രിഫത്തു സ്വഹാബ: അബൂ നുഐം: 1391)
لا تَجْني أمٌّ على ولَدٍ
“മാതാവിന്റെ കുറ്റം സന്താനത്തിനു മേൽ ചുമത്തപ്പെടില്ല.” (സുനനു നസാഈ: 2/ 251, സുനനു ഇബ്നുമാജ: 2/ 147, സുനനു ഇബ്നു ഹിബ്ബാൻ: 1683)
ഇബ്റാഹിം നബിയുമായി ബന്ധപ്പെടുത്തി കൊണ്ട് ബുഖാരിയിൽ വന്ന ഹദീസ് ഇപ്രകാരമാണ്: “പല്ലികളെ കൊല്ലാൻ (അനുവാദം നൽകി കൊണ്ട്) പ്രവാചകൻ (സ) കൽപ്പന പുറപ്പെടുവിച്ചു. അദ്ദേഹം (സ) പറഞ്ഞു: അത് ഇബ്റാഹിം നബിയുടെ (അ) മേൽ തീ ഊതാൻ ശ്രമിച്ചിരുന്നു.” (സ്വഹീഹുൽ ബുഖാരി: 2628)
ഈ ഹദീസുമായി ബന്ധപ്പെട്ട വളരെ പ്രധാനപ്പെട്ട ചില കാര്യങ്ങൾ മനസ്സിലാക്കുന്നതോടെ ഹദീസ് സംബന്ധമായ തെറ്റിദ്ധാരണകൾ ഇല്ലാതാവുന്നതാണ്:
* പല്ലികളെ കൊല്ലാൻ അനുവദിച്ചതിലെ മുഖ്യ കാരണം അവയിലെ ഉപദ്രവങ്ങൾ തന്നെയാണ്. ആ മുഖ്യ കാരണം ധാരാളം ഹദീസുകളിലൂടെ പ്രവാചകൻ (സ) പഠിപ്പിച്ചു കഴിഞ്ഞു. അതിനു പുറമെ ഒരു അധിക കാരണം കൂടി പങ്കു വെക്കുകയാണ് ഈ ഹദീസിലൂടെ അദ്ദേഹം ചെയ്യുന്നത്.
* വീട്ടിലെ പല്ലികളെയല്ല ഹദീസിൽ തീയിലേക്ക് ഊതാൻ ശ്രമിച്ചു എന്ന് പറഞ്ഞത്. കാരണം, ഇബ്റാഹിം നബിയെ(അ) തീക്കുണ്ടാരമുണ്ടായി എറിഞ്ഞത് വീട്ടിനുള്ളിൽ അല്ലല്ലൊ. മരുഭൂമിലെ വിശാല മൈതാനത്താണ്.
* പല്ലികൾക്കും മറ്റു ഉരഗങ്ങൾക്കും – മനുഷ്യരെ പോലെ തന്നെ – ശ്വാസകോശമുണ്ട്. മനുഷ്യന്റെ ശ്വാസകോശം പോലെ തന്നെ അവയുടെ ശ്വാസകോശം വാരിയെല്ലുകൾക്കും വയറിലെ പേശികൾക്കുമിടയിലായാണ് സ്ഥിതി ചെയ്യുന്നത്. അവയും ഓക്സിജൻ ഉള്ളിലേക്ക് എടുക്കുകയും കാർബൺ ഡൈ ഓക്സൈഡ് പുറത്തുവിടുകയുമാണ് ചെയ്യുന്നത്. അതുകൊണ്ട് തന്നെ ഊതുക എന്നത് അവക്ക് ശാരീരികമായി അസാധ്യമായ ഒരു കാര്യമൊന്നുമല്ല.
‘ആക്രമണാത്മക ഉരഗങ്ങളിൽ ഒന്നായ ‘ഡെസേർട്ട് മോണിറ്റർ’ ഭീഷണി നേരിടുമ്പോൾ ‘ശരീരം, വായു കൊണ്ട് വീർപ്പിക്കുകയും ഉച്ചത്തിൽ ഊതുകയുകയും’ ചെയ്യും. മെഡിറ്ററേനിയൻ വീട്ടു പല്ലികൾ വഴക്കിനിടയിലും ഇണകളെ ആകർഷിക്കാനും ‘ചിലക്കുക’ പതിവാണ്. മധ്യ, തെക്കേ അമേരിക്കയിലെ ടേണിപ്പ്-ടെയിൽഡ് പല്ലികൾ തങ്ങളുടെ വിഹാര പരിധി അടയാളപ്പെടുത്തുന്നതിനായി പ്രാണികളെ അനുകരിക്കുന്ന ശബ്ദത്തിൽ ശബ്ദം പുറപ്പെടുവിക്കുന്നു. 14 ഇഞ്ച് (36 സെന്റീമീറ്റർ) ഉയരമുള്ള ഗെക്കോയായ ന്യൂ കാലിഡോണിയൻ പല്ലി ഉച്ചത്തിൽ അലറുന്നത് കാരണം, അത് “മരങ്ങളിലെ രാക്ഷസൻ” എന്ന പ്രാദേശിക വിളിപ്പേര് നേടി. ഏഷ്യയിൽ കാണപ്പെടുന്ന പുരുഷ ടോക്കെയ് പല്ലികൾ, ഇണചേരാൻ സൂചിപ്പിച്ചു കൊണ്ട് “ടോകേ-ടോക്കേ!” എന്ന് അത്യുച്ചത്തിൽ ശബ്ദിക്കുന്നു. ‘വായു ശ്വാസകോശങ്ങളിൽ നിന്ന് ഗ്ലോട്ടിസിലൂടെ പുറന്തള്ളുന്നതിലൂടെ’യാണ് പല്ലികൾ ഈ ശബ്ദങ്ങൾ പുറപ്പെടുവിക്കുന്നത്.’ (https://www.nationalgeographic.com/animals/article/151024-animal-behavior-lizards-reptiles-geckos-science-anatomy )
ഗൗളിവർഗത്തിന് വായു ഊതാനും വായു പുറം തള്ളി ചീറ്റാനും ചീറാനും അലറാനുമൊക്കെ കഴിയുമെന്ന് ചുരുക്കം.
* പല്ലികൾ തീ ഊതുന്ന ഡ്രാഗണാണോ എന്ന് പരിഹസിക്കുന്നവരുണ്ട്. പല്ലികൾ തീ ഊതി എന്ന് ഹദീസിൽ വന്നിട്ടില്ല എന്നതാണ് അവർക്കുള്ള മറുപടി. പല്ലി തീയിലേക്ക് ഊതിയാൽ എന്ത് സംഭവിക്കാനാണ്? അത് എങ്ങനെ ആളിക്കത്താനാണ്? എന്ന് ചോദിക്കുന്നവരോട് പറയാനുള്ളത് രണ്ട് കാര്യങ്ങളാണ്. ഒന്ന്, ശക്തമായി ഊതാൻ കഴിയുന്ന പല്ലികളുടെ ചില സവിശേഷതകളെ സംബന്ധിച്ച് നാം വിവരിച്ചു കഴിഞ്ഞു. രണ്ട്, പല്ലിയുടെ ഊത്തിന് വല്ല സ്വാധീനവും ആ തീയിൽ വരുത്താൻ കഴിഞ്ഞുവെന്ന് ഹദീസിൽ ഇല്ല. മറിച്ച്, ഹദീസ് സംസാരിക്കുന്നത് തീ ഊതി ആളിക്കത്തിക്കാൻ ആ ജീവികൾ ആശിക്കുകയും ശ്രമിക്കുകയും ചെയ്തുവെന്ന മനസ്ഥിതിയുടെ ജീർണതയെ മാത്രമാണ്.
ശാഹ് വലിയുല്ലാഹ് ദഹ്ലവി പറഞ്ഞു: “‘അവയുടെ ഊത്ത് തീയിൽ യാതൊരു സ്വാധീനവും സൃഷ്ടിക്കില്ലാ എന്നിരുന്നിട്ടും’ പിശാചിന്റെ പ്രേരണയോട് പ്രകൃത്യാ ഉള്ളതായ അവയുടെ അടുപ്പം കാരണം അവ ഇബ്റാഹിമിന്റെ തീ ഊതാൻ ശ്രമിച്ചു. പല്ലികളുടെ ഈ പ്രകൃതത്തിലെ പൈശാചികത പ്രവാചകൻ (സ) ദിവ്യബോധത്തിലൂടെ അറിഞ്ഞു. അതിനെ പറ്റി ഉണർത്തുകയും ചെയ്തു.” (ഹുജ്ജത്തുല്ലാഹിൽ ബാലിഗ: 2:282)
* പല്ലികളെ കൊല്ലാൻ അനുവദിച്ചതിലെ ‘ഭൗതീകമായ’ മുഖ്യ കാരണം പങ്കു വെച്ചതിന് ശേഷം ‘അഭൗതീകമായ’ ഒരു അറിവു കൂടി അനുബന്ധമായി പഠിപ്പിക്കുക മാത്രമാണ് ഈ ഹദീസിലൂടെ അദ്ദേഹം ചെയ്യുന്നത്. മറ്റു ജീവജാലങ്ങളിൽ നിന്ന് വ്യത്യസ്ഥമായി പല്ലികളുടെ പ്രകൃതത്തിലുള്ള മാനസികവും സ്വഭാവപരവുമായ വ്യതിയാനവും നീചതയുമാണ് ആ ‘അഭൗതീക’ജ്ഞാനം. ആ ജ്ഞാനം പല്ലികളെ സൃഷ്ടിച്ച, പല്ലികളുടെ ജൈവ പ്രകൃതിയും മനോ വിഹാരങ്ങളും രഹസ്യങ്ങളുമെല്ലാം അറിയുന്ന സ്രഷ്ടാവിന് മാത്രം ലഭ്യമാകുന്ന അറിവാണ്.
“ആകാശങ്ങളിലും ഭൂമിയിലുമുള്ളവരും, ചിറക് നിവര്ത്തിപ്പിടിച്ചു കൊണ്ട് പക്ഷികളും അല്ലാഹുവിന്റെ മഹത്വം പ്രകീര്ത്തിച്ചു കൊണ്ടിരിക്കുന്നു എന്ന് നീ കണ്ടില്ലേ? ഓരോരുത്തര്ക്കും തന്റെ പ്രാര്ത്ഥനയും കീര്ത്തനവും എങ്ങനെയെന്ന് അറിവുണ്ട്. അവര് പ്രവര്ത്തിക്കുന്നതിനെപ്പറ്റി അല്ലാഹു അറിയുന്നവനത്രെ.” (കുർആൻ: 24:41)
“കണ്ണുകളുടെ കള്ളനോട്ടവും, ഹൃദയങ്ങള് മറച്ച് വെക്കുന്നതും അവന് (അല്ലാഹു) അറിയുന്നു.” (കുർആൻ: 40:19)
“അവന്റെ പക്കലാകുന്നു അദൃശ്യകാര്യത്തിന്റെ ഖജനാവുകള്. അവനല്ലാതെ അവ അറിയുകയില്ല. കരയിലും കടലിലുമുള്ളത് അവന് അറിയുന്നു. അവനറിയാതെ ഒരു ഇല പോലും വീഴുന്നില്ല. ഭൂമിയിലെ ഇരുട്ടുകള്ക്കുള്ളിലിരിക്കുന്ന ഒരു ധാന്യമണിയാകട്ടെ, പച്ചയോ, ഉണങ്ങിയതോ ആയ ഏതൊരു വസ്തുവാകട്ടെ, വ്യക്തമായ ഒരു രേഖയില് എഴുതപ്പെട്ടതായിട്ടല്ലാതെ ഉണ്ടാവില്ല.” (കുർആൻ: 6:59)
പല്ലികളുടെ ജൈവ പ്രകൃതിയെയും മനോ വിഹാരങ്ങളെയും സംബന്ധിച്ച അവയുടെ സ്രഷ്ടാവിന്റെ ഈ ‘അഭൗതീക’ജ്ഞാനം യാതാർഥ്യമല്ലെന്ന് വിമർശകർക്ക് എന്തിന്റെ അടിസ്ഥാനത്തിലാണ് വാദിക്കാനാവുക?! ആ ആദൃശ്യവും അഭൗതീകവുമായ ഒരു വിവരം തെറ്റാണെന്ന് ഭൗതീകമായ ഏത് ശാസ്ത്രം കൊണ്ടാണ് വിമർശകർ തെളിയിക്കുക ?!!
* ഏതൊ ഒരു പല്ലി, ഇബ്റാഹിം നബിയെ(അ) ശത്രുക്കൾ തീക്കുണ്ടാരത്തിൽ എറിഞ്ഞപ്പോൾ തീ ആളിക്കത്തിക്കാൻ ശ്രമിച്ചു അതു കാരണം എല്ലാ തലമുറയിലുള്ള പല്ലികളെയും കൊല്ലണം എന്നല്ല ഹദീസിൽ ഉള്ളത്. ഒരു പല്ലിയെ പറ്റിയുള്ള നിരൂപണമല്ല ഈ ഹദീസ്. പല്ലി വർഗത്തെ സംബന്ധിച്ചാണ് ഹദീസ്. ഒരാളെ പോലും കൊന്നിട്ടില്ലാത്ത ഒരു നിരപരാധിയായ മനുഷ്യനെ ആദർശത്തിന്റെ പേരിൽ മാത്രം തീക്കുണ്ടാരത്തിലേക്ക് എറിഞ്ഞപ്പോൾ ജീവജാലങ്ങളിൽ പലതും ആ തീ കെടാൻ വേണ്ടി ആശിച്ചു. എന്നാൽ പല്ലി വർഗം (ഒരു പല്ലിയല്ല) അത് ആളിക്കത്തിക്കാൻ ആശിക്കുകയും അതിന് വേണ്ടി പരിശ്രമിക്കുകയും ചെയ്തു എന്ന ഒരു സംഭവത്തെ ആ ജീവവർഗത്തിന്റെ പ്രകൃതിപരവും മാനസികവുമായ നീചതക്ക് തെളിവായി അവയെ പടച്ച സ്രഷ്ടാവ് പഠിപ്പിച്ചു. ഇബ്റാഹിം നബിയുമായി ബന്ധപ്പെട്ട ഈ സംഭവം, പല്ലികളുടെ പ്രകൃതവുമായി ബന്ധപ്പെട്ട് അവയുടെ സ്രഷ്ടാവായ അല്ലാഹു അറിയുന്ന ഒരു വസ്തുതക്കുള്ള ഒരു ഉദാഹരണം മാത്രമാണ്. അല്ലാതെ മൂല കാരണമല്ല.
അപ്പോൾ പിന്നെ ഈ നീച വർഗത്തെ എന്തിന് സൃഷ്ടിച്ചു എന്നതാണ് മറ്റൊരു ചോദ്യം. പിശാചിനെ എന്തിന് സൃഷ്ടിച്ചു ? എന്ന് ചോദിക്കും പോലെ നിരർത്ഥകമാണ് ഈ ചോദ്യം. അല്ലാഹുവിന്റെ സൃഷ്ടികളിൽ നീച സൃഷ്ടികൾക്കും സ്ഥാനവും പ്രസക്തിയുമുണ്ട്. കൃത്യമായ യുക്തിയും തേട്ടവുമുണ്ട്. ഉദാഹരണത്തിന്, പല്ലിയെ കൊണ്ടുള്ള ഭൗതീകമായ ചില ഉപകാരങ്ങളും മാറ്റി വെച്ചാൽ തന്നെ, പല്ലിയുടെ ഈ പ്രകൃതത്തെ സംബന്ധിച്ച ഹദീസ് ആരെല്ലാം വിശ്വസിച്ച് സത്യവിശ്വാസിയാവും ആരെല്ലാം പരിഹസിച്ച് തള്ളി സത്യനിഷേധിയാവും എന്ന പരീക്ഷണം തന്നെ പല്ലിയുടെ സൃഷ്ടിപ്പിനു പിന്നിലെ യുക്തികളിൽ ഒന്നാണ്.
“തീര്ച്ചയായും അതിനെ നാം അക്രമകാരികള്ക്ക് ഒരു പരീക്ഷണമാക്കിയിരിക്കുന്നു.” (കുർആൻ: 37:63)
മുല്ലാ അലിയുൽകാരി പറഞ്ഞു: “പല്ലി ഉപദ്രവങ്ങൾ ചെയ്യുന്ന ഒരു ചെറു ജീവിയാണ്… പല്ലി ഇബ്റാഹിമിന്റെ മേൽ തീ ഊതാൻ ശ്രമിച്ചിരുന്നു എന്ന് പറഞ്ഞതിന്റെ ഉദ്ദേശം ഇബ്റാഹീമിന്റെ ശരീരത്തിന് താഴെ ശത്രുക്കൾ കത്തിച്ച തീയിൽ അത് ഊതി എന്നാണ്.
കാദി പറഞ്ഞു: ഇത് പ്രവാചകൻ (സ) പറയാൻ കാരണം പല്ലി വർഗത്തിന്റെ (സ്വഭാവപരമായ) നികൃഷ്ടത കൂടി വ്യക്തമാക്കാനാണ്. ഇബ്റാഹിം നബിയെ (അ) ശത്രുക്കൾ തീക്കുണ്ടാരത്തിൽ എറിഞ്ഞ സമയത്ത് പിശാച് പല്ലികളുടെ (പ്രകൃതത്തിലെ) നീചത കാരണം, ആ തീ ആളിക്കത്തിക്കാൻ (പലതിനേയും ഉപയോഗപ്പെടുത്തുന്ന കൂട്ടത്തിൽ) അവയെയും ഉപയോഗപ്പെടുത്താൻ ശ്രമിച്ചു. (ഈ മാനസികമായ നീച പ്രകൃതിക്ക് പുറമെ) അവ ശാരീരികമായും ഉപദ്രവകാരികളാണ്.
ഇബ്നുൽ മലക്ക് പറഞ്ഞു: അവയുടെ ഉപദ്രവത്തിൽ പെട്ടതാണ് അവ ഭക്ഷണങ്ങൾ കേടു വരുത്തുകയും പലയിടത്തും കാഷ്ടിച്ചിട്ട് വൃത്തികേടാക്കുകയും ചെയ്യുക എന്നത്. പ്രകൃത്യാ അവ ഉപദ്രവകാരികളാണ് എന്നർത്ഥം.” (മിർക്കാത്തുൽ മഫാത്തീഫ്: 7:2671)
ശൈഖ് മുനജ്ജിദ് പറഞ്ഞു: “ഇബ്റാഹിം നബിയെ(അ) ശത്രുക്കൾ തീക്കുണ്ടാരത്തിൽ എറിഞ്ഞ സമയത്ത് പിശാച് പല്ലി തീയിൽ ഊതാൻ ശ്രമിച്ചു എന്ന് പ്രവാചകൻ (സ) പറയാൻ കാരണം പല്ലി വർഗത്തിന്റെ (ആത്മീയമായവും മാനസികവുമായ) നീചതയെയും നികൃഷ്ടതയെയും അറിയിക്കാൻ വേണ്ടിയാണ്. എന്നാൽ അവയെ കൊല്ലാൻ അനുവാദം നൽകിയത് അക്കാരണത്താൽ മാത്രമല്ല. (അവയിലെ ഉപദ്രവങ്ങൾ കാരണമാണ്.)” (ഇസ്ലാം: സുആൽ വൽജവാബ്: 289055)
ഇനി, പല്ലികളെ കൊല്ലുന്നതിന്റെ മികവിനനുസരിച്ച് ഇനാം പ്രഖ്യാപിക്കുക വഴി ആ ജീവിയോട് ക്രൂരതയല്ലെ ചെയ്യുന്നത് എന്നാണ് മറ്റൊരു വിമർശനം. പല്ലിയെ അടിക്കുന്നതിന്റെ പ്രതിഫലത്തെ സംബന്ധിച്ച ഹദീസ് ഇപ്രകാരമാണ്:
“പല്ലിയെ ആരെങ്കിലും ഒരു അടിക്ക് കൊന്നാൽ അവന് നൂറ് നന്മ രേഖപ്പെടുത്തപ്പെടും. രണ്ടാമത്തെ അടിയിൽ കൊല്ലുന്നവന് (ആദ്യത്തെ അടിയിൽ തന്നെ കൊല്ലുന്നവനേക്കാൾ) താഴെ പ്രതിഫലമാണ് ലഭിക്കുക. മൂന്നാമത്തെ അടിയിൽ കൊല്ലുന്നവന് (രണ്ടാമത്തെ അടിയിൽ തന്നെ കൊല്ലുന്നവനേക്കാൾ) താഴെ പ്രതിഫലമാണ് ലഭിക്കുക.” (സ്വഹീഹു മുസ്ലിം: 3359)
അടി മത്സരത്തിനുള്ള ആഹ്വാനമല്ല. യഥാർത്ഥത്തിൽ, ജീവജാലങ്ങളോടുള്ള പ്രവാചകന്റെ(സ) കാരുണ്യത്തിനുള്ള ഏറ്റവും നല്ല ഉദാഹരണമാണ് ഈ ഹദീസ്. ക്രൂരതയായിരുന്നു ഈ വാചകത്തിന്റെ ഉൾപ്രേരണയെങ്കിൽ ആ ജീവിയെ ഇഞ്ചിഞ്ചായി കൊല്ലാനാണ് അഹ്വാനം നൽകപ്പെടുമായിരുന്നത്. അതിന് പകരം അവയെ കൊല്ലുകയാണെങ്കിൽ ഒറ്റ അടിക്ക് കൊന്ന് വേദനയിൽ നിന്ന് പൊടുന്നനെ ആശ്വാസം നൽകാനും അതിനാണ് കൂടുതൽ പ്രതിഫലമെന്നുമാണ് ഹദീസ്. രണ്ടാമതൊരടി ആവശ്യമുണ്ടെങ്കിൽ അതിൽ കൊന്നിരിക്കണം എന്നതിനാലും വീണ്ടും ആ ജീവിയെ വേദനയിൽ തളച്ചിടരുത് എന്നതിനാലും മൂന്നാമത്തെ അടിയേക്കാൾ പ്രതിഫലം രണ്ടാമത്തെ അടിക്ക് നിശ്ചയിച്ചു. അടിയുടെ എണ്ണം കൂടും തോറും പ്രതിഫലം കുറയുമ്പോൾ ഏറ്റവും കുറഞ്ഞ എണ്ണത്തിൽ അതിനെ കൊല്ലാൻ ആളുകൾ ശ്രദ്ധിക്കുകയാണ് സംഭവിക്കുക.
പല്ലികൾ ഉപദ്രവകാരികളാണ് എന്ന മുഖ്യ കാരണത്തിന് പുറമെ അവ പ്രകൃത്യാ നീച ചിന്തയുള്ളവയാണ് എന്ന അധിക കാരണവും ഉണ്ടായിട്ടും അവയെ കൊല്ലേണ്ടി വന്നാൽ, ഇഞ്ചിഞ്ചായി ക്രൂരമായി കൊല്ലരുത് എന്ന് നിഷ്കർഷിച്ചത് കാരുണ്യമല്ലെ.
പ്രവാചകൻ (സ) പറഞ്ഞു: “അല്ലാഹു സുകൃതവാനാണ്. നന്മയെ അവൻ ഇഷ്ടപ്പെടുന്നു. അതിനാൽ നിങ്ങൾ വിധിക്കുകയാണെങ്കിൽ നീതിയോടെ വിധിക്കുക. നിങ്ങൾ കൊല്ലുകയാണെങ്കിൽ (പോലും) അതിലും (കാരുണ്യമായ) നന്മ കാണിക്കണം. (മുഅ്ജമുൽ അവ്സത്ത്: ത്വബ്റാനി: 5735)
“അല്ലാഹു സുകൃതവാനാണ്. നന്മയെ അവൻ ഇഷ്ടപ്പെടുന്നു. അതിനാൽ നിങ്ങൾ വിധിക്കുകയാണെങ്കിൽ നീതിയോടെ വിധിക്കുക. നിങ്ങൾ കൊല്ലുകയാണെങ്കിൽ (പോലും) ഉരുവിനോട് നന്മ (കരുണ) പ്രവർത്തിക്കുക. (മൃഗത്തെ) അറുക്കുകയാണെങ്കിൽ നല്ല രീതിയിൽ അറുക്കുക. ആയുധത്തിന് മൂർച്ച കൂട്ടി ഉരുവിന് (വേദനയിൽ നിന്ന്) പെട്ടെന്ന് ആശ്വാസം നൽകുക.” (ത്വബ്റാനി: 7121, സ്വഹീഹുൽ ജാമിഅ്: 1824)
വല്ല ജീവികളും മനുഷ്യരെ ഉപദ്രവിക്കുകയും അപായപ്പെടുത്തുകയും അവയെ കൊല്ലൽ ആവശ്യമായി വരികയും ചെയ്താൽ ആ കൊലയിൽ പോലും കരുണയുണ്ടാകണം എന്നാണ് പ്രവാചകൻ (സ) പഠിപ്പിച്ചത്.
മനുഷ്യരെ ഉപദ്രവിക്കാത്ത മൃഗങ്ങളെയും ജീവികളേയും തിരിച്ച് ഉപദ്രവിക്കാനൊ കൊല്ലാനൊ പ്രവാചകൻ (സ) അനുവാദം നൽകിയിട്ടില്ല. എന്നു മാത്രമല്ല ഒരു ജീവി ഉപദ്രവിച്ചു എന്നതിന്റെ പേരിൽ ആ വർഗത്തിൽപ്പെട്ട ഉപദ്രവിക്കാത്ത മറ്റു അംഗങ്ങളെ കൊല്ലുന്നതു പോലും അല്ലാഹു വിലക്കി. ഫവാസിക്കുകളിൽ എല്ലാ നായകളെയും പ്രവാചകൻ (സ) ഉൾപ്പെടുത്തിയില്ല, “കടിക്കുന്ന നായ” യെയാണ് കൊല്ലാൻ അനുവധിച്ചത് എന്ന് ശ്രദ്ധിക്കുക.
ഒരിക്കൽ ഒരു പ്രവാചകനെ ഉറുമ്പ് കടിച്ചു. അപ്പോൾ അദ്ദേഹം ഉറുമ്പും കൂട്ടിലെ മുഴുവൻ ഉറുമ്പുകളേയും കൊന്നു. അതിന്റെ പേരിൽ അല്ലാഹു ആ പ്രവാചകനെ ചോദ്യം ചെയ്യുകയുണ്ടായി.
أنْ قَرَصَتْكَ نَمْلَةٌ أحْرَقْتَ أُمَّةً مِنَ الأُمَمِ تُسَبِّحُ!
“ഒരു ഉറുമ്പ് കടിച്ചു എന്നതിന്റെ പേരിൽ അല്ലാഹുവെ സ്തുതിക്കുന്ന ഒരു സമൂഹത്തെ തന്നെ നീ ചുട്ട് ചാമ്പലാക്കിയൊ !” (സ്വഹീഹുൽ ബുഖാരി: 3019)
മൃഗങ്ങളോടുള്ള കാരുണ്യം പ്രവാചകനോളം ഊന്നിപ്പറഞ്ഞ മറ്റൊരു മത വ്യക്തിത്വങ്ങൾ വിരളമാണ്. ചില ഉദാഹരണങ്ങൾ കാണുക:
അബ്ദുല്ലാഹിബ്നു അബ്ബാസ് (റ) നിവേദനം: അല്ലാഹുവിന്റെ ദൂതൽ ഒരു വ്യക്തിയുടെ അടുത്തു കൂടെ കടന്നുപോയി; അയാൾ തന്റെ കാൽ ഒരു ആടിന്റെ പുറത്തു വെച്ച് കത്തി മൂർച്ച കൂട്ടുകയാണ്. അടാകട്ടെ അയാളിലേക്ക് തുറിച്ച് നോക്കി കൊണ്ടിരിക്കുകയുമാണ്. പ്രവാചകൻ (സ) പറഞ്ഞു: ഇതിന് മുമ്പ് (കത്തിക്ക് മൂർച്ച കൂട്ടുക എന്ന പണി) ചെയ്യാമായിരുന്നില്ലേ ? (ഉരുവിന്റെ മുമ്പിൽ വെച്ചു തന്നെ അത് ചെയ്യണമായിരുന്നോ ?) അതിന് രണ്ട് വട്ടം കൊല്ലാനാണോ നീ ഉദ്ദേശിക്കുന്നത് ?! (മുസ്തദ്റക് ഹാകിം: 7570)
ഭക്ഷിക്കുവാനായി അറുക്കുക എന്ന ന്യായമായ കാരണത്താലാണെങ്കിലും അവയെ അതിയായി ഭയപ്പെടുത്തുന്നത് ക്രൂരതയാണെന്ന് പ്രവാചകൻ (സ) പ്രഖ്യാപിച്ചു.
മൃഗങ്ങൾക്കും അവയുടെ പ്രകൃതത്തിന് യോജിച്ച അവകാശങ്ങളുണ്ടെന്ന് അദ്ദേഹം നൂറ്റാണ്ടുകൾക്ക് മുമ്പ് പഠിപ്പിച്ചു :
അബ്ദുല്ലാഹിബ്നു ജഅ്ഫർ (റ) പറഞ്ഞു: അൻസ്വാരികളിൽ പെട്ട ഒരാളുടെ തോട്ടത്തിൽ പ്രവാചകൻ (സ) പ്രവേശിച്ചു. അപ്പോൾ അവിടെയതാ ഒരു ഒട്ടകം; അല്ലാഹുവിന്റെ ദൂതനെ (സ) കണ്ടതും അത് തേങ്ങി, അതിന്റെ കണ്ണിൽ നിന്നും കണ്ണുനീർ ഒഴുകി. അപ്പോൾ പ്രവാചകൻ (സ) അതിനടുത്ത് ചെന്ന് അതിന്റെ പൂഞ്ഞയും തലയുടെ പിൻഭാഗം തലോടി. അപ്പോൾ അത് ശാന്തമായി. അദ്ദേഹം ചോദിച്ചു: ഈ ഒട്ടകത്തിന്റെ ഉടമ ആരാണ് ? ആരുടേതാണ് ഈ ഒട്ടകം? അൻസ്വാരികളിൽ പെട്ട ഒരു യുവാവ് വന്ന് അദ്ദേഹത്തോട് പറഞ്ഞു: അല്ലാഹുവിന്റെ ദൂതരേ, ഈ ഒട്ടകം എന്റേതാണ്. അപ്പോൾ പ്രവാചകൻ (സ) പറഞ്ഞു: “അല്ലാഹു താങ്കൾക്ക് ഉടമപ്പെടുത്തി തന്ന ഈ മൃഗത്തിന്റെ കാര്യത്തിൽ താങ്കൾ അല്ലാഹുവെ സൂക്ഷിക്കുന്നില്ലേ ? താങ്കൾ അതിനെ പട്ടിണിക്കിടുന്നതായും (പ്രയാസകരമായ ജോലികൾ നൽകി) ക്ഷീണിപ്പിക്കുന്നതായും അത് എന്നോട് പരാതിപ്പെടുന്നു.” (സുനനു അബൂദാവൂദ്: 2549, മുസ്നദു അഹ്മദ്: 1745 )
ഏതു മൃഗത്തോടും കരുണ കാണിക്കൽ പുണ്യമാണ് എന്നതാണ് ഇസ്ലാമിലെ അടിസ്ഥാന തത്ത്വം.
അബൂഹുറൈറയിൽ നിന്ന്: (പ്രവാചകാനുചരന്മാർ) ചോദിച്ചു: അല്ലാഹുവിന്റെ ദൂതരേ, മൃഗങ്ങളോട് നന്മ ചെയ്യുന്നതിൽ ഞങ്ങൾക്ക് പ്രതിഫലമുണ്ടോ ? അദ്ദേഹം പറഞ്ഞു: പച്ച കരളുള്ള എന്തിനോടും നന്മ ചെയ്യുന്നതിന് പ്രതിഫലമുണ്ട്. (സ്വഹീഹുൽ ബുഖാരി: 2363)
സ്വാഭാവികമായും പല്ലിയും ഇതിൽ ഉൾപ്പെടും. അതുകൊണ്ട് തന്നെ ആവശ്യമില്ലാതെ, തിരഞ്ഞു നടന്ന് പല്ലികളെ കൊല്ലാനൊന്നും ഹദീസുകളിലില്ല. ഇനി കൊല്ലേണ്ടി വന്നാൽ തന്നെ പെട്ടെന്ന് കൊല്ലുവാനും അദ്ദേഹം കൽപ്പിച്ചു.
കൊല്ലൽ അനുവദനീയമായ ഫവാസിക്കുകളിൽ കടിക്കുന്ന നായയെ എണ്ണിയ അതേ പ്രവാചകൻ (സ) കടിക്കാൻ വരാത്ത നായ്ക്കളോട് പുണ്യം ചെയ്യാൻ പഠിപ്പിച്ചു:
അബൂഹുറൈറ (റ) നിവേദനം: പ്രവാചകൻ (സ) പറഞ്ഞു: ഒരു നായ ഒരു കിണറിന് ചുറ്റും ചുറ്റിനടക്കുകയായിരുന്നു; ദാഹം കൊണ്ട് അത് ചാകാറായിട്ടുണ്ടായിരുന്നു. അപ്പോൾ ഇസ്റാഈല്യരിലെ ഒരു അഭിസാരിക അതിനെ കണ്ടു. അവർ അവരുടെ ചെരുപ്പിന്റെ മേലാവരണമൂരി (അതിൽ കിണറ്റിലെ വെള്ളം നിറച്ച്,) നായയെ കുടിപ്പിച്ചു. അത് മൂലം അവരുടെ പാപങ്ങൾ അവർക്ക് പൊറുത്തു കൊടുക്കപ്പെട്ടു. (സ്വഹീഹുൽ ബുഖാരി: 3308)
അബ്ദുർ റഹ്മാനിബ്നു അബ്ദുല്ല തന്റെ പിതാവിൽ നിന്നും ഉദ്ധരിക്കുന്നു. അദ്ദേഹം പറഞ്ഞു:
ഞങ്ങൾ അല്ലാഹുവിന്റെ ദൂതനോടൊപ്പം(സ) ഒരു യാത്രയിലായിരിക്കേ അദ്ദേഹം അൽപ്പ നേരം വിശ്രമിക്കാൻ പോയി. ഈ സമയം ഞങ്ങൾ ഒരു പക്ഷിയേയും അതിന്റെ രണ്ട് കുഞ്ഞുങ്ങളേയും കണ്ടു. ഞങ്ങൾ അതിന്റെ കുഞ്ഞുങ്ങളെ എടുത്തു കൊണ്ടുപോയി. തള്ള പക്ഷി വന്ന് ചിറകുവിരിച്ച് വട്ടമിട്ട് പറക്കാൻ തുടങ്ങി. അപ്പോൾ അല്ലാഹുവിന്റെ ദൂതൻ (സ) പറഞ്ഞു: ആരാണ് കുഞ്ഞുങ്ങളെ എടുത്ത് അതിനെ സങ്കടത്തിലാക്കിയത്. അതിന്റെ കുഞ്ഞുങ്ങളെ അതിന് തിരിച്ച് നൽകുക. (സുനനു അബൂദാവൂദ് : 2675)
ജീവജാലങ്ങളെ അനാവശ്യമായും വിനോധത്തിനായും കൊല്ലുന്നത് പോയി അവയെ ശകാരിക്കുന്നതും ശപിക്കുന്നതും വരെ പ്രവാചകൻ (സ) വിലക്കി:
“നിങ്ങൾ കോഴിയെ ശകാരിക്കരുത്; തീർച്ചയായും അത് നമ്മെ നമസ്ക്കാരത്തിന് (പ്രഭാതവേളയിൽ) ഉണർത്തുന്നു.” (സുനനു അബൂദാവൂദ്: 5101, സുനനുൽ കുബ്റാ: നസാഈ: 10781)
ചുരുക്കത്തിൽ, പല്ലിയെ കൊല്ലാൻ അനുവാദം നൽകി കൊണ്ടുള്ള ഹദീസ് വിവാദവൽക്കരിക്കുന്നത്, മൃഗശാലയിലെ Do not feed Monkeys’ ഫലകം കുരങ്ങുകളെ പട്ടിണിക്കിട്ട് കൊല്ലാൻ ആഹ്വാനം ചെയ്യുന്ന ക്രൂരതയാണെന്ന് മുദ്രാവാക്യം വിളിക്കുന്നതു പോലെ ബാലിശമാണ്.
സ്ത്രീകളുടെ യോനീസ്രവം മഞ്ഞ നിറത്തിലുള്ളതാണെന്നും അതാണ് കുഞ്ഞിന്റെ രൂപീകരണത്തിൽ പങ്കാളിയാകുന്നത് എന്നുമെല്ലാം വ്യക്തമാക്കുന്ന നിരവധി ഹദീഥുകളുണ്ട്. പ്രത്യുത്പാദനത്തിൽ പങ്കെടുക്കുന്ന സ്ത്രീയുടെ സ്രവം യോനിയിൽ പുറത്തേക്ക് കാണാൻ സാധ്യമല്ല. സ്ത്രീയുടെ സ്ഖലനത്തെയും അതിന്റെ നിറത്തെയുമെല്ലാം കുറിച്ച പരാമർശങ്ങൾ മുഹമ്മദ് നബിയുടെ തെറ്റിദ്ധാരണയിൽ നിന്നുണ്ടായതാണെന്നതല്ലേ ശരി? യോനീഭാഗത്തെ അണുബാധ നിമിത്തം യോനീസ്രവം ചിലപ്പോൾ മഞ്ഞ നിറത്തിലാകാറുണ്ട്. അത് കണ്ട് തെറ്റിദ്ധരിച്ച മുഹമ്മദ് നബി പറഞ്ഞപ്പോൾ വന്ന അബദ്ധമല്ലേ ഹദീഥുകളിലെ ഈ മഞ്ഞ ദ്രാവകം ??
സിൽഷിജ്.K
അല്ല. കുഞ്ഞിന്റെ നിർമ്മാണത്തിൽ പങ്കെടുക്കുന്ന സ്ത്രീദ്രാവകത്തിന് മഞ്ഞ നിറമാണെന്ന് പറഞ്ഞ മുഹമ്മദ് നബി(സ)ക്ക് തെറ്റുകളൊന്നും പറ്റിയിട്ടില്ല. ദൈവികബോധനത്തിന്റെ അടിസ്ഥാനത്തിൽ നബി (സ) പറഞ്ഞ കാര്യങ്ങൾ തെറ്റാണെന്ന് തെളിയിക്കാൻ ഒരു ശാസ്ത്രത്തിനും കഴിയില്ല. ഈ രംഗത്തെ പുതിയ ഗവേഷണങ്ങൾ വ്യക്തമാക്കുന്നത് ഈ സത്യമാണ്.
ഥൗബാനിൽ(റ) നിന്ന് ഇമാം മുസ്ലിം തന്റെ സ്വഹീഹിൽ നിവേദനം ചെയ്ത, ജൂത പണ്ഡിതന്റെ ചോദ്യങ്ങള്ക്കുള്ള പ്രവാചകന്റെ(സ) ഉത്തരത്തെപ്പറ്റി വിശദീകരിക്കുന്ന ദീര്ഘമായ ഒരു ഹദീഥുണ്ട്. ആ ഹദീഥിൽ ശിശുവിന്റെ സൃഷ്ടിയെക്കുറിച്ച ചോദ്യത്തിനുള്ള വിശദമായ ഉത്തരം ആരംഭിക്കുന്നത് 'പുരുഷസ്രവം വെളുത്തനിറത്തിലുള്ളതും സ്ത്രീസ്രവം മഞ്ഞനിറത്തിലുള്ളതുമാണ്' എന്നു പറഞ്ഞുകൊണ്ടാണ്. ജൂത ചോദ്യങ്ങള്ക്കെല്ലാം മറുപടി പറഞ്ഞ ശേഷം 'അയാള് എന്നോട് ചോദിച്ച കാര്യങ്ങളെക്കുറിച്ചൊന്നും അല്ലാഹു അറിയിച്ചുതരുന്നതുവരെ എനിക്ക് യാതൊരു വിവരവുമുണ്ടായിരുന്നില്ല' എന്ന് പറഞ്ഞതായുള്ള ഥൗബാനി(റ)ന്റെ പരാമര്ശം ശ്രദ്ധേയമാണ്. സ്വന്തം സ്രവത്തെക്കുറിച്ച് അറിയാത്ത സ്ത്രീകള്ക്കടക്കം നിങ്ങളുടെ സ്രവം മഞ്ഞനിറത്തിലുള്ളതാണ് എന്ന് പ്രവാചകന് (സ) പറഞ്ഞുകൊടുത്തത് വ്യക്തമായ ദൈവബോധനത്തിന്റെ അടിസ്ഥാനത്തിലാണെന്ന് വ്യക്തമാക്കുന്നതാണീ പ്രവാചകപരാമര്ശം. ബാഹ്യമായി കാണുന്ന ഏതെങ്കിലും സ്രവത്തെക്കുറിച്ചുള്ളതായിക്കൊള്ളണമെന്നില്ല ഈ പ്രവാചകപരാമർശമെന്ന് ഇതിൽ നിന്ന് വ്യക്തമാവുന്നുണ്ട്. ഇതിൽ നിന്ന് വ്യത്യസ്തമായി ബാഹ്യമായി കാണുന്ന ദ്രാവകത്തെക്കുറിച്ച് തന്നെയാണ് മഞ്ഞദ്രാവകമെന്ന് പ്രവാചകൻ(സ) പറഞ്ഞതെന്ന് ഖണ്ഡിതമായി മനസ്സിലാക്കിത്തരുന്ന നബിവചനങ്ങളൊന്നും തന്നെയില്ല.
ഏതാണീ മഞ്ഞ ദ്രാവകം? കുഞ്ഞിന്റെ സൃഷ്ടിയില് പങ്കെടുക്കുന്ന പുരുഷസ്രവത്തിന്റെ നിറം 'അബ്യദ്വ്' (വെള്ള) ആണെന്നു പറഞ്ഞതിനുശേഷമാണ് സ്ത്രീ സ്രവത്തിന്റെ നിറം 'അസ്വ്ഫര്' (മഞ്ഞ) ആണെന്ന് പ്രവാചകന് (സ) പറഞ്ഞത്. രണ്ടും കൂടിച്ചേര്ന്നാണ് കുഞ്ഞുണ്ടാകുന്നതെന്നും അതിനുശേഷം അദ്ദേഹം വ്യക്തമാക്കി. വെള്ള നിറത്തിലുള്ള പുരുഷസ്രവത്തെപോലെതന്നെ ബീജസങ്കലനത്തില് പങ്കെടുക്കുന്ന സ്ത്രീസ്രവത്തിന്റെ നിറം മഞ്ഞയാണെന്നാണ് പ്രവാചകന് (സ) ഇവിടെ പഠിപ്പിക്കുന്നതെന്നുറപ്പാണ്. സ്ത്രീശരീരത്തില്നിന്ന് നിര്ഗളിക്കു ന്ന ഏതു സ്രവത്തിനാണ് മഞ്ഞനിറമുള്ളതെന്ന കാര്യത്തില് കര്മശാസ്ത്ര പണ്ഡിതന്മാര് ഏറെ ചര്ച്ച ചെയ്തതായി കാണാന് കഴിയും. സ്ത്രീജനനേന്ദ്രിയത്തില്നിന്ന് നിര്ഗളിക്കുന്ന കാണാനാവുന്ന സ്രവങ്ങള്ക്കൊന്നും തന്നെ മഞ്ഞ നിറമില്ലെന്ന വസ്തുതയാണ് വിശാലമായ ഇത്തരം ചര്ച്ചകളുടെ ഉല്ഭവത്തിന് നിമിത്തമായത്.
സ്ത്രീകളുടെ ജനനേന്ദ്രിയത്തില്നിന്ന് പുറത്തുവരുന്ന സ്രവങ്ങള് മൂന്നെണ്ണമാണ്. തന്റെ ശരീരം ലൈംഗികബന്ധത്തിന് സജ്ജമായിയെന്ന് അറിയിച്ചുകൊണ്ട് സ്ത്രീജനനേന്ദ്രിയത്തില്നിന്ന് കിനിഞ്ഞിറങ്ങുന്ന ബര്ത്തോലിന് സ്രവം (Bartholin fluid) ആണ് ഒന്നാമത്തേത്. യോനീമുഖത്തിനകത്തായി സ്ഥിതി ചെയ്യുന്ന പയര്വിത്തിന്റെ വലിപ്പത്തിലുള്ള രണ്ട് ബര്ത്തോലിന് ഗ്രന്ഥികള് സ്ത്രീശരീരം ലൈംഗികമായി ഉത്തേജിപ്പിക്കപ്പെടുമ്പോള് പുറപ്പെടുവിക്കുന്ന ഈ സ്രവത്തിന് നിറമില്ല. രതിമൂര്ച്ചയുടെ അവസരത്തില് ചില സ്ത്രീകളുടെ ജനനേന്ദ്രിയത്തില്നിന്ന് പുറത്തുവരുന്ന പാരായുറിത്രല് സ്രവമാണ് (Para urethral fluid) രണ്ടാമത്തെ യോനീ സ്രവം. യോനിയുടെ ആന്തരികഭിത്തിയില് സ്ഥിതി ചെയ്യുന്ന പാരായുറിത്രല് ഗ്രന്ഥികളില്നിന്നു വളരെ ചെറിയ അളവില്മാത്രം പുറത്തുവരുന്ന ഈ സ്രവം താരതമ്യേന കട്ടിയുള്ളതും വെള്ള നിറത്തിലുള്ളതുമായിരിക്കും. സ്ത്രീ ജനനേന്ദ്രിയത്തെ എല്ലായ്പ്പോഴും വരളാതെ സൂക്ഷിക്കുന്ന സെര്വിക്കല് ശ്ലേഷ്മ (Cervical mucus) ആണ് മൂന്നാമത്തെ യോനീ സ്രവം. അണ്ഡോല്സര്ജനസമയമല്ലെങ്കില് ഈ സ്രവം വഴുവഴുപ്പുള്ളതും നല്ല വെളുത്ത ക്രീം നിറത്തിലുള്ളതുമായിരിക്കും. അണ്ഡോല്സര്ജനത്തോടടുക്കുമ്പോള് വെള്ളനിറം മങ്ങുകയും വഴുവഴുപ്പ് കുറയുകയും ചെയ്യുന്ന ഈ സ്രവം ഉല്സര്ജനസമയമാകുമ്പോഴേക്ക് ജലത്തെപ്പോലെ വര്ണരഹിതമാവുകയും മുട്ടയുടെ വെള്ളക്കരുവിനെപ്പോലെയായിത്തീരുകയും ചെയ്യും. അണുബാധയുണ്ടാകുമ്പോള് മാത്രമാണ് സെല്വിക്കല് ശ്ലേഷ്മത്തിന് മങ്ങിയ മഞ്ഞനിറമുണ്ടാകുന്നത്. സ്ത്രീജനനേന്ദ്രിയത്തില്നിന്ന് സാധാരണഗതിയില് നിര്ഗളിക്കപ്പെടുന്ന മൂന്ന് സ്രവങ്ങളും വെളുത്തതോ നിറമില്ലാത്തതോ ആണെന്നും ഹദീഥുകളില് പറഞ്ഞ മഞ്ഞസ്രവമല്ല ഇവയെന്നും വ്യക്തമാണ്. ഇവയ്ക്കൊന്നുംതന്നെ കുഞ്ഞിന്റെ രൂപീകരണത്തില് നേരിട്ട് പങ്കൊന്നുമില്ലതാനും.
കുഞ്ഞിന്റെ രൂപീകരണത്തിന് നിമിത്തമാകുന്ന സ്രവമെന്താണ് എന്ന ചോദ്യത്തിന് ഉത്തരം കാണാന് ശ്രമിക്കുമ്പോഴാണ് ഹദീഥുകളില് പറഞ്ഞ മഞ്ഞ സ്രവമേതാണെന്ന് നമുക്ക് മനസ്സിലാവുക. ആര്ത്തവചക്രത്തിന്റെ പതിനാലാം ദിവസം അണ്ഡാശയത്തിനകത്തെ പൂര്ണ വളര്ച്ചയെത്തിയ ഫോളിക്കിളില് പ്രത്യക്ഷപ്പെടുന്ന ദ്വാരത്തിലൂടെ പ്രായപൂര്ത്തിയെത്തിയ അണ്ഡത്തെവഹിച്ചുകൊണ്ട് ഫോളിക്കുളാര് ദ്രവവും ക്യൂമുലസ് കോശങ്ങളും പുറത്തേക്ക് തെറിച്ച് ഫലോപ്പിയന് നാളിയുടെ അറ്റത്തുള്ള ഫിംബ്രയകളില് പതിക്കുന്നതിനാണ് അണ്ഡോല്സര്ജനം (Ovulation) എന്നു പറയുന്നത്. രതിമൂര്ച്ചയോടനുബന്ധിച്ച് പുരുഷശരീരത്തില് നടക്കുന്ന ശുക്ലസ്ഖലന(Ejaculation)ത്തിന് തുല്യമായി സ്ത്രീശരീരത്തില് നടക്കുന്ന പ്രക്രിയയാണ് ഇതെങ്കിലും ഒരു ആര്ത്തവചക്രത്തില് ഒരു തവണ മാത്രമാണ് ഇത് സംഭവിക്കുന്നത്. ശുക്ല സ്ഖലനവും അണ്ഡോല്സര്ജനവുമാണ് കുഞ്ഞിന്റെ സൃഷ്ടിക്ക് നിദാനമായി പുരുഷശരീരത്തിലും സ്ത്രീശരീരത്തിലും സംഭവിക്കുന്ന രണ്ട് പ്രക്രിയകള്. പുരുഷബീജങ്ങളെ വഹിക്കുന്ന ശുക്ലദ്രാവകത്തെപ്പോലെ സ്ത്രീയുടെ അണ്ഡത്തെ വഹിക്കുന്ന ഫോളിക്കുളാര് ദ്രവവും കുഞ്ഞിന്റെ നിര്മാണത്തിന് നിമിത്തമാകുന്ന ദ്രാവകമാണ്. ഹദീഥുകളില് പറഞ്ഞ കുഞ്ഞിന്റെ സൃഷ്ടിക്ക് കാരണമായ സ്ത്രീസ്രവം അണ്ഡത്തെ വഹിക്കുന്ന ഫോളിക്കുളാര് ദ്രാവകമാണെന്നാണ് ഇത് വ്യക്തമാക്കുന്നത്. അങ്ങനെയാണെങ്കില് പുരുഷദ്രാവകം വെളുത്തതും സ്ത്രീദ്രാവകം മഞ്ഞയുമെന്ന് പരാമര്ശത്തിന്റെ വെളിച്ചത്തില് പരിശോധിക്കുമ്പോള് ഫോളിക്കുളാര് ദ്രാവകത്തിന്റെ നിറം മഞ്ഞയായിരിക്കണം. എന്നാല് എന്താണ് വസ്തുത?
പ്രായപൂര്ത്തിയെത്തുന്നതിനുമുമ്പുള്ള അണ്ഡാവസ്ഥയായ അണ്ഡത്തെ (Oocyte) സംരക്ഷിക്കുകയും വളര്ത്തിക്കൊണ്ടുവന്ന് ബീജസങ്കലനത്തിന് പറ്റിയ അണ്ഡമാക്കിത്തീര്ക്കുകയും ചെയ്യുകയാണ് ഫോളിക്കിളിന്റെ ധര്മം. പെണ്കുഞ്ഞ് ജനിക്കുമ്പോള് തന്നെ അവളുടെ അണ്ഡാശയത്തിലുള്ള പ്രായപൂര്ത്തിയെത്താത്ത അണ്ഡകങ്ങളെ പൊതിഞ്ഞ് ആദിമ ഫോളിക്കിളുകളുണ്ടാവും (Primordial follicles). അവള് പ്രായപൂര്ത്തിയാകുന്നതോടെ ഇതില് ചില ഫോളിക്കിളുകള് വളര്ന്നുവരികയും ഓരോ ആര്ത്തവചക്രത്തിന്റെയും ശരാശരി 14-16 ദിവസങ്ങള് കഴിഞ്ഞ് പൊട്ടി പൂര്ണവളര്ച്ചയെത്തിയ അണ്ഡത്തെ (Ovum) പുറത്തുവിടുന്നതോടെ അവയുടെ ധര്മം അവസാനിക്കുകയും ചെയ്യുന്നു. ജനനസമയത്തുള്ള ഏകദേശം 1,80,000 ഫോളിക്കിളുകളില് നാനൂറെണ്ണത്തോളം മാത്രമാണ് അണ്ഡോല്സര്ജനത്തിനുമുമ്പത്തെ വളര്ച്ചയെത്തുവാനുള്ള ഭാഗ്യമുണ്ടാകുന്നത്. പ്രസ്തുത വളര്ച്ചയ്ക്ക് വ്യത്യസ്തങ്ങളായ ഘട്ടങ്ങളുണ്ട്. ഇതിലെ ഓരോ ഘട്ടങ്ങളിലും അതു കടന്നുപോകാന് കഴിയാത്ത ഫോളിക്കിളുകള് മരിച്ചുപോകുന്നുണ്ട്. ഓരോ ആര്ത്തവചക്രത്തിലും ഇരുപതോളം ഫോളിക്കിളുകള് വളര്ച്ചയെത്തുന്നുവെങ്കിലും ഒരെണ്ണത്തിന് മാത്രമാണ് ഫോളിക്കിള് മരണമായ അട്രീഷ്യ(atresia)യില്നിന്ന് രക്ഷപ്പെട്ട് അണ്ഡോല്സര്ജനത്തിന് കഴിയുന്നത്. അട്രീഷ്യയില് നിന്ന് രക്ഷപ്പെട്ട് അണ്ഡോല്സര്ജനത്തിന് കഴിയുന്ന ഫോളിക്കിളുകള് രണ്ട് ദശകളിലൂടെയാണ് കടന്നു പോകുന്നത്. അണ്ഡോല്സര്ജനത്തിലൂടെ അവസാനിക്കുന്ന ഒന്നാമത്തെ ദശയെ ഫോളിക്കുളാര് ദശ (follicular phase) എന്നും അതിനുശേഷ മുള്ള ദശയെ ലൂടിയല് ദശ (luteal phase) എന്നുമാണ് വിളിക്കുക. ആര്ത്തവം മുതല് അണ്ഡോല്സര്ജനം വരെയുള്ള ഫോളിക്കുളാര് ദശയില് അണ്ഡകം പൂര്ണവളര്ച്ചയെത്തിയ അണ്ഡമായിത്തീരുന്നതിനും യഥാരൂപത്തിലുള്ള അണ്ഡോല്സര്ജനം നടക്കുന്നതിനും വേണ്ടി വ്യത്യസ്തങ്ങളായ പ്രക്രിയകള് നടക്കേണ്ടതുണ്ട്. ഈ പ്രക്രിയകളുടെ അവസാനമായി ശരീരത്തിലെ ഈസ്ട്രജന് നില പരമാവധി ഉയരുകയും ലൂറ്റിനൈസിംഗ് ഹോര്മോണ് (LH), ഫോളിക്കിള് സ്റ്റിമുലേറ്റിംഗ് ഹോര്മോണ് (FSH) എന്നീ ഹോര്മോണുകളെ ഇതിന്റെ ഫലമായി ഉത്പാദിപ്പിക്കുകയും ചെയ്യുന്നു. 24 മുതല് 36 വരെ മണിക്കൂറുകള് നീണ്ടുനില്ക്കുന്ന ഈ പ്രക്രിയയുടെ അന്ത്യം കുറിച്ചുകൊണ്ടാണ് അണ്ഡം വഹിക്കുന്ന പൂര്ണവളര്ച്ചയെത്തിയ ഫോളിക്കിളില് (Ovarian follicle) സ്റ്റിഗ്മയെന്ന് പേരുള്ള ദ്വാരമുണ്ടാവുകയും അത് പൊട്ടി അണ്ഡത്തെ വഹിച്ചുകൊണ്ട് ഫോളിക്കുളാര് ദ്രവം പുറത്തേക്ക് തെറിക്കുകയും ചെയ്യുന്നത്. ഈ പുറത്തേക്കു തെറിക്കല് പ്രക്രിയക്കാണ് അണ്ഡോല്സര്ജനം (Ovulation) എന്നു പറയുക
ഫോളിക്കുളാര് ദശയിലുടനീളം നടക്കുന്ന അണ്ഡവളര്ച്ചയ്ക്കും അതിന് ഉല്സര്ജിക്കാനാവശ്യമായസംവിധാനങ്ങളൊരുക്കുന്നതിനും നിമിത്തമാകുന്നത് FSHന്റെ പ്രവര്ത്തനങ്ങളാണ്. പ്രസ്തുത ഉത്പാദനത്തോടനുബന്ധിച്ചാണ് ഹൈപ്പോതലാമസില്നിന്നുള്ള ഗൊണാടോട്രോ പിന് റിലീസിംഗ് ഹോര്മോണിന്റെ (GnRH) പ്രേരണയാല് പിറ്റിയൂട്ടറിയില്നിന്ന് LHന്റെ ഉത്പാദനം നടക്കുന്നത്. ഈ ഹോര്മോണ് ഉത്പാദിപ്പിക്കുന്ന പ്രോട്ടീന് വിഘാടക രസങ്ങളായ പ്രോട്ടിയോലിറ്റിക് എന്സൈമുകളാണ് (Proteolytic enzymes) ഫോളിക്കിളിലുണ്ടാവുന്ന ദ്വാരമായ സ്റ്റിഗ്മക്ക് കാരണമാകുന്നത്. അണ്ഡോല്സര്ജനത്തിനുശേഷമുള്ള ഫോളിക്കിള് അവശിഷ്ടങ്ങളെ നിയന്ത്രിക്കുന്നതും പ്രധാനമായി ഈ ഹോര്മോണാണ്. ലൂട്ടിയല് ദശയില് അണ്ഡം നഷ്ടപ്പെട്ട ഫോളിക്കിള് അവശിഷ്ടങ്ങള് കോര്പസ് ലൂടിയം (Lorpus Luteum) ആയിത്തീരുകയും മാതൃസ്വഭാവങ്ങളെ ഉദ്ദീപിക്കുന്ന പ്രോജസ്റ്ററോണ് (Progesterone) ഹോര്മോണിന്റെ വര്ധിതമായ ഉത്പാദനത്തിന് നിമിത്തമാവുകയും ചെയ്യുന്നു.
എന്താണീ ലൂറ്റിനൈസിംഗ് ഹോര്മോണ്? മഞ്ഞയെന്ന് അര്ത്ഥം വരുന്ന ലൂറ്റിയസ് (Luteus) എന്ന ലാറ്റിന് പദത്തിന്റെ നപുംസകരൂപമായ ലൂറ്റിയത്തില്നിന്നാണ് (Luteum) ലൂറ്റിനൈസ് (Luteinize) എന്ന ക്രിയയുണ്ടായിരിക്കുന്നത്. കോര്പ്പസ് ലൂടിയത്തിന്റെ നിര്മിതിക്ക് നിമിത്ത മായ പ്രവര്ത്തനങ്ങള്ക്കാണ് സാങ്കേതികമായി ലൂറ്റിനൈസ് എന്ന് പറയുന്നതെങ്കിലും പദപരമായി അതിനര്ത്ഥം 'മഞ്ഞയാക്കുന്നത്' എന്നാണ്. ലൂറ്റിനൈസിംഗ് ഹോര്മോണിന്റെ പ്രവര്ത്തനഫലമായാണ് ഫോളിക്കുളാര് ദശ പിന്നിട്ട ഫോളിക്കിള് അവശിഷ്ടങ്ങള് കോര് പസ് ലൂടിയം ആയിത്തീരുന്നത്. കോര്പസ് ലൂടിയം എന്ന പദദ്വയത്തിനര്ത്ഥം മഞ്ഞ വസ്തുവെന്നാണ് (Yellow body). ലൂടിയല് ദശയിലേക്ക് കടന്ന അണ്ഡം നഷ്ടപ്പെട്ട ഫോളിക്കിള് അവശിഷ്ടങ്ങളെല്ലാം കൂടി രണ്ടു മുതല് അഞ്ചു സെന്റീമീറ്റര് വരെ വ്യാസത്തില് ശരീരത്തില് ഏതാ നും ദിവസങ്ങള് കൂടി അവശേഷിക്കും. മനുഷ്യരില് ഇത് ഓറഞ്ചു നിറത്തിലാണ് കാണപ്പെടുന്നത്. അണ്ഡോല്സര്ജനത്തിന്റെ അവസാനഘട്ടത്തില് ഉത്പാദിപ്പിക്കപ്പെടുന്ന LH അതിന്റെ പ്രവര്ത്തനമാരംഭിക്കുകയും ഫോളിക്കുളാര് ദ്രവത്തെ മഞ്ഞവല്ക്കരിക്കുകയും ചെയ്യും. ഫോളിക്കിളിലെ സ്റ്റിഗ്മ പൊട്ടി അണ്ഡത്തോടെ പുറത്തേക്ക് തെറിക്കുന്ന ഫോളിക്കുളാര് ദ്രാവകത്തിന്റെ നിറം മഞ്ഞയായിരിക്കും. പുരുഷ ശുക്ലവുമായി താരതമ്യം ചെയ്യുമ്പോള് കട്ടിയില്ലാത്തതും മഞ്ഞ നിറത്തിലുള്ളതുമായ ദ്രാവകമാണ് ഫോളിക്കിള് പൊട്ടി പുറത്തേക്കൊഴുകുന്ന കുഞ്ഞിന്റെ നിര്മാണത്തിന് നിമിത്തമാകുന്ന സ്ത്രീസ്രവം എന്നര്ത്ഥം.
മഞ്ഞ നിറത്തിലുള്ള സ്ത്രീയുടെ സ്രവമാണ് വെളുത്ത നിറത്തിലുള്ള പുരുഷന്റെ സ്രവവുമായി ചേർന്ന് കുഞ്ഞുണ്ടാവുന്നത് എന്ന പ്രവാചകന്റെ പരാമർശം തന്നെയാണ് ശരിയെന്ന് ഇവ വ്യക്തമാക്കുന്നു. ഈ നൂറ്റാണ്ടിന്റെ തുടക്കത്തിൽ മാത്രം ഭ്രൂണശാസ്ത്രജ്ഞർ മനസ്സിലാക്കിയ കാര്യങ്ങൾ പതിനാലു നൂറ്റാണ്ടുകൾക്ക് മുമ്പ് പ്രസ്താവിച്ചതിനു ശേഷം നബി (സ) വ്യക്തമാക്കിയ കാര്യം ഇവിടെ എടുത്ത് പറയേണ്ടതുണ്ടെന്ന് തോന്നുന്നു. "അയാള് എന്നോട് ചോദിച്ച കാര്യങ്ങളെക്കുറിച്ചൊന്നും അല്ലാഹു അറിയിച്ചുതരുന്നതുവരെ എനിക്ക് യാതൊരു വിവരവുമുണ്ടായിരുന്നില്ല”.
വിഷയവുമായി ബന്ധപ്പെട്ട വീഡിയോ
പാരമ്പര്യ സ്വഭാവങ്ങളുടെ സംപ്രേഷണത്തെപ്പറ്റി ക്വുര്ആനില് വ്യക്തമായ പരാമര്ശങ്ങളൊന്നുമില്ല. സ്വഹീഹു മുസ്ലിമിലെ കിതാബുല് ഹയ്ദിലുള്ള സ്ത്രീയുടെ സ്രവത്തെക്കുറിച്ച ഒരു നബിവചനത്തില് പാരമ്പര്യത്തെക്കുറിച്ച കൃത്യവും വ്യക്തവുമായ സൂചനകളുണ്ട്. പ്രസ്തുത ഹദീഥിന്റെ സാരം ഇങ്ങനെയാണ്:
'സ്രവം കാരണമായിട്ടാണ് കുട്ടിക്ക് സാദൃശ്യമുണ്ടാകുന്നത്. സ്ത്രീയുടെ സ്രവം പുരുഷന്റെ സ്രവത്തിന് മുകളില് വന്നാല് കുട്ടിക്ക് മാതൃ സഹോദരന്മാരോട് സാദൃശ്യമുണ്ടാകും. പുരുഷന്റെ സ്രവം സ്ത്രീയുടെ സ്രവത്തിന് മുകളില് വന്നാല് കുട്ടിക്ക് അവന്റെ പിതൃവ്യന്മാ രോട് സാദൃശ്യമുണ്ടാകും.''(സ്വഹീഹു മുസ്ലിം, കിതാബുല് ഹൈദ്വ്, ബാബു വുജുബില് ഗസ്ലി അലല് മര്അത്തി ബി ഖുറൂജില് മനിയ്യി മിന്ഹ, ഹദീഥ് 314.)
പുരുഷന്റെയും സ്ത്രീയുടെയും സ്രവങ്ങളാണ് കുഞ്ഞിലേക്ക് പാരമ്പര്യ സ്വഭാവങ്ങള് പകര്ത്തുന്നെന്ന് വ്യക്തമാക്കുന്ന ഈ ഹദീഥ് ജനി തക സംപ്രേഷണവുമായി ബന്ധപ്പെട്ട പുതിയ വിവരങ്ങളുമായി പൂര്ണമായും പൊരുത്തപ്പെടുന്നതാണെന്ന വസ്തുത അത്ഭുതകരമാണ്. ഓരോ അവയവങ്ങളില് നിന്നും ഊര്ന്നിറങ്ങുന്നതാണ് ബീജമെന്നും ആണില് നിന്നോ പെണ്ണില് നിന്നോ ആരില്നിന്നാണോ ശക്തബീജമുണ്ടാ കുന്നത് അവരുടെ സവിശേഷതയായിരിക്കും കുഞ്ഞിലേക്ക് പകര്ത്തപ്പെടുന്നെന്നും ആര്ജ്ജിത സ്വഭാവങ്ങള് കുഞ്ഞിലേക്കു പകരുമെന്നു മുള്ള ഹിപ്പോക്രാറ്റസ് മുതല് ഡാര്വിന് വരെയുള്ളവരുടെ വീക്ഷണങ്ങളെ ഈ ഹദീഥ് അനുകൂലിക്കുന്നില്ല. രക്തത്തിലൂടെയാണ് പാരമ്പ ര്യത്തിന്റെ സംപ്രേഷണം നടക്കുന്നതെന്ന അരിസ്റ്റോട്ടിലിന്റെ വീക്ഷണത്തെ ഇത് നിരാകരിക്കുകയും ചെയ്യുന്നു. സ്ത്രീപുരുഷസ്രവങ്ങളു ടെ പ്രത്യക്ഷീകരണമാണ് കുഞ്ഞിന്റെ സവിശേഷതകള് നിര്ണയിക്കുന്നതെന്ന ഈ ഹദീഥ് മുന്നോട്ടുവെക്കുന്ന ആശയം ആധുനികകാലം വരെയുള്ള ശാസ്ത്രജ്ഞരൊന്നും മനസ്സിലാക്കിയിട്ടില്ലാത്തതാണ്. അതിശക്തമായ സൂക്ഷ്മദര്ശനികളുടെ സഹായത്താല് നടത്തിയ ഗവേഷ ണങ്ങള് വെളിപ്പെടുത്തിയ യാഥാര്ത്ഥ്യങ്ങളുമായി ഈ നബിവചനം യോജിച്ചുവരുന്നുവെന്ന വസ്തുത എന്തുമാത്രം ആശ്ചര്യകരമല്ല!
ഈ ഹദീഥില് പുരുഷന്റെ സ്രവത്തെ സ്ത്രീയുടെ സ്രവം അതിജയിച്ചാല് എന്നു പരിഭാഷപ്പെടുത്തിയിരിക്കുന്നത് 'ഇദാ അലാ മാഉഹാ മാഉര് റജൂലി'യെന്ന അറബി വചനത്തെയാണ്. പെണ്സ്രവം പുരുഷസ്രവത്തെ അതിജയിക്കുന്നതിന് ഇവിടെ 'അലാ'യെന്നാണ് പ്രയോഗി ച്ചിരിക്കുന്നത്. ഒന്നിനു മുകളില് മറ്റൊന്ന് ആധിപത്യം പുലര്ത്തുന്നതിനാണ് 'അലാ'യെന്നു പ്രയോഗിക്കുകയെന്ന് സൂറത്തുല് മുഅ്മിനൂ നിലെ 91-ാം വചനത്തില് നിന്ന് നാം നേരത്തെ മനസ്സിലാക്കിയിട്ടുള്ളതാണ്. പ്രത്യക്ഷീകരണ(dominance)ത്തെ ദ്യോതിപ്പിക്കുന്ന കൃത്യമായ പദമാണിത്. പുരുഷസ്രവം പെണ്സ്രവത്തിനുമേല് പ്രത്യക്ഷീകരിക്കുമ്പോള് പിതൃസഹോദരങ്ങളോടും, പെണ്സ്രവമാണ് പ്രത്യക്ഷീകരി ക്കുന്നതെങ്കില് മാതൃസഹോദരങ്ങളോടുമായിരിക്കും കുഞ്ഞിനു സാദൃശ്യമെന്നാണ് ഈ ഹദീഥ് പഠിപ്പിക്കുന്നത്. ഏതെങ്കിലുമൊരു സവി ശേഷതയുമായി ബന്ധപ്പെട്ട പെണ്സ്രവത്തിലെ ജീനാണ് പ്രത്യക്ഷമാവുന്നതെങ്കില് മാതൃസഹോദരങ്ങളിലാരുടെയെങ്കിലും സവിശേഷ തയാണ് കുഞ്ഞിനുണ്ടാവുകയെന്നും ആണ്സ്രവത്തിലെ ജീനാണ് പ്രത്യക്ഷമാവുന്നതെങ്കില് പിതൃസഹോദരങ്ങളില് ആരുടെയെങ്കിലും സവിശേഷതയാണ് കുഞ്ഞിനുണ്ടാവുകയെന്നുമുള്ള വസ്തുതകള് -ഇരുപതാം നൂറ്റാണ്ടിന്റെ തുടക്കത്തില് മാത്രം നാം മനസ്സിലാക്കിയ സത്യങ്ങള്- എത്ര കൃത്യമായാണ് ഈ ഹദീഥില് പ്രസ്താവിക്കുന്നത്!
ഹദീഥില് പിതൃസഹോദരങ്ങള് എന്നു പരിഭാഷപ്പെടുത്തിയിരിക്കുന്നത് 'അഅ്മാം' എന്ന പദത്തെയും മാതൃസഹോദരങ്ങള് എന്നു പരി ഭാഷപ്പെടുത്തിയിരിക്കുന്നത് 'അഖ്ലാല്' എന്ന പദത്തെയുമാണ്. 'അമ്മി'ന്റെ ബഹുവചനമാണ് 'അഅ്മാം'; 'ഖാലി'ന്റേത് 'അഖ്ലാലും'. പിതൃസഹോദരങ്ങളെ മൊത്തത്തില് അഅ്മാം എന്നും, മാതൃസഹോദരങ്ങളെ മൊത്തത്തില് അഖ്ലാല് എന്നും വിളിക്കുന്നു. പുരുഷ സ്രവം പെണ് സ്രവത്തെ അതിജയിച്ചാല് പിതാവിന്റെയും പെണ്സ്രവമാണ് അതിജയിക്കുന്നതെങ്കില് മാതാവിന്റെയും സാദൃശ്യമാണ് കുഞ്ഞിനുണ്ടാവുകയെന്നായിരുന്നു ഈ ഹദീഥിലുള്ളതെങ്കില് പാരമ്പര്യത്തെക്കുറിച്ച പുതിയ വിവരങ്ങളുമായി അത് വൈരുദ്ധ്യം പുലര്ത്തുന്നുവെന്ന് പറയാന് കഴിയുമായിരുന്നു; എന്നാല് സദൃശ്യപ്പെടാനുള്ള സാധ്യത പിതാവിലോ മാതാവിലോ പരിമിതപ്പെടുത്തു ന്നില്ല. ഏതെങ്കിലുമൊരു ജീനിന്റെ പ്രത്യക്ഷീകരണം നടക്കുമ്പോള് അത് പിതാവില് പ്രത്യക്ഷമായതു തന്നെയാകണമെന്നില്ലെന്നും പിതൃസ ഹോദരങ്ങളിലാരിലെങ്കിലും പ്രത്യക്ഷമായതാകാമെന്നുമാണല്ലോ ജനിതകം നമ്മെ പഠിപ്പിക്കുന്നത്. മാതൃസഹോദരങ്ങള്, പിതൃസഹോദ രങ്ങള് തുടങ്ങിയ ബഹുവചന പ്രയോഗങ്ങളിലൂടെ ഓരോ സവിശേഷതകളുടെയും ജീനുകള് പ്രത്യക്ഷീകരിക്കുന്നതിനുള്ള സാധ്യതകള് ഈ ഹദീഥില് തുറന്നിട്ടിരിക്കുകയാണ്. മാതൃശരീരത്തില് നിന്നുള്ള ജീനാണ് കുഞ്ഞില് പ്രത്യക്ഷമാകുന്നതെങ്കില് അതേ ജീന് മാതാവില് പ്രത്യ ക്ഷമല്ലെങ്കിലും മാതൃസഹോദരങ്ങളില് ആരിലെങ്കിലും പ്രത്യക്ഷമായിരിക്കുമെന്നും പിതാവില് നിന്നുള്ളതാണെങ്കില് പിതൃസഹോദരന് മാരിലാരിലെങ്കിലും അത് പ്രത്യക്ഷമായിരിക്കുമെന്നുമുള്ള ജനിതക ശാസ്ത്രം നമുക്ക് നല്കുന്ന അറിവുകള് എത്ര സമര്ത്ഥമായാണ് ഈ ഹദീഥിലെ പരാമര്ശങ്ങള്ക്കിടയില് ഒളിപ്പിച്ചുവെച്ചിരിക്കുന്നത്! പാരമ്പര്യത്തെക്കുറിച്ച പുതിയ വിവരങ്ങളുമായി പൂര്ണമായും പൊരുത്തപ്പെടുന്നതാണീ ഹദീഥ്. ഇതിലെ പദപ്രയോഗങ്ങളുടെ കൃത്യത ആരെയും അത്ഭുതപ്പെടുത്തുന്നതാണ്.
ലിംഗനിര്ണയവുമായി ബന്ധപ്പെട്ട ക്വുര്ആന് പരാമര്ശങ്ങള് ശ്രദ്ധിക്കുക.
''ആണ്, പെണ് എന്നീ രണ്ട് ഇണകളെ അവനാണ് സൃഷ്ടിച്ചതെന്നും. ഒരു ബീജം സ്രവിക്കപ്പെടുമ്പോള് അതില് നിന്ന്.'' (53: 45-46) (1)
''പിന്നെ അവന് ഒരു ഭ്രൂണമായി. എന്നിട്ട് അല്ലാഹു (അവനെ) സൃഷ്ടിച്ചു സംവിധാനിച്ചു. അങ്ങനെ അതില് നിന്ന് ആണും പെണ്ണുമാകുന്ന രണ്ടു ഇണകളെ അവന് ഉണ്ടാക്കി. അങ്ങനെയുള്ളവന് മരിച്ചവരെ ജീവിപ്പിക്കാന് കഴിവുള്ളവനല്ലെ?'' (75: 38-40)(2)
ഹദീഥുകളിലാണ് ലിംഗനിര്ണയത്തെപ്പറ്റി കുറേക്കൂടി വ്യക്തമായ പരാമര്ശമുള്ളത്.
- അനസില് നിന്ന്: പ്രവാചകന് മദീനയില് വന്ന വിവരം അബ്ദുല്ലാഹിബ്നു സലാമിനു കിട്ടി. അദ്ദേഹം നബിയുടെ അടുത്തുവന്ന് പറഞ്ഞു: 'ഒരു പ്രവാചകനു മാത്രം അറിയാവുന്ന മൂന്നു കാര്യങ്ങള് ഞാന് താങ്കളോട് ചോദിക്കുകയാണ്..... ഇനി കുട്ടിക്ക് സാദൃശ്യം ലഭിക്കുന്ന കാര്യം; പുരുഷന് സ്ത്രീയുമായി വേഴ്ച നടത്തുന്ന വേളയില് അവന്റെ സ്രവം അവളുടെ സ്രവത്തെ അതിജയിച്ചാല് കുട്ടിക്ക് സാദൃശ്യം അയാളോടായി. അവളുടെ സ്രവം അവന്റെ സ്രവത്തെയാണ് അതിജയിക്കുന്നതെങ്കില് അവളോടും.' അബ്ദുല്ല പറഞ്ഞു: 'താങ്കള് അല്ലാഹുവിന്റെ ദൂതനാണെന്നു ഞാന് സാക്ഷ്യപ്പെടുത്തുന്നു.'(3)
- അനസ് ബ്നുമാലികി(റ)ല് നിന്ന്: പുരുഷന് സ്വപ്നസ്ഖലനമുണ്ടാവുന്നതുപോലെ സ്ത്രീക്കും സ്വപ്നസ്ഖലനമുണ്ടായാല് അവള് എന്താണ് ചെേയ്യണ്ടത് എന്നതിനെ സംബന്ധിച്ച് ഉമ്മുസുലൈം പ്രവാചകനോട് ചോദിച്ചു. ...........നിശ്ചയമായും പുരുഷന്റെ‚ഇന്ദ്രിയം വെളുത്തതും കട്ടിയുള്ളതുമാണ്. സ്ത്രീയുടെ ഇന്ദ്രിയം മഞ്ഞനിറമുള്ളതും നേര്മയുള്ളതുമാണ്. ഏത് മുകളില് വരുന്നുവോ അല്ലെങ്കില് മുന്കടക്കുന്നുവോ അതിനോടാണ് കുട്ടിക്ക് സാദൃശ്യമുണ്ടാവുക.'(4)
- നബി (സ) സ്വാതന്ത്ര്യം നല്കിയ ഥൗബാനി(റ)ല് നിന്ന്: ഞാന് നബി(സ)യുടെ അടുക്കല് നില്ക്കുമ്പോള്ƒജൂത പണ്ഡിതന്മാരില് നിന്നുള്ള ഒരു പണ്ഡിതന് വരികയും 'അസ്സലാമു അലൈക്ക യാ മുഹമ്മദ് (മുഹമ്മദ്, നിനക്ക് സമാധാനമുണ്ടാകട്ടെ)' എന്ന് പറയുകയും ചെയ്തു. ............ അയാള് തുടര്ന്നു പറഞ്ഞു: 'ഭൂനിവാസികളില്നിന്നും ഒരു പ്രവാചകനോ അല്ലെങ്കില് ഒന്നോ രണ്ടോ ആളുകള്ക്കോ അല്ലാതെ മറ്റൊരാക്കും അറിയാത്ത ഒരു കാര്യത്തെ സംബന്ധിച്ച് ചോദിക്കുവാനാണ് ഞാന് വന്നിട്ടുള്ളത്.' നബി (സ) ചോദിച്ചു: 'ഞാനത് പറഞ്ഞാ ല് നിനക്കത് ഉപകരിക്കുമോ?'. 'ഞാന് എന്റ ചെവികള് കൊണ്ട് കേള്ക്കും'. അയാള് പറഞ്ഞു: '(പ്രസവിക്കപ്പെടുന്ന) ശിശുവിനെക്കു റിച്ച് ചോദിക്കുവാനാണ് ഞാന് വന്നത്' നബി (സ) പറഞ്ഞു: 'പുരുഷന്റെ‚ ഇന്ദ്രിയം വെളുത്ത നിറത്തിലുളളതും സ്ത്രീയുടെ ഇന്ദ്രിയം മഞ്ഞനിറത്തിലുള്ളതുമാണ്. അത് രണ്ടും ഒരുമിച്ച് ചേരുകയും പുരുഷ ഇന്ദ്രിയം സ്ത്രീ ഇന്ദ്രിയത്തെ അതിജയിക്കുകയും ചെയ്താല് അല്ലാഹുവിന്റെ അനുമതിയോടെ അത് ആണ് കുട്ടിയായിതീരുന്നു. സ്ത്രീയുടെ ഇന്ദ്രിയം പുരുഷ ഇന്ദ്രിയത്തെ അതിജയിച്ചാല് അല്ലാഹു വിന്റെ‚അനുമതിയോടെ അത് പെണ്കുട്ടിയായി തീരുന്നു.' ജൂതന് പറഞ്ഞു: 'തീര്ച്ചയായും താങ്കള് പറഞ്ഞത് സത്യമാണ്. തീര്ച്ചയായും താങ്കള് ഒരു പ്രവാചകന് തന്നെയാണ്'. പിന്നെ അയാള് തിരിച്ചുപോയി. അപ്പോള് നബി (സ) പറഞ്ഞു: 'അയാള് എന്നോടു ചോദിച്ച കാര്യങ്ങളെക്കുറിച്ചൊന്നും അല്ലാഹു അറിയിച്ചുതരുന്നതുവരെ എനിക്ക് യാതൊരു വിവരവും ഉണ്ടായിരുന്നില്ല.'(5)
- ഹുദൈഫത്ത് ബ്നുഅസീദി(റ)ണ് നിന്ന്: നബി (സ) പറഞ്ഞു: 'ഗര്ഭാശയത്തിണ് ബീജം നാല്പത് ദിവസം അല്ലെങ്കില് നാല്പത്തഞ്ച് ദിവസം ആയിത്തീരുമ്പോള് അതിന്മേല് ഒരു മലക്ക് പ്രവേശിക്കും. എന്നിട്ടവന് ചോദിക്കും: രക്ഷിതാവേ, ദൗര്ഭാഗ്യവാനോ അതോ സൗഭാഗ്യവാനോ? എന്നിട്ട് അത് രേഖപ്പെടുത്തും. പിന്നെ ചോദിക്കും: രക്ഷിതാവേ, ആണോ അതോ പെണ്ണോ? എന്നിട്ട് അതും രേഖപ്പെ ടുത്തും. അവന്റെ കര്മവും അവന്റെ‚ ഫലവും, അവന്റെ‚അവധിയും, അവന്റെ‚ ഉപജീവനവും എഴുതപ്പെടും. പിന്നീട് ഏടുകള് ചുരുട്ടപ്പെടും. അതില് ഒന്നും വര്ദ്ധിപ്പിക്കപ്പെടുകയില്ല; ഒന്നും ചുരുട്ടപ്പെടുകയുമില്ല.'(6)
- അബ്ദാഹി ബ്നുമസ്ഊദി(റ)ല് നിന്ന്: നബി (സ) പറയുന്നത് ഞാന് കേട്ടു: 'ബീജത്തിന്മേല് നാല്പത്തിരണ്ട് ദിവസം കഴിഞ്ഞാല് അല്ലാഹു ഒരു മലക്കിനെ നിയോഗിക്കും. എന്നിട്ട് അവന് അതിനെ രൂപപ്പെടുത്തുകയും, അതിന് കേള്വിയും കാഴ്ചയും ചര്മവും മാംസവും അസ്ഥിയും രൂപപ്പൈടുത്തുകയും ചെയ്യും. പിന്നീട് ആ മലക്ക് ചോദിക്കും: രക്ഷിതാവേ, ആണോ അതോ പെണ്ണോ? അപ്പോള് നിന്റെ രക്ഷിതാവ് അവന് ഉദ്ദേശിക്കുന്നത് വിധിക്കും. മലക്ക് അത് രേഖപ്പെടുത്തും. പിന്നീട് മലക്ക് ചോദിക്കും: രക്ഷിതാവേ ഇവന്റെ അവധി? അപ്പോള് നിന്റെ രക്ഷിതാവ് അവന് ഉദ്ദേശിച്ചത് പറയുകയും മലക്ക് അത് രേഖപ്പെടുത്തുകയും ചെയ്യും. പിന്നെ മലക്ക് ചോദി ക്കും: രക്ഷിതാവേ, ഇവന്റെ ഉപജീവനം? അപ്പോള് നിന്റെ രക്ഷിതാവ് അവന് ഉദ്ദേശിച്ചത് വിധിക്കുകയും മലക്ക് അത് രേഖപ്പെടുത്തു കയും ചെയ്യും. പിന്നീട് മലക്ക് തന്റെ‚കയ്യില് ആ ഏടുമായി പോകും. കല്പിക്കപ്പെട്ടതിനേക്കാള് വര്ദ്ധിപ്പിക്കുകയോ ചുരുക്കുകയോ ഇല്ല.'(7)
- അനസ് ബ്നുമാലികില് (റ) നിന്ന്: നബി (സ) പറഞ്ഞു: 'പ്രതാപവാനും മഹാനുമായ അല്ലാഹു ഗര്ഭാശയത്തിന്റെ കാര്യം ഒരു മലക്കിനെ ഏല്പിച്ചിട്ടുണ്ട്. ആ മലക്ക് പറയും: രക്ഷിതാവേ, ബീജമാണ്. രക്ഷിതാവേ സിക്താണ്ഡമാണ്. രക്ഷിതാവേ മാംസപിണ്ഡമാണ്. അല്ലാഹു ഒരു സൃഷ്ടിയില് വിധിക്കാന് ഉദ്ദേശിച്ചാല് മലക്ക് പറയും: രക്ഷിതാവേ, ആണോ പെണ്ണോ? ദൗര്ഭാഗ്യവാനോ അതോ സൗഭാഗ്യവാനോ? ഉപജീവനം എങ്ങനെയാണ്? അവധി എത്രയാണ്? അങ്ങനെ അവയെല്ലാം തന്റെ മാതാവിന്റെ വയറ്റിലായിരിക്കെ തന്നെ രേഖപ്പെടുത്ത പ്പെടും.(8)
- (നബി(സ)യോട് ചോദിക്കപ്പെട്ടു:) സ്വപ്നസ്ഖലനമുണ്ടായാല് സ്ത്രീ കുളിക്കേണ്ടതുണ്ടോ? അപ്പോള് നബി(സ) പറഞ്ഞു: 'അതെ; അവള് ഇന്ദ്രിയം കണ്ടാല്'. അപ്പോള് ഉമ്മുസുലൈം (റ) ചോദിച്ചു: 'സ്ത്രീക്ക് സ്ഖലനമുണ്ടാകുമോ?' അപ്പോള് അദ്ദേഹം പറഞ്ഞു: 'എന്തൊരു കഷ്ടം! പിന്നെ? എങ്ങനെയാണ് കുട്ടിക്ക് അവളോട് സാദൃശ്യം ഉണ്ടാകുന്നത്?' മറ്റൊരു നിവേദനത്തില് ആഇശ (റ) ഉമ്മുസുലൈം(റ)യോട് 'ഛെ! സ്ത്രീക്ക് അതുണ്ടാകുമോ?' എന്ന് ചോദിച്ചുവെന്നാണുള്ളത്. മറ്റൊരു റിപ്പോര്ട്ടില്, ഈ ഹദീഥിന്റെ‚അവസാന ഭാഗത്ത് ഇങ്ങനെ കൂടിയുണ്ട്. 'ഇന്ദ്രിയം കാരണമായിട്ടാണ് കുട്ടിക്ക് സാദൃശ്യമുണ്ടാകുന്നത്. സ്ത്രീയുടെ ഇന്ദ്രിയം പുരുഷന്റെ ഇന്ദ്രിയത്തിന് മുകളില് വന്നാല് കുട്ടിക്ക് മാതൃ സഹോദരന്മാരോട് സാദൃശ്യമുണ്ടാകും. പുരുഷന്റെ‚ ഇന്ദ്രിയം സ്ത്രീയുടെ ഇന്ദ്രിയത്തിന് മുകളില് വന്നാല് കുട്ടിക്ക് അവന്റെ പിതൃവ്യന്മാരോട് സാദൃശ്യമുണ്ടാകും.'(9)
മുകളില് പറഞ്ഞ അല്ലാഹുവിന്റെയും അവന്റെ ദൂതന്റെയും വചനങ്ങളിലൊന്നും തന്നെ ആശാസ്ത്രീയമായ പരാമര്ശങ്ങളൊന്നുമില്ല. ലിംഗനിര്ണയത്തെപ്പറ്റിയുള്ള ഏറ്റവും പുതിയ വിവരങ്ങളുമായിപ്പോലും അവ പൂര്ണമായും യോജിച്ചു വരുന്നുവെന്നത് അത്ഭുതകരം തന്നെയാണ്.
- സ്രവിക്കപ്പെടുന്ന ശുക്ലത്തില് നിന്നാണ് ആണും പെണ്ണുമുണ്ടാകുന്നതെന്ന് സൂറത്തുല് ഖിയാമയിലെ 38 മുതല് 40 വരെയുള്ള വചനങ്ങ ളില് പറയുന്നു. ശുക്ലദ്രാവകത്തിലെ പുരുഷബീജം X ക്രോമസോം വഹിക്കുന്നതാണെങ്കില് അത് അണ്ഡവുമായി ചേര്ന്നാല് പെണ്കു ഞ്ഞും Y ക്രോമസോം വഹിക്കുന്നതാണെങ്കില് അത് അണ്ഡവുമായി ചേര്ന്നാല് ആണ്കുഞ്ഞുമുണ്ടാകുന്നു. ശുക്ലദ്രാവകമാണ് കുഞ്ഞ് ആണോ പെണ്ണോ എന്നു തീരുമാനിക്കുന്നത് എന്നര്ത്ഥം.
- സ്രവിക്കപ്പെടുന്ന ബീജത്തില് തന്നെ ആണ്, പെണ് എന്നിവയുണ്ടെന്നും അതാണ് ആണ്-പെണ് ഇണകളുടെ ഉല്പത്തിക്ക് കാരണമാകുന്ന തെന്നും സൂറത്തുന്നജ്മിലെ 45,46 വചനങ്ങള് വ്യക്തമാക്കുന്നു. സ്രവിക്കപ്പെടുന്ന ബീജത്തില് തന്നെ ആണ് ക്രോമസോമായ Yയെ വഹി ക്കുന്ന ബീജാണുക്കളും പെണ്ക്രോമസോമായ Xനെ വഹിക്കുന്ന ബീജാണുക്കുളുമുണ്ട്. ബീജദ്രാവകത്തിലെ Y ആണ്ബീജം അണ്ഡവു മായി ചേര്ന്നാല് ആണ്കുട്ടിയും X പെണ്ബീജമാണ് അണ്ഡവുമായി ചേരുന്നതെങ്കില് പെണ്കുട്ടിയുമാണുണ്ടാവുക.
- അനസില് നിന്ന് ബുഖാരി നിവേദനം ചെയ്ത അബ്ദുല്ലാഹിബ്നു സലാമുമായി പ്രവാചകന് (സ) നടത്തിയ സംഭാഷണത്തെക്കുറിച്ച് വിവരിക്കുന്ന ഹദീഥിലും അദ്ദേഹത്തില് നിന്നുതന്നെ മുസ്ലിം നിവേദനം ചെയ്ത സ്വപ്നസ്ഖലനത്തെക്കുറിച്ച ഹദീഥിലും ഥൗബാ നി(റ)ല് നിന്ന് മുസ്ലിം നിവേദനം ചെയ്ത ജൂതപുരോഹിതനു നല്കിയ മറുപടികയെക്കുറിച്ച് വിശദീകരിക്കുന്ന ഹദീഥിലും പുരുഷ ബീജം സ്ത്രീബീജത്തെ അതിജയിച്ചാല് ആണ്കുഞ്ഞും, സ്ത്രീബീജം പുരുഷബീജത്തെയാണ് അതിജയിക്കുന്നതെങ്കില് പെണ്കുട്ടിയുമാ ണുണ്ടാവുകയെന്ന് പ്രവാചകന് (സ) പറഞ്ഞതായി ഉദ്ധരിച്ചിരിക്കുന്നു. ഈ പരാമര്ശത്തെ സുരതക്രിയയില് പുരുഷനാണ് ആദ്യം സ്ഖലിക്കുന്നതെങ്കില് ആണ്കുട്ടിയും സ്ത്രീക്കാണ് ആദ്യം സ്ഖലിക്കുകയെങ്കില് പെണ്കുട്ടിയുമാണുണ്ടാവുകയെന്നാണ് പല പണ്ഡിത ന്മാരും വ്യാഖ്യാനിച്ചിരിക്കുന്നത്. രതിമൂര്ച്ചയോടനുബന്ധിച്ച് ചില സ്ത്രീകള്ക്ക് പാരായൂറിത്രല് നാളിയില് നിന്ന് പുറത്തേക്കുവ രുന്ന ദ്രാവകത്തിന് കുഞ്ഞിന്റെ ജനനത്തില് യാതൊരു പങ്കുമില്ല എന്ന് ഇന്ന് നമുക്കറിയാം. പെണ്ണിന്റെ സ്ഖലനത്തിന് കുഞ്ഞിന്റെ ഉല്പത്തി പ്രക്രിയയില് യാതൊരു പങ്കും വഹിക്കുവാനില്ലെങ്കില് അതോടനുബന്ധിച്ചുണ്ടാകുന്ന ദ്രാവകം ആദ്യമോ പിന്നെയോ ഉണ്ടാകുന്നതെന്നത് ലിംഗനിര്ണയത്തെ ബാധിക്കുവാന് സാധ്യതയൊന്നുമില്ല. ഈ ഹദീഥുകളില് ബീജത്തിന്റെ അധീശത്വത്തെക്കുറിക്കു വാന് പ്രയോഗിച്ചിരിക്കുന്നത് 'സബഖ'യെന്നും 'അലാ' എന്നുമുള്ള ക്രിയകളാണ്. ഒന്നിനുമേല് മറ്റൊന്ന് മുന്കടക്കുന്നതിനോ ആദ്യമാകു ന്നതിനോ വിജയിക്കുന്നതിനോ അധികാരം സ്ഥാപിക്കുന്നതിനോ ആണ് 'സബഖ'യെന്നു പറയുകയെന്ന് അംഗീകൃത ഭാഷാ നിഘണ്ടുക്കള് പരിശോധിച്ചാല് വ്യക്തമാകും(10).
ഒന്നിനുമുകളില് മറ്റൊന്ന് ആധിപത്യം സ്ഥാപിക്കുന്നതിനാണ് 'അലാ'യെന്ന് പ്രയോഗിക്കുകയെന്ന് ക്വുര്ആനില്നിന്നു തന്നെ വ്യക്തമാകു ന്നുണ്ട്. സൂറത്തുല് മുഅ്മിനൂനിലെ 91-ാം വചനം നോക്കുക.
''അല്ലാഹു യാതൊരു സന്താനത്തെയും സ്വീകരിച്ചിട്ടില്ല. അവനോടൊപ്പം യാതൊരു ദൈവവുമുണ്ടായിട്ടില്ല. അങ്ങനെയായിരുന്നുവെങ്കില് ഓരോ ദൈവവും താന് സൃഷ്ടിച്ചതുമായി പോയിക്കളയുകയും, അവരില് ചിലര് ചിലരെ അടിച്ചമര്ത്തുകയും ചെയ്യുമായിരുന്നു. അവര് പറഞ്ഞുണ്ടാക്കുന്നതില് നിന്നെല്ലാം അല്ലാഹു എത്ര പരിശുദ്ധന്!'' (23: 91)(11)
ഈ വചനത്തില് 'ചിലര് ചിലരെ അടിച്ചമര്ത്തുകയും ചെയ്യുമായിരുന്നു'വെന്നു പരിഭാഷപ്പെടുത്തിയിരിക്കുന്നത് 'വ ലഅലാ ബഅദുഹും അലാ ബഅദിന്' എന്ന പ്രയോഗത്തെയാണ്. 'അലാ'യെന്നാല് ആധിപത്യം സ്ഥാപിക്കുക, അടിച്ചമര്ത്തുക എന്നിങ്ങനെയാണ് യഥാര്ത്ഥത്തിലുള്ള സാരമെന്നര്ത്ഥം.
പുരുഷബീജത്തിലെ Y പെണ്ബീജത്തിലെ Xനുമേല് ആധിപത്യം സ്ഥാപിക്കുമ്പോഴാണ് ആണ്കുഞ്ഞുണ്ടാകുന്നത് എന്നും പെണ്ബീജത്തിലെ X പുരുഷബീജത്തിലെ Yക്കുമേല് ആധിപത്യം സ്ഥാപിക്കുമ്പോഴാണ് പെണ്കുഞ്ഞുണ്ടാകുന്നത് എന്നുമുള്ള ജനിതകശാസ്ത്ര വസ്തുതകളു മായി ഈ ഹദീഥുകള് പൂര്ണമായും പൊരുത്തപ്പെടുന്നു. മധ്യകാലഘട്ടത്തിലുണ്ടായിരുന്ന അറിവിന്റെ അടിസ്ഥാനത്തില് ഹദീഥ് മനസ്സിലാക്കിയവര് ആണ്സ്ഖലനം ആദ്യം നടന്നാല് ആണ്കുഞ്ഞും പെണ്സ്ഖലനം നടന്നാല് പെണ്കുഞ്ഞുമുണ്ടാകുമെന്ന് ഇതില്നിന്ന് മനസ്സിലാക്കിയെന്നത് നബിവചനത്തിന്റെ ആശാസ്ത്രീയതയല്ല, അറിവിന്റെ കാലനിബന്ധതയെയാണ് വെളിപ്പെടുത്തുന്നത്. 'സബഖ' യെന്ന ക്രിയയെ വ്യാഖ്യാനിച്ചാല് ആദ്യമുണ്ടാകുന്നത് ഏത് ദ്രവമാണോ അതിന്റെ അടിസ്ഥാനത്തിലാണ് ലിംഗനിര്ണയമെന്ന് വേണമെ ങ്കില് പറയാനാകുമെങ്കിലും 'അലാ'യെന്ന പ്രയോഗം അത്തരമൊരു വ്യാഖ്യാനത്തിന് പഴുതുകളൊന്നും നല്കുന്നില്ല. ഈ ഹദീഥുകളെ ഒന്നിച്ചു പരിഗണിച്ചുകൊണ്ട്, നിലനില്ക്കുന്ന അറിവിന്റെ അടിസ്ഥാനത്തില് വ്യാഖ്യാനിച്ചപ്പോഴാണ് പുരുഷ-പെണ് സ്ഖലനങ്ങളുടെ ക്രമമാണ് ലിംഗനിര്ണയത്തിന് നിദാനമെന്നാണ് ഈ ഹദീഥുകള് പഠിപ്പിക്കുന്നതെന്ന നിഗമനത്തില് വ്യാഖ്യാതാക്കള് എത്തിച്ചേര്ന്നത്. ഹദീഥുകളെ മൊത്തത്തിലെടുത്ത് പരിശോധിച്ചാല് ഒരു ദ്രവത്തിനു മേലുള്ള മറ്റേ ദ്രവത്തിന്റെ ആധിപത്യം തന്നെയാണ് അവയില് വ്യക്തമാക്കപ്പെട്ടിരിക്കുന്നതെന്ന് മനസ്സിലാകും. കഴിഞ്ഞ നൂറ്റാണ്ടിന്റെ പകുതിയില് മാത്രം ശാസ്ത്രലോകത്തിന് മനസ്സിലായ ബീജത്തി ന്റെ ആധിപത്യമാണ് ലിംഗനിര്ണയത്തിന് കാരണമാകുന്നതെന്ന വസ്തുത എത്ര കൃത്യമായാണ് ഈ ഹദീഥുകള് വരച്ച് കാണിക്കുന്നത്!
- മുസ്ലിം ഹുദൈഫത്തു ബ്നു അസീദില് (റ) നിന്നും അബ്ദുല്ലാഹിബ്നു മസ്ഊദില് (റ) നിന്നും നിവേദനം ചെയ്ത രണ്ട് വ്യത്യസ്ത ഹദീഥുകളില് നിന്ന് ഗര്ഭസ്ഥശിശുവിലുളള ലിംഗമാറ്റത്തിനുവേണ്ടിയുള്ള മലക്ക് പ്രത്യക്ഷപ്പെടുന്നതും കുട്ടി ആണോ പെണ്ണോയെന്ന് ആത്യന്തികമായി തീരുമാനിക്കപ്പെടുന്നതും ബീജസങ്കലനത്തിന് ശേഷം നാല്പത് ദിവസങ്ങള്ക്കും നാല്പത്തിയഞ്ച് ദിവസങ്ങള്ക്കുമിടയിലാണെന്ന് വ്യക്തമാവുന്നു.
SRY ജീന് പ്രവര്ത്തനക്ഷമമാകുന്നത് ആറാമത്തെ ആഴ്ചയാണെന്ന വിവരം നമുക്ക് ലഭിച്ചത് മൂന്നു പതിറ്റാണ്ടുകള്ക്ക് മുമ്പു മാത്രമാണ്. XX സിക്താണ്ഡമാണെങ്കിലും XY സിക്താണ്ഡമാണെങ്കിലും അപൂര്വമായുണ്ടാകുന്ന സിക്താണ്ഡങ്ങളാണെങ്കിലുമെല്ലാം അവയുടെ ലിംഗമെന്താ ണെന്ന് ആത്യന്തികമായി തീരുമാനിക്കപ്പെടുക SRY ജീന് പ്രവര്ത്തനക്ഷമമാകുന്നതിന്റെ അടിസ്ഥാനത്തിലാണ്. ആറാമത്തെ ആഴ്ചയാണ് SRY ജീന് പ്രവര്ത്തനക്ഷമമാവുന്നതെന്ന ഭ്രൂണശാസ്ത്രം 1985ല് മാത്രം നമുക്കു പറഞ്ഞുതന്ന വിവരവും നാല്പതു ദിവസങ്ങള്ക്കും നല്പത്തിയഞ്ച് ദിവസങ്ങള്ക്കുമിടയിലാണ് ലിംഗതീരുമാനവുമായി മലക്ക് നിയോഗിക്കപ്പെടുന്നതെന്ന പതിനാലു നൂറ്റാണ്ടുകള്ക്ക് മുമ്പ് നബി (സ) പറഞ്ഞ വിവരവും എത്ര ക്രൃത്യമായാണ് ഇവിടെ യോജിച്ചുവരുന്നത്! എന്തുകൊണ്ടാണ് ഹദീഥുകളിലെ പരാമര്ശങ്ങള് ഇത്ര യും കൃത്യമാകുന്നതെന്ന ചോദ്യത്തിന് ക്വുര്ആന് തന്നെ ഉത്തരം നല്കിയിട്ടുണ്ട്.
''നിങ്ങളുടെ കൂട്ടുകാരന് വഴിതെറ്റിയിട്ടില്ല. ദുര്മാര്ഗിയായിട്ടുമില്ല. അദ്ദേഹം തന്നിഷ്ടപ്രകാരം സംസാരിക്കുന്നുമില്ല. അത് അദ്ദേഹത്തിന് ദിവ്യസമ്പേശമായി നല്കപ്പെടുന്ന ഒരു ഉല്ബോധനം മാത്രമാകുന്നു.'' (53: 2-4)
കുറിപ്പുകൾ
- ക്വുര്ആന് 53: 45-46
- ക്വുര്ആന് 75: 38-40
- സ്വഹീഹുല് ബുഖാരി, കിതാബു അഹാദീഥുല് അംബിയാഅ്, ബാബു ഖല്ഖി ആദം വ ദുര്റിയ്യത്തിഹി, ഹദീഥ്
- സ്വഹീഹു മുസ്ലിം, കിതാബുല് ഹൈദ്വ്, ബാബു വുജുബില് ഗസ്ലി അലല് മര്അത്തി ബി ഖുറൂജില് മനിയ്യി മിന്ഹ, ഹദീഥ്
- സ്വഹീഹു മുസ്ലിം, കിതാബുല് ഹൈദ്വ്, ബാബു ബയാനി സ്വിഫത്തില് മനിയിര്റജുലി വല് മര്അത്തി വ അന്നല് വലദ മഖ്ലൂഖുന് മിന് മാഇ.
- സ്വഹീഹു മുസ്ലിം, കിതാബുല് ക്വദ്ര്, ബാബു കൈഫിയ്യത്തില് ഖല്ബില് ആദമിയ്യി ഫീ ബദനി ഉമ്മിഹി വ കിതാബത്തി രിസ്കിഹി വ അജലിഹി, വ അമലിഹി വ ശകാവത്തിഹി വ സഅദത്തിഹി, ഹദീഥ്
- സ്വഹീഹു മുസ്ലിം, കിതാബുല് ക്വദ്ര്, ബാബു കൈഫിയ്യത്തില് ഖല്ബില് ആദമിയ്യി ഫീ ബദനി ഉമ്മിഹി വ കിതാബത്തി രിസ്കിഹി വ അജലിഹി, വ അമലിഹി വ ശകാവത്തിഹി വ സഅദത്തിഹി, ഹദീഥ്
- സ്വഹീഹു മുസ്ലിം, കിതാബുല് ക്വദ്ര്, ബാബു കൈഫിയ്യത്തില് ഖല്ബില് ആദമിയ്യി ഫീ ബദനി ഉമ്മിഹി വ കിതാബത്തി രിസ്കിഹി വ അജലിഹി, വ അമലിഹി വ ശകാവത്തിഹി വ സഅദത്തിഹി, ഹദീഥ്
- സ്വഹീഹു മുസ്ലിം, കിതാബുല് ഹൈദ്വ്, ബാബു വുജുബില് ഗസ്ലി അലല് മര്അത്തി ബി ഖുറൂജില് മനിയ്യി മിന്ഹ, ഹദീഥ്
- Edward William Lane : Arabic-English Lexicon, London, 1863, Book 1, Page 1300.
- ക്വുര്ആന് 23: 91
ഭ്രൂണവളര്ച്ചയുടെ ആദ്യത്തെ നാല്പതു ദിവസങ്ങള് തികച്ചും ജൈവികവും യാന്ത്രികവുമായ മൂന്നു ഘട്ടങ്ങളുടേതാണെന്നും അതിനുശേ ഷമാണ് അല്ലാഹുവിന്റെ അനുഗ്രഹങ്ങളുമായി മലക്കുകളുടെ നിയോഗമുണ്ടാവുകയും വൈയക്തികമായ സവിശേഷതകള് തീരുമാനിക്ക പ്പെടുകയും ചെയ്യുന്നതെന്ന് വ്യക്തമാക്കുന്ന നിരവധി ഹദീഥുകളുണ്ട്.
- അബ്ദുല്ലാഹിബ്നു മസ്ഊദില് (റ) നിന്ന്: സത്യസന്ധനും വിശ്വസ്തനുമായ അല്ലാഹുവിന്റെ ദൂതന് ഞങ്ങളെ അറിയിച്ചു: നിങ്ങളിലുള്ള ഒരാളുടെ സൃഷ്ടികര്മം തന്റെ മാതാവിന്റെ ഉദരത്തില് സംയോജിക്കുന്നത് നാല്പതു ദിവസങ്ങളിലായാണ്. പിന്നെ, അതേപോലെത്ത ന്നെ അത് അലഖയായിത്തീരുന്നു. അതേപോലെ പിന്നീടത് മുദ്വ്അയായി മാറുന്നു. അനന്തരം അല്ലാഹു ഒരു മലക്കിനെ അയക്കുകയും നാലുകാര്യങ്ങള് രേഖപ്പെടുത്താന് കല്പിക്കുകയും ചെയ്യുന്നു. അവന്റെ കര്മവും ആഹാരവും ആയുസ്സും സൗഭാഗ്യവാനോ നിര്ഭാഗ്യവാനോ എന്നീ കാര്യങ്ങള്. ശേഷം അവനിലേക്ക് ആത്മാവ് ഊതപ്പെടുന്നു.(1)
- ഹുദൈഫത്തുബ്നു ഉസൈദ് (റ)യില് നിന്ന്: നുത്വ്ഫ ഗര്ഭാശയത്തിലെത്തി നാല്പതോ നാല്പത്തിയഞ്ചോ ദിവസങ്ങള് കഴിഞ്ഞാല് അതിന്മേല് ഒരു മലക്ക് എത്തുകയും ഇങ്ങനെ ചോദിക്കുകയും ചെയ്യുന്നു. 'നാഥാ, സൗഭാഗ്യവാനോ നിര്ഭാഗ്യവാനോ?' അത് രേഖപ്പെ ടുത്തുന്നു. അവന്റെ കര്മങ്ങളും പ്രവൃത്തികളും അന്ത്യവും ആഹാരവുമെല്ലാം രേഖപ്പെടുത്തുന്നു. പിന്നെ ആ രേഖ ചുരുട്ടപ്പെടുന്നു. ശേഷം അതില് കൂട്ടിച്ചര്ക്കലുകളോ കിഴിക്കലുകളോ ഇല്ല.(2)
- അബ്ദുല്ലാഹിബ്നു മസ്ഊദില് നിന്ന്: അല്ലാഹുവിന്റെ ദൂതന് ഇങ്ങനെ പറയുന്നത് താന് കേട്ടതായി ഹുദൈഫത്തുബ്നു ഉസൈദുല് ഗിഫാരി (റ) പറഞ്ഞു: നുത്വ്ഫയെത്തി നാല്പത്തിരണ്ടു ദിവസങ്ങള് കഴിഞ്ഞാല് അല്ലാഹു ഒരു മലക്കിനെ അയക്കുകയും അവന് ആകൃതി നല്കുകയും ചെയ്യുന്നു. ശേഷം അവന് കേള്വിശക്തിയും കാഴ്ചശക്തിയും ത്വക്കും മാംസപേശികളും അസ്ഥികളുമെല്ലാം നല്കുന്നു. അങ്ങനെ മലക്ക് ചോദിക്കുന്നു: നാഥാ, പുരുഷനോ സ്ത്രീയോ? നിന്റെ നാഥന് ഉദ്ദേശിക്കുന്നത് തീരുമാനിക്കുകയും മലക്ക് അത് രേഖപ്പെടുത്തുകയും ചെയ്യുന്നു. പിന്നെ മലക്ക് ചോദിക്കുന്നു: നാഥാ, അവന്റെ അന്ത്യമെന്നാണ്? നിന്റെ നാഥന് ഉദ്ദേശിക്കുന്നത് പറയുകയും അത് രേഖപ്പെടുത്തുകയും ചെയ്യുന്നു. പിന്നെ മലക്ക് ചോദിക്കുന്നു: അവന്റെ ഉപജീവനമെങ്ങനെയാണ്? നിന്റെ നാഥന് ഇച്ഛിക്കുന്ന പോലെ തീരുമാനിക്കുകയും മലക്ക് അത് രേഖപ്പെടുത്തുകയും ചെയ്യുന്നു. ശേഷം മലക്ക് തന്റെ കയ്യില് ചുരുട്ടിയ രേഖയു മായി പോകുന്നു. പിന്നെ അതില് കൂട്ടിച്ചേര്ക്കലുകളോ കിഴിക്കലുകളോ ഇല്ല.(3)
- അബൂ തുഫൈലി(റ)ല് നിന്ന്: ഞാന് ഹുദൈഫത്തുബ്നു ഉസൈദ് അല് ഗിഫാരി(റ)യെ സന്ദര്ശിച്ചപ്പോള് അദ്ദേഹം പറഞ്ഞു: അല്ലാഹുവി ന്റെ ദൂതന് (സ) ഇങ്ങനെ പറയുന്നതായി ഞാനെന്റെ രണ്ടു ചെവികള് കൊണ്ടു കേട്ടിട്ടുണ്ട്. നുത്വ്ഫ ഗര്ഭാശയത്തിലെത്തി നാല്പതു രാവുകള് കഴിഞ്ഞാല് പിന്നെ ഒരു മലക്കെത്തി അതിന് രൂപം നല്കും. സുഹൈര് (റ) പറഞ്ഞു: അദ്ദേഹം ഇങ്ങനെകൂടി പറഞ്ഞുവെന്ന് ഞാന് കരുതുന്നു: കണക്കാക്കുകയും രൂപപ്പെടുത്തുകയും ചെയ്യുന്നവനോട് അവന് ചോദിക്കും: നാഥാ, സ്ത്രീയോ പുരുഷനോ? അങ്ങനെ അല്ലാഹു അതിനെ ആണോ പെണ്ണോ ആക്കിത്തീര്ക്കുന്നു. പിന്നെ അവന് ചോദിക്കുന്നു: നാഥാ, വൈകല്യമുള്ളവനോ ഇല്ലാത്ത വനോ? അങ്ങനെ അല്ലാഹു അവനെ പൂര്ണനോ വൈകല്യമുള്ളവനോ ആക്കുന്നു. പിന്നെ അവന് ചോദിക്കുന്നു: നാഥാ, അവന്റെ ഉപജീ വനമെന്താണ്? അന്ത്യമെന്നാണ്? പ്രകൃതമെന്താണ്? അങ്ങനെ അവന് സൗഭാഗ്യവാനോ നിര്ഭാഗ്യവാനോയെന്ന് അല്ലാഹു തീരുമാനി ക്കുന്നു.(4)
നുത്വ്ഫയുണ്ടായി നാല്പതുരാവുകള്ക്കു ശേഷമാണ് ഒരാളുടെ അവയവങ്ങള് രൂപീകരിക്കപ്പെടുകയും വ്യക്തിത്വം തീരുമാനിക്കപ്പെ ടുകയും ചെയ്യുന്നതെന്ന് വ്യക്തമാക്കുന്നതാണ് ഈ ഹദീഥുകള്. നാല്പത് ദിവസങ്ങള് കഴിഞ്ഞശേഷം സംഭവിക്കുന്നതായി ഈ ഹദീഥുകള് വ്യക്തമാക്കുന്ന കാര്യങ്ങള് താഴെ പറയുന്നവയാണ്:-
- ലിംഗ നിര്ണയം
- വ്യക്തിത്വ നിര്ണയം
- അവയവ രൂപീകരണം
- വൈകല്യങ്ങളുണ്ടെങ്കില് അവയുടെ പ്രകടനം
- വിധി നിര്ണയത്തിന്റെ രേഖീകരണം
മുദ്വ്അഃയെന്നു ക്വുര്ആന് വിളിക്കുന്ന ചര്വ്വിതമാംസത്തിന്റെ പൂര്ണരൂപം പ്രാപിക്കുന്ന ആറാമത്തെ ആഴ്ചയുടെ അവസാനത്തിലുള്ള ഭ്രൂണത്തെയും അതിന്റെ രൂപത്തെയും അതിലുള്ള അവയവാടയാളങ്ങളെയും മുകുളങ്ങളെയും കുറിച്ചെല്ലാം നാം മനസ്സിലാക്കിക്കഴിഞ്ഞ താണ്. ആറാം ആഴ്ചക്കു മുമ്പുനടക്കുന്ന മൂന്ന് ഘട്ടങ്ങളും ക്രമപ്രവൃദ്ധമായി സംഭവിക്കുന്നതാണെന്നും നാം കണ്ടു. ഈ ഘട്ടങ്ങളുടെ നൈര ന്തര്യം സൂചിപ്പിക്കുന്ന ഒരു ഹദീഥുമുണ്ട്.
അനസുബ്നു മാലിക് (റ)യില് നിന്ന്: അത്യുന്നതനായ അല്ലാഹു ഗര്ഭാശയത്തിന്റെ സംരക്ഷകനായി ഒരു മലക്കിനെ നിയോഗിക്കും. അപ്പോള് മലക്ക് പറയും: നാഥാ, നുത്വ്ഫ; നാഥാ, അലഖഃ; മുദ്വ്അഃ; അതിന്റെ സൃഷ്ടി പൂര്ത്തിയാക്കുവാന് അല്ലാഹു ഉദ്ദേശിച്ചിട്ടുണ്ടെ ങ്കില് മലക്ക് ചോദിക്കും: നാഥാ, ആണോ അതല്ല പെണ്ണോ? സൗഭാഗ്യവാനോ അതല്ല നിര്ഭാഗ്യവാനോ? ഉപജീവനമെന്താണ്? അന്ത്യമെങ്ങനെയാണ്? മാതാവിന്റെ ഗര്ഭാശയത്തില് വെച്ചുതന്നെ അതെല്ലാം എഴുതപ്പെടും.(5)
ഈ ഹദീഥുകളില് നിന്നെല്ലാം തന്നെ നുത്വ്ഫ, അലഖഃ; മുദ്വ്അഃ എന്നിങ്ങനെ ക്വുര്ആന് പരാമര്്വശിച്ച മൂന്ന് ഭ്രൂണഘട്ടങ്ങളും നടക്കു ന്നത് ആദ്യത്തെ ആറ് ആഴ്ചകളിലാണെന്നു വ്യക്തമാണ്. എന്നാല് മുകളില് ആദ്യമായി ഉദ്ധരിച്ചിരിക്കുന്ന ബുഖാരിയും മുസ്ലിമും അബ്ദുല്ലാഹിബ്നു മസ്ഊദില് (റ) നിന്ന് നിവേദനം ചെയ്തിരിക്കുന്ന ഹദീഥില് നിന്ന് ഭ്രൂണത്തിന്റെ നുത്വ്ഫ ഘട്ടവും അലഖ ഘട്ടവും മുദ്വ്അ ഘട്ടവും നാല്പത് ദിവസം വീതം ദിവസങ്ങളുള്ക്കൊള്ളുന്നതാണെന്ന ഒരു ധാരണ ആദ്യകാല ഹദീഥ് പണ്ഡിതന്മാര് മുതല് തന്നെ വെച്ചുപുലര്ത്തിയിരുന്നതായി നമുക്ക് കാണാന് കഴിയും. പ്രസ്തുത ധാരണ എടുത്തുദ്ധരിച്ചുകൊണ്ട് ക്വുര്ആനിലും ഹദീഥുകളിലും പ്രതിപാദിച്ചിട്ടുള്ള നുത്വ്ഫ, അലഖ, മുദ്വ്അ പരാമര്ശങ്ങളെല്ലാം അശാസ്ത്രീയവും അബദ്ധജഡിലവുമാണെന്ന് വിമര്ശിക്കുന്നവരുണ്ട്. ഭ്രൂണഘടനയെക്കുറിച്ച് പരാമര്ശിക്കുന്ന ക്വുര്ആന് വചനങ്ങളും മുഴുവന് ഹദീഥുകളും താരതമ്യം ചെയ്തുകൊണ്ട് പരിശോധിച്ചാല് ഈ വിമര്ശനങ്ങളിലൊന്നും യാതൊരുവിധ കഴമ്പുമില്ലെന്നു മനസ്സിലാകും. താഴെ പറയുന്ന വസ്തുതകള് ശ്രദ്ധിക്കുക.
- ഹദീഥുകള് പ്രവാചകനില് നിന്നുള്ളതാണെന്ന് ഉറപ്പാണെങ്കില് അതിലെ ആശയങ്ങള് അല്ലാഹുവില് നിന്നുള്ളതും അതുകൊണ്ടു തന്നെ അമാനുഷികവുമായിരിക്കും. എന്നാല് ക്വുര്ആനിലെ പദങ്ങള്ക്കുള്ളതുപോലെ ഹദീഥുകളുടെ പദങ്ങള്ക്ക് അമാനുഷികതയൊന്നുമില്ല. അല്ലാഹുവില് നിന്നുള്ള ബോധനത്തിന്റെ വെളിച്ചത്തില് പ്രവാചകന് (സ) പറഞ്ഞ കാര്യങ്ങള് അതുകേട്ട സ്വഹാബിമാര് അവരുടെ ഭാഷയില് ആവിഷ്കരിച്ചതാണ് ഹദീഥുകളിലെ പ്രവാചകവചനങ്ങള്. മാറ്റാന് പാടില്ലെന്ന് ദൈവദൂതരാല് നിഷ്കര്ഷിക്കപ്പെട്ട പ്രാര് ത്ഥനകളെപ്പോലെയുള്ളവ യാതൊരുവിധ മാറ്റവുമില്ലാതെ നിവേദനം ചെയ്ത സ്വഹാബിമാര് തന്നെ പ്രവാചകനില് (സ) നിന്ന് ലഭിച്ച ആശയങ്ങള് തങ്ങളുടേതായ ഭാഷയില് മറ്റുള്ളവര്ക്ക് പറഞ്ഞുകൊടുത്തതായി കാണാന് കഴിയും. പ്രവാചകന് ഉപയോഗിച്ച പദങ്ങ ള്ക്ക് അമാനുഷികതയില്ലെന്നും അതിലെ ആശയങ്ങള് മാത്രമാണ് ദൈവപ്രോക്തമെന്നുമായിരുന്നു അവര് മനസ്സിലാക്കിയിരുന്നതെന്ന് ഇതില് നിന്ന് വ്യക്തമാണ്. പ്രവാചകനില് (സ) നിന്നുകേട്ട ഒരേകാര്യം തന്നെ വ്യത്യസ്ത സ്വഹാബിമാര് വ്യത്യസ്ത പദങ്ങളും ശൈലിയു മുപയോഗിച്ച് അടുത്ത തലമുറക്ക് പറഞ്ഞുകൊടുത്തതിന് നിരവധി ഉദാഹരണങ്ങളുണ്ട്. ഹദീഥുകളില് ഉപയോഗിക്കപ്പെട്ടിരിക്കുന്ന പദങ്ങള് പലപ്പോഴും സ്വഹാബിമാരുടേതായിരിക്കുമെന്നും അവയുള്ക്കൊള്ളുന്ന ആശയം മാത്രമാണ് ദൈവികമെന്നുമാണ് ഇസ്ലാം പഠിപ്പിക്കുന്നതെന്ന വസ്തുത മനസ്സിലാക്കാതെയാണ് ഇതടക്കമുള്ള പല വിമര്ശനങ്ങളും ഉന്നയിക്കപ്പെടുന്നത്.
- നുത്വ്ഫ, അലഖ, മുദ്വ്അ തുടങ്ങിയ ഘട്ടങ്ങളിലോരോന്നും നാല്പത് ദിവസം വീതം ഉള്ക്കൊള്ളുന്നുവെന്ന് സ്വഹീഹുല് ബുഖാരി, സ്വഹീഹുമുസ്ലിം, സുനനു അബൂദാവൂദ്, ജാമിഉത്തിര്മിദി, സുനനു ഇബ്നു മാജ, മുസ്നദ് അഹ്മദ് തുടങ്ങിയ ഹദീഥ് ഗ്രന്ഥങ്ങളിലൊ ന്നും തന്നെയില്ല. പ്രസ്തുത ഗ്രന്ഥങ്ങളിലെല്ലാം അബ്ദുല്ലാഹിബ്നു മസ്ഊദില് (റ) നിന്ന് നിവേദനം ചെയ്യുന്ന ഹദീഥ് തുടങ്ങുന്നത് ഇങ്ങ നെയാണ്: 'നിങ്ങളിലുള്ള ഒരാളുടെ സൃഷ്ടികര്മം തന്റെ മാതാവിന്റെ ഉദരത്തില് സംയോജിക്കുന്നത് നാല്പതു ദിവസങ്ങളിലായാണ്. പിന്നെ അതേപോലെ അത് അലഖയാകുന്നു; പിന്നെ അതേപോലെ അത് മുദ്വ്അയാകുന്നു. ഇവിടെ നാല്പതു ദിവസത്തില് നടക്കുമെന്ന് പറഞ്ഞത് മാതൃശരീരത്തിലെ സംയോജനമാണ് (ജംഉല് ഖല്ഖ്); അത് നുത്വ്ഫയല്ല. ജംഉല് ഖല്ഖ് എന്നുപറഞ്ഞത് നുത്വ്ഫയെ ഉദ്ദേശി ച്ചുകൊണ്ടാണെന്ന തെറ്റിദ്ധാരണ കൊണ്ടാണ് ഈ ഹദീഥിന്റെ അടിസ്ഥാനത്തില് നുത്വ്ഫ, അലഖ, മുദ്വ്അ എന്നീ മൂന്നുഘട്ടങ്ങള്ക്കും നാല്പതുവീതം ദിവസങ്ങളാണെന്ന നിഗമനത്തില് ഹദീഥ് പണ്ഡിതന്മാരില് ചിലര് എത്തിച്ചേര്ന്നത്.
- സ്വഹീഹുല് ബുഖാരിയിലും സ്വഹീഹുല് മുസ്ലിമിലും നിരവധി തവണ ഉദ്ധരിക്കപ്പെട്ടതാണ് ഈ ഹദീഥ്. സ്വഹീഹുല് ബുഖാരിയില് തന്നെ കിതാബു ബദ്ഉല് ഖല്ഖ്, കിതാബു അഹാദീഥുല് അന്ബിയാഅ്, കിതാബുല് ഖദ്റ്, കിതാബുത്തൗഹീദ് എന്നീ നാല് അധ്യായങ്ങ ളില് ഈ ഹദീഥ് ഉദ്ധരിക്കപ്പെട്ടിട്ടുണ്ട്. സ്വഹീഹു മുസ്ലിമിലെ കിതാബുല് ഖദ്റില് ഉദ്ധരിക്കപ്പെട്ട ഇതേ ഹദീഥിന് ഇമാം നവവി നല്കു ന്ന വ്യാഖ്യാനത്തിലാണ് നാല്പത്, നാല്പത്തിരണ്ട്, നാല്പത്തിയഞ്ച് ദിവസങ്ങള് കഴിഞ്ഞാല് നുത്വ്ഫക്കുമേല് മലക്ക് വരുമെന്ന് പ്രസ്താവിക്കുന്ന നടേ ഉദ്ധരിക്കപ്പെട്ട രണ്ടു മുതല് നാലുവരെയുള്ള ഹദീഥുകളിലെ ആശയങ്ങളെയും ഈ ഹദീഥിലെ ആശയത്തെയും കൂട്ടിയോജിപ്പിച്ചുകൊണ്ട് ആദ്യത്തെ നാല്പതുദിവസത്തില് നടക്കുമെന്ന് പറഞ്ഞ ജംഉല് ഖല്ഖ് കൊണ്ടു വിവക്ഷിക്കുന്നത് നുത്വ്ഫ യാണെന്നും നാല്പതു ദിവസങ്ങള്കൂടി കഴിഞ്ഞ് അലഖയും വീണ്ടും നാല്പതു ദിവസങ്ങള്കൂടി കഴിഞ്ഞ് മുദ്വ്അയുമുണ്ടാവുകയാണ് ചെയ്യുന്നതെന്നും അതുകൊണ്ടുതന്നെ ആത്മാവ് ഊതുന്നത് നാല് മാസങ്ങള്ക്കുശേഷമാണ് എന്നുമുള്ള അഭിപ്രായങ്ങളുന്നയിക്കുന്നത്.(6) ഈ അഭിപ്രായം പ്രവാചകന്റേതായി നിവേദനം ചെയ്യപ്പെട്ടതല്ല എന്നതുകൊണ്ടുതന്നെ അത് നിര്ബന്ധമായും സ്വീകരിക്കേണ്ടതായി മുസ്ലിംകളാരും കരുതുന്നില്ല. ഗര്ഭാശയത്തിലെന്ത് നടക്കുന്നുവെന്നറിയാന് ശാസ്ത്രീയമായ മാര്ഗങ്ങളൊന്നുമില്ലാതിരുന്ന കാലത്ത് ഹദീഥിലെ പദങ്ങളുടെ വിവക്ഷയെക്കുറിച്ച് നടത്തിയ ഊഹങ്ങള് മാത്രമാണ് ഇമാം നവവിയുടേത്. പ്രസ്തുത ഊഹം തെറ്റാണെന്ന് ശാസ്ത്രീയമായ പഠനങ്ങളിലൂടെ മനസ്സിലായാല് അത് സ്വീകരിക്കുവാന് മുഹമ്മദ് നബി(സ)യെ പിന്പറ്റുന്ന മുസ്ലിംകള്ക്ക് യാതൊരു വൈമനസ്യവുമുണ്ടാവില്ല.
- ഭ്രൂണഘട്ടങ്ങളുടെ നാല്പത് ദിവസത്തെക്കുറിച്ച് പറയുന്ന മുകളിലെ ഹദീഥുകളും ഭ്രൂണപരിണാമത്തിന്റെ ഘട്ടങ്ങളെ പ്രതിപാദിക്കുന്ന ക്വുര്ആന് ആയത്തുകളും മുന്നില്വെച്ചുകൊണ്ട് പരിശോധിച്ചാല് നുത്വ്ഫ മുതല് മുദ്വ്അ വരെയുള്ള മൂന്നുഘട്ടങ്ങളും നാല്പതു ദിവസത്തിനകം പൂര്ണമാകുന്നതാണ് എന്നു മനസ്സിലാകും.
- മുദ്വ്അയില് നിന്നാണ് ഇദ്വാം അഥവാ അസ്ഥികള് ഉണ്ടാകുന്നതെന്നാണ് ക്വുര്ആന് പറയുന്നത് (23:14). നാല്പത്തിരണ്ട് ദിവസങ്ങള് ക്കുശേഷമാണ് ഭ്രൂണത്തിന് അസ്ഥിയുണ്ടാകുന്നതെന്ന് ഹുദൈഫ(റ)യില് നിന്ന് അബ്ദുല്ലാഹിബ്നു മസ്ഊദ് (റ) നിവേദനം ചെയ്തതായി സ്വഹീഹു മുസ്ലിമിലുള്ള ഹദീഥിലുണ്ട് (മുകളിലത്തെ മൂന്നാമത്തെ ഹദീഥ്). ഈ ഹദീഥ് മുഹദ്ദിസുകളായ അബൂദാവൂദും ത്വബ്റാനി യും തങ്ങളുടെ ഹദീഥ് ഗ്രന്ഥങ്ങളില് ഉദ്ധരിച്ചിട്ടുമുണ്ട്. നുത്വ്ഫ മുതല് മുദ്വ്അ വരെയുള്ള ഓരോ ഘട്ടങ്ങള്ക്കും നാല്പതു ദിവസങ്ങള് വീതമുണ്ടെങ്കില് ക്വുര്ആന് വചനപ്രകാരം നാലുമാസങ്ങള്ക്കുശേഷമാണ് അസ്ഥിയുണ്ടാവുക. ഈ ഹദീഥിലാണെങ്കില് നാല്പത്തി രണ്ടു രാത്രികള്ക്കുശേഷം അസ്ഥികളുണ്ടാവാന് തുടങ്ങുന്നുവെന്നാണുള്ളത്. നാല്പത്തിരണ്ടു ദിവസങ്ങള്ക്കു മുമ്പ് മുദ്വ്അയെന്ന ഘട്ടം കഴിഞ്ഞുപോയിയെന്നാണ് ഇതിനര്ത്ഥം. ഈ ഹദീഥും ആയത്തും യോജിപ്പിച്ചാല് ഇക്കാര്യം വ്യക്തമായി മനസ്സിലാകും.
- മുദ്വ്അയുണ്ടായതിനു ശേഷമാണ് അവയവ രൂപീകരണങ്ങളെല്ലാം നടക്കുന്നതെന്ന് മുകളിലെ ഹദീഥുകള് വ്യക്തമാക്കുന്നു. മുദ്വ്അയാ യിത്തീര്ന്ന് നാല്പതു ദിവസം കഴിഞ്ഞാണ് അവയവ രൂപീകരണം നടക്കുന്നതെന്നു പറഞ്ഞാല് നാലു മാസങ്ങള്ക്കുശേഷമാണ് അവയ വ രൂപീകരണം നടക്കുന്നതെന്നാണ് അതിനര്ത്ഥം. മൂന്നുമാസം പ്രായമായ സന്ദര്ഭത്തില് ഗര്ഭം അലസിപ്പോകുമ്പോള് ആ പ്രായത്തി ലുള്ള കുഞ്ഞുങ്ങള്ക്ക് ഒരുവിധം എല്ലാ ബാഹ്യാവയവങ്ങളും രൂപീകരിക്കപ്പെട്ടിട്ടുണ്ടെന്ന് പ്രവാചകകാലത്ത് ജീവിച്ചിരുന്നവര്ക്ക് മനസ്സിലാവുമായിരുന്നു. നൂറ്റിയിരുപത് ദിവസങ്ങള്ക്കുശേഷമാണ് അവയവ രൂപീകരണമെന്ന് അവര് പ്രവാചകനില് (സ) നിന്ന് മനസ്സിലാക്കിയിരുന്നുവെങ്കില് തങ്ങളുടെ അനുഭവത്തിന്റെ അടിസ്ഥാനത്തില് അവര് അദ്ദേഹത്തോട് സംശയമുന്നയിക്കുമായിരുന്നു. ഇക്കാര്യത്തില് അനുയായികളാരെങ്കിലും സംശയമുന്നയിച്ചതായോ എതിരിളികളാരെങ്കിലും വിമര്ശനമുന്നയിച്ചതായോ യാതൊരു വിധ നിവേദനങ്ങളുമില്ല. നൂറ്റിയിരുപത് ദിവസങ്ങള് കഴിഞ്ഞ ശേഷമേ മുദ്വ്അ ഘട്ടം കഴിയൂവെന്ന് അവരാരും പ്രവാചകനില് (സ) നിന്ന് മനസ്സിലാക്കിയിട്ടില്ലെന്നാണ് ഇതിനര്ത്ഥം.
- ബുഖാരിയിലും മുസ്ലിമിലുമുള്ള ഈ ഹദീഥിലെ വാചകങ്ങളില് പ്രധാനപ്പെട്ട ഒരു വ്യത്യാസമുണ്ട്. നിങ്ങളോരോരുത്തരും മാതൃവ യറ്റില് നാല്പതു ദിവസം കൊണ്ടാണ് സംയോജിക്കുന്നത് എന്നുപറഞ്ഞശേഷം അങ്ങനെ അതേപോലെത്തന്നെ അലഖയാകുന്നു; അങ്ങ നെ അതേപോലെത്തന്നെ മുദ്വ്അയാകുന്നു' (ഥുമ്മ യകൂനു അലഖത്തുന് മിഥ്ല ദാലിക്ക; ഥുമ്മ യകൂനു മുദ്വ്അത്തുന് മിഥ്ല ദാലിക്ക) എന്നാണ് ബുഖാരിയിലുള്ളത്. മുസ്ലിമിലാകട്ടെ 'അങ്ങനെ അതേപോലെത്തന്നെ അതില്വെച്ചുതന്നെ അലഖയാകുന്നു; അങ്ങനെ അതേ പോലെത്തന്നെ അതില്വെച്ചുതന്നെ മുദ്വ്അയാകുന്നു' (ഥുമ്മ യകൂനു ഫീദാലിക അലഖത്തുന് മിഥ്ല ദാലിക്ക; ഥുമ്മ യകൂനു മുദ്വ്അത്തു ന് ഫീദാലിക മിഥ്ല ദാലിക്ക)(7) എന്നാണുള്ളത്. രണ്ടുതവണ 'ഫീദാലിക്ക'യെന്ന് ആവര്ത്തിച്ചു വന്നിട്ടുണ്ട്, ഇവിടെ. 'അതില് വെച്ചുത ന്നെ'യെന്നു പരിഭാഷപ്പെടുത്തിയിരിക്കുന്ന ഫീദാലിക്കെയന്ന് ഉപയോഗിച്ചത് നാല്പതു ദിവസത്തെക്കുറിക്കുവാനാകാനാണ് കൂടുതല് സാധ്യതയെന്നാണ് വാചകത്തിന്റെ ഘടനയില് നിന്ന് മനസ്സിലാവുക. അങ്ങനെയാണെങ്കില് ഈ ഹദീഥിന്റെ നേര്ക്കുനേരെയുള്ള വിവ ക്ഷ നാല്പതു ദിവസങ്ങള്ക്കകത്താണ് അലഖ, മുദ്വ്അ എന്നീ ഘട്ടങ്ങള് എന്നായിരിക്കും. മുസ്ലിമിലുള്ള ഈ ഹദീഥിന്റെ വാചകഘട നയോടും മറ്റുസമാനമായ ഹദീഥുകളിലെ ആശയങ്ങളോടും നിരീക്ഷിച്ച് മനസ്സിലാക്കാന് കഴിയുന്ന വസ്തുതകളോടും പൊരുത്തപ്പെടുന്ന അര്ത്ഥമതാണ്.
- പരാമര്ശിക്കപ്പെട്ട ഹദീഥിലെ മിഥ്ല ദാലിക്ക (അതേപോലെത്തന്നെ)യെന്ന അലഖയെയും മുദ്വ്അയെയും കുറിച്ച പരാമര്ശങ്ങള്ക്കു ശേഷം ആവര്ത്തിക്കപ്പെട്ടിരിക്കുന്ന പദപ്രയോഗം, 'നാല്പതു ദിവസം' എന്ന ആദ്യഭാഗത്തിന്റെ ആവര്ത്തനത്തെയാണ് കുറിക്കുന്ന തെന്ന വീക്ഷണത്തില് നിന്നാണ് അലഖ, മുദ്വ്അ എന്നീ ഘട്ടങ്ങളില് ഓരോന്നിനും നാല്പതു ദിവസങ്ങള് വീതം ഉണ്ടെന്ന നിഗമനത്തിലെ ത്തുവാന് ഇമാം നവവിയെപ്പോലെയുള്ള ഹദീഥ് വിശാദരന്മാരെ പ്രേരിപ്പിച്ചത്. എന്നാല് ഈ വിഷയം പറയുന്ന മറ്റു ഹദീഥുകളുമാ യി താരതമ്യം ചെയ്യുകയും വാചകഘടനയെ സൂക്ഷ്മമായി വിശകലനം നടത്തുകയും ചെയ്താല് അത് മാതൃശരീരത്തിലെ സംയോജന ത്തെയാണ് (ജംഉല് ഖല്ഖ്) കുറിക്കുന്നതെന്നാണ് മനസ്സിലാവുകയെന്ന് പല പണ്ഡിതന്മാരും സൂചിപ്പിച്ചിട്ടുണ്ട്. ക്രിസ്താബ്ദം പതി മൂന്നാം നൂറ്റാണ്ടില് ജീവിച്ച കമാല് അബ്ദുല് വാഹിദ് ബ്നു അബ്ദുല് കരീം അസ്സംലക്കാനിയെന്ന ക്വുര്ആന് പണ്ഡിതന് തന്റെ അല് ബുര്ഹാനല് കാശിഫ് അന്ഇഅ്ജാസുല് ക്വുര്ആന്(8) എന്ന കൃതിയില് ഇക്കാര്യം സമര്ത്ഥിക്കുന്നുണ്ട്. ഭ്രൂണശാസ്ത്ര വസ്തുതകള് മനസ്സിലാക്കിയതിനുശേഷം ഇസ്ലാമിക പ്രബോധകര് നടത്തുന്ന ദുര്വ്യാഖ്യാനമല്ല ഇതെന്നര്ത്ഥം.
ആറ് ആഴ്ചകള്ക്കുശേഷമാണ് ഭ്രൂണത്തിന്റെ ലിംഗനിര്ണയവും വ്യക്തിത്വത്തിന്റെ പാരമ്പര്യനിര്ണയവും അവയവ രൂപീകരണവും വൈകല്യങ്ങളുണ്ടെങ്കില് അവയുടെ സ്ഥിരീകരണവും നടക്കുകയെന്ന ഹദീഥുകള് സൂചിപ്പിക്കുന്ന കാര്യങ്ങളെല്ലാം കൃത്യമാണെന്ന് ആധു നിക ശാസ്ത്രം നമുക്ക് പറഞ്ഞുതരുന്നു. ബീജസങ്കലനം, പ്രതിഷ്ഠാപനം, സോമൈറ്റ് രൂപീകരണം എന്നീ ഭ്രണവളര്ച്ചയുടെ പ്രാഥമികമായ മൂന്ന് ഘട്ടങ്ങളും ക്രമപ്രവൃദ്ധമായി നടക്കുന്നത് ആദ്യത്തെ നാല്പതു ദിവസങ്ങള്ക്കുള്ളിലാണെന്ന് ഭ്രൂണശാസ്ത്രം പറയുമ്പോള് അത് ഹദീഥുകളുടെ സത്യതയുടെ പ്രഘോഷണമാണെന്നതാണ് സത്യം; ക്വുര്ആന് വിമര്ശകര് എത്രതന്നെ ശക്തമായി നിഷേധിച്ചാലും അതാണ് വസ്തുത. സത്യാന്വേഷികള്ക്കെല്ലാം അത് സുതരാം മനസ്സിലാവും, തീര്ച്ച.
കുറിപ്പുകള്:
- സ്വഹീഹുല് ബുഖാരി, കിതാബുല് ഖദ്ര്, ഹദീഥ്
- സ്വഹീഹ് മുസ്ലിം, കിതാബുല് ഖദ്ര്, ഹദീഥ്
- സ്വഹീഹ് മുസ്ലിം, കിതാബുല് ഖദ്ര്, ഹദീഥ്
- സ്വഹീഹ് മുസ്ലിം, കിതാബുല് ഖദ്ര്, ഹദീഥ്
- സ്വഹീഹുല് ബുഖാരി, കിതാബുല് ഖദ്ര്, ഹദീഥ് 6595, സ്വഹീഹ് മുസ്ലിം, കിതാബുല് ഖദ്ര്, ഹദീഥ്
- ഇമാം നവവി: സ്വഹീഹ് മുസ്ലിം http://hadith.al-islam.com/.
- സ്വഹീഹ് മുസ്ലിം, കിതാബുല് ഖദ്ര്, ഹദീഥ്
- Abdul-Majeed A. Zindani, Mustafa A. Ahmed, Joe Leigh Simpson: Embryogenesis and Human Development in the first 40 days in Abdul-Majeed A. Zindani: Human Development as Described in the Quran and Sunnah, Riyadh, 1983, Page 122.
ജൂത പണ്ഡിതന്റെ ചോദ്യങ്ങള്ക്കുള്ള പ്രവാചകന്റെ(സ) ഉത്തരത്തെപ്പറ്റി വിശദീകരിക്കുന്ന ഥൗബാന്(റ) നിവേദനം ചെയ്ത ദീര്ഘമായ ഹദീഥിലും ശിശുവിന്റെ സൃഷ്ടിയെക്കുറിച്ച ചോദ്യത്തിനുള്ള വിശദമായ ഉത്തരം ആരംഭിക്കുന്നത് 'പുരുഷസ്രവം വെളുത്തനിറത്തിലു ള്ളതും സ്ത്രീസ്രവം മഞ്ഞനിറത്തിലുള്ളതുമാണ്' എന്നു പറഞ്ഞുകൊണ്ടാണ്.(സ്വഹീഹ്മുസ്ലിം) ജൂത ചോദ്യങ്ങള്ക്കെല്ലാം മറുപടി പറഞ്ഞ ശേഷം 'അയാള് എന്നോട് ചോദിച്ച കാര്യങ്ങളെക്കുറിച്ചൊന്നും അല്ലാഹു അറിയിച്ചുതരുന്നതുവരെ എനിക്ക് യാതൊരു വിവരവുമുണ്ടാ യിരുന്നില്ല' എന്ന് പറഞ്ഞതായുള്ള ഥൗബാനി (റ)ന്റെ പരാമര്ശം ശ്രദ്ധേയമാണ്. സ്വന്തം സ്രവത്തെക്കുറിച്ച് അറിയാത്ത സ്ത്രീകള്ക്കടക്കം നിങ്ങളുടെ സ്രവം മഞ്ഞനിറത്തിലുള്ളതാണ് എന്ന് പ്രവാചകന്(സ) പറഞ്ഞുകൊടുത്തത് വ്യക്തമായ ദൈവബോധനത്തിന്റെ അടിസ്ഥാന ത്തിലാണെന്ന് വ്യക്തമാക്കുന്നതാണീ പ്രവാചകപരാമര്ശം.
ഏതാണീ മഞ്ഞ ദ്രാവകം? കുഞ്ഞിന്റെ സൃഷ്ടിയില് പങ്കെടുക്കുന്ന പുരുഷസ്രവത്തിന്റെ നിറം 'അബ്യദ്വ്' ആണെന്നു പറഞ്ഞതിനുശേഷ മാണ് സ്ത്രീ സ്രവത്തിന്റെ നിറം 'അസ്വ്ഫര്' (മഞ്ഞ) ആണെന്ന് പ്രവാചകന് (സ) പറഞ്ഞത്. രണ്ടും കൂടിച്ചേര്ന്നാണ് കുഞ്ഞുണ്ടാകുന്ന തെന്നും അതിനുശേഷം അദ്ദേഹം വ്യക്തമാക്കി. വെള്ള നിറത്തിലുള്ള പുരുഷസ്രവത്തെപോലെതന്നെ ബീജ സങ്കലനത്തില് പങ്കെടുക്കുന്ന സ്ത്രീസ്രവത്തിന്റെ നിറം മഞ്ഞയാണെന്നാണ് പ്രവാചകന് (സ) ഇവിടെ പഠിപ്പിക്കുന്നതെന്നുറപ്പാണ്. സ്ത്രീശരീരത്തില്നിന്ന് നിര്ഗളിക്കു ന്ന ഏതു സ്രവത്തിനാണ് മഞ്ഞനിറമുള്ളതെന്ന കാര്യത്തില് കര്മശാസ്ത്ര പണ്ഡിതന്മാര് ഏറെ ചര്ച്ച ചെയ്തതായി കാണാന് കഴിയും. സ്ത്രീജനനേന്ദ്രിയത്തില്നിന്ന് നിര്ഗളിക്കുന്ന കാണാനാവുന്ന സ്രവങ്ങള്ക്കൊന്നും തന്നെ മഞ്ഞനിറമില്ലെന്ന വസ്തുതയാണ് വിശാലമായ ഇത്തരം ചര്ച്ചകളുടെ ഉല്ഭവത്തിന് നിമിത്തമായത്.
സ്ത്രീകളുടെ ജനനേന്ദ്രിയത്തില്നിന്ന് പുറത്തുവരുന്ന സ്രവങ്ങള് മൂന്നെണ്ണമാണ്. തന്റെ ശരീരം ലൈംഗികബന്ധത്തിന് സജ്ജമായിയെന്ന് അറിയിച്ചുകൊണ്ട് സ്ത്രീജനനേന്ദ്രിയത്തില്നിന്ന് കിനിഞ്ഞിറങ്ങുന്ന ബര്ത്തോലിന് സ്രവം (Bartholin fluid) ആണ് ഒന്നാമത്തേത്. യോനീമുഖ ത്തിനകത്തായി സ്ഥിതി ചെയ്യുന്ന പയര്വിത്തിന്റെ വലിപ്പത്തിലുള്ള രണ്ട് ബര്ത്തോലിന്ഗ്രന്ഥികള് സ്ത്രീശരീരം ലൈംഗികമായി ഉത്തേ ജിപ്പിക്കപ്പെടുമ്പോള് പുറപ്പെടുവിക്കുന്ന ഈ സ്രവത്തിന് നിറമില്ല. രതിമൂര്ച്ചയുടെ അവസരത്തില് ചില സ്ത്രീകളുടെ ജനനേന്ദ്രിയത്തി ല്നിന്ന് പുറത്തുവരുന്ന പാരായുറിത്രല് സ്രവമാണ് (Para urethral fluid) രണ്ടാമത്തെ യോനീ സ്രവം. യോനിയുടെ ആന്തരികഭിത്തിയില് സ്ഥിതി ചെയ്യുന്ന പാരായുറിത്രല് ഗ്രന്ഥികളില്നിന്നു വളരെ ചെറിയ അളവില്മാത്രം പുറത്തുവരുന്ന ഈ സ്രവം താരതമ്യേന കട്ടിയുള്ള തും വെള്ള നിറത്തിലുള്ളതുമായിരിക്കും. സ്ത്രീ ജനനേന്ദ്രിയത്തെ എല്ലായ്പ്പോഴും വരളാതെ സൂക്ഷിക്കുന്ന സെര്വിക്കല് ശ്ലേഷ്മ (Cervical mucus) ആണ് മൂന്നാമത്തെ യോനീ സ്രവം. അണ്ഡോല്സര്ജനസമയമല്ലെങ്കില് ഈ സ്രവം വഴുവഴുപ്പുള്ളതും നല്ല വെളുത്ത ക്രീം നിറത്തിലു ള്ളതുമായിരിക്കും. അണ്ഡോല്സര്ജനത്തോടടുക്കുമ്പോള് വെള്ളനിറം മങ്ങുകയും വഴുവഴുപ്പ് കുറയുകയും ചെയ്യുന്ന ഈ സ്രവം ഉല്സ ര്ജനസമയമാകുമ്പോഴേക്ക് ജലത്തെപ്പോലെ വര്ണരഹിതമാവുകയും മുട്ടയുടെ വെള്ളക്കരുവിനെപ്പോലെയായിത്തീരുകയും ചെയ്യും. (Fritz K. Beller&Gebhard F.B. Schumacher (Editors): Biology of the Fluids of the Female Genital Tract, Amsterdam, 1979 Pages 312-388) അണുബാധയുണ്ടാ കുമ്പോള് മാത്രമാണ് സെല്വിക്കല് ശ്ലേഷ്മത്തിന് മങ്ങിയ മഞ്ഞനിറമുണ്ടാകുന്നത്. സ്ത്രീജനനേന്ദ്രിയത്തില്നിന്ന് സാധാരണഗതിയില് നിര്ഗളിക്കപ്പെടുന്ന മൂന്ന് സ്രവങ്ങളും വെളുത്തതോ നിറമില്ലാത്തതോ ആണെന്നും ഹദീഥുകളില് പറഞ്ഞ മഞ്ഞസ്രവമല്ല ഇവയെന്നും വ്യക്തമാണ്. ഇവയ്ക്കൊന്നുംതന്നെ കുഞ്ഞിന്റെ രൂപീകരണത്തില് നേരിട്ട് പങ്കൊന്നുമില്ലതാനും.
കുഞ്ഞിന്റെ രൂപീകരണത്തിന് നിമിത്തമാകുന്ന സ്രവമെന്താണ് എന്ന ചോദ്യത്തിന് ഉത്തരം കാണാന് ശ്രമിക്കുമ്പോഴാണ് ഹദീഥുകളില് പറഞ്ഞ മഞ്ഞ സ്രവമേതാണെന്ന് നമുക്ക് മനസ്സിലാവുക. ആര്ത്തവചക്രത്തിന്റെ പതിനാലാം ദിവസം അണ്ഡാശയത്തിനകത്തെ പൂര്ണ വളര്ച്ചയെത്തിയ ഫോളിക്കിളില് പ്രത്യക്ഷപ്പെടുന്ന ദ്വാരത്തിലൂടെ പ്രായപൂര്ത്തിയെത്തിയ അണ്ഡത്തെവഹിച്ചുകൊണ്ട് ഫോളിക്കുളാര് ദ്രവവും ക്യൂമുലസ് കോശങ്ങളും പുറത്തേക്ക് തെറിച്ച് ഫലോപ്പിയന് നാളിയുടെ അറ്റത്തുള്ള ഫിംബ്രയകളില് പതിക്കുന്നതിനാണ് അണ്ഡോല്സര്ജനം (Ovulation) എന്നു പറയുന്നത്. രതിമൂര്ച്ചയോടനുബന്ധിച്ച് പുരുഷശരീരത്തില് നടക്കുന്ന ശുക്ലസ്ഖലന (Ejaculation) ത്തിന് തുല്യമായി സ്ത്രീശരീരത്തില് നടക്കുന്ന പ്രക്രിയയാണ് ഇതെങ്കിലും ഒരു ആര്ത്തവചക്രത്തില് ഒരു തവണ മാത്രമാണ് ഇത് സംഭവി ക്കുന്നത്. ശുക്ല സ്ഖലനവും അണ്ഡോല്സര്ജനവുമാണ് കുഞ്ഞിന്റെ സൃഷ്ടിക്ക് നിദാനമായി പുരുഷശരീരത്തിലും സ്ത്രീശരീരത്തിലും യഥാക്രമം സംഭവിക്കുന്ന രണ്ട് പ്രക്രിയകള്. പുരുഷബീജങ്ങളെ വഹിക്കുന്ന ശുക്ലദ്രാവകത്തെപ്പോലെ സ്ത്രീയുടെ അണ്ഡത്തെ വഹിക്കുന്ന ഫോളിക്കുളാര് ദ്രവവും കുഞ്ഞിന്റെ നിര്മാണത്തിന് നിമിത്തമാകുന്ന ദ്രാവകമാണ്. ഹദീഥുകളില് പറഞ്ഞ കുഞ്ഞിന്റെ സൃഷ്ടിക്ക് കാരണമായ സ്ത്രീസ്രവം അണ്ഡത്തെ വഹിക്കുന്ന ഫോളിക്കുളാര് ദ്രാവകമാണെന്നാണ് ഇത് വ്യക്തമാക്കുന്നത്. അങ്ങനെയാണെങ്കില് പുരുഷദ്രാവകം വെളുത്തതും സ്ത്രീദ്രാവകം മഞ്ഞയുമെന്ന് പരാമര്ശത്തിന്റെ വെളിച്ചത്തില് പരിശോധിക്കുമ്പോള് ഫോളിക്കുളാര് ദ്രാവകത്തിന്റെ നിറം മഞ്ഞയായിരിക്കണം. എന്നാല് എന്താണ് വസ്തുത?
പ്രായപൂര്ത്തിയെത്തുന്നതിനുമുമ്പുള്ള അണ്ഡാവസ്ഥയായ അണ്ഡത്തെ (Oocyte) സംരക്ഷിക്കുകയും വളര്ത്തിക്കൊണ്ടുവന്ന് ബീജസങ്കല നത്തിന് പറ്റിയ അണ്ഡമാക്കിത്തീര്ക്കുകയും ചെയ്യുകയാണ് ഫോളിക്കിളിന്റെ ധര്മം. പെണ്കുഞ്ഞ് ജനിക്കുമ്പോള് തന്നെ അവളുടെ അണ്ഡാശയത്തിലുള്ള പ്രായപൂര്ത്തിയെത്താത്ത അണ്ഡകങ്ങളെ പൊതിഞ്ഞ് ആദിമ ഫോളിക്കിളുകളുണ്ടാവും (Primordial follicles). അവള് പ്രായപൂര്ത്തിയാകുന്നതോടെ ഇതില് ചില ഫോളിക്കിളുകള് വളര്ന്നുവരികയും ഓരോ ആര്ത്തവചക്രത്തിന്റെയും ശരാശരി 14-16 ദിവസങ്ങള് കഴിഞ്ഞ് പൊട്ടി പൂര്ണവളര്ച്ചയെത്തിയ അണ്ഡത്തെ (Ovum) പുറത്തുവിടുന്നതോടെ അവയുടെ ധര്മം അവസാനിക്കുകയും ചെയ്യുന്നു. ജനനസമയത്തുള്ള ഏകദേശം 1,80,000 ഫോളിക്കിളുകളില് നാനൂറെണ്ണത്തോളം മാത്രമാണ് അണ്ഡോല്സര്ജനത്തിനുമുമ്പത്തെ വളര്ച്ചയെത്തുവാനുള്ള ഭാഗ്യമുണ്ടാകുന്നത്. പ്രസ്തുത വളര്ച്ചയ്ക്ക് വ്യത്യസ്തങ്ങളായ ഘട്ടങ്ങളുണ്ട്. ഇതിലെ ഓരോ ഘട്ടങ്ങളിലും അതു കടന്നുപോകാന് കഴിയാത്ത ഫോളിക്കിളുകള് മരിച്ചുപോകുന്നുണ്ട്. ഓരോ ആര്ത്തവചക്രത്തിലും ഇരുപതോളം ഫോളിക്കിളുകള് വളര്ച്ചയെത്തുന്നുവെങ്കിലും ഒരെണ്ണത്തിന് മാത്രമാണ് ഫോളിക്കിള് മരണമായ അട്രീഷ്യ(atresia)യില്നിന്ന് രക്ഷപ്പെട്ട് അണ്ഡോല്സര്ജന ത്തിന് കഴിയുന്നത്. അട്രീഷ്യയില് നിന്ന് രക്ഷപ്പെട്ട് അണ്ഡോല്സര്ജനത്തിന് കഴിയുന്ന ഫോളിക്കിളുകള് രണ്ട് ദശകളിലൂടെയാണ് കടന്നു പോകുന്നത്. അണ്ഡോല്സര്ജനത്തിലൂടെ അവസാനിക്കുന്ന ഒന്നാമത്തെ ദശയെ ഫോളിക്കുളാര് ദശ (follicular phase) എന്നും അതിനുശേഷ മുള്ള ദശയെ ലൂടിയല് ദശ (luteal phase) എന്നുമാണ് വിളിക്കുക. ആര്ത്തവം മുതല് അണ്ഡോല്സര്ജനം വരെയുള്ള ഫോളിക്കുളാര് ദശയില് അണ്ഡകം പൂര്ണവളര്ച്ചയെത്തിയ അണ്ഡമായിത്തീരുന്നതിനും യഥാരൂപത്തിലുള്ള അണ്ഡോല്സര്ജനം നടക്കുന്നതിനും വേണ്ടി വ്യത്യ സ്തങ്ങളായ പ്രക്രിയകള് നടക്കേണ്ടതുണ്ട്. ഈ പ്രക്രിയകളുടെ അവസാനമായി ശരീരത്തിലെ ഈസ്ട്രജന് നില പരമാവധി ഉയരുകയും ലൂറ്റിനൈസിംഗ് ഹോര്മോണ് (LH), ഫോളിക്കിള് സ്റ്റിമുലേറ്റിംഗ് ഹോര്മോണ് (FSH) എന്നീ ഹോര്മോണുകളെ ഇതിന്റെ ഫലമായി ഉത്പാദിപ്പിക്കുകയും ചെയ്യുന്നു. 24 മുതല് 36 വരെ മണിക്കൂറുകള് നീണ്ടുനില്ക്കുന്ന ഈ പ്രക്രിയയുടെ അന്ത്യം കുറിച്ചുകൊണ്ടാണ് അണ്ഡം വഹിക്കുന്ന പൂര്ണവളര്ച്ചയെത്തിയ ഫോളിക്കിളില് (Ovarian follicle) സ്റ്റിഗ്മയെന്ന് പേരുള്ള ദ്വാരമുണ്ടാവുകയും അത് പൊട്ടി അണ്ഡത്തെ വഹിച്ചുകൊണ്ട് ഫോളിക്കുളാര് ദ്രവം പുറത്തേക്ക് തെറിക്കുകയും ചെയ്യുന്നത്. ഈ പുറത്തേക്കു തെറിക്കല് പ്രക്രിയക്കാണ് അണ്ഡോല്സര്ജനം (Ovulation) എന്നു പറയുക.(Anand Kumar&Amal K. Mukhopadhyay (Editors): Follicular Growth, Ovulation And Fertilization: Molecular And Clinical Basis, New Delhi, 2002 Pages 207-265)
ഫോളിക്കുളാര് ദശയിലുടനീളം നടക്കുന്ന അണ്ഡവളര്ച്ചയ്ക്കും അതിന് ഉല്സര്ജിക്കാനാവശ്യമായസംവിധാനങ്ങളൊരുക്കുന്നതിനും നിമി ത്തമാകുന്നത് FSHന്റെ പ്രവര്ത്തനങ്ങളാണ്. പ്രസ്തുത ഉത്പാദനത്തോടനുബന്ധിച്ചാണ് ഹൈപ്പോതലാമസില്നിന്നുള്ള ഗൊണാടോട്രോ പിന് റിലീസിംഗ് ഹോര്മോണിന്റെ (GnRH) പ്രേരണയാല് പിറ്റിയൂട്ടറിയില്നിന്ന് LHന്റെ ഉത്പാദനം നടക്കുന്നത്. ഈ ഹോര്മോണ് ഉത്പാദിപ്പിക്കുന്ന പ്രോട്ടീന് വിഘാടക രസങ്ങളായ പ്രോട്ടിയോലിറ്റിക് എന്സൈമുകളാണ് (Proteolytic enzymes) ഫോളിക്കിളിലുണ്ടാവുന്ന ദ്വാരമായ സ്റ്റിഗ്മക്ക് കാരണമാകുന്നത്. അണ്ഡോല്സര്ജനത്തിനുശേഷമുള്ള ഫോളിക്കിള് അവശിഷ്ടങ്ങളെ നിയന്ത്രിക്കുന്നതും പ്രധാനമായി ഈ ഹോര്മോണാണ്. ലൂട്ടിയല് ദശയില് അണ്ഡം നഷ്ടപ്പെട്ട ഫോളിക്കിള് അവശിഷ്ടങ്ങള് കോര്പസ് ലൂടിയം (Lorpus Luteum) ആയിത്തീരു കയും മാതൃസ്വഭാവങ്ങളെ ഉദ്ദീപിക്കുന്ന പ്രോജസ്റ്ററോണ് (Progesterone) ഹോര്മോണിന്റെ വര്ധിതമായ ഉത്പാദനത്തിന് നിമിത്തമാവുകയും ചെയ്യുന്നു.
എന്താണീ ലൂറ്റിനൈസിംഗ് ഹോര്മോണ്? മഞ്ഞയെന്ന് അര്ത്ഥം വരുന്ന ലൂറ്റിയസ് (Luteus) എന്ന ലാറ്റിന് പദത്തിന്റെ നപുംസകരൂപമായ ലൂറ്റിയത്തില്നിന്നാണ് (Luteum) ലൂറ്റിനൈസ് (Luteinize) എന്ന ക്രിയയുണ്ടായിരിക്കുന്നത്. കോര്പ്പസ് ലൂടിയത്തിന്റെ നിര്മിതിക്ക് നിമിത്ത മായ പ്രവര്ത്തനങ്ങള്ക്കാണ് സാങ്കേതികമായി ലൂറ്റിനൈസ് എന്ന് പറയുന്നതെങ്കിലും പദപരമായി അതിനര്ത്ഥം 'മഞ്ഞയാക്കുന്നത്' എന്നാണ്. ലൂറ്റിനൈസിംഗ് ഹോര്മോണിന്റെ പ്രവര്ത്തനഫലമായാണ് ഫോളിക്കുളാര് ദശ പിന്നിട്ട ഫോളിക്കിള് അവശിഷ്ടങ്ങള് കോര് പസ് ലൂടിയം ആയിത്തീരുന്നത്. കോര്പസ് ലൂടിയം എന്ന പദദ്വയത്തിനര്ത്ഥം മഞ്ഞ വസ്തുവെന്നാണ് (Yellow body). ലൂടിയല് ദശയിലേക്ക് കടന്ന അണ്ഡം നഷ്ടപ്പെട്ട ഫോളിക്കിള് അവശിഷ്ടങ്ങളെല്ലാം കൂടി രണ്ടു മുതല് അഞ്ചു സെന്റീമീറ്റര് വരെ വ്യാസത്തില് ശരീരത്തില് ഏതാ നും ദിവസങ്ങള് കൂടി അവശേഷിക്കും. മനുഷ്യരില് ഇത് ഓറഞ്ചു നിറത്തിലാണ് കാണപ്പെടുന്നത്.(Jno C Dalton Jr.: Prize Essay on the Corpus Luteum, Charleston, South Carolina, 2009, Pages 45-67) അണ്ഡോല്സര്ജനത്തിന്റെ അവസാനഘട്ടത്തില് ഉത്പാദിപ്പിക്കപ്പെടുന്ന LH അതിന്റെ പ്രവര്ത്തനമാരംഭിക്കുകയും ഫോളിക്കുളാര് ദ്രവത്തെ മഞ്ഞവല്ക്കരിക്കുകയും ചെയ്യും. ഫോളിക്കിളിലെ സ്റ്റിഗ്മ പൊട്ടി അണ്ഡത്തോടെ പുറത്തേക്ക് തെറിക്കുന്ന ഫോളിക്കുളാര് ദ്രാവകത്തിന്റെ നിറം മഞ്ഞയായിരിക്കും. പുരുഷ ശുക്ലവുമായി താരതമ്യം ചെയ്യുമ്പോള് കട്ടിയില്ലാത്തതും മഞ്ഞ നിറത്തിലുള്ളതുമായ ദ്രാവകമാണ് ഫോളിക്കിള് പൊട്ടി പുറത്തേക്കൊഴുകുന്ന കുഞ്ഞിന്റെ നിര്മാണത്തിന് നിമിത്തമാകുന്ന സ്ത്രീസ്രവം എന്നര്ത്ഥം.
ഹദീഥുകളിൽ പറഞ്ഞതെത്ര ശരി !!
ഇല്ല. മുഹമ്മദ് നബിക്കു (സ) ശേഷം രചിക്കപ്പെട്ടതാണെന്ന് വ്യക്തമാക്കുന്ന പരാമർശങ്ങളും ശാസ്ത്രീയമായ അബദ്ധങ്ങളും ഹദീഥുകളിലുണ്ടെന്ന് സ്ഥാപിക്കുവാൻ വേണ്ടിയുള്ള ശ്രമങ്ങൾ നടന്നിട്ടുണ്ട് എന്നത് ശരിയാണ്. അങ്ങനെ ഉണ്ടെന്നത് അടിസ്ഥാനരഹിതമായ ഒരു ആരോപണമാണ്.
ചരിത്രവിമര്ശന രീതിയില് മത്നിനെ അപഗ്രഥിച്ച് അത് നബി(സ) പറഞ്ഞതല്ലെന്ന് സ്ഥാപിക്കണമെങ്കില് അതില് കാലാനുക്രമ പ്രമാദ (anarchonism) ങ്ങളെന്തെങ്കിലുമുണ്ടാവണം. നബി(സ)ക്കുശേഷം നിലവില് വന്ന എന്തെങ്കി ലും കാര്യങ്ങളെക്കുറിച്ച പരാമര്ശങ്ങളുണ്ടാവുകയും പ്രസ്തുത പരാമര്ശങ്ങള് നബി(സ)ക്കു ശേഷം രചിക്കപ്പെട്ടവയാണ് അതെന്ന് വ്യക്തമാക്കുകയും ചെയ്യുമ്പോഴാണ് അവയെ കാലാനുക്രമ പ്രമാദങ്ങളായി പരിഗണിക്കുന്നത്. അങ്ങനെയുള്ള എന്തെങ്കിലും സ്വഹീഹായ ഹദീഥുകളിലുള്ളതായി തെളിയിക്കുവാന് വിമര്ശകര്ക്കൊന്നും കഴിഞ്ഞിട്ടില്ല. അത്തരം വല്ല പരാമര്ശങ്ങളും മത്ന് ഉള്ക്കൊള്ളുന്നു ണ്ടോയെന്ന പരിശോധന കൂടി കഴിഞ്ഞ ശേഷമാണ് ഒരു ഹദീഥ് സ്വഹീഹാണെന്ന് തീരുമാനിക്കുന്നത് എന്നതിനാലാണിത്. കാലാനുക്രമ പ്രമാദങ്ങളെ പരതിപ്പരിശോധിച്ച് ബൈബിളിനെപ്പോലെയുള്ള രചനകളുടെ ചരിത്രപരതയെ ചോദ്യം ചെയ്യുന്നതു പോലെ ഹദീഥുകളുടെ ചരിത്രപരതയെ നിഷേധിക്കുവാന് ചരിത്ര വിമര്ശകര്ക്ക് കഴിയില്ല. ചരിത്ര വിമര്ശന രീതിയുടെ വക്താക്കള് കാലാനുക്രമപ്രമാദങ്ങളെ പഠനവിധേയമാക്കി ഏതെങ്കിലുമൊരു പുരാതന സ്രോതസ്സിന്റെ ചരിത്രപരത നിര്ണയിക്കുവാനുള്ള സങ്കേതങ്ങള് കണ്ടെത്തുന്നതിന് നൂറ്റാണ്ടുകള്ക്കു മുമ്പുതന്നെ പ്രസ്തുത സങ്കേതങ്ങള്ക്ക് പിടികൊടുക്കാത്തവണ്ണം ഹദീഥുകളെ അന്യൂനമാക്കുവാന് ഹദീഥ് നിദാന ശാസ്ത്ര ജ്ഞര്ക്ക് കഴിഞ്ഞിരുന്നുവെന്നര്ഥം.
നബി(സ)യില് നിന്നുള്ളതാണെന്ന രീതിയില് നിവേദനം ചെയ്യപ്പെട്ട ഹദീഥുകളിലൊന്നും കാലാനുക്രമ പ്രമാദങ്ങളുണ്ടായിരുന്നില്ല എന്ന് ഇതിനര്ഥമില്ല. അന്തിമ പ്രവാചകന്റെ പേരില് ആരോപിക്കപ്പെട്ടിരുന്ന വ്യാജ ഹദീഥുകളുടെ മത്നുകളില് അത്തരം പ്രമാദങ്ങളുണ്ടാ യിരുന്നു. നബി(സ)ക്ക് പതിറ്റാണ്ടുകള്ക്കു ശേഷം രചിക്കപ്പെടുകയും നബി(സ)യുടെ പേരില് ആരോപിക്കപ്പെടുകയും ചെയ്യുന്ന വൃത്താന്തങ്ങളില് അത് സ്വാഭാവികമാണ്. വ്യാജഹദീഥുകള് നിര്മിച്ചയാളുടെ ചരിത്രപരമായ അജ്ഞതയുടെ തോതനുസരിച്ച് അത്തരം പ്രമാദ ങ്ങളുടെ എണ്ണത്തില് ഏറ്റക്കുറച്ചിലുകളുണ്ടാകുമെന്നേയുള്ളൂ. 'ഇരുനൂറു വര്ഷങ്ങള്ക്കു ശേഷമാണ് ലോകാവസാനത്തിന്റെ അടയാള ങ്ങള് പ്രത്യക്ഷപ്പെടുക' എന്ന ഹദീഥ് തള്ളുന്നതിന് ഇരുനൂറു വര്ഷം കഴിഞ്ഞിട്ടും അത്തരം ലക്ഷണങ്ങളൊന്നും കണ്ടിട്ടില്ല എന്ന കാരണം കൂടി ഇമാം ബുഖാരി പറഞ്ഞതായി ഇമാം ദഹബി വ്യക്തമാക്കുന്നുണ്ട്.(അദ്ദഹബി: മീസാനുല് ഇഅ്തിദാല്, വാല്യം 3, പുറം 306) 'പ്രചാരത്തിലിരിക്കുന്ന മുസ്ലിം നാണയങ്ങളെ പൊട്ടിക്കുന്നത് പ്രവാചകന്(സ) വിലക്കിയിരിക്കുന്നു' വെന്ന ഹദീഥ് നിവേദനം ചെയ്ത മുഹമ്മദ് ബിന് ഫദാ സ്വീകരിക്കുവാന് കൊള്ളാത്തവനാണെന്ന് വിധിക്കുവാനുള്ള കാരണങ്ങളിലൊന്നായി അമവിയ്യാക്കളുടെ കാലത്ത് മാത്രമാണ് മുസ്ലിംകള് നാണയങ്ങള് അടിച്ചിറക്കാനാരംഭിച്ചത് എന്ന വസ്തുത എടുത്തുപറയുകയും നബി(സ)യുടെ കാലത്ത് നാണയം നിലനിന്നിരുന്നില്ലെന്നതിനാല് അദ്ദേഹം അസ്വീകാര്യനാണെന്ന് ഇമാം ബുഖാരി വ്യക്തമാക്കുകയും ചെയ്തിട്ടുണ്ട്.(ഇമാം ബുഖാരിയുടെ അത്താരീഖുല് ഔസത്തില് നിന്ന് (വാല്യം 2, പുറം 110 -119) ഖൈബറിലെ യഹൂദന്മാരോട് നികുതിവാങ്ങിയതിന് സാക്ഷി സഅദ് ബ്നു മുആദും (റ)കരാര് എഴുതിയത് മുആവിയയേുമായിരുന്നു' എന്ന ഹദീഥ് തള്ളിക്കളയുന്നതിനു ള്ള കാരണമായി ഖൈബര് യുദ്ധകാലത്ത് കപ്പം വാങ്ങുന്ന സമ്പ്രദായമുണ്ടായിരുന്നില്ലെന്നും സഅദ്ബ്നു മുആദ് (റ)പ്രസ്തുത യുദ്ധത്തിന് മുമ്പു തന്നെ മരണപ്പെട്ടിട്ടുണ്ടെന്നും മുആവിയ (റ)മുസ്ലിമായത് യുദ്ധത്തിന് ശേഷവുമാണെന്ന വസ്തുതകള് കൂടി പണ്ഡിതന്മാര് എടുത്തു പറഞ്ഞതായി നടേ വ്യക്തമാക്കിയിട്ടുണ്ട്. ഇത്തരം പ്രമാദങ്ങള് കണ്ടെത്തുകയും അവയെ അപഗ്രഥിച്ച് മാറ്റിവെക്കുകയും ചെയ്ത ശേഷ മാണ് ഹദീഥ് നിദാന ശാസ്ത്രജ്ഞന്മാര് സ്വഹീഹായ ഹദീഥുകളെ വേര്തിരിച്ച് രേഖപ്പെടുത്തിയത്. സ്വഹീഹായ ഹദീഥുകളുടെ മത്നില് ഇനിയുമൊരു ചരിത്ര വിമര്ശനാപഗ്രഥനം ആവശ്യമില്ലെന്നര്ഥം.
സ്വഹീഹായ ഹദീഥുകളില് ആരോപിക്കപ്പെടുന്ന ശാസ്ത്രാബദ്ധങ്ങളുടെ സ്ഥിതിയും ഇതുതന്നെയാണ്. ഖണ്ഡിതമായി തെളിയിക്കപ്പെട്ട ശാസ്ത്രവസ്തുതകളൊന്നും സ്വഹീഹായ ഹദീഥുകള് നിഷേധിക്കുന്നില്ല. എന്നാല് ശാസ്ത്രീയമായി തെളിയിക്കപ്പെടാത്ത കാര്യങ്ങളെക്കു റിച്ച പരാമര്ശങ്ങള് അവയിലുണ്ടാവാം. നിലനില്ക്കുന്ന അറിവിന്റെ മാത്രം അടിസ്ഥാനത്തില് അത്തരം പരാമര്ശങ്ങളെ അബദ്ധമായി ഗണിച്ച് ഹദീഥുകള് അസ്വീകാര്യമാണെന്ന് വിധിക്കുന്നത് അപകടകരമാണ്. മനുഷ്യന്റെ വൈജ്ഞാനിക മണ്ഡലം ഇനിയെത്ര വളരാനിരി ക്കുന്നു?! പ്രാപഞ്ചിക പ്രതിഭാസങ്ങളെക്കുറിച്ച അറിവിന്റെ ചുരുളുകള് ഇനിയുമെത്ര നിവരാനിരിക്കുന്നു?! വിജ്ഞാനമഹാസാഗര ത്തിലെ തുള്ളികള് മാത്രം ആസ്വദിക്കാന് അവസരം ലഭിച്ച മനുഷ്യര്ക്കെങ്ങനെയാണ് പ്രസ്തുത സാഗരത്തിന്റെ ഉടമസ്ഥനായ അല്ലാഹു വിന്റെ ബോധനത്തെ തനിക്കു ലഭിച്ച ചെറിയ അറിവിന്റെ വെളിച്ചത്തില് തള്ളിക്കളയാനാവുക? നമുക്ക് മനസ്സിലായിക്കഴിഞ്ഞിട്ടില്ലാത്ത കാര്യങ്ങളെക്കുറിച്ച് സ്വഹീഹായ ഹദീഥുകള് എന്തെങ്കിലും പറയുന്നുവെങ്കില്, നമ്മുടെ വൈജ്ഞാനിക മണ്ഡലം അവിടെയെത്തിക്കഴി ഞ്ഞിട്ടില്ലെന്ന് കരുതി കാത്തിരിക്കുന്നതല്ലേ ശരി?! സ്വഹീഹായ ഹദീഥുകളില് അശാസ്ത്രീയത പരതുന്നവര്ക്ക്, അറിയാനുള്ളതെല്ലാം അറിഞ്ഞുകഴിഞ്ഞവരാണ് തങ്ങളെന്ന് അഭിപ്രായമുണ്ടോ? ഇല്ലെങ്കില്, എങ്ങനെയാണ് ഹദീഥുകളെ തള്ളിക്കളയാന് ശാസ്ത്രത്തിന്റെ ഉപകരണങ്ങള് ഉപയുക്തമാവുക?
അല്ല. കഞ്ഞിന്റെ സൃഷ്ടിയിൽ പുരുഷസ്രവത്തിനും സ്ത്രീസ്രവത്തിനും പങ്കുണ്ടെന്നും അവ കൂട്ടിച്ചെർന്നാണ് കുഞ്ഞുണ്ടാവുന്നത് എന്നും തന്നെയാണ് ഖുർആനും ഹദീഥുകളും വ്യക്തമാക്കുന്നത്. മനുഷ്യനെ ജലത്തില്നിന്നാണ് സൃഷ്ടിച്ചിരിക്കുന്നതെന്ന് പ്രസ്താവിക്കുന്ന ക്വുര്ആന് വചനങ്ങള് സൂചിപ്പിക്കുന്നത് പുരുഷസ്രവത്തില്നിന്നുള്ള മനുഷ്യ സൃഷ്ടിയാണെന്നാണ് പ്രമുഖരായ ക്വുര്ആന് വ്യാഖ്യാതക്കളെല്ലാം അഭിപ്രായപ്പെട്ടിരിക്കുന്നതെന്നത് ശരിയാണ് . ജലത്തില് നിന്ന് മനുഷ്യനെ സൃഷ്ടിച്ചതായി പരാമര്ശിക്കുന്ന സുറത്തുല് ഫുര്ക്വാനിലെ 25ാം വചനത്തിന് വ്യാഖ്യാനമായി നിസ്സാരമായ ജലത്തില്നിന്നാണ് മനുഷ്യ സൃഷ്ടി നടന്നതെന്ന സൂറത്തുല് മുര്സലാത്തിലെ 20ാം വചനവും 'നിസാരമായ ഒരു ജലത്തിന്റെ സത്തില്' നിന്നാണ് അത് നടന്നതെന്ന സൂറത്തുസ്സജദയിലെ എട്ടാം വചനവും നിലകൊള്ളുന്നുണ്ട്. ഈ വചനങ്ങള് താരതമ്യം ചെയ്ത് പരിശോധിച്ചാല് മനുഷ്യനെ സൃഷ്ടിച്ച ജലമായി ക്വുര്ആന് പരിചയപ്പെടുത്തുന്നത് പുരുഷസ്രവമാണെന്നു തന്നെയാണ് മനസ്സിലാവുക.
സ്ത്രീയുടെ സ്രവത്തെക്കുറിച്ച് ക്വുര്ആനില് നേര്ക്കുനേരെയുള്ള പരാമര്ശങ്ങളൊന്നുമില്ലെങ്കിലും സ്വുല്ബിന്റെയും തറാഇബിന്റെയും ഇടയില്നിന്ന് പുറപ്പെടുന്ന തെറിച്ചുവീഴുന്ന ദ്രാവകത്തില്നിന്നാണ് മനുഷ്യനെ സൃഷ്ടിച്ചതെന്ന് പറയുന്ന സൂറത്തുത്ത്വാരിഖിലെ ആറും ഏഴും വചനങ്ങളെ വ്യാഖ്യാനിച്ച പ്രവാചകാനുചരന്മാരില് ക്വുര്ആന് വ്യാഖ്യാനത്തിന് പ്രസിദ്ധനായ ഇബ്നു അബ്ബാസും(റ) മറ്റൊരു സ്വഹാബിയായ ഇക്രിമ(റ)യും പുരുഷന്റെ സ്വുല്ബില്നിന്ന് പുറപ്പെടുന്ന ദ്രാവകവും സ്ത്രീയുടെ തറാഇബില്നിന്ന് പുറപ്പെടുന്ന ദ്രാവകവും ഒരുമിച്ചു ചേര്ന്നാണ് കുഞ്ഞുണ്ടാകുന്നതെന്ന് വ്യാഖ്യാനിച്ചതായി ഇമാം ത്വബരി രേഖപ്പെടുത്തുന്നുണ്ട്.(തഫ്സീര് അത്ത്വബ്രി) പ്രസിദ്ധ ക്വുര്ആന് വ്യാഖ്യാതക്കളായ ത്വബ്രി, സമഖ്ശരി, ത്വബ്റാനി, റാസി, ക്വുര്തുബി, ഇബ്നുകഥീര്, ജലാലൈനി, ശൗക്വാനി തുടങ്ങിയവരെല്ലാം പുരുഷന്റെ സ്വുല്ബില്നിന്നും സ്ത്രീയുടെ തറാഇബില്നിന്നും പുറപ്പെടുന്ന ദ്രാവകങ്ങളുടെ മിശ്രണത്തില്നിന്നാണ്് കുഞ്ഞുണ്ടാവുന്നതെന്നാണ് ഈ ആയത്ത് അര്ത്ഥമാക്കുന്നതെന്നാണ് അഭിപ്രായപ്പെട്ടിരിക്കുന്നത്. സ്ത്രീസ്രവവും പുരുഷസ്രവവും കൂടിച്ചേര്ന്നാണ് കുഞ്ഞുണ്ടാകുന്നതെന്ന് പ്രവാചകാനുചരന്മാര് പരിശുദ്ധ ക്വുര്ആനില് നിന്നു മനസ്സിലാക്കിയിരുന്നുവെന്ന് ഇത് വ്യക്തമാക്കുന്നു.
ഹദീഥുകള് ഇവ്വിഷയകമായ കൂടുതല് വിശദീകരണങ്ങള് നല്കുന്നുണ്ട്. 'സ്ത്രീകള്ക്ക് സ്രവമുണ്ടാകുമോ?'യെന്ന ഉമ്മുസുലൈമി (റ)ന്റെ ചോദ്യത്തിന് പ്രവാചകന്(സ) നല്കിയ മറുപടിയില്നിന്ന് അക്കാലത്തെ പൊതുവിശ്വാസവും അതിലെ കൃത്യമായ പ്രവാചകതിരുത്തലും നമുക്ക് ലഭിക്കുന്നു. സ്വഹീഹുല് ബുഖാരിയില് ഉമ്മുസലമ(റ)യില്നിന്ന് നിവേദനം ചെയ്യപ്പെട്ട ഈ ഹദീഥില്നിന്ന് സ്ത്രീയുടെ സ്രവത്തെക്കുറിച്ച് അക്കാലത്തെ സ്ത്രീകള്ക്കുതന്നെ അറിയില്ലായിരുന്നുവെന്ന് മനസ്സിലാക്കാം.
അത്ഭുതത്തോടുകൂടിയാണ് ഉമ്മുസുലൈം 'സ്ത്രീകള്ക്ക് സ്രവമുണ്ടാകുമോ?'യെന്ന് ചോദിക്കുന്നത്. സംശയം ചോദിക്കുകയെന്നതിലുപരി അങ്ങനെ ഉണ്ടാവില്ലല്ലോയെന്ന് ദ്യോതിപ്പിച്ചുകൊണ്ടുള്ള പ്രസ്തുത ചോദ്യത്തിന് 'അതെ! ഇതെന്തൊരു ചോദ്യം? പിന്നെയെങ്ങനെയാണ് കുട്ടിക്ക് അവളോട് സാദൃശ്യമുണ്ടാവുക?' എന്ന മറുചോദ്യമാണ് പ്രവാചകന് (സ) മറുപടിയായി നല്കുന്നത്. സ്ത്രീകള്ക്ക് സ്രവമുണ്ടെന്ന് വ്യക്തമാക്കുക മാത്രമല്ല, അത് കുട്ടിയുടെ പാരമ്പര്യദാതാവുകൂടിയാണെന്ന് പഠിപ്പിക്കുകകൂടി ചെയ്യുന്നുണ്ട് ഈ പ്രവാചകവചനം. ഉമ്മുസുലൈമും(റ) പ്രവാചകനും(സ) തമ്മില് നടന്ന ഈ സംഭാഷണം കൂറേക്കൂടി വിശദമായി ഇമാം മുസ്്ലിം(റ) അനസുബ്നു മാലിക്കില് (റ) നിന്ന് നിവേദനം ചെയ്തിട്ടുണ്ട്. 'പുരുഷന്റെ സ്രവം വെളുത്തതും കട്ടിയുള്ളതുമാണ്; സ്ത്രീയുടെ സ്രവം മഞ്ഞ നിറത്തിലുള്ളതും നേര്മയുള്ളതുമാണ്. ഏത് സ്രവമാണോ മുന്കടക്കുന്നത് അതിനോടാണ് കുഞ്ഞിന് സാദൃശ്യമുണ്ടാവുക' എന്നുകൂടി ഉമ്മുസുലൈമിനോട്(റ) പ്രവാചകന്(സ) പറഞ്ഞതായി ഈ നിവേദനത്തിലുണ്ട്. വെളുത്ത, കട്ടിയായ പുരുഷസ്രവത്തോട് മഞ്ഞ, നേര്മയായ സ്ത്രീസ്രവം കൂടിച്ചേര്ന്നാണ് കുഞ്ഞുണ്ടാകുന്നതെന്നാണ് ഇവിടെ പ്രവാചകന്(സ) പഠിപ്പിക്കുന്നത്.
ഒരു ജൂത പണ്ഡിതന്റെ ചോദ്യങ്ങള്ക്കുള്ള പ്രവാചകന്റെ(സ) ഉത്തരത്തെപ്പറ്റി വിശദീകരിക്കുന്ന ഥൗബാന്(റ) നിവേദനം ചെയ്ത സ്വഹീഹ് മുസ്ലിമിലുള്ള ദീര്ഘമായ ഹദീഥിലും ശിശുവിന്റെ സൃഷ്ടിയെക്കുറിച്ച ചോദ്യത്തിനുള്ള വിശദമായ ഉത്തരം ആരംഭിക്കുന്നത് 'പുരുഷസ്രവം വെളുത്തനിറത്തിലുള്ളതും സ്ത്രീസ്രവം മഞ്ഞനിറത്തിലുള്ളതുമാണ്; അവ രണ്ടും കൂട്ടിച്ചെരുമ്പോൾ....' എന്നു പറഞ്ഞുകൊണ്ടാണ്. ജൂത ചോദ്യങ്ങള്ക്കെല്ലാം മറുപടി പറഞ്ഞശേഷം 'അയാള് എന്നോട് ചോദിച്ച കാര്യങ്ങളെക്കുറിച്ചൊന്നും അല്ലാഹു അറിയിച്ചുതരുന്നതുവരെ എനിക്ക് യാതൊരു വിവരവുമുണ്ടായിരുന്നില്ല' എന്ന് പറഞ്ഞതായുള്ള ഥൗബാനി (റ)ന്റെ പരാമര്ശം ശ്രദ്ധേയമാണ്. സ്വന്തം സ്രവത്തെക്കുറിച്ച് അറിയാത്ത സ്ത്രീകള്ക്കടക്കം നിങ്ങളുടെ സ്രവം മഞ്ഞനിറത്തിലുള്ളതാണ് എന്ന് പ്രവാചകന്(സ) പറഞ്ഞുകൊടുത്തത് വ്യക്തമായ ദൈവബോധനത്തിന്റെ അടിസ്ഥാനത്തിലാണെന്ന് വ്യക്തമാക്കുന്നതാണീ പ്രവാചകപരാമര്ശം.
ഏതാണീ മഞ്ഞ ദ്രാവകം? കുഞ്ഞിന്റെ സൃഷ്ടിയില് പങ്കെടുക്കുന്ന പുരുഷസ്രവത്തിന്റെ നിറം 'അബ്യദ്വ്' ആണെന്നു പറഞ്ഞതിനുശേഷമാണ് സ്ത്രീ സ്രവത്തിന്റെ നിറം 'അസ്വ്ഫര്' (മഞ്ഞ) ആണെന്ന് പ്രവാചകന് (സ) പറഞ്ഞത്. രണ്ടും കൂടിച്ചേര്ന്നാണ് കുഞ്ഞുണ്ടാകുന്നതെന്നും അതിനുശേഷം അദ്ദേഹം വ്യക്തമാക്കി. വെള്ള നിറത്തിലുള്ള പുരുഷസ്രവത്തെപോലെതന്നെ ബീജ സങ്കലനത്തില് പങ്കെടുക്കുന്ന സ്ത്രീസ്രവത്തിന്റെ നിറം മഞ്ഞയാണെന്നാണ് പ്രവാചകന് (സ) ഇവിടെ പഠിപ്പിക്കുന്നതെന്നുറപ്പാണ്. സ്ത്രീശരീരത്തില്നിന്ന് നിര്ഗളിക്കുന്ന ഏതു സ്രവത്തിനാണ് മഞ്ഞനിറമുള്ളതെന്ന കാര്യത്തില് കര്മശാസ്ത്ര പണ്ഡിതന്മാര് ഏറെ ചര്ച്ച ചെയ്തതായി കാണാന് കഴിയും. സ്ത്രീജനനേന്ദ്രിയത്തില്നിന്ന് നിര്ഗളിക്കുന്ന കാണാനാവുന്ന സ്രവങ്ങള്ക്കൊന്നും തന്നെ മഞ്ഞനിറമില്ലെന്ന വസ്തുതയാണ് വിശാലമായ ഇത്തരം ചര്ച്ചകളുടെ ഉല്ഭവത്തിന് നിമിത്തമായത്.
സ്ത്രീകളുടെ ജനനേന്ദ്രിയത്തില്നിന്ന് പുറത്തുവരുന്ന സ്രവങ്ങള് മൂന്നെണ്ണമാണ്. തന്റെ ശരീരം ലൈംഗികബന്ധത്തിന് സജ്ജമായിയെന്ന് അറിയിച്ചുകൊണ്ട് സ്ത്രീജനനേന്ദ്രിയത്തില്നിന്ന് കിനിഞ്ഞിറങ്ങുന്ന ബര്ത്തോലിന് സ്രവം(Bartholin fluid) ആണ് ഒന്നാമത്തേത്. യോനീമുഖത്തിനകത്തായി സ്ഥിതി ചെയ്യുന്ന പയര്വിത്തിന്റെ വലിപ്പത്തിലുള്ള രണ്ട് ബര്ത്തോലിന്ഗ്രന്ഥികള് സ്ത്രീശരീരം ലൈംഗികമായി ഉത്തേജിപ്പിക്കപ്പെടുമ്പോള് പുറപ്പെടുവിക്കുന്ന ഈ സ്രവത്തിന് നിറമില്ല. രതിമൂര്ച്ചയുടെ അവസരത്തില് ചില സ്ത്രീകളുടെ ജനനേന്ദ്രിയത്തില്നിന്ന് പുറത്തുവരുന്ന പാരായുറിത്രല് സ്രവമാണ്(Para urethral fluid) രണ്ടാമത്തെ യോനീ സ്രവം. യോനിയുടെ ആന്തരികഭിത്തിയില് സ്ഥിതി ചെയ്യുന്ന പാരായുറിത്രല് ഗ്രന്ഥികളില്നിന്നു വളരെ ചെറിയ അളവില്മാത്രം പുറത്തുവരുന്ന ഈ സ്രവം താരതമ്യേന കട്ടിയുള്ളതും വെള്ള നിറത്തിലുള്ളതുമായിരിക്കും. സ്ത്രീ ജനനേന്ദ്രിയത്തെ എല്ലായ്പ്പോഴും വരളാതെ സൂക്ഷിക്കുന്ന സെര്വിക്കല് ശ്ലേഷ്മ (Cervical mucus) ആണ് മൂന്നാമത്തെ യോനീ സ്രവം. അണ്ഡോല്സര്ജനസമയമല്ലെങ്കില് ഈ സ്രവം വഴുവഴുപ്പുള്ളതും നല്ല വെളുത്ത ക്രീം നിറത്തിലുള്ളതുമായിരിക്കും. അണ്ഡോല്സര്ജനത്തോടടുക്കുമ്പോള് വെള്ളനിറം മങ്ങുകയും വഴുവഴുപ്പ് കുറയുകയും ചെയ്യുന്ന ഈ സ്രവം ഉല്സര്ജനസമയമാകുമ്പോഴേക്ക് ജലത്തെപ്പോലെ വര്ണരഹിതമാവുകയും മുട്ടയുടെ വെള്ളക്കരുവിനെപ്പോലെയായിത്തീരുകയും ചെയ്യും. അണുബാധയുണ്ടാകുമ്പോള് മാത്രമാണ് സെല്വിക്കല് ശ്ലേഷ്മത്തിന് മങ്ങിയ മഞ്ഞനിറമുണ്ടാകുന്നത്. സ്ത്രീജനനേന്ദ്രിയത്തില്നിന്ന് സാധാരണഗതിയില് നിര്ഗളിക്കപ്പെടുന്ന മൂന്ന് സ്രവങ്ങളും വെളുത്തതോ നിറില്ലാത്തതോ ആണെന്നും ഹദീഥുകളില് പറഞ്ഞ മഞ്ഞസ്രവമല്ല ഇവയെന്നും വ്യക്തമാണ്. ഇവയ്ക്കൊന്നുംതന്നെ കുഞ്ഞിന്റെ രൂപീകരണത്തില് നേരിട്ട് പങ്കൊന്നുമില്ലതാനും.
കുഞ്ഞിന്റെ രൂപീകരണത്തിന് നിമിത്തമാകുന്ന സ്രവമെന്താണ് എന്ന ചോദ്യത്തിന് ഉത്തരം കാണാന് ശ്രമിക്കുമ്പോഴാണ് ഹദീഥുകളില് പറഞ്ഞ മഞ്ഞ സ്രവമേതാണെന്ന് നമുക്ക് മനസ്സിലാവുക. ആര്ത്തവചക്രത്തിന്റെ പതിനാലാം ദിവസം അണ്ഡാശയത്തിനകത്തെ പൂര്ണ വളര്ച്ചയെത്തിയ ഫോളിക്കിളില് പ്രത്യക്ഷപ്പെടുന്ന ദ്വാരത്തിലൂടെ പ്രായപൂര്ത്തിയെത്തിയ അണ്ഡത്തെവഹിച്ചുകൊണ്ട് ഫോളിക്കുളാര് ദ്രവവും ക്യൂമുലസ് കോശങ്ങളും പുറത്തേക്ക് തെറിച്ച് ഫലോപ്പിയന് നാളിയുടെ അറ്റത്തുള്ള ഫിംബ്രയകളില് പതിക്കുന്നതിനാണ് അണ്ഡോല്സര്ജനം (Ovulation) എന്നു പറയുന്നത്. രതിമൂര്ച്ചയോടനുബന്ധിച്ച് പുരുഷശരീരത്തില് നടക്കുന്ന ശുക്ലസ്ഖലന(Ejaculation) ത്തിന് തുല്യമായി സ്ത്രീശരീരത്തില് നടക്കുന്ന പ്രക്രിയയാണ് ഇതെങ്കിലും ഒരു ആര്ത്തവചക്രത്തില് ഒരു തവണ മാത്രമാണ് ഇത് സംഭവിക്കുന്നത്. ശുക്ല സ്ഖലനവും അണ്ഡോല്സര്ജനവുമാണ് കുഞ്ഞിന്റെ സൃഷ്ടിക്ക് നിദാനമായി പുരുഷശരീരത്തിലും സ്ത്രീശരീരത്തിലും യഥാക്രമം സംഭവിക്കുന്ന രണ്ട് പ്രക്രിയകള്. പുരുഷബീജങ്ങളെ വഹിക്കുന്ന ശുക്ലദ്രാവകത്തെപ്പോലെ സ്ത്രീയുടെ അണ്ഡത്തെ വഹിക്കുന്ന ഫോളിക്കുളാര് ദ്രവവും കുഞ്ഞിന്റെ നിര്മാണത്തിന് നിമിത്തമാകുന്ന ദ്രാവകമാണ്. ഹദീഥുകളില് പറഞ്ഞ കുഞ്ഞിന്റെ സൃഷ്ടിക്ക് കാരണമായ സ്ത്രീസ്രവം അണ്ഡത്തെ വഹിക്കുന്ന ഫോളിക്കുളാര് ദ്രാവകമാണെന്നാണ് ഇത് വ്യക്തമാക്കുന്നത്. അങ്ങനെയാണെങ്കില് പുരുഷദ്രാവകം വെളുത്തതും സ്ത്രീദ്രാവകം മഞ്ഞയുമെന്ന് പരാമര്ശത്തിന്റെ വെളിച്ചത്തില് പരിശോധിക്കുമ്പോള് ഫോളിക്കുളാര് ദ്രാവകത്തിന്റെ നിറം മഞ്ഞയായിരിക്കണം. എന്നാല് എന്താണ് വസ്തുത?
പ്രായപൂര്ത്തിയെത്തുന്നതിനുമുമ്പുള്ള അണ്ഡാവസ്ഥയായ അണ്ഡത്തെ(Oocyte) സംരക്ഷിക്കുകയും വളര്ത്തിക്കൊണ്ടുവന്ന് ബീജസങ്കലനത്തിന് പറ്റിയ അണ്ഡമാക്കിത്തീര്ക്കുകയും ചെയ്യുകയാണ് ഫോളിക്കിളിന്റെ ധര്മം. പെണ്കുഞ്ഞ് ജനിക്കുമ്പോള് തന്നെ അവളുടെ അണ്ഡാശയത്തിലുള്ള പ്രായപൂര്ത്തിയെത്താത്ത അണ്ഡകങ്ങളെ പൊതിഞ്ഞ് ആദിമ ഫോളിക്കിളുകളുണ്ടാവും (Primordial follicles). അവള് പ്രായപൂര്ത്തിയാകുന്നതോടെ ഇതില് ചില ഫോളിക്കിളുകള് വളര്ന്നുവരികയും ഓരോ ആര്ത്തവചക്രത്തിന്റെയും ശരാശരി 14-16 ദിവസങ്ങള് കഴിഞ്ഞ് പൊട്ടി പൂര്ണവളര്ച്ചയെത്തിയ അണ്ഡത്തെ (Ovum) പുറത്തുവിടുന്നതോടെ അവയുടെ ധര്മം അവസാനിക്കുകയും ചെയ്യുന്നു. ജനനസമയത്തുള്ള ഏകദേശം 1,80,000 ഫോളിക്കിളുകളില് നാനൂറെണ്ണത്തോളം മാത്രമാണ് അണ്ഡോല്സര്ജനത്തിനുമുമ്പത്തെ വളര്ച്ചയെത്തുവാനുള്ള ഭാഗ്യമുണ്ടാകുന്നത്. പ്രസ്തുത വളര്ച്ചയ്ക്ക് വ്യത്യസ്തങ്ങളായ ഘട്ടങ്ങളുണ്ട്. ഇതിലെ ഓരോ ഘട്ടങ്ങളിലും അതു കടന്നുപോകാന് കഴിയാത്ത ഫോളിക്കിളുകള് മരിച്ചുപോകുന്നുണ്ട്. ഓരോ ആര്ത്തവചക്രത്തിലും ഇരുപതോളം ഫോളിക്കിളുകള് വളര്ച്ചയെത്തുന്നുവെങ്കിലും ഒരെണ്ണത്തിന് മാത്രമാണ് ഫോളിക്കിള് മരണമായ അട്രീഷ്യ(atresia)യില്നിന്ന് രക്ഷപ്പെട്ട് അണ്ഡോല്സര്ജനത്തിന് കഴിയുന്നത്. അട്രീഷ്യയില് നിന്ന് രക്ഷപ്പെട്ട് അണ്ഡോല്സര്ജനത്തിന് കഴിയുന്ന ഫോളിക്കിളുകള് രണ്ട് ദശകളിലൂടെയാണ് കടന്നുപോകുന്നത്. അണ്ഡോല്സര്ജനത്തിലൂടെ അവസാനിക്കുന്ന ഒന്നാമത്തെ ദശയെ ഫോളിക്കുളാര് ദശfollicular phase) എന്നും അതിനുശേഷമുള്ള ദശയെ ലൂടിയല് ദശ (luteal phase) എന്നുമാണ് വിളിക്കുക. ആര്ത്തവം മുതല് അണ്ഡോല്സര്ജനം വരെയുള്ള ഫോളിക്കുളാര് ദശയില് അണ്ഡകം പൂര്ണവളര്ച്ചയെത്തിയ അണ്ഡമായിത്തീരുന്നതിനും യഥാരൂപത്തിലുള്ള അണ്ഡോല്സര്ജനം നടക്കുന്നതിനും വേണ്ടി വ്യത്യസ്തങ്ങളായ പ്രക്രിയകള് നടക്കേണ്ടതുണ്ട്. ഈ പ്രക്രിയകളുടെ അവസാനമായി ശരീരത്തിലെ ഈസ്ട്രജന് നില പരമാവധി ഉയരുകയും ലൂറ്റിനൈസിംഗ് ഹോര്മോണ് (LH), ഫോളിക്കിള് സ്റ്റിമുലേറ്റിംഗ് ഹോര്മോണ് (FSH) എന്നീ ഹോര്മോണുകളെ ഇതിന്റെ ഫലമായി ഉത്പാദിപ്പിക്കുകയും ചെയ്യുന്നു. 24 മുതല് 36 വരെ മണിക്കൂറുകള് നീണ്ടുനില്ക്കുന്ന ഈ പ്രക്രിയയുടെ അന്ത്യം കുറിച്ചുകൊണ്ടാണ് അണ്ഡം വഹിക്കുന്ന പൂര്ണവളര്ച്ചയെത്തിയ ഫോളിക്കിളില്(Ovarian follicle) സ്റ്റിഗ്മയെന്ന് പേരുള്ള ദ്വാരമുണ്ടാവുകയും അത് പൊട്ടി അണ്ഡത്തെ വഹിച്ചുകൊണ്ട് ഫോളിക്കുളാര് ദ്രവം പുറത്തേക്ക് തെറിക്കുകയും ചെയ്യുന്നത്. ഈ പുറത്തേക്കു തെറിക്കല് പ്രക്രിയക്കാണ് അണ്ഡോല്സര്ജനം (Ovulation)എന്നു പറയുക.
ഫോളിക്കുളാര് ദശയിലുടനീളം നടക്കുന്ന അണ്ഡവളര്ച്ചയ്ക്കും അതിന് ഉല്സര്ജിക്കാനാവശ്യമായസംവിധാനങ്ങളൊരുക്കുന്നതിനും നിമിത്തമാകുന്നത് FSHപ്രവര്ത്തനങ്ങളാണ്. പ്രസ്തുത ഉത്പാദനത്തോടനുബന്ധിച്ചാണ് ഹൈപ്പോതലാമസില്നിന്നുള്ള ഗൊണാടോട്രോപിന് റിലീസിംഗ് ഹോര്മോണിന്റെ(GnRH) പ്രേരണയാല് പിറ്റിയൂട്ടറിയില്നിന്ന് LHന്റെ ഉത്പാദനം നടക്കുന്നത്. ഈ ഹോര്മോണ് ഉത്പാദിപ്പിക്കുന്ന പ്രോട്ടീന് വിഘാടക രസങ്ങളായ പ്രോട്ടിയോലിറ്റിക് എന്സൈമുകളാണ്ഫോ(Proteolytic enzymes) ളിക്കിളിലുണ്ടാവുന്ന ദ്വാരമായ സ്റ്റിഗ്മക്ക് കാരണമാകുന്നത്. അണ്ഡോല്സര്ജനത്തിനുശേഷമുള്ള ഫോളിക്കിള് അവശിഷ്ടങ്ങളെ നിയന്ത്രിക്കുന്നതും പ്രധാനമായി ഈ ഹോര്മോണാണ്. ലൂട്ടിയല് ദശയില് അണ്ഡം നഷ്ടപ്പെട്ട ഫോളിക്കിള് അവശിഷ്ടങ്ങള് കോര്പസ് ലൂടിയം(Lorpus Luteum) ആയിത്തീരുകയും മാതൃസ്വഭാവങ്ങളെ ഉദ്ദീപിക്കുന്ന പ്രോജസ്റ്ററോണ് (Progesterone) ഹോര്മോണിന്റെ വര്ധിതമായ ഉത്പാദനത്തിന് നിമിത്തമാവുകയും ചെയ്യുന്നു.
എന്താണീ ലൂറ്റിനൈസിംഗ് ഹോര്മോണ്? മഞ്ഞയെന്ന് അര്ത്ഥം വരുന്ന ലൂറ്റിയസ് (Luteus) എന്ന ലാറ്റിന് പദത്തിന്റെ നപുംസകരൂപമായ ലൂറ്റിയത്തില്നിന്നാണ് (Luteum) ലൂറ്റിനൈസ് (Luteinize)എന്ന ക്രിയയുണ്ടായിരിക്കുന്നത്. കോര്പ്പസ് ലൂടിയത്തിന്റെ നിര്മിതിക്ക് നിമിത്തമായ പ്രവര്ത്തനങ്ങള്ക്കാണ് സാങ്കേതികമായി ലൂറ്റിനൈസ് എന്ന് പറയുന്നതെങ്കിലും പദപരമായി അതിനര്ത്ഥം 'മഞ്ഞയാക്കുന്നത്' എന്നാണ്. ലൂറ്റിനൈസിംഗ് ഹോര്മോണിന്റെ പ്രവര്ത്തനഫലമായാണ് ഫോളിക്കുളാര് ദശ പിന്നിട്ട ഫോളിക്കിള് അവശിഷ്ടങ്ങള് കോര്പസ് ലൂടിയം ആയിത്തീരുന്നത്. കോര്പസ് ലൂടിയം എന്ന പദദ്വയത്തിനര്ത്ഥം മഞ്ഞ വസ്തുവെന്നാണ് (Yellow body). ലൂടിയല് ദശയിലേക്ക് കടന്ന അണ്ഡം നഷ്ടപ്പെട്ട ഫോളിക്കിള് അവശിഷ്ടങ്ങളെല്ലാം കൂടി രണ്ടു മുതല് അഞ്ചു സെന്റീമീറ്റര് വരെ വ്യാസത്തില് ശരീരത്തില് ഏതാനും ദിവസങ്ങള് കൂടി അവശേഷിക്കും. മനുഷ്യരില് ഇത് ഓറഞ്ചു നിറത്തിലാണ് കാണപ്പെടുന്നത്. അണ്ഡോല്സര്ജനത്തിന്റെ അവസാനഘട്ടത്തില് ഉത്പാദിപ്പിക്കപ്പെടുന്ന LH അതിന്റെ പ്രവര്ത്തനമാരംഭിക്കുകയും ഫോളിക്കുളാര് ദ്രവത്തെ മഞ്ഞവല്ക്കരിക്കുകയും ചെയ്യും. ഫോളിക്കിളിലെ സ്റ്റിഗ്മ പൊട്ടി അണ്ഡത്തോടെ പുറത്തേക്ക് തെറിക്കുന്ന ഫോളിക്കുളാര് ദ്രാവകത്തിന്റെ നിറം മഞ്ഞയായിരിക്കും. പുരുഷ ശുക്ലവുമായി താരതമ്യം ചെയ്യുമ്പോള് കട്ടിയില്ലാത്തതും മഞ്ഞ നിറത്തിലുള്ളതുമായ ദ്രാവകമാണ് ഫോളിക്കിള് പൊട്ടി പുറത്തേക്കൊഴുകുന്ന കുഞ്ഞിന്റെ നിര്മാണത്തിന് നിമിത്തമാകുന്ന സ്ത്രീസ്രവം എന്നര്ത്ഥം.
കുഞ്ഞിന്റെ സൃഷ്ടിക്ക് നിമിത്തമാകുന്ന സ്ത്രീസ്രവം മഞ്ഞനിറത്തിലുള്ളതും കട്ടി കുറഞ്ഞതുമാണെന്ന പ്രവാചകവചനം എത്രമാത്രം കൃത്യമാണെന്ന് നമുക്ക് ബോധ്യപ്പെടുന്നത് ഫോളിക്കിള് രൂപാന്തീകരണത്തെക്കുറിച്ച (folliculogenesis) പുതിയ പഠനങ്ങളുടെ വെളിച്ചത്തിലാണ്. കോര്പ്പസ് ലൂടിയത്തെയും ലൂറ്റിനൈസിംഗ് ഹോര്മോണിന്റെ ധര്മത്തെയുമെല്ലാം കുറിച്ച് കൃത്യമായി മനസ്സിലാക്കാന് കഴിഞ്ഞത് കഴിഞ്ഞ നൂറ്റാണ്ടിന്റെ അന്ത്യത്തിലും ഈ നൂറ്റാണ്ടിന്റെ തുടക്കത്തിലുമായി നടന്ന സാങ്കേതിക വിപ്ലവങ്ങളുടെ ഫലമായി ഉണ്ടായിവന്ന സൂക്ഷ്മദര്ശിനികളുപയോഗിച്ചുള്ള പഠനങ്ങള് വഴിയാണ്. ഇപ്പോള് മാത്രം നമുക്ക് മനസ്സിലായ ഇക്കാര്യം എങ്ങനെ പ്രവാചകന്(സ) അറിഞ്ഞുവെന്നതിന് അദ്ദേഹം തന്നെ മറുപടി പറഞ്ഞിട്ടുണ്ട്. 'അയാള് എന്നോട് ചോദിച്ച കാര്യങ്ങളെക്കുറിച്ചൊന്നും അല്ലാഹു അറിയിച്ചുതരുന്നതുവരെ എനിക്ക് യാതൊരു വിവരവുമുണ്ടായിരുന്നില്ല' എന്ന പ്രവാചക പ്രസ്താവനയില്നിന്ന് നബിവചനങ്ങളുടെ സ്രോതസ് എന്താണെന്ന് മനസ്സിലാക്കാനാവും. തന്റെ ചോദ്യങ്ങള്ക്ക് കൃത്യമായി മറുപടി നല്കിയ നബി (സ)യോട് 'താങ്കള് പറഞ്ഞത് സത്യമാണ്; താങ്കളൊരു ദൈവദൂതന് തന്നെയാണ്'(സ്വഹീഹ്മുസ്ലിം) എന്ന് സാക്ഷ്യപ്പെടുത്തിക്കൊണ്ടാണ് ജൂതപണ്ഡിതന് തിരിച്ചുപോയതെന്ന വസ്തുത ശ്രദ്ധേയമാണ്. പൂര്വവേദങ്ങളെക്കുറിച്ച് അറിയാവുന്നവര്ക്ക് മുഹമ്മദ് നബി(സ)യെപ്പറ്റി സ്വന്തം മക്കളെ അറിയുന്നതുപോലെ അറിയാന് കഴിഞ്ഞിരുന്നുവെന്ന ക്വുര്ആന് പ്രസ്താവനയുടെ സത്യത കൂടി ഇവിടെ വെളിപ്പെടുന്നുണ്ട്: ''നാം വേദം നല്കിയിട്ടുള്ളവര്ക്ക് സ്വന്തം മക്കളെ അറിയാവുന്നത് പോലെ അദ്ദേഹത്തെ (റസൂലിനെ) അറിയാവുന്നതാണ്. തീര്ച്ചയായും അവരില് ഒരു വിഭാഗം അറിഞ്ഞുകൊണ്ടുതന്നെ സത്യം മറച്ചുവെക്കുകയാകുന്നു.'' (ക്വുര്ആന് 2:146)
വിഷയവുമായി ബന്ധപ്പെട്ട വീഡിയോ
അല്ല. ആർത്തവരക്തത്തിന് കുഞ്ഞിന്റെ രൂപീകരണത്തിൽ എന്തെങ്കിലും പങ്കുള്ളതായി ഖുർആൻ പഠിപ്പിക്കുന്നില്ല. ആര്ത്തവരക്തത്തെക്കുറിച്ച് ക്വുര്ആന് പരാമര്ശിക്കുന്നത് രണ്ടു തവണയാണ്.
അവ ഇങ്ങനെയാണ്:
''ആര്ത്തവത്തെപ്പറ്റി അവര് നിന്നോട് ചോദിക്കുന്നു. പറയുക; അതൊരു മാലിന്യമാകുന്നു. അതിനാല് ആര്ത്തവഘട്ടത്തില് നിങ്ങള് സ്ത്രീകളില് നിന്ന് അകന്നു നില്ക്കേണ്ടതാണ്. അവര് ശുദ്ധിയാകുന്നത് വരെ അവരെ സമീപിക്കുവാന് പാടില്ല. എന്നാല് അവര് ശുചീകരിച്ചു കഴിഞ്ഞാല് അല്ലാഹു നിങ്ങളോട് കല്പിച്ച വിധത്തില് നിങ്ങള് അവരുടെ അടുത്ത് ചെന്നുകൊള്ളുക. തീര്ച്ചയായും അല്ലാഹു പശ്ചാതപിക്കുന്നവരെ ഇഷ്ടപ്പെടുന്നു. ശുചിത്വം പാലിക്കുന്നവരെയും ഇഷ്ടപ്പെടുന്നു.''(ക്വുര്ആന് 2:222)
''നിങ്ങളുടെ സ്ത്രീകളില് നിന്നും ആര്ത്തവത്തെ സംബന്ധിച്ച് നിരാശപ്പെട്ടിട്ടുള്ളവരെ സംബന്ധിച്ചിടത്തോളം നിങ്ങള് അവരുടെ ഇദ്ദയുടെ കാര്യത്തില് സംശയത്തിലാണെങ്കില് അത് മൂന്ന് മാസമാകുന്നു. ആര്ത്തവമുണ്ടായിട്ടില്ലാത്തവരുടേതും അങ്ങനെ തന്നെ. ഗര്ഭവതികളായ സ്ത്രീകളാകട്ടെ, അവരുടെ അവധി അവര് തങ്ങളുടെ ഗര്ഭം പ്രസവിക്കലാകുന്നു. വല്ലവനും അല്ലാഹുവെ സൂക്ഷിക്കുന്ന പക്ഷം അവന്ന് അവന്റെ കാര്യത്തില് അല്ലാഹു എളുപ്പമുണ്ടാക്കികൊടുക്കുന്നതാണ്.''(ക്വുര്ആന് 65:4)
ആര്ത്തവത്തെക്കുറിച്ച സംശയത്തിന് മറുപടി പറയുമ്പോള് സൂറത്തുല് ബക്വറയിലെ സൂക്തത്തില് അതൊരു മാലിന്യമാണെന്നും അത് പുറപ്പെടുന്ന സന്ദര്ഭത്തില് സ്ത്രീകളുമായി ശാരീരികബന്ധം പാടില്ലെന്നും മാത്രമാണ് പറയുന്നതെന്ന കാര്യം ശ്രദ്ധേയമാണ്. ഇവിടെ കുഞ്ഞിന്റെ രൂപീകരണവുമായി അതിന് ഏതെങ്കിലും തരത്തിലുള്ള ബന്ധമുണ്ടെന്ന് സൂചിപ്പിക്കുന്ന യാതൊരു പരാമര്ശവുമില്ല. സൂറത്തുത്ത്വലാക്വിലെ വചനമാകട്ടെ, ആര്ത്തവവിരാമക്കാരുടെയും ആര്ത്തവമുണ്ടായിട്ടില്ലാത്തവരുടെയും ഇദ്ദ കാലത്തെക്കുറിച്ചുള്ളതാണ്. അവിടെയും ഗര്ഭധാരണത്തെയോ കുഞ്ഞിന്റെ രൂപീകരണത്തെയോ കുറിക്കുന്ന യാതൊന്നും തന്നെ പറഞ്ഞിട്ടില്ല. ആര്ത്തവകാലത്തെയും ആര്ത്തവരക്തത്തെയും കുറിച്ച നിരവധി പരാമര്ശങ്ങള് ഹദീഥുകളിലുണ്ട്. സ്വഹീഹുല് ബുഖാരിയിലെ ആറാമത്തെ അധ്യായവും സ്വഹീഹു മുസ്്ലിമിലെ മൂന്നാം അധ്യായവും 'കിതാബുല് ഹൈദ്വ്' അഥവാ ആര്ത്തവത്തെക്കുറിച്ച അധ്യായങ്ങളാണ്. ബുഖാരി 37 ഹദീഥുകളും മുസ്്ലിം 158 ഹദീഥുകളും ഈ അധ്യായത്തില് നല്കിയിട്ടുണ്ട്. ഇവയില് മിക്കതും കര്മശാസ്ത്ര സംബന്ധിയായ വിഷയങ്ങളാണ് കൈകാര്യം ചെയ്യുന്നത്. സുനനുന്നസാഇയിലെ മൂന്നാം അധ്യായമായ 'കിതാബുല് ഹൈദ്വു വല് ഇസ്തിഹാദ്വ', സുനനു അബൂദാവൂദിലെ ഒന്നാം അധ്യായമായ 'കിതാബുത്ത്വഹാറ', ജാമിഉത്തിര്മിദിയിലെ ഒന്നാം അധ്യായമായ 'കിതാബുത്ത്വഹാറത്തു അന് റസൂലുല്ലാഹി സ്വല്ലല്ലാഹു അലൈഹിവസല്ലം', സുനനു ഇബ്നുമാജയിലെ ഒന്നാം അധ്യായമായ 'കിതാബുത്ത്വഹാറത്തു വസുനനുഹാ', മുവത്വാ മാലിക്കിലെ രണ്ടാം അധ്യായമായ 'കിതാബുത്ത്വഹാറ' എന്നിവയില് ഉദ്ധരിച്ചിരിക്കുന്ന ആര്ത്തവ സംബന്ധിയായ ഹദീഥുകളിലും പ്രധാനമായി പരാമര്ശിച്ചിരിക്കുന്നത് കര്മപരമായ കാര്യങ്ങളെക്കുറിച്ചാണ്. ആര്ത്തവരക്തത്തെക്കുറിച്ചുള്ള നൂറിലധികം വരുന്ന ഹദീഥുകള്ക്കിടയിലെവിടെയും അതിന് കുഞ്ഞിന്റെ രൂപീകരണത്തില് എന്തെങ്കിലും വിധത്തിലുള്ള പങ്കുണ്ടെന്ന് സൂചിപ്പിക്കുന്ന ഒരു പരാമര്ശം പോലുമില്ല.
വിഷയവുമായി ബന്ധപ്പെട്ട വീഡിയോ
ബീജസങ്കലനമെന്ന പദം ഖുർആൻ പ്രയോഗിച്ചിട്ടില്ലെന്നത് ശരിയാണ്. എന്നാൽ, സ്ത്രീ-പുരുഷ ബീജങ്ങളുടെ സംയോജനത്തിൽ നിന്നാണ് കുഞ്ഞുണ്ടാവുന്നതെന്ന വസ്തുതയിലേക്ക് ക്വുർആൻ വെളിച്ചം വീശിയിട്ടുണ്ട്. ഇവ്വിഷയകമായ ഖുർആൻ പരാമർശങ്ങളുടെ കൃത്യതയും സൂക്ഷ്മതയുമറിയണമെങ്കിൽ അക്കാലത്ത് നിലവിലുണ്ടായിരുന്ന സങ്കൽപ്പങ്ങൾ എന്തൊക്കെയായിരുന്നുവെന്ന് മനസ്സിലാക്കണം. പുരുഷ ശുക്ലവും ആര്ത്തവരക്തവും ചേര്ന്നാണ് കുഞ്ഞുണ്ടാവുന്നതെന്ന് കരുതിയ പിപ്പിലാദ ഋഷി മുതല്(1) പാലില്നിന്ന് തൈരുണ്ടാവുന്നതുപോലെ ശുക്ലദ്രാവകം ഘനീഭവിച്ചാണ് ശിശുനിര്മിതി നടക്കുന്നതെന്ന് വിചാരിച്ച ബൈബിളിലെ ഇയ്യോബ്(2) പുസ്തകത്തിന്റെ കര്ത്താവ് വരെയുള്ളവരുടെ വീക്ഷണങ്ങള് വ്യത്യസ്ത അറ്റങ്ങളിലുള്ളവയായിരുന്നു. പുരുഷന്റെയും സ്ത്രീയുടെയും ശുക്ലങ്ങളിലുള്ള ബീജങ്ങള് കൂടിച്ചേര്ന്നാണ് കുഞ്ഞുണ്ടാവുന്നതെന്ന് കരുതിയ ഹിപ്പോക്രാറ്റസ്,(3) മാതൃരക്തത്തെ പുരുഷശുക്ലം ഘനീഭവിപ്പിച്ചാണ് ശിശുവുണ്ടാകുന്നതെന്ന് കരുതിയ അരിസ്റ്റോട്ടില്,(4) ശുക്ലത്തെ മാതൃരക്തം പരിപോഷിപ്പിക്കുമ്പോഴാണ് അതിന്റെ നിര്മിതി നടക്കുന്നതെന്ന് വിചാരിച്ച ഗാലന്(5) എന്നിവരുടെ വീക്ഷണങ്ങള് പാശ്ചാത്യന് വൈജ്ഞാനിക മണ്ഡലത്തില് സജീവമായിരുന്ന കാലത്താണ് ക്വുര്ആന് അവതരിക്കുന്നത്.
“നുത്വ്ഫ (ബീജം) യില് നിന്നാണ് കുഞ്ഞുണ്ടാവുന്നതെന്ന് പറഞ്ഞതോടൊപ്പം തന്നെ കൂടിച്ചേര്ന്നുണ്ടായ നുത്വ്ഫയാണ് ശിശുനിര്മിതിക്ക് നിമിത്തമാവുന്നതെന്നു കൂടി ക്വുര്ആന് വ്യക്തമാക്കുന്നുണ്ട്. ക്വുര്ആന് പറയുന്നത് നോക്കുക: ”നുത്വ്ഫതുന് അംശാജിൽ (കൂട്ടിച്ചെർന്നുണ്ടായ ബീജം) നിന്ന് തീർച്ചയായും നാം മനുഷ്യനെ സൃഷ്ടിച്ചിരിക്കുന്നു; നമുക്ക് അവനെ പരീക്ഷിക്കുവാന്. അങ്ങനെ നാം അവനെ കേള്ക്കുന്നവനും കാണുന്നവനുമാക്കിയിരിക്കുന്നു.’‘(6)
മനുഷ്യസൃഷ്ടി നടന്നത് ‘നുത്വ്ഫതുന് അംശാജി’ല് നിന്നാണെന്നാണ് ഈ വചനത്തില് വ്യക്തമാക്കിയിരിക്കുന്നത്. മീം, ശീന്, ജീം അക്ഷരത്രയത്തില്നിന്ന് നിഷ്പന്നമായ മാശിജിന്റെ ബഹുവചനമാണ് അംശാജ്. കൂട്ടിച്ചേര്ക്കുക, ആശയക്കുഴപ്പത്തിലാക്കുക, ഒന്നിനെ മറ്റൊന്നുമായി ഒന്നിച്ചുചേര്ക്കുക എന്നീ അര്ത്ഥങ്ങളിലാണ് ഈ അക്ഷരത്രയം ഉപയോഗിക്കാറുള്ളത്.(7) ‘നുത്വ്ഫതുന് അംശാജുന്’ എന്നാല് കൂടിച്ചേര്ന്നുണ്ടായ നുത്വ്ഫയെന്നാണ് അര്ത്ഥമെന്ന് ഇത് വ്യക്തമാക്കുന്നു. ക്വുര്ആനില് ഈ വചനത്തിലല്ലാതെ മറ്റൊരിടത്തും ഈ പദം പ്രയോഗിച്ചിട്ടില്ല. പുരുഷബീജവും അണ്ഡവും ചേര്ന്ന സിക്താണ്ഡത്തെ കുറിക്കാനാണ് ക്വുര്ആന് ഇങ്ങനെ പ്രയോഗിച്ചതെന്നാണ് മനസ്സിലാവുന്നത്.
പുരുഷ-സ്ത്രീ സ്രവങ്ങളുടെ സംയോജനത്തില്നിന്നാണ് കുഞ്ഞുണ്ടാവുന്നതെന്ന വസ്തുത പ്രവാചകന്(സ) വ്യക്തമാക്കിയിട്ടുണ്ട്. ശിശുനിര്മിതിയെക്കുറിച്ച ജൂത ചോദ്യത്തിനുള്ള പ്രവാചക മറുപടിയില് ”പുരുഷസ്രവം വെളുത്തതും സ്ത്രീസ്രവം മഞ്ഞയുമാണ്; അവ രണ്ടും കൂടിച്ചേര്ന്നാല്…” എന്നു കാണാം.(8) പുരുഷന്റെ നുത്വ്ഫയും സ്ത്രീയുടെ നുത്വ്ഫയും കൂടിച്ചേര്ന്നുണ്ടാവുന്ന “നുത്വ്ഫയെക്കുറിച്ചാണ് ക്വുര്ആനില് ‘നുത്വ്ഫതിന് അംശാജിന്’’എന്ന് പറഞ്ഞിരിക്കുന്നതെന്ന് ഇതില്നിന്ന് വ്യക്തമാണ്. പ്രത്യുല്പാദനത്തെയും കുഞ്ഞിന്റെ ലിംഗനിര്ണയം, വിധി എന്നിവയെയുമെല്ലാം കുറിച്ച് പ്രതിപാദിക്കുന്ന ഹദീഥുകളിലും സ്ത്രീ-പുരുഷ സ്രവങ്ങളുടെ സംയോജനത്തെക്കുറിച്ച പരാമര്ശങ്ങള് കാണാം. (9)
പ്രവാചകനില്നിന്ന് മതം പഠിച്ച സ്വഹാബിമാര് സ്ത്രീ-പുരുഷ സ്രവങ്ങളുടെ സംയോജനമാണ് “നുത്വ്ഫതിന് അംശാജിന്’ എന്നതുകൊണ്ട് മനസ്സിലാക്കിയതെന്ന് ക്വുര്ആന് വ്യാഖ്യാന ഗ്രന്ഥങ്ങള് വ്യക്തമാക്കുന്നുണ്ട്. ”പുരുഷസ്രവവും സ്ത്രീസ്രവവും; അവ യോജിക്കുമ്പോള്”’ എന്നാണ് ഇബ്നുഅബ്ബാസ്(റ) ഈ വചനത്തെ വ്യാഖ്യാനിച്ചതെന്ന് ഇമാം ത്വബ്രി തന്റെ ജാമിഉല് ബയാന് ഫീ തഫ്സീറില് ക്വുര്ആനില് സമര്ത്ഥിക്കുന്നു.(10) ഇക്രിമ(റ)യാകട്ടെ, “”പുരുഷസ്രവവും സ്ത്രീസ്രവവും; അതിലൊന്ന് മറ്റേതുമായി കൂടിച്ചേരുമ്പോള്” എന്നാണ് ഈ വചനത്തെ വ്യാഖ്യാനിക്കുന്നത്. റബീഉബ്നു അനസ് (റ), ഹസന്(റ), മുജാഹിദ്(റ) എന്നിവരും ഇതേപോലെ തന്നെയാണ് ഈ വചനത്തെ വ്യാഖ്യാനിച്ചതെന്ന് ഇമാം ത്വബ്രി(റ) വിശദീകരിക്കുന്നുണ്ട്. സ്ത്രീയുടെയും പുരുഷന്റെയും നുത്വ്ഫകളുടെ സംയോജനത്തില്നിന്നാണ് കുഞ്ഞുണ്ടാകുന്നതെന്നായിരുന്നു സ്വഹാബിമാരും താബിഉകളുമെല്ലാം മനസ്സിലാക്കിയതെന്ന് ഇമാം റാസി(റ) തന്റെ ക്വുര്ആന് വ്യാഖ്യാനഗ്രന്ഥമായ മഫാതീഹുല് ഗൈബില്, ഈ വചനത്തെ വ്യാഖ്യാനിച്ചുകൊണ്ട് പരാമര്ശിക്കുന്നു.(11)
“നുത്വ്ഫതുന് അംശാജുന് എന്നാല് കൂടിച്ചേര്ന്നുണ്ടായ ബീജം എന്നു തന്നെയാണ് അര്ത്ഥമെന്ന് മുസ്്ലിംകളല്ലാത്ത ക്വുര്ആന് വ്യാഖ്യാതാക്കള് പോലും സമ്മതിക്കുന്നതാണ്. നടേ പറഞ്ഞ സൂക്തത്തിന് പതിനെട്ടാം നൂറ്റാണ്ടുകാരനായ ഓറിയന്റലിസ്റ്റ് ക്വുര്ആന് പരിഭാഷകന് ജോര്ജ് സെയില് നല്കുന്ന പരിഭാഷ ”Verily, we have created man of mingled seed of btoh sexes” എന്നാണ്.(12) ഇരുപതാം നൂറ്റാണ്ടുകാരനായ ബ്രിട്ടീഷ് ഓറിയന്റലിസ്റ്റ് എ. ജെ. ആര്ബെറി ഈ വചനത്തെ പരിഭാഷപ്പെടുത്തിയിരിക്കുന്നത് ”we created man of a spermdrop, a mingling” എന്നാണ്.(13) സ്ത്രീയുടെയും പുരുഷന്റെയും ബീജങ്ങളുടെ സങ്കലനത്തില്നിന്നാണ് അതുണ്ടാവുന്നതെന്ന് ജോര്ജ് സെയില് ഈ വചനത്തില്നിന്ന് മനസ്സിലാക്കിയത് ഏതെങ്കിലും ഇസ്്ലാമിക പ്രബോധകരുടെ സ്വാധീനം കൊണ്ടല്ല, പ്രത്യുത അറബിഭാഷയിലൂടെ ക്വുര്ആന് പഠിച്ചപ്പോള് അദ്ദേഹത്തിന് അങ്ങനെ മനസ്സിലായതാണ്. മുന്ധാരണയില്ലാതെ ക്വുര്ആനെ സമീപിക്കുന്നവര്ക്കെല്ലാം ഈ വചനത്തില്നിന്ന് സ്ത്രീ-പുരുഷ ബീജങ്ങളുടെ സംഗമമാണ് കുഞ്ഞുണ്ടാവുന്നതിന് നിമിത്തമാകുകയെന്നാണ് മനസ്സിലാവുകയെന്ന് സെയ്ലിന്റെ പരിഭാഷ തെര്യപ്പെടുത്തുന്നുണ്ട്.
പുരുഷബീജവും അണ്ഡവും കൂടിച്ചേരുന്ന ബീജസങ്കലന(fertilization)മെന്ന പ്രക്രിയയെക്കുറിച്ച് കൂടുതല് മനസ്സിലാക്കുമ്പോഴാണ് കൂടിച്ചേര്ന്നുണ്ടായ നുത്വ്ഫയെന്ന പ്രയോഗം എത്രമാത്രം കൃത്യമാണെന്ന് ബോധ്യപ്പെടുക. ആര്ത്തവചക്രത്തിന്റെ മധ്യത്തില് നടക്കുന്ന അണ്ഡോല്സര്ജ്ജന(Ovulation)മാണ് പെണ്ശരീരത്തില് നടക്കുന്ന ബീജസങ്കലനത്തിലേക്കുള്ള ആദ്യപടി. അണ്ഡോല്സര്ജ്ജനം കഴിഞ്ഞാല് ഒരു ദിവസത്തിലധികം അണ്ഡം ജീവിച്ചിരിക്കില്ല. അതിനകം ബീജസങ്കലനം നടന്നില്ലെങ്കില് അണ്ഡം നശിച്ചുപോകും. അണ്ഡാശയത്തില്നിന്ന് പുറത്തുവന്ന് ഫലോപ്പിയന് നാളിയിലെത്തി ബീജത്തെ പ്രതീക്ഷിച്ചുകൊണ്ട് നില്ക്കുന്ന അണ്ഡത്തിനടുത്തെത്തുന്ന ഇരുന്നൂറോളം വരുന്ന പുരുഷബീജങ്ങളില് ഒരെണ്ണത്തിന് മാത്രമാണ് അതിന്റെ ‘ഭിത്തി ഭേദിച്ച് അകത്തുകടക്കാനാവുക. ഒരു തവണ സ്ഖലിക്കുന്ന കോടിക്കണക്കിന് ബീജങ്ങളില്നിന്ന് നീന്തി അണ്ഡത്തിനടുത്തെത്തുന്നതില് വിജയിക്കുന്ന ഇരുനൂറോളമെണ്ണത്തില്നിന്ന് ഒരേ ഒരെണ്ണത്തിനുമാത്രം! അണ്ഡത്തെ പൊതിഞ്ഞുനില്ക്കുന്ന മോളിക്കുളാര് കോശങ്ങളുടെ നിരയായ കൊറോണ റേഡിയാറ്റ(Corona radiata)യിലൂടെ വലിഞ്ഞ് അകത്തുകയറി അണ്ഡഭിത്തിയായ സോണ പെല്ലുസിഡ(zona pellucida)യെ ഭേദിച്ച് അണ്ഡകോശദ്രവ്യത്തിനകത്തെത്തുവാന് കെല്പുള്ള ഒരേയൊരു ബീജത്തിനുമാത്രം ലഭിക്കുന്ന അവസരം! ഇങ്ങനെ ഒരു ബീജാണു അകത്തു കയറിക്കഴിഞ്ഞാല് ഉടന് നടക്കുന്ന കോര്ട്ടിക്കല് പ്രതിപ്രവര്ത്തനങ്ങള് (cortical reactions) വഴി പിന്നെയൊരു ബീജവും അണ്ഡത്തിനകത്തേക്ക് കടക്കാത്ത സ്ഥിതി സംജാതമാവുന്നു. അതിനുശേഷമാണ് ബീജകോശകേന്ദ്രത്തിലെ ജനിതക വസ്തുക്കള് അണ്ഡകോശത്തിന്റെ കോശദ്രവ്യത്തില് കലരുകയും അവയും അണ്ഡജനതിക വസ്തുക്കളും തമ്മില് യോജിക്കുകയും ചെയ്യുന്നത്. അണ്ഡകോശത്തിലെ 23 ക്രോമോസോമുകളും ബീജകോശത്തിലെ 23 ക്രോമോസോമുകളും കൂടിച്ചേര്ന്ന് 46 ക്രോമോസോമുകളുള്ള ഒരു പൂര്ണകോശമായിത്തീരുന്ന പ്രക്രിയയാണ് ബീജസങ്കലനം.
ബീജത്തിന്റെയും അണ്ഡത്തിന്റെയും ന്യൂക്ലിയസ്സുകള് ഒരുമിച്ചുചേര്ന്ന് 46 ക്രോമോസോമുകളുള്ള ഒരു പൂര്ണ്ണ ന്യൂക്ലിയസ് ആകുന്നതിന് മുമ്പ് അണ്ഡത്തിന്റെ കോശദ്രവ്യത്തിനകത്ത് രണ്ട് പ്രോന്യൂക്ലിയസുകള് (pronuclei) ഉള്ള ഒരു ഘട്ടമുണ്ട്. ആണ് പ്രോന്യൂക്ലിയസും പെണ് പ്രോന്യൂക്ലിയസും അണ്ഡകോശദ്രവ്യത്തിനകത്ത് സ്ഥിതി ചെയ്യുന്ന ഘട്ടം. ഈ സമയത്തെ സംയോജിത കോശത്തിന്റെ പുരുഷ പ്രോന്യൂക്ലിയസ് ഒഴികെയുള്ള ഭാഗങ്ങളെല്ലാം പഴയ അണ്ഡത്തിന്റേതിനു സമാനമായിരിക്കും. ശുക്ലകോശത്തിന്റെ കോശസ്തരം അണ്ഡത്തിന്റെ സ്തരവുമായി ചേര്ന്ന് അപ്രത്യക്ഷമാവും. ശുക്ലത്തിന്റെ വാലും കോശദ്രവ്യത്തിലുള്ള മൈറ്റോകോണ്ട്രിയയുമെല്ലാം പുരുഷ പ്രോന്യൂക്ലിയസ് ഉണ്ടാകുന്നതോടെ നശിച്ചുപോവും. അതുകൊണ്ടാണ് നമ്മുടെയെല്ലാം -പുരുഷനായാലും സ്ത്രീയായാലും- കോശങ്ങള്ക്കകത്തെ മൈറ്റോകോണ്ട്രിയ നമുക്ക് മാതാവില്നിന്ന് ലഭിച്ചതാണെന്ന് പറയുന്നത്. അഥവാ പുരുഷബീജവും സ്ത്രീബീജവും കൂടിച്ചേര്ന്ന് ഒരു മൂന്നാം വസ്തുവുണ്ടാവുകയല്ല, സ്ത്രീ ബീജത്തിനകത്ത് പുരുഷന്റെ ജനിതകവസ്തുവിന്റെ കൂടിച്ചേരല് നടക്കുക മാത്രമാണ് ബീജസങ്കലനത്തില് സംഭവിക്കുന്നത്. അണ്ഡത്തിന്റെ കോശദ്രവ്യവും കോശസ്തരവും മൈറ്റോകോണ്ട്രിയയുമെല്ലാം തന്നെയാണ് സിക്താണ്ഡത്തിനുമുണ്ടാവുക. അതിന്റെ ന്യൂക്ലിയസിലേക്ക് പുരുഷബീജത്തിന്റെ ജനിതക വസ്തു കൂടിച്ചേരുക മാത്രമാണ് ബീജസങ്കലനത്തില് നടക്കുന്നത്.(14) രണ്ട് അര്ധകോശങ്ങള് ചേര്ന്ന് പൂര്ണകോശമാകുന്ന പ്രക്രിയയെന്ന, ബീജസങ്കലനത്തിന് സാധാരണയായി പറയാറുള്ള നിര്വചനത്തിനുപകരം പൂര്ണകോശത്തിന്റെ കോശദ്രവ്യവും അര്ധന്യൂക്ലിയസുമുള്ള അണ്ഡത്തിലേക്ക് പുരുഷബീജത്തിനകത്തെ അര്ധന്യൂക്ലിയസിലെ ജനിതക വസ്തുവിനെ കടത്തിവിട്ട് അതിനെ പൂര്ണകോശമാക്കുന്ന പ്രക്രിയയാണ് ബീജസങ്കലനം എന്നു പറയുന്നതാകും കൃത്യമായ നിര്വചനം.
ന്യൂക്ലിയസിനെ മാറ്റിനിര്ത്തിയാല് അണ്ഡം ഒരു പൂര്ണകോശം തന്നെയാണ്. പൂര്ണകോശത്തിന്റേതുപോലെയുള്ള ദ്രവ്യവും സ്തരവും മൈറ്റോകോണ്ട്രിയയുമെല്ലാമാണ് അണ്ഡകോശത്തിലുമുള്ളത്. അതിനെ പൂര്ണകോശമാക്കിതീര്ക്കുന്നതിന് ഒരു അര്ധന്യൂക്ലിയസ് കൂടി മാത്രം മതി. പ്രസ്തുത അര്ധന്യൂക്ലിയസാണ് പുരുഷബീജം നല്കുന്നത്. അങ്ങനെ നോക്കുമ്പോള് അണ്ഡത്തിലേക്ക് അര്ധന്യൂക്ലിയസ് കൂട്ടിച്ചേര്ക്കുന്ന പ്രക്രിയയാണ് ബീജസങ്കലനമെന്ന് പറയാം. സ്ത്രീ നുത്വ്ഫയിലേക്ക് പുരുഷ ജനിതകവസ്തുവിനെ കൂട്ടിച്ചേര്ക്കുന്ന പ്രക്രിയ. ഇങ്ങനെ കൂടിച്ചേര്ന്നു കഴിഞ്ഞാലും സ്ത്രീ നുത്വ്ഫ, നുത്വ്ഫ തന്നെയായിരിക്കും. അതിന്റെ കോശദ്രവ്യത്തിനോ സ്തരത്തിനോ ആകൃതിക്കോ മാറ്റങ്ങളൊന്നുമുണ്ടാവുകയില്ല. പുരുഷന്റെ ജനിതക വസ്തു കൂടിച്ചേര്ന്ന് അംശാജ് ആയിത്തീര്ന്നതായിരിക്കും ആ നുത്വ്ഫയെന്നതുമാത്രമാണ് വ്യത്യാസം. മനുഷ്യനെ സൃഷ്ടിച്ചിരിക്കുന്നത് നുത്വ്ഫതുന് അംശാജില്’ നിന്നാണെന്ന ക്വുര്ആന് പരാമര്ശത്തിന്റെ കൃത്യതയാണ് നമുക്കിവിടെ ബോധ്യപ്പെടുന്നത്. പുരുഷജനിതക വസ്തു കൂട്ടിച്ചേര്ത്ത സ്ത്രീ നുത്വ്ഫയില് നിന്നാണല്ലോ നമ്മുടെയെല്ലാം തുടക്കം. പ്രസ്തുത നുത്വ്ഫ വിഭജിക്കപ്പെട്ടാണ് നമ്മുടെ ശരീരവും ഇന്ദ്രിയങ്ങളുമെല്ലാം ഉണ്ടായിട്ടുള്ളത്. ”നുത്വ്ഫത്തിന് അംശാജിന്” എന്ന പ്രയോഗത്തില്നിന്ന് പുരുഷസ്രവത്തിന്റെയും സ്ത്രീസ്രവത്തിന്റെയും സമ്മേളനം വഴിയാണ് കുഞ്ഞുണ്ടാവുന്നതെന്നാണ് സ്വഹാബിമാര് മനസ്സിലാക്കിയതെന്ന് പറയുമ്പോള് അവരാരും തന്നെ ബീജത്തെയും അണ്ഡത്തെയും കുറിച്ച് പറഞ്ഞിട്ടില്ലല്ലോയെന്ന് തര്ക്കിക്കുന്നത് ശുദ്ധ അസംബന്ധമാണ്. നുത്വ്ഫയെന്നാല് പുരുഷ സ്രവത്തിന്റെയോ സ്ത്രീ സ്രവത്തിന്റെയോ ഒരു ഒരു തുള്ളിയോ ചെറിയ അളവോയെന്നാണ് അവര് മനസ്സിലാക്കിയിരുന്നതെന്ന് ഹദീഥുകളില് നിന്ന് നമുക്ക് വ്യക്തമായി. അണ്ഡത്തെ വഹിച്ചുകൊണ്ടുള്ള ഫോളിക്കുളാര് ദ്രാവകത്തിന്റെ ചെറിയൊരു അംശമാണ് അണ്ഡമെന്നും ശുക്ലദ്രാവകത്തിന്റെ ചെറിയൊരു അംശമായ ശുക്ലാണുവാണ് അതുമായി യോജിക്കുന്നതെന്നും ഇന്ന് നമുക്കറിയാം. പരീക്ഷണങ്ങളിലൂടെ ആധുനിക മനുഷ്യര് കണ്ടെത്തിയതെല്ലാം പൗരാണികര്ക്ക് അറിയാമായിരുന്നുവെന്ന് ആരും വാദിക്കുന്നില്ല. ദിവ്യവെളിപാടുകളുടെ അടിസ്ഥാനത്തില് അന്തിമപ്രവാചകന് പറഞ്ഞതൊന്നുംതന്നെ ആധുനികശാസ്ത്രം കണ്ടെത്തുന്ന വസ്തുതകള്ക്ക് എതിരാവുകയില്ലെന്ന് മാത്രമാണ് മുസ്്ലിംകളുടെ വാദം. പുരുഷസ്രവത്തിന്റെയോ സ്ത്രീസ്രവത്തിന്റെയോ ചെറിയൊരു അംശമാണ് നുത്വ്ഫയെന്ന് മനസ്സിലാക്കിയവര് നുത്വ്ഫത്തിന് അംശാജിന് എന്ന പ്രയോഗത്തെ വ്യാഖ്യാനിക്കുമ്പോള് സ്ത്രീസ്രവവും പുരുഷസ്രവവും കൂടിച്ചേര്ന്നുണ്ടാവുന്ന നുത്വ്ഫയില്നിന്നുള്ള മനുഷ്യസൃഷ്ടിയെ കുറിച്ചാണ് ഇവിടെ പറയുന്നതെന്ന് സ്വാഭാവികമായും പരാമര്ശിക്കും. അതില്നിന്ന് പുരുഷസ്രവവും സ്ത്രീസ്രവവും പൂര്ണമായാണ് ശിശുനിര്മിതിയില് പങ്കെടുക്കുന്നതെന്നാണ് അവര് മനസ്സിലാക്കിയതെന്ന് കരുതിക്കൂടാത്തതാണ്. സ്ത്രീസ്രവത്തിന്റെ ഭാഗമായ നുത്വ്ഫയും പുരുഷസ്രവത്തിന്റെ ഭാഗമായ നുത്വ്ഫയും കൂടിച്ചേര്ന്ന നുത്വ്ഫത്തിന് അംശാജിനില്നിന്നാണ് കുഞ്ഞുണ്ടാവുന്നതെന്നാണ് അവര് കരുതിയിരുന്നതെന്നുതന്നെയാണ് അവരുടെ പരാമര്ശങ്ങള് വ്യക്തമാക്കുന്നത്.
പുരുഷബീജത്തെയും അണ്ഡത്തെയും കുറിച്ച് നുത്വ്ഫയെന്ന് പ്രയോഗിച്ച ക്വുര്ആന് സിക്താണ്ഡത്തെ (zygote) കുറിക്കാന് “നുത്വ്ഫത്തിന് അംശാജിന്’ എന്നാണ് പ്രയോഗിച്ചതെന്ന വസ്തുത ശ്രദ്ധേയമാണ്. സ്ത്രീയുടെ നുത്വ്ഫയിലേക്ക് പുരുഷ ജനിതകവസ്തുവിനെ കൂട്ടിച്ചേര്ക്കുന്ന പ്രക്രിയയാണ് ബീജസങ്കലനമെന്ന് നാം മനസ്സിലാക്കി. പ്രസ്തുത പ്രക്രിയ കഴിഞ്ഞ ശേഷമുള്ള ബീജത്തെ കുറിക്കാന് ഏറ്റവും കൃത്യമായ പദം തന്നെയാണ് ക്വുര്ആന് ഉപയോഗിച്ചിരിക്കുന്നത്. കൂട്ടിച്ചേര്ക്കപ്പെട്ട ബീജം-നുത്വ്ഫത്തിന് അംശാജിന്! സര്വജ്ഞനായ അല്ലാഹുവിനല്ലാതെ ആര്ക്കാണ് ഇത്ര കൃത്യമായി പദങ്ങള് പ്രയോഗിക്കാന് കഴിയുക!
- കുറിപ്പുകള്:
- ഗര്ഭോപനിഷത്ത്, വചനങ്ങള് 2,3; ഉപനിഷദ്സര്വസ്വം, തൃശൂര്, 2001, പുറം 63-68.
- ഇയ്യോബ് 10: 9-11.
- Hippocrates: ‘The Seed’, Sections 5-7, Hippocratic Writings, Page 319-320.
- Aristotle: On the Generation of Animals, Montana, 2004, page 3-229.
- Phillip de Lacy: Corpus Medicorum Graecorum: Galeni de Semine (Galen: On Semen) (Greek text with Englisht yrans), Akademie Verlag, 20-Nov-1992, section I: 9:1-10, page 107-109.
- വിശുദ്ധ ഖുര്ആന് 76: 2.
- ലിസാനുല് അറബ്.
- സ്വഹീഹു മുസ്ലിം, കിതാബുല് ഹൈദ്വ്.
- സ്വഹീഹു മുസ്ലിം, കിതാബുണ് ക്വദ്യ്യ.
- തഫ്സീര് അത്ത്വബ്രി 76: 2.
- ഇമാം റാസി: ജാമിഉല് ബയാന് ഫീ തഫ്സീറില് ഖുര്ആന് (http://www.altafsir.com/)
- George Sale : The Koran (Al-Qur’an) (http://www.gutenberg.org/).
- Arthur John Arberry: The Koran Interpreted, Page 315.
- Elaine N. Marieb& Katja Hoehn: Anatomy & Physiology, London, 2012, Pages 1119- 1121.
വിഷയവുമായി ബന്ധപ്പെട്ട വീഡിയോ
വിഷയവുമായി ബന്ധപ്പെട്ട വീഡിയോ
ഏതൊരു വൈജ്ഞാനിക മേഖലയിലേക്കും ക്വുര്ആനും ഹദീഥുകളും നല്കുന്ന വെളിച്ചത്തെക്കുറിച്ച് ചര്ച്ച ചെയ്യുമ്പോള് നാം പ്രാഥമികമായി മനസ്സിരുത്തേണ്ട വസ്തുത, ശാസ്ത്രത്തെക്കുറിച്ചോ ഭൗതിക വിജ്ഞാനീയങ്ങളെകുറിച്ചോ അറിവു നല്കുന്നതിനുവേണ്ടി അവതരിപ്പിക്കപ്പെട്ട വെളിപാടുകളല്ല ഇവയെന്നുള്ളതാണ്. മനുഷ്യരുടെ ജീവിതവിജയത്തിനാവശ്യമായ മാര്ഗനിര്ദേശങ്ങള് നല്കുകയും മരണാനന്തര ജീവിതത്തിലെ ശാശ്വത ശാന്തിയിലേക്ക് അവരെ നയിക്കുകയുമാണ് വെളിപാടുകള് നിര്വഹിക്കുന്ന ധര്മം. പ്രസ്തുത ധര്മ നിര്വഹണത്തിനിടയില്, ചുറ്റുപാടുകളെയും തന്നെ തന്നെയും നിരീക്ഷിച്ചുകൊണ്ട് സര്വ്വലോക സ്രഷ്ടാവിന്റെ അനുഗ്രഹങ്ങളെയും മാര്ഗദര്ശനത്തിന്റെ അനിവാര്യതയെയും കുറിച്ച് സ്വയം ബോധ്യപ്പെടുത്തുവാന് മനുഷ്യരോട് ആഹ്വാനം ചെയ്യുന്നതിനിടയിലാണ് ഭൗതിക വിജ്ഞാനീയങ്ങളിലേക്ക് പ്രധാനമായും ക്വുര്ആനും ഹദീഥുകളും വെളിച്ചം വീശുന്നത്. തലച്ചോറിന്റെ ഉപയോഗത്തിലൂടെ മനുഷ്യര് നേടിയെടുക്കേണ്ട വിവരങ്ങളോ പ്രസ്തുത വിവരങ്ങളുടെ വെളിച്ചത്തില് വികസിപ്പിച്ചെടുക്കേണ്ട സാങ്കേതികവിദ്യയെയോ കുറിച്ച് പഠിപ്പിക്കുകയല്ല, പ്രത്യുത തലച്ചോറിന് മാത്രമായി മനസ്സിലാക്കിയെടുക്കാനാവാത്ത യഥാര്ത്ഥമായ അറിവു നല്കുകയാണ് വെളിപാടുകളുടെ ധര്മം എന്നതുകൊണ്ടുതന്നെ ഭൗതിക വിജ്ഞാനീയങ്ങളുടെ ഏതെങ്കിലുമൊരു ശാഖയെക്കുറിച്ച പൂര്ണമായ വിവരങ്ങളോ വിവരണങ്ങളോ തേടി ക്വുര്ആനിലോ ഹദീഥുകളിലോ പരതുന്നത് വിഡ്ഢിത്തമാണ്.
മസ്തിഷ്കത്തിന് മനസ്സിലാക്കാനാവുന്ന വസ്തുതകളെ ചൂണ്ടിക്കാണിച്ച് അവയുടെ അപഗ്രഥനത്തിലൂടെ മനസ്സിലാക്കാനാവാത്ത ലോകത്തെക്കുറിച്ച് മനസ്സിലാക്കുവാന് മനുഷ്യരോട് പറയുമ്പോള്, പ്രസ്തുത വസ്തുതകളെക്കുറിച്ച പരാമര്ശങ്ങളിലൊന്നും അബദ്ധങ്ങള് കടന്നുവരുന്നില്ലെന്നതാണ് ഈ വെളിപാടുകളുടെ സവിശേഷത. എഴുതപ്പെട്ട കാലത്തെ അറിവില്ലായ്മയുടെ സ്വാധീനമില്ലാത്ത മതപരമോ മതേതരമോ ആയ ഗ്രന്ഥങ്ങളൊന്നുമില്ലെന്ന സ്വാഭാവികതയ്ക്ക് അപവാദമാണ് ക്വുര്ആനും സ്വഹീഹായ ഹദീഥുകളുമെന്ന വസ്തുത വ്യത്യസ്ത വൈജ്ഞാനിക മേഖലകളെക്കുറിച്ച് ഈ വെൡപാടുകളിലുള്ള പരാമര്ശങ്ങളെ ഇന്നു നിലനില്ക്കുന്ന തെളിയിക്കപ്പെട്ട യാഥാര്ത്ഥ്യങ്ങളുമായി താരതമ്യം ചെയ്താല് സുതരാം ബോധ്യപ്പെടും. തെറ്റുപറ്റാത്തവനില്നിന്നുള്ളതാണ് ഈ വെളിപാടുകളെന്ന വസ്തുത വ്യക്തമാക്കുവാന് ഇത്തരം താരതമ്യങ്ങള് നിമിത്തമാകുമെന്നാണ് ഇവ്വിഷയകമായ ഇസ്ലാമിക പ്രബോധകരുടെ അവകാശവാദം.
മനുഷ്യരെ സ്വന്തത്തെപ്പറ്റി ചിന്തിക്കുവാന് പ്രേരിപ്പിച്ചുകൊണ്ട് സ്രഷ്ടാവിന്റെ അസ്തിത്വത്തെയും പുനരുത്ഥാനത്തിന്റെ സത്യതയെയും കുറിച്ച് ബോധ്യപ്പെടുത്തുന്ന ക്വുര്ആന് സൂക്തങ്ങളിലും താന് പ്രവാചകനാണെന്നുള്ള യാഥാര്ത്ഥ്യം വ്യക്തമാക്കിക്കൊണ്ടുള്ള നബിവചനങ്ങളിലുമാണ് മനുഷ്യഭ്രൂണത്തിന്റെ ഉല്പത്തിയെയും പരിണാമത്തെയും കുറിച്ച പരാമര്ശങ്ങളിലധികവും കടന്നുവരുന്നത്. ക്വുര്ആനിലും സ്വഹീഹായ ഹദീഥുകളിലും പ്രതിപാദിക്കപ്പെട്ട ഭ്രൂണശാസ്ത്ര വസ്തുതകളെ ആധുനിക പഠനങ്ങള് വെളിച്ചത്തുകൊണ്ടുവന്ന യാഥാര്ത്ഥ്യങ്ങളുമായി താരതമ്യം ചെയ്തു പഠിക്കുന്നവര്ക്കൊന്നും തന്നെ ഈ സ്രോതസുകളിലുള്ളത് ദൈവിക വെളിപാടാണെന്ന യാഥാര്ത്ഥ്യത്തെ നിഷേധിക്കാനാവുകയില്ല.അതുകൊണ്ടാണല്ലോ, കാനഡയില് ടൊറന്റോ സര്വകലാശാലയിലെ പ്രൊഫസറും അറിയപ്പെടുന്ന ഭ്രൂണശാസ്ത്രജ്ഞനും മെഡിക്കല് കോളേജുകളില് പഠിപ്പിക്കുന്ന ഗ്രന്ഥങ്ങളുടെ കര്ത്താവുമായ ഡോക്ടര് കീത്ത് മൂര് ഇങ്ങനെ പറഞ്ഞത്: ”മനുഷ്യ പ്രത്യുല്പാദനത്തെയും ഭ്രൂണവളര്ച്ചയെയും സംബന്ധിച്ച് വിവരിക്കുന്ന ക്വുര്ആനിലെയും സുന്നത്തിലെയും വചനങ്ങളെ വ്യാഖ്യാനിക്കുവാനായി സുഊദി അറേബ്യയിലെ ജിദ്ദ കിംഗ് അബ്ദുല് അസീസ് സര്വകലാശാലയിലെ ഭ്രൂണശാസ്ത്ര സമിതിയെ സഹായിക്കുവാനും അവരോടൊപ്പം പ്രവര്ത്തിക്കുവാനും കഴിഞ്ഞ മൂന്ന് വര്ഷമായി എനിക്കു സാധിച്ചു. ഭ്രൂണശാസ്ത്രം തന്നെ സ്ഥാപിക്കപ്പെട്ടിട്ടില്ലാത്ത ക്രിസ്താബ്ദം ഏഴാം നൂറ്റാണ്ടില് രേഖപ്പെടുത്തപ്പെട്ട കാര്യങ്ങളുടെ കൃത്യത കണ്ട് ആദ്യമേ തന്നെ അത്ഭുതപരതന്ത്രനായിതീര്ന്നു. ക്രിസ്താബ്ദം പത്താം നൂറ്റാണ്ടില് ജീവിച്ച മുസ്്ലിം ശാസ്ത്രജ്ഞന്മാരുടെ മഹത്തായ ചരിത്രത്തെക്കുറിച്ചും രോഗശുശ്രൂഷാരംഗത്തെ അവരുടെ സംഭാവനകളെക്കുറിച്ചും എനിക്ക് അറിയാമായിരുന്നുവെങ്കിലും ക്വുര്ആനിലും സുന്നത്തിലുമടങ്ങിയിരിക്കുന്ന മതപരമായ കാര്യങ്ങളെപ്പറ്റി എനിക്ക് യാതൊരുവിധ അറിവുമുണ്ടായിരുന്നില്ല.”(L. Keith Moore and Abdul-Majeed al-Zindani: The Developing Human with Islamic Additions, Third Edition, Philadelphia, 1982.)
”മനുഷ്യവളര്ച്ചയെക്കുറിച്ച ക്വുര്ആന് പരാമര്ശങ്ങളെ വ്യക്തമാക്കുവാനായി സഹായിക്കാനാവുകയെന്നത് എന്നെ സംബന്ധിച്ചിടത്തോളം ഏറെ സന്തോഷകരമാണ്. ക്വുര്ആനില് പറഞ്ഞ ഈ വിജ്ഞാനങ്ങളില് ഭൂരിഭാഗവും അതിന്റെ അവതരണത്തിന് ശേഷം നൂറ്റാണ്ടുകള് കഴിഞ്ഞുമാത്രം കണ്ടുപിടിക്കപ്പെട്ടവയാണ് എന്നതുകൊണ്ടുതന്നെ അവ മുഹമ്മദിന് ദൈവത്തില്നിന്ന് അഥവാ അല്ലാഹുവില്നിന്ന് ലഭിച്ചതായിരിക്കുവാനേ നിര്വാഹമുള്ളു. മുഹമ്മദ് ദൈവത്തിന്റെ അഥവാ അല്ലാഹുവിന്റെ ദൂതന് തന്നെയാണെന്ന കാര്യമാണ് ഇത് സമര്ത്ഥിക്കുന്നത്.”(Abdul-Majeed al-Zindani: This is the Truth (video tape).)ഭ്രൂണത്തിന്റെ ഉല്പത്തിയെയും പരിണാമത്തെയും കുറിച്ച് ആധുനികശാസ്ത്രം നമുക്ക് നല്കുന്ന അറിവുകളുടെ വെളിച്ചത്തില് ഈ പരാമര്ശങ്ങള് പഠനവിധേയമാക്കുമ്പോള് ഇതിലെ കൃത്യതയും സൂക്ഷ്മതയും ആരെയും ആശ്ചര്യഭരിതരാക്കുമെന്ന കാര്യത്തില് സംശയമില്ല. ഏതെങ്കിലുമൊരു മനുഷ്യന്റെ തലച്ചോറിനകത്ത് രൂപീകരിക്കപ്പെട്ട ആശയങ്ങളുടെ സമാഹാരമാണ് ക്വുര്ആനെങ്കില് മുഹമ്മദ് നബി (സ)യുടെ കാലത്ത് നിലനിന്നിരുന്ന അബദ്ധധാരണകളിലേതെങ്കിലും ക്വുര്ആനില് ഉണ്ടാവേണ്ടിയിരുന്നു. അത്തരം അബദ്ധങ്ങളൊന്നുമില്ലെന്നു മാത്രമല്ല, ആധുനിക യന്ത്രങ്ങളുടെ സഹായത്തോടെ മാത്രം നാം മനസ്സിലാക്കിയ കാര്യങ്ങള് പോലും വളരെ കൃത്യമായി ക്വുര്ആനിലും ഹദീഥുകളിലും പരാമര്ശിക്കപ്പെടുന്നുവെന്ന വസ്തുത എന്തുമാത്രം അത്ഭുതകരമല്ല! ആധുനികഭ്രൂണശാസ്ത്രത്തിന്റെ കണ്ണടയിലൂടെ ക്വുര്ആനിലും ഹദീഥുകളിലും പരാമര്ശിക്കപ്പെട്ട ഭ്രൂണഘട്ടങ്ങളെ നോക്കുന്ന സത്യസന്ധരായ ആര്ക്കും ഈ സ്രോതസുകളുടെ ദൈവികത നിഷേധിക്കാനാവില്ല. അതുകൊണ്ടാണല്ലോ ജീവിച്ചിരിക്കുന്നവരില് ഏറ്റവും പ്രഗത്ഭനായ ഡോ. കീത്ത് മൂറിനെപ്പോലുള്ള ഒരു ഭ്രൂണശാസ്ത്രജ്ഞനുപോലും അത് സമ്മതിക്കേണ്ടിവന്നത്!